Site Loader

Π‘ΠΎΠ΄Π΅Ρ€ΠΆΠ°Π½ΠΈΠ΅

ВСория(Π‘ΠΈΠ»Π° Π›ΠΎΡ€Π΅Π½Ρ†Π°) — Π‘ΠΈΠ»Π° АмпСра ΠΈ Π‘ΠΈΠ»Π° Π›ΠΎΡ€Π΅Π½Ρ†Π°

Π›ΠžΠ Π•ΠΠ¦, Π₯Π•ΠΠ”Π Π˜Πš АНВОН
НидСрландский Ρ„ΠΈΠ·ΠΈΠΊ

Π‘ΠΈΠ»Π° Π›ΠΎΡ€Π΅Π½Ρ†Π°


Π£Π³ΠΎΠ» Ξ± Π² этом Π²Ρ‹Ρ€Π°ΠΆΠ΅Π½ΠΈΠΈ Ρ€Π°Π²Π΅Π½ ΡƒΠ³Π»Ρƒ ΠΌΠ΅ΠΆΠ΄Ρƒ ΡΠΊΠΎΡ€ΠΎΡΡ‚ΡŒΡŽ ΠΈ Π²Π΅ΠΊΡ‚ΠΎΡ€ΠΎΠΌ ΠΌΠ°Π³Π½ΠΈΡ‚Π½ΠΎΠΉ ΠΈΠ½Π΄ΡƒΠΊΡ†ΠΈΠΈΒ  НаправлСниС силы Π›ΠΎΡ€Π΅Π½Ρ†Π°, Π΄Π΅ΠΉΡΡ‚Π²ΡƒΡŽΡ‰Π΅ΠΉ Π½Π° ΠΏΠΎΠ»ΠΎΠΆΠΈΡ‚Π΅Π»ΡŒΠ½ΠΎ Π·Π°Ρ€ΡΠΆΠ΅Π½Π½ΡƒΡŽ частицу, Ρ‚Π°ΠΊ ΠΆΠ΅, ΠΊΠ°ΠΊ ΠΈ Π½Π°ΠΏΡ€Π°Π²Π»Π΅Π½ΠΈΠ΅ силы АмпСра, ΠΌΠΎΠΆΠ΅Ρ‚ Π±Ρ‹Ρ‚ΡŒ Π½Π°ΠΉΠ΄Π΅Π½ΠΎ ΠΏΠΎ ΠΏΡ€Π°Π²ΠΈΠ»Ρƒ Π»Π΅Π²ΠΎΠΉ Ρ€ΡƒΠΊΠΈ ΠΈΠ»ΠΈ ΠΏΠΎ ΠΏΡ€Π°Π²ΠΈΠ»Ρƒ Π±ΡƒΡ€Π°Π²Ρ‡ΠΈΠΊΠ°. Π’Π·Π°ΠΈΠΌΠ½ΠΎΠ΅ располоТСниС Π²Π΅ΠΊΡ‚ΠΎΡ€ΠΎΠ²

, ΠΈ для ΠΏΠΎΠ»ΠΎΠΆΠΈΡ‚Π΅Π»ΡŒΠ½ΠΎ заряТСнной частицы ΠΏΠΎΠΊΠ°Π·Π°Π½ΠΎ Π½Π° рис.

(Π’Π·Π°ΠΈΠΌΠ½ΠΎΠ΅ располоТСниС Π²Π΅ΠΊΡ‚ΠΎΡ€ΠΎΠ² , ΠΈ ΠœΠΎΠ΄ΡƒΠ»ΡŒ силы Π›ΠΎΡ€Π΅Π½Ρ†Π° числСнно Ρ€Π°Π²Π΅Π½ ΠΏΠ»ΠΎΡ‰Π°Π΄ΠΈ ΠΏΠ°Ρ€Π°Π»Π»Π΅Π»ΠΎΠ³Ρ€Π°ΠΌΠΌΠ°, построСнного Π½Π° Π²Π΅ΠΊΡ‚ΠΎΡ€Π°Ρ… ΠΈ ΠΏΠΎΠΌΠ½ΠΎΠΆΠ΅Π½Π½ΠΎΠΉ Π½Π° заряд q)

Π‘ΠΈΠ»Π° Π›ΠΎΡ€Π΅Π½Ρ†Π° Π½Π°ΠΏΡ€Π°Π²Π»Π΅Π½Π° пСрпСндикулярно Π²Π΅ΠΊΡ‚ΠΎΡ€Π°ΠΌ ΠΈ

ΠŸΡ€ΠΈ Π΄Π²ΠΈΠΆΠ΅Π½ΠΈΠΈ заряТСнной частицы Π² ΠΌΠ°Π³Π½ΠΈΡ‚Π½ΠΎΠΌ ΠΏΠΎΠ»Π΅ сила Π›ΠΎΡ€Π΅Π½Ρ†Π° Ρ€Π°Π±ΠΎΡ‚Ρ‹ Π½Π΅ ΡΠΎΠ²Π΅Ρ€ΡˆΠ°Π΅Ρ‚.
Угловая ΡΠΊΠΎΡ€ΠΎΡΡ‚ΡŒ двиТСния заряТСнной частицы ΠΏΠΎ ΠΊΡ€ΡƒΠ³ΠΎΠ²ΠΎΠΉ Ρ‚Ρ€Π°Π΅ΠΊΡ‚ΠΎΡ€ΠΈΠΈ
называСтся Ρ†ΠΈΠΊΠ»ΠΎΡ‚Ρ€ΠΎΠ½Π½ΠΎΠΉ частотой. Циклотронная частота Π½Π΅ зависит ΠΎΡ‚ скорости (ΡΠ»Π΅Π΄ΠΎΠ²Π°Ρ‚Π΅Π»ΡŒΠ½ΠΎ, ΠΈ ΠΎΡ‚ кинСтичСской энСргии) частицы. Π­Ρ‚ΠΎ ΠΎΠ±ΡΡ‚ΠΎΡΡ‚Π΅Π»ΡŒΡΡ‚Π²ΠΎ ΠΈΡΠΏΠΎΠ»ΡŒΠ·ΡƒΠ΅Ρ‚ΡΡ Π² Ρ†ΠΈΠΊΠ»ΠΎΡ‚Ρ€ΠΎΠ½Π°Ρ… – ускоритСлях тяТСлых частиц (ΠΏΡ€ΠΎΡ‚ΠΎΠ½ΠΎΠ², ΠΈΠΎΠ½ΠΎΠ²). ΠŸΡ€ΠΈΠ½Ρ†ΠΈΠΏΠΈΠ°Π»ΡŒΠ½Π°Ρ схСма Ρ†ΠΈΠΊΠ»ΠΎΡ‚Ρ€ΠΎΠ½Π° ΠΏΡ€ΠΈΠ²Π΅Π΄Π΅Π½Π° Π½Π° рис.
(Π”Π²ΠΈΠΆΠ΅Π½ΠΈΠ΅ заряТСнных частиц Π² Π²Π°ΠΊΡƒΡƒΠΌΠ½ΠΎΠΉ ΠΊΠ°ΠΌΠ΅Ρ€Π΅ Ρ†ΠΈΠΊΠ»ΠΎΡ‚Ρ€ΠΎΠ½Π°)ΠœΠ΅ΠΆΠ΄Ρƒ полюсами сильного элСктромагнита помСщаСтся вакуумная ΠΊΠ°ΠΌΠ΅Ρ€Π°, Π² ΠΊΠΎΡ‚ΠΎΡ€ΠΎΠΉ находятся Π΄Π²Π° элСктрода Π² Π²ΠΈΠ΄Π΅ ΠΏΠΎΠ»Ρ‹Ρ… мСталличСских ΠΏΠΎΠ»ΡƒΡ†ΠΈΠ»ΠΈΠ½Π΄Ρ€ΠΎΠ² (Π΄ΡƒΠ°Π½Ρ‚ΠΎΠ²). К Π΄ΡƒΠ°Π½Ρ‚Π°ΠΌ ΠΏΡ€ΠΈΠ»ΠΎΠΆΠ΅Π½ΠΎ ΠΏΠ΅Ρ€Π΅ΠΌΠ΅Π½Π½ΠΎΠ΅ элСктричСскоС напряТСниС, частота ΠΊΠΎΡ‚ΠΎΡ€ΠΎΠ³ΠΎ Ρ€Π°Π²Π½Π° Ρ†ΠΈΠΊΠ»ΠΎΡ‚Ρ€ΠΎΠ½Π½ΠΎΠΉ частотС.ΠžΠ΄Π½ΠΎΡ€ΠΎΠ΄Π½Ρ‹Π΅ ΠΌΠ°Π³Π½ΠΈΡ‚Π½Ρ‹Π΅ поля ΠΈΡΠΏΠΎΠ»ΡŒΠ·ΡƒΡŽΡ‚ΡΡ Π²ΠΎ ΠΌΠ½ΠΎΠ³ΠΈΡ… ΠΏΡ€ΠΈΠ±ΠΎΡ€Π°Ρ… ΠΈ, Π² частности, Π² масс-спСктромСтрах – устройствах, с ΠΏΠΎΠΌΠΎΡ‰ΡŒΡŽ ΠΊΠΎΡ‚ΠΎΡ€Ρ‹Ρ… ΠΌΠΎΠΆΠ½ΠΎ ΠΈΠ·ΠΌΠ΅Ρ€ΡΡ‚ΡŒ массы заряТСнных частиц – ΠΈΠΎΠ½ΠΎΠ² ΠΈΠ»ΠΈ ядСр Ρ€Π°Π·Π»ΠΈΡ‡Π½Ρ‹Ρ… Π°Ρ‚ΠΎΠΌΠΎΠ². Масс-спСктромСтры ΠΈΡΠΏΠΎΠ»ΡŒΠ·ΡƒΡŽΡ‚ΡΡ для раздСлСния ΠΈΠ·ΠΎΡ‚ΠΎΠΏΠΎΠ², Ρ‚ΠΎ Π΅ΡΡ‚ΡŒ ядСр Π°Ρ‚ΠΎΠΌΠΎΠ² с ΠΎΠ΄ΠΈΠ½Π°ΠΊΠΎΠ²Ρ‹ΠΌ зарядом, Π½ΠΎ Ρ€Π°Π·Π½Ρ‹ΠΌΠΈ массами (Π½Π°ΠΏΡ€ΠΈΠΌΠ΅Ρ€,
20
Ne ΠΈ 22Ne). ΠŸΡ€ΠΎΡΡ‚Π΅ΠΉΡˆΠΈΠΉ масс-спСктромСтр ΠΏΠΎΠΊΠ°Π·Π°Π½ Π½Π° рис.

Β§ 35. Π—ΠΠšΠžΠ ΠΠœΠŸΠ•Π Π. Π‘Π˜Π›Π Π›ΠžΠ Π•ΠΠ¦Π

Π—Π°ΠΊΠΎΠ½ Π‘ΠΈΠΎ-Π‘Π°Π²Π°Ρ€Π°-Лапласа–АмпСра ΡΠΊΡΠΏΠ΅Ρ€ΠΈΠΌΠ΅Π½Ρ‚Π°Π»ΡŒΠ½ΠΎ ΠΏΡ€ΠΎΠ²Π΅Ρ€ΠΈΡ‚ΡŒ нСльзя, Π½ΠΎ слСдствия ΠΈΠ· Π½Π΅Π³ΠΎ ΠΏΠΎΠ΄Ρ‚Π²Π΅Ρ€ΠΆΠ΄Π°ΡŽΡ‚ΡΡ Π½Π° ΠΏΡ€Π°ΠΊΡ‚ΠΈΠΊΠ΅.

Π’ΠΎ всСх Ρ‚ΠΎΡ‡ΠΊΠ°Ρ… пространства, ΠΎΠΊΡ€ΡƒΠΆΠ°ΡŽΡ‰Π΅Π³ΠΎ ΠΏΡ€ΠΎΠΈΠ·Π²ΠΎΠ»ΡŒΠ½Ρ‹ΠΉ Ρ‚ΠΎΠΊ, всСгда сущСствуСт обусловлСнноС этим Ρ‚ΠΎΠΊΠΎΠΌ ΠΏΠΎΠ»Π΅ сил, ΠΊΠΎΡ‚ΠΎΡ€ΠΎΠ΅ ΠΏΠΎ слоТившСйся историчСски Ρ‚Π΅Ρ€ΠΌΠΈΠ½ΠΎΠ»ΠΎΠ³ΠΈΠΈ называСтся ΠΌΠ°Π³Π½ΠΈΡ‚Π½Ρ‹ΠΌ ΠΏΠΎΠ»Π΅ΠΌ.

По Π°Π½Π°Π»ΠΎΠ³ΠΈΠΈ с элСктростатикой ΠΌΠΎΠΆΠ½ΠΎ ввСсти ΡΠΈΠ»ΠΎΠ²ΡƒΡŽ характСристику Ρ‚ΠΎΡ‡ΠΊΠΈ ΠΌΠ°Π³Π½ΠΈΡ‚Π½ΠΎΠ³ΠΎ поля – Π²Π΅ΠΊΡ‚ΠΎΡ€ ΠΌΠ°Π³Π½ΠΈΡ‚Π½ΠΎΠΉ ΠΈΠ½Π΄ΡƒΠΊΡ†ΠΈΠΈ:

β€” Π·Π°ΠΊΠΎΠ½ Π‘ΠΈΠΎ-Π‘Π°Π²Π°Ρ€Π°-Лапласа для расчСта ΠΈΠ½Π΄ΡƒΠΊΡ†ΠΈΠΈ ΠΌΠ°Π³Π½ΠΈΡ‚Π½ΠΎΠ³ΠΎ поля, создаваСмого элСмСнтом Ρ‚ΠΎΠΊΠ° Π² Π½Π΅ΠΊΠΎΡ‚ΠΎΡ€ΠΎΠΉ Ρ‚ΠΎΡ‡ΠΊΠ΅ (рис.69). Π­ΠΊΡΠΏΠ΅Ρ€ΠΈΠΌΠ΅Π½Ρ‚Π°Π»ΡŒΠ½ΠΎ ΠΏΡ€ΠΎΠ²Π΅Ρ€ΠΈΡ‚ΡŒ эту Ρ„ΠΎΡ€ΠΌΡƒΠ»Ρƒ нСльзя, Π½ΠΎ ΠΌΠΎΠΆΠ½ΠΎ Ρ€Π°ΡΡΡ‡ΠΈΡ‚Π°Ρ‚ΡŒ ΠΈΠ½Π΄ΡƒΠΊΡ†ΠΈΡŽ ΠΌΠ°Π³Π½ΠΈΡ‚Π½ΠΎΠ³ΠΎ поля, созданного всСм ΠΊΠΎΠ½Ρ‚ΡƒΡ€ΠΎΠΌ с Ρ‚ΠΎΠΊΠΎΠΌ, ΠΈΡΠΏΠΎΠ»ΡŒΠ·ΡƒΡ установлСнный Π½Π° ΠΎΠΏΡ‹Ρ‚Π΅ ΠΏΡ€ΠΈΠ½Ρ†ΠΈΠΏ супСрпозиции ΠΌΠ°Π³Π½ΠΈΡ‚Π½Ρ‹Ρ… ΠΏΠΎΠ»Π΅ΠΉ: . -лишь Ρ„ΠΎΡ€ΠΌΠ°Π»ΡŒΠ½Π°Ρ запись, Π½Π° ΠΏΡ€Π°ΠΊΡ‚ΠΈΠΊΠ΅ ΠΈΠ½Ρ‚Π΅Π³Ρ€ΠΈΡ€ΠΎΠ²Π°Π½ΠΈΠ΅ Π²ΠΎΠ·ΠΌΠΎΠΆΠ½ΠΎ лишь для ΠΏΡ€ΠΎΠ΅ΠΊΡ†ΠΈΠΉ Π²Π΅ΠΊΡ‚ΠΎΡ€Π° ΠΌΠ°Π³Π½ΠΈΡ‚Π½ΠΎΠΉ ΠΈΠ½Π΄ΡƒΠΊΡ†ΠΈΠΈ.Π’Π» (ВСсла).

Если Π·Π°Π΄Π°Π½Π° объСмная ΠΏΠ»ΠΎΡ‚Π½ΠΎΡΡ‚ΡŒ Ρ‚ΠΎΠΊΠ°,

Ρ‚ΠΎ:

. Π’ΠΎΠ³Π΄Π°

ΠœΠ°Π³Π½ΠΈΡ‚Π½ΠΎΠ΅ ΠΏΠΎΠ»Π΅ пороТдаСтся двиТущимися зарядами(Ρ‚ΠΎΠΊΠ°ΠΌΠΈ). Если ΡΠΊΠΎΡ€ΠΎΡΡ‚ΡŒ Π½Π°ΠΏΡ€Π°Π²Π»Π΅Π½Π½ΠΎΠ³ΠΎ двиТСния зарядов Π² ΠΏΡ€ΠΎΠ²ΠΎΠ΄Π½ΠΈΠΊΠ΅

, Ρ‚ΠΎ . Π’ΠΎΠ³Π΄Π°:

Π˜Π½Π΄ΡƒΠΊΡ†ΠΈΡŽ ΠΌΠ°Π³Π½ΠΈΡ‚Π½ΠΎΠ³ΠΎ поля Ρ‚ΠΎΡ‡Π΅Ρ‡Π½ΠΎΠ³ΠΎ заряда, двиТущСгося с постоянной нСрСлятивистской ΡΠΊΠΎΡ€ΠΎΡΡ‚ΡŒΡŽ (рис.70) ΠΌΠΎΠΆΠ½ΠΎ ΠΎΠΏΡ€Π΅Π΄Π΅Π»ΠΈΡ‚ΡŒ ΠΏΠΎ Ρ„ΠΎΡ€ΠΌΡƒΠ»Π΅:

Π’Π·Π΅ΠΌΠ»ΠΈ~5*10-5Π’Π», Π’ΠΌΠΎΠ·Π³Π°~10-11Π’Π».

Π’max ~150 Π’Π» β€” ΠΏΠΎΠ»ΡƒΡ‡Π΅Π½Π° Π² Π²ΠΈΠ΄Π΅ ΠΈΠΌΠΏΡƒΠ»ΡŒΡΠ°.

БАМОБВОЯВ. XI: Ρ€Π°ΡΡΡ‡ΠΈΡ‚Π°Ρ‚ΡŒ ΠΈΠ½Π΄ΡƒΠΊΡ†ΠΈΡŽ ΠΌΠ°Π³Π½ΠΈΡ‚Π½ΠΎΠ³ΠΎ поля: 1)бСсконСчного Π΄Π»ΠΈΠ½Π½ΠΎΠ³ΠΎ прямого ΠΏΡ€ΠΎΠ²ΠΎΠ΄Π½ΠΈΠΊΠ° с Ρ‚ΠΎΠΊΠΎΠΌ I Π² Ρ‚ΠΎΡ‡ΠΊΠ΅ Π½Π° расстоянии b ΠΎΡ‚ Π½Π΅Π³ΠΎ;

2)полубСсконСчного Π΄Π»ΠΈΠ½Π½ΠΎΠ³ΠΎ прямого ΠΏΡ€ΠΎΠ²ΠΎΠ΄Π½ΠΈΠΊΠ° с Ρ‚ΠΎΠΊΠΎΠΌ I Π² Ρ‚ΠΎΡ‡ΠΊΠ΅ Π½Π° расстоянии b ΠΎΡ‚ Π½Π΅Π³ΠΎ.

Π’ Π·Π°ΠΊΠΎΠ½Π΅ Π‘ΠΈΠΎ-Π‘Π°Π²Π°Ρ€Π°-Лапласа-АмпСра Ρ€Π°ΡΡΠΌΠ°Ρ‚Ρ€ΠΈΠ²Π°Π»ΠΎΡΡŒ взаимодСйствиС элСмСнтов Ρ‚ΠΎΠΊΠΎΠ² Π΄Π²ΡƒΡ… ΠΊΠΎΠ½Ρ‚ΡƒΡ€ΠΎΠ².

Π’Ρ‹Ρ€Π°ΠΆΠ΅Π½ΠΈΠ΅

опрСдСляСт ΠΈΠ½Π΄ΡƒΠΊΡ†ΠΈΡŽ ΠΌΠ°Π³Π½ΠΈΡ‚Π½ΠΎΠ³ΠΎ поля, созданного элСмСнтом Ρ‚ΠΎΠΊΠ° Π² мСстС располоТСния элСмСнта Ρ‚ΠΎΠΊΠ° .

Π˜ΡΠΏΠΎΠ»ΡŒΠ·ΡƒΡ ΠΏΡ€ΠΈΠ½Ρ†ΠΈΠΏ супСрпозиции ΠΌΠ°Π³Π½ΠΈΡ‚Π½Ρ‹Ρ… ΠΏΠΎΠ»Π΅ΠΉ, ΠΌΠΎΠΆΠ½ΠΎ Π½Π°ΠΉΡ‚ΠΈ ΠΈΠ½Π΄ΡƒΠΊΡ†ΠΈΡŽ ΠΌΠ°Π³Π½ΠΈΡ‚Π½ΠΎΠ³ΠΎ поля, создаваСмого всСм ΠΏΠ΅Ρ€Π²Ρ‹ΠΌ ΠΊΠΎΠ½Ρ‚ΡƒΡ€ΠΎΠΌ с Ρ‚ΠΎΠΊΠΎΠΌ Π² мСстС располоТСния Π²Ρ‚ΠΎΡ€ΠΎΠ³ΠΎ элСмСнта Ρ‚ΠΎΠΊΠ°. Π’ этом случаС Π½Π° Π²Ρ‚ΠΎΡ€ΠΎΠΉ элСмСнт Ρ‚ΠΎΠΊΠ° Π±ΡƒΠ΄Π΅Ρ‚ Π΄Π΅ΠΉΡΡ‚Π²ΠΎΠ²Π°Ρ‚ΡŒ сила

.

Π‘ΠΈΠ»Π°, Π΄Π΅ΠΉΡΡ‚Π²ΡƒΡŽΡ‰Π°Ρ Π½Π° элСмСнт Ρ‚ΠΎΠΊΠ° Π² ΠΌΠ°Π³Π½ΠΈΡ‚Π½ΠΎΠΌ ΠΏΠΎΠ»Π΅, называСтся силой АмпСра, Π° Ρ„ΠΎΡ€ΠΌΡƒΠ»Π°, ΠΏΠΎΠ·Π²ΠΎΠ»ΡΡŽΡ‰Π°Ρ Ρ€Π°ΡΡΡ‡ΠΈΡ‚Π°Ρ‚ΡŒ эту силу – Π·Π°ΠΊΠΎΠ½ АмпСра:

Π’Π°ΠΊ ΠΊΠ°ΠΊ

, Ρ‚ΠΎ Π·Π°ΠΊΠΎΠ½ АмпСра ΠΌΠΎΠΆΠ΅Ρ‚ Π±Ρ‹Ρ‚ΡŒ записан Π² Π²ΠΈΠ΄Π΅:

Π˜Π½Ρ‚Π΅Π³Ρ€ΠΈΡ€ΡƒΡ эти выраТСния ΠΏΠΎ ΠΎΠ±ΡŠΠ΅ΠΌΠ½Ρ‹ΠΌ ΠΈΠ»ΠΈ Π»ΠΈΠ½Π΅ΠΉΠ½Ρ‹ΠΌ элСмСнтам Ρ‚ΠΎΠΊΠ°, ΠΌΠΎΠΆΠ½ΠΎ Π½Π°ΠΉΡ‚ΠΈ силу, Π΄Π΅ΠΉΡΡ‚Π²ΡƒΡŽΡ‰ΡƒΡŽ Π½Π° Ρ‚ΠΎΡ‚ ΠΈΠ»ΠΈ ΠΈΠ½ΠΎΠΉ объСм ΠΏΡ€ΠΎΠ²ΠΎΠ΄Π½ΠΈΠΊΠ° ΠΈΠ»ΠΈ Π΅Π³ΠΎ Π»ΠΈΠ½Π΅ΠΉΠ½Ρ‹ΠΉ участок.

Π­ΠΊΡΠΏΠ΅Ρ€ΠΈΠΌΠ΅Π½Ρ‚Π°Π»ΡŒΠ½ΠΎ ΠΏΠΎΠΊΠ°Π·Π°Π½ΠΎ, Ρ‡Ρ‚ΠΎ ΠΌΠ°Π³Π½ΠΈΡ‚Π½ΠΎΠ΅ ΠΏΠΎΠ»Π΅ Ρ‚Π°ΠΊΠΆΠ΅ дСйствуСт Π½Π° двиТущиСся заряды. Π‘ΠΈΠ»Π°, Π΄Π΅ΠΉΡΡ‚Π²ΡƒΡŽΡ‰Π°Ρ Π½Π° двиТущийся элСктричСский заряд со стороны элСктромагнитного поля, называСтся силой Π›ΠΎΡ€Π΅Π½Ρ†Π°.

ΠŸΠΎΠ»ΡƒΡ‡ΠΈΠΌ Ρ„ΠΎΡ€ΠΌΡƒΠ»Ρƒ для ΠΌΠ°Π³Π½ΠΈΡ‚Π½ΠΎΠΉ ΡΠΎΡΡ‚Π°Π²Π»ΡΡŽΡ‰Π΅ΠΉ силы Π›ΠΎΡ€Π΅Π½Ρ†Π°. Π˜ΡΠΏΠΎΠ»ΡŒΠ·ΡƒΠ΅ΠΌ для этого Ρ„ΠΎΡ€ΠΌΡƒΠ»Ρƒ для силы АмпСра, Π΄Π΅ΠΉΡΡ‚Π²ΡƒΡŽΡ‰Π΅ΠΉ Π½Π° элСмСнт Ρ‚ΠΎΠΊΠ° Π² ΠΌΠ°Π³Π½ΠΈΡ‚Π½ΠΎΠΌ ΠΏΠΎΠ»Π΅:

Если Ρ‚ΠΎΠΊ прСкращаСтся, Ρ‚ΠΎ исчСзаСт сила АмпСра, Π½ΠΎ сила Ρ‚ΠΎΠΊΠ°

, Π³Π΄Π΅ q0 β€” Π²Π΅Π»ΠΈΡ‡ΠΈΠ½Π° свободного заряда, Π° N – число свободных зарядов, проходящих Ρ‡Π΅Ρ€Π΅Π· ΠΏΠΎΠΏΠ΅Ρ€Π΅Ρ‡Π½ΠΎΠ΅ сСчСниС ΠΏΡ€ΠΎΠ²ΠΎΠ΄Π½ΠΈΠΊΠ° Π·Π° dt.

Если срСдняя ΡΠΊΠΎΡ€ΠΎΡΡ‚ΡŒ Π½Π°ΠΏΡ€Π°Π²Π»Π΅Π½Π½ΠΎΠ³ΠΎ двиТСния свободных зарядов

, Ρ‚ΠΎ β€” сила, Π΄Π΅ΠΉΡΡ‚Π²ΡƒΡŽΡ‰Π°Ρ со стороны ΠΌΠ°Π³Π½ΠΈΡ‚Π½ΠΎΠ³ΠΎ поля Π½Π° двиТущийся заряд.

Если заряд двигаСтся Π² пространствС, Π² ΠΊΠΎΡ‚ΠΎΡ€ΠΎΠΌ ΡΡƒΡ‰Π΅ΡΡ‚Π²ΡƒΡŽΡ‚ ΠΎΠ΄Π½ΠΎΠ²Ρ€Π΅ΠΌΠ΅Π½Π½ΠΎ элСктричСскоС ΠΈ ΠΌΠ°Π³Π½ΠΈΡ‚Π½ΠΎΠ΅ ΠΏΠΎΠ»Π΅, Ρ‚ΠΎ Π½Π° Π½Π΅Π³ΠΎ дСйствуСт сила Π›ΠΎΡ€Π΅Π½Ρ†Π°:

Π‘ΠΈΠ»Π° Π›ΠΎΡ€Π΅Π½Ρ†Π° являСтся ΠΏΡ€ΠΈΡ‡ΠΈΠ½ΠΎΠΉ появлСния силы АмпСра.

Π’Π΅ΠΊΡ‚ΠΎΡ€ ΠΌΠ°Π³Π½ ΠΈΠ½Π΄ΡƒΠΊΡ†ΠΈΠΈ | ΠžΡ‚ ΡƒΡ€ΠΎΠΊΠ° Π΄ΠΎ экзамСна

ΠŸΡ€ΠΈ ΠΏΡ€ΠΎΡ…ΠΎΠΆΠ΄Π΅Π½ΠΈΠΈ Ρ‚ΠΎΠΊΠ° ΠΏΠΎ ΠΏΡ€ΠΎΠ²ΠΎΠ΄Π½ΠΈΠΊΡƒ Π²ΠΎΠΊΡ€ΡƒΠ³ Π½Π΅Π³ΠΎ образуСтся ΠΌΠ°Π³Π½ΠΈΡ‚Π½ΠΎΠ΅ ΠΏΠΎΠ»Π΅. Π’Π΅ΠΊΡ‚ΠΎΡ€Π½ΡƒΡŽ характСристику ΠΌΠ°Π³Π½ΠΈΡ‚Π½ΠΎΠ³ΠΎ поля Π½Π°Π·Ρ‹Π²Π°ΡŽΡ‚ Π²Π΅ΠΊΡ‚ΠΎΡ€ΠΎΠΌ ΠΌΠ°Π³Π½ΠΈΡ‚Π½ΠΎΠΉ ΠΈΠ½Π΄ΡƒΠΊΡ†ΠΈΠΈ . Π­Ρ‚ΠΎ ΠΏΠΎΠ»Π΅ ΠΎΠΊΠ°Π·Ρ‹Π²Π°Π΅Ρ‚ Π½Π° Ρ€Π°ΠΌΠΊΡƒ с Ρ‚ΠΎΠΊΠΎΠΌ, ΠΏΠΎΠΌΠ΅Ρ‰Π΅Π½Π½ΡƒΡŽ Π² ΠΏΠΎΠ»Π΅, ΠΎΡ€ΠΈΠ΅Π½Ρ‚ΠΈΡ€ΡƒΡŽΡ‰Π΅Π΅ дСйствиС. Π’Π°ΠΊΠΎΠ΅  дСйствиСм ΠΌΠ°Π³Π½ΠΈΡ‚Π½ΠΎΠ³ΠΎ поля Π½Π° Ρ€Π°ΠΌΠΊΡƒ с Ρ‚ΠΎΠΊΠΎΠΌ ΠΈΠ»ΠΈ ΠΌΠ°Π³Π½ΠΈΡ‚Π½ΡƒΡŽ стрСлку ΠΌΠΎΠΆΠ½ΠΎ ΠΈΡΠΏΠΎΠ»ΡŒΠ·ΠΎΠ²Π°Ρ‚ΡŒ для опрСдСлСния направлСния Π²Π΅ΠΊΡ‚ΠΎΡ€Π° ΠΌΠ°Π³Π½ΠΈΡ‚Π½ΠΎΠΉ ΠΈΠ½Π΄ΡƒΠΊΡ†ΠΈΠΈ. Π—Π°  принимаСтся Π½Π°ΠΏΡ€Π°Π²Π»Π΅Π½ΠΈΠ΅, ΠΊΠΎΡ‚ΠΎΡ€Ρ‹ΠΉ ΠΏΠΎΠΊΠ°Π·Ρ‹Π²Π°Π΅Ρ‚ сСвСрный полюс

N ΠΌΠ°Π³Π½ΠΈΡ‚Π½ΠΎΠΉ стрСлки. Для опрСдСлСния направлСния Π²Π΅ΠΊΡ‚ΠΎΡ€Π° ΠΌΠ°Π³Π½ΠΈΡ‚Π½ΠΎΠΉ ΠΈΠ½Π΄ΡƒΠΊΡ†ΠΈΠΈ поля, созданного прямолинСйным ΠΏΡ€ΠΎΠ²ΠΎΠ΄Π½ΠΈΠΊΠΎΠΌ с Ρ‚ΠΎΠΊΠΎΠΌ, ΠΏΠΎΠ»ΡŒΠ·ΡƒΡŽΡ‚ΡΡ ΠΏΡ€Π°Π²ΠΈΠ»ΠΎΠΌ Π±ΡƒΡ€Π°Π²Ρ‡ΠΈΠΊΠ°: Ссли Π½Π°ΠΏΡ€Π°Π²Π»Π΅Π½ΠΈΠ΅ ΠΏΠΎΡΡ‚ΡƒΠΏΠ°Ρ‚Π΅Π»ΡŒΠ½ΠΎΠ³ΠΎΒ Π΄Π²ΠΈΠΆΠ΅Π½ΠΈΡ Π±ΡƒΡ€Π°Π²Ρ‡ΠΈΠΊΠ° совпадаСт с Π½Π°ΠΏΡ€Π°Π²Π»Π΅Π½ΠΈΠ΅ΠΌ Ρ‚ΠΎΠΊΠ° Π² ΠΏΡ€ΠΎΠ²ΠΎΠ΄Π½ΠΈΠΊΠ΅, Ρ‚ΠΎ Π½Π°ΠΏΡ€Π°Π²Π»Π΅Π½ΠΈΠ΅ вращСния Ρ€ΡƒΡ‡ΠΊΠΈ Π±ΡƒΡ€Π°Π²Ρ‡ΠΈΠΊΠ° ΡƒΠΊΠ°Π·Ρ‹Π²Π°Π΅Ρ‚ Π½Π°ΠΏΡ€Π°Π²Π»Π΅Π½ΠΈΠ΅ Π²Π΅ΠΊΡ‚ΠΎΡ€Π° ΠΌΠ°Π³Π½ΠΈΡ‚Π½ΠΎΠΉ ΠΈΠ½Π΄ΡƒΠΊΡ†ΠΈΠΈ.

Π½Π°ΠΏΡ€Π°Π²Π»Π΅Π½ΠΈΠ΅ Π²Π΅ΠΊΡ‚ΠΎΡ€Π° ΠΌΠ°Π³Π½ΠΈΡ‚Π½ΠΎΠ³ΠΎ поля прямого ΠΏΡ€ΠΎΠ²ΠΎΠ΄Π½ΠΈΠΊΠ° с Ρ‚ΠΎΠΊΠΎΠΌ.

Если ΠΌΠ΅ΠΆΠ΄Ρƒ полюсами ΠΏΠΎΠ΄ΠΊΠΎΠ²ΠΎΠΎΠ±Ρ€Π°Π·Π½ΠΎΠ³ΠΎ ΠΌΠ°Π³Π½ΠΈΡ‚Π° ΠΏΠΎΠΌΠ΅ΡΡ‚ΠΈΡ‚ΡŒ ΠΏΡ€ΠΎΠ²ΠΎΠ΄Π½ΠΈΠΊ с Ρ‚ΠΎΠΊΠΎΠΌ, Ρ‚ΠΎ ΠΎΠ½ Π±ΡƒΠ΄Π΅Ρ‚ Π²Ρ‚ΡΠ³ΠΈΠ²Π°Ρ‚ΡŒΡΡ ΠΈΠ»ΠΈ Π²Ρ‹Ρ‚Π°Π»ΠΊΠΈΠ²Π°Ρ‚ΡŒΡΡ ΠΈΠ· поля ΠΌΠ°Π³Π½ΠΈΡ‚Π°. Π—Π°ΠΊΠΎΠ½, ΠΎΠΏΡ€Π΅Π΄Π΅Π»ΡΡŽΡ‰ΠΈΠΉ силу, Π΄Π΅ΠΉΡΡ‚Π²ΡƒΡŽΡ‰ΡƒΡŽ Π½Π° ΠΎΡ‚Π΄Π΅Π»ΡŒΠ½Ρ‹ΠΉ нСбольшой участок ΠΏΡ€ΠΎΠ²ΠΎΠ΄Π½ΠΈΠΊΠ°, Π±Ρ‹Π» установлСн Π² 1820 Π³. А. АмпСром.Β 

Π‘ΠΈΠ»Π° дСйствия ΠΎΠ΄Π½ΠΎΡ€ΠΎΠ΄Π½ΠΎΠ³ΠΎ ΠΌΠ°Π³Β­Π½ΠΈΡ‚Π½ΠΎΠ³ΠΎ поля Π½Π° ΠΏΡ€ΠΎΠ²ΠΎΠ΄Π½ΠΈΠΊ с Ρ‚ΠΎΠΊΠΎΠΌ прямо ΠΏΡ€ΠΎΠΏΠΎΡ€Ρ†ΠΈΠΎΠ½Π°Π»ΡŒΠ½Π° силС Ρ‚ΠΎΠΊΠ°, Π΄Π»ΠΈΠ½Π΅ ΠΏΡ€ΠΎΠ²ΠΎΠ΄Π½ΠΈΠΊΠ°, ΠΌΠΎΠ΄ΡƒΠ»ΡŽ Π²Π΅ΠΊΡ‚ΠΎΡ€Π° ΠΈΠ½Π΄ΡƒΠΊΡ†ΠΈΠΈ ΠΌΠ°Π³Π½ΠΈΡ‚Π½ΠΎΠ³ΠΎ поля, синусу ΡƒΠ³Π»Π° ΠΌΠ΅ΠΆΠ΄Ρƒ Π²Π΅ΠΊΡ‚ΠΎΡ€ΠΎΠΌ ΠΈΠ½Π΄ΡƒΠΊΡ†ΠΈΠΈ ΠΌΠ°Π³Π½ΠΈΡ‚Π½ΠΎΠ³ΠΎ поля ΠΈ ΠΏΡ€ΠΎΠ²ΠΎΠ΄Π½ΠΈΠΊΠΎΠΌ:

F=B.I.β„“.Β sinΒ Ξ± Β β€”Β Π·Π°ΠΊΠΎΠ½ АмпСра.

  • Π‘ΠΈΠ»Π° АмпСра максимальна, Ссли Π²Π΅ΠΊΡ‚ΠΎΡ€ ΠΌΠ°Π³Π½ΠΈΡ‚Π½ΠΎΠΉ ΠΈΠ½Π΄ΡƒΠΊΡ†ΠΈΠΈ пСрпСндикулярСн ΠΏΡ€ΠΎΠ²ΠΎΠ΄Π½ΠΈΠΊΡƒ.
  • Если Π²Π΅ΠΊΡ‚ΠΎΡ€ ΠΌΠ°Π³Π½ΠΈΡ‚Π½ΠΎΠΉ ΠΈΠ½Π΄ΡƒΠΊΡ†ΠΈΠΈ ΠΏΠ°Ρ€Π°Π»Π»Π΅Π»Π΅Π½ ΠΏΡ€ΠΎΠ²ΠΎΠ΄Π½ΠΈΠΊΡƒ, Ρ‚ΠΎ ΠΌΠ°Π³Π½ΠΈΡ‚Π½ΠΎΠ΅ ΠΏΠΎΠ»Π΅ Π½Π΅ ΠΎΠΊΠ°Π·Ρ‹Π²Π°Π΅Ρ‚ Π½ΠΈΠΊΠ°ΠΊΠΎΠ³ΠΎ дСйствия Π½Π° ΠΏΡ€ΠΎΠ²ΠΎΠ΄Π½ΠΈΠΊ с Ρ‚ΠΎΠΊΠΎΠΌ, Ρ‚.Π΅. сила АмпСра Ρ€Π°Π²Π½Π° Π½ΡƒΠ»ΡŽ.

НаправлСниС силы АмпСра (ΠΏΡ€Π°Π²ΠΈΠ»ΠΎ Π»Π΅Π²ΠΎΠΉ Ρ€ΡƒΠΊΠΈ) Если Π»Π΅Π²ΡƒΡŽ Ρ€ΡƒΠΊΡƒ Ρ€Π°ΡΠΏΠΎΠ»ΠΎΠΆΠΈΡ‚ΡŒ Ρ‚Π°ΠΊ, Ρ‡Ρ‚ΠΎΠ±Ρ‹ пСрпСндикулярная ΡΠΎΡΡ‚Π°Π²Π»ΡΡŽΡ‰Π°Ρ Π²Π΅ΠΊΡ‚ΠΎΡ€Π°Β Π’Β Π²Ρ…ΠΎΠ΄ΠΈΠ»Π° Π² ладонь, Π° Ρ‡Π΅Ρ‚Ρ‹Ρ€Π΅ вытянутых ΠΏΠ°Π»ΡŒΡ†Π° Π±Ρ‹Π»ΠΈ Π½Π°ΠΏΡ€Π°Π²Π»Π΅Π½Ρ‹ ΠΏΠΎ Π½Π°ΠΏΡ€Π°Π²Π»Π΅Π½ΠΈΡŽ Ρ‚ΠΎΠΊΠ°, Ρ‚ΠΎ ΠΎΡ‚ΠΎΠ³Π½ΡƒΡ‚Ρ‹ΠΉ Π½Π°Β 90° большой ΠΏΠ°Π»Π΅Ρ† ΠΏΠΎΠΊΠ°ΠΆΠ΅Ρ‚ Π½Π°ΠΏΡ€Π°Π²Π»Π΅Π½ΠΈΠ΅ силы, Π΄Π΅ΠΉΡΡ‚Π²ΡƒΡŽΡ‰Π΅ΠΉ Π½Π° ΠΏΡ€ΠΎΠ²ΠΎΠ΄Π½ΠΈΠΊ с Ρ‚ΠΎΠΊΠΎΠΌ.

ΠœΠ°ΠΊΡ€ΠΎΡΠΊΠΎΠΏΠΈΡ‡Π΅ΡΠΊΠΈΠΌ проявлСниСм силы Π›ΠΎΡ€Π΅Π½Ρ†Π° являСтся сила АмпСра. Π—Π°ΠΏΠΈΡˆΠ΅ΠΌΒ ΡΠΈΠ»Ρƒ, Π΄Π΅ΠΉΡΡ‚Π²ΡƒΡŽΡ‰ΡƒΡŽ Π½Π° ΠΎΠ΄Π½Ρƒ частицу. Если заряТСнная частица Π²Π»Π΅Ρ‚Π°Π΅Ρ‚ Π² ΠΌΠ°Π³Π½ΠΈΡ‚Π½ΠΎΠ΅ ΠΏΠΎΠ»Π΅ со ΡΠΊΠΎΡ€ΠΎΡΡ‚ΡŒΡŽΒ , Π½Π° Π½Π΅Π΅ со стороны ΠΌΠ°Π³Π½ΠΈΡ‚Π½ΠΎΠ³ΠΎ поля дСйствуСт сила, ΠΊΠΎΡ‚ΠΎΡ€ΡƒΡŽ Π½Π°Π·Ρ‹Π²Π°ΡŽΡ‚ силой Π›ΠΎΡ€Π΅Π½Ρ†Π°:Β ,Β 

a – ΡƒΠ³ΠΎΠ» ΠΌΠ΅ΠΆΠ΄Ρƒ Π²Π΅ΠΊΡ‚ΠΎΡ€Π°ΠΌΠΈ ΠΈ .Β 

  • Π’ ΠΎΠ΄Π½ΠΎΡ€ΠΎΠ΄Π½ΠΎΠΌ ΠΌΠ°Π³Π½ΠΈΡ‚Π½ΠΎΠΌ ΠΏΠΎΠ»Π΅, Π½Π°ΠΏΡ€Π°Π²Π»Π΅Π½Π½ΠΎΠΌ пСрпСндикулярно Π²Π΅ΠΊΡ‚ΠΎΡ€Ρƒ скорости, ΠΏΠΎΠ΄ дСйствиСм силы Π›ΠΎΡ€Π΅Π½Ρ†Π° заряТСнная частица Π±ΡƒΠ΄Π΅Ρ‚ Ρ€Π°Π²Π½ΠΎΠΌΠ΅Ρ€Π½ΠΎ Π΄Π²ΠΈΠ³Π°Ρ‚ΡŒΡΡ ΠΏΠΎ окруТности постоянного радиуса r. Π‘ΠΈΠ»Π° Π›ΠΎΡ€Π΅Π½Ρ†Π° Π² этом случаС являСтся Ρ†Π΅Π½Ρ‚Ρ€ΠΎΡΡ‚Ρ€Π΅ΠΌΠΈΡ‚Π΅Π»ΡŒΠ½ΠΎΠΉ силой:
  • Если заряТСнная частица двиТСтся Π² ΠΌΠ°Π³Π½ΠΈΡ‚Π½ΠΎΠΌ ΠΏΠΎΠ»Π΅ Ρ‚Π°ΠΊ, Ρ‡Ρ‚ΠΎ Π²Π΅ΠΊΡ‚ΠΎΡ€ скорости   составляСт с Π²Π΅ΠΊΡ‚ΠΎΡ€ΠΎΠΌ ΠΌΠ°Π³Π½ΠΈΡ‚Π½ΠΎΠΉ ΠΈΠ½Π΄ΡƒΠΊΡ†ΠΈΠΈΒ Β ΡƒΠ³ΠΎΠ» aΒ , Ρ‚ΠΎ Ρ‚Ρ€Π°Π΅ΠΊΡ‚ΠΎΡ€ΠΈΠ΅ΠΉ двиТСния частицы являСтся винтовая линия с радиусом r.

Если Ρ€Π°ΡΠΏΠΎΠ»ΠΎΠΆΠΈΡ‚ΡŒ Π»Π΅Π²ΡƒΡŽ Ρ€ΡƒΠΊΡƒ Ρ‚Π°ΠΊ, Ρ‡Ρ‚ΠΎΠ±Ρ‹ Β ΡΠΎΡΡ‚Π°Π²Π»ΡΡŽΡ‰Π°ΡΒ ΠΌΠ°Π³Π½ΠΈΡ‚Π½ΠΎΠΉΒ ΠΈΠ½Π΄ΡƒΠΊΡ†ΠΈΠΈΒ Β , пСрпСндикулярная скорости заряда, Π²Ρ…ΠΎΠ΄ΠΈΠ»Π°Β Π² ладонь, Π° Ρ‡Π΅Ρ‚Ρ‹Ρ€Π΅ вытянутых ΠΏΠ°Π»ΡŒΡ†Π° Π±Ρ‹Π»ΠΈ Π½Π°ΠΏΡ€Π°Π²Π»Π΅Π½Ρ‹ ΠΏΠΎ двиТСнию ΠΏΠΎΠ»ΠΎΠΆΠΈΡ‚Π΅Π»ΡŒΠ½ΠΎΠ³ΠΎ заряда, Ρ‚ΠΎ ΠΎΡ‚ΠΎΠ³Π½ΡƒΡ‚Ρ‹ΠΉ Π½Π° 90

0 большой ΠΏΠ°Π»Π΅Ρ†Β Β ΡƒΠΊΠ°ΠΆΠ΅Ρ‚ Π½Π°ΠΏΡ€Π°Π²Π»Π΅Π½ΠΈΠ΅ Π΄Π΅ΠΉΡΡ‚Π²ΡƒΡŽΡ‰Π΅ΠΉ Π½Π° заряд силы Π›ΠΎΡ€Π΅Π½Ρ†Π°Β FΠ»

.

Π‘Π°ΠΌΠΎΡΡ‚ΠΎΡΡ‚Π΅Π»ΡŒΠ½Π°Ρ Ρ€Π°Π±ΠΎΡ‚Π° ΠΏΠΎ Ρ‚Π΅ΠΌΠ΅ «Π‘ΠΈΠ»Π° АмпСра. Π‘ΠΈΠ»Π° Π›ΠΎΡ€Π΅Π½Ρ†Π°» | ΠœΠ°Ρ‚Π΅Ρ€ΠΈΠ°Π» ΠΏΠΎ Ρ„ΠΈΠ·ΠΈΠΊΠ΅ (11 класс):

Π‘Π°ΠΌΠΎΡΡ‚ΠΎΡΡ‚Π΅Π»ΡŒΠ½Π°Ρ Ρ€Π°Π±ΠΎΡ‚Π° ΠΏΠΎ Ρ‚Π΅ΠΌΠ΅ «Π‘ΠΈΠ»Π° АмпСра. Π‘ΠΈΠ»Π° Π›ΠΎΡ€Π΅Π½Ρ†Π°»

Π’Π°Ρ€ΠΈΠ°Π½Ρ‚ 1.

1. Β Π’ ΠΌΠ°Π³Π½ΠΈΡ‚Π½ΠΎΠΌ ΠΏΠΎΠ»Π΅ находится ΠΏΡ€ΠΎΠ²ΠΎΠ΄Π½ΠΈΠΊ с Ρ‚ΠΎΠΊΠΎΠΌ (рис. 25). Каково Π½Π°ΠΏΡ€Π°Π²Π»Π΅Π½ΠΈΠ΅ силы АмпСра, Π΄Π΅ΠΉΡΡ‚Π²ΡƒΡŽΡ‰Π΅ΠΉ Π½Π° ΠΏΡ€ΠΎΠ²ΠΎΠ΄Π½ΠΈΠΊ?

А. ΠžΡ‚ читатСля.Β Β Β Β Β Β Β Β Π‘. К Ρ‡ΠΈΡ‚Π°Ρ‚Π΅Π»ΡŽ.Β Β Β Β Β Β Β Β Β Β Β Β Β Β Β Β Π’. Π Π°Π²Π½Π° Π½ΡƒΠ»ΡŽ.

2. ΠŸΡ€ΡΠΌΠΎΠ»ΠΈΠ½Π΅ΠΉΠ½Ρ‹ΠΉ ΠΏΡ€ΠΎΠ²ΠΎΠ΄Π½ΠΈΠΊ Π΄Π»ΠΈΠ½ΠΎΠΉ 10 см находится Π² ΠΎΠ΄Π½ΠΎΡ€ΠΎΠ΄Π½ΠΎΠΌ ΠΌΠ°Π³Π½ΠΈΡ‚Π½ΠΎΠΌ ΠΏΠΎΠ»Π΅ с ΠΈΠ½Π΄ΡƒΠΊΡ†ΠΈΠ΅ΠΉ 4 Π’Π» ΠΈ располоТСн ΠΏΠΎΠ΄ ΡƒΠ³Π»ΠΎΠΌ 30Β° ΠΊ Π²Π΅ΠΊΡ‚ΠΎΡ€Ρƒ ΠΌΠ°Π³Π½ΠΈΡ‚Π½ΠΎΠΉ ΠΈΠ½Π΄ΡƒΠΊΡ†ΠΈΠΈ. Π§Π΅ΠΌΡƒ Ρ€Π°Π²Π½Π° сила, Π΄Π΅ΠΉΡΡ‚Π²ΡƒΡŽΡ‰Π°Ρ Π½Π° ΠΏΡ€ΠΎΠ²ΠΎΠ΄Π½ΠΈΠΊ со стороны ΠΌΠ°Π³Π½ΠΈΡ‚Π½ΠΎΠ³ΠΎ поля, Ссли сила Ρ‚ΠΎΠΊΠ° Π² ΠΏΡ€ΠΎΠ²ΠΎΠ΄Π½ΠΈΠΊΠ΅ 3 А?

3. Π’ ΠΌΠ°Π³Π½ΠΈΡ‚Π½ΠΎΠΌ ΠΏΠΎΠ»Π΅ с ΠΈΠ½Π΄ΡƒΠΊΡ†ΠΈΠ΅ΠΉ 5 Π’Π» двиТСтся элСктрон со ΡΠΊΠΎΡ€ΠΎΡΡ‚ΡŒΡŽ 104Β ΠΌ/с, Π½Π°ΠΏΡ€Π°Π²Π»Π΅Π½Π½ΠΎΠΉ пСрпСндикулярно линиям ΠΈΠ½Π΄ΡƒΠΊΡ†ΠΈΠΈ ΠΌΠ°Π³Π½ΠΈΡ‚Π½ΠΎΠ³ΠΎ поля. Π§Π΅ΠΌΡƒ Ρ€Π°Π²Π΅Π½ ΠΌΠΎΠ΄ΡƒΠ»ΡŒ силы, Π΄Π΅ΠΉΡΡ‚Π²ΡƒΡŽΡ‰Π΅ΠΉ Π½Π° элСктрон со стороны ΠΌΠ°Π³Π½ΠΈΡ‚Π½ΠΎΠ³ΠΎ поля?

4. На рисункС 77 ΠΏΠΎΠΊΠ°Π·Π°Π½ΠΎ сСчСниС ΠΏΡ€ΠΎΠ²ΠΎΠ΄Π½ΠΈΠΊΠ° с Ρ‚ΠΎΠΊΠΎΠΌ. ЭлСктричСский Ρ‚ΠΎΠΊ Π½Π°ΠΏΡ€Π°Π²Π»Π΅Π½ пСрпСндикулярно плоскости рисунка. Π’ ΠΊΠ°ΠΊΠΎΠΌ случаС ΠΏΡ€Π°Π²ΠΈΠ»ΡŒΠ½ΠΎ ΡƒΠΊΠ°Π·Π°Π½ΠΎ Π½Π°ΠΏΡ€Π°Π²Π»Π΅Π½ΠΈΠ΅ Π»ΠΈΠ½ΠΈΠΉ ΠΈΠ½Π΄ΡƒΠΊΡ†ΠΈΠΈ ΠΌΠ°Π³Π½ΠΈΡ‚Π½ΠΎΠ³ΠΎ поля, созданного этим Ρ‚ΠΎΠΊΠΎΠΌ?

1) рис А Β Β Β Β Β Β Β Β 2) рис Π‘Β Β Β Β Β Β Β Β  3) Π½Π΅Ρ‚ ΠΏΡ€Π°Π²ΠΈΠ»ΡŒΠ½ΠΎΠ³ΠΎ ΠΎΡ‚Π²Π΅Ρ‚Π°

5. Π’ ΠΊΠ°ΠΊΡƒΡŽ сторону отклоняСтся ΠΏΡ€ΠΎΡ‚ΠΎΠ½ ΠΏΠΎΠ΄ дСйствиСм ΠΌΠ°Π³Π½ΠΈΡ‚Π½ΠΎΠ³ΠΎ поля (рис. 29)?

A. Π’Π»Π΅Π²ΠΎ. Π‘. Π’ΠΏΡ€Π°Π²ΠΎ. B. Π’Π²Π΅Ρ€Ρ….

6. Π’ΠΎ всСх Ρ‡Π΅Ρ‚Ρ‹Ρ€Π΅Ρ… Π²Π°Ρ€ΠΈΠ°Π½Ρ‚Π°Ρ… ΠΎΠΏΡ€Π΅Π΄Π΅Π»ΠΈΡ‚Π΅ Π½Π°ΠΏΡ€Π°Π²Π»Π΅Π½ΠΈΠ΅ силы АмпСра.

A. Π’Π»Π΅Π²ΠΎ. Π‘. Π’ΠΏΡ€Π°Π²ΠΎ. B. Π’Π²Π΅Ρ€Ρ….Β  Π“. ΠžΡ‚ читатСля.Β Β Β Β Β Β Β Β Π”. К Ρ‡ΠΈΡ‚Π°Ρ‚Π΅Π»ΡŽ.Β  Β  Β Π•. Π Π°Π²Π½Π° Π½ΡƒΠ»ΡŽ. Π–) Π²Π½ΠΈΠ·

7. Π’ΠΎ всСх Ρ‡Π΅Ρ‚Ρ‹Ρ€Π΅Ρ… Π²Π°Ρ€ΠΈΠ°Π½Ρ‚Π°Ρ… ΠΎΠΏΡ€Π΅Π΄Π΅Π»ΠΈΡ‚Π΅ Π½Π°ΠΏΡ€Π°Π²Π»Π΅Π½ΠΈΠ΅ силы Π›ΠΎΡ€Π΅Π½Ρ†Π°

A. Π’Π»Π΅Π²ΠΎ. Π‘. Π’ΠΏΡ€Π°Π²ΠΎ. B. Π’Π²Π΅Ρ€Ρ….Β  Π“. ΠžΡ‚ читатСля.Β Β Β Β Β Β Β Β Π”. К Ρ‡ΠΈΡ‚Π°Ρ‚Π΅Π»ΡŽ.Β  Β  Β Π•. Π Π°Π²Π½Π° Π½ΡƒΠ»ΡŽ. Π–) Π²Π½ΠΈΠ·

Π‘Π°ΠΌΠΎΡΡ‚ΠΎΡΡ‚Π΅Π»ΡŒΠ½Π°Ρ Ρ€Π°Π±ΠΎΡ‚Π° ΠΏΠΎ Ρ‚Π΅ΠΌΠ΅ «Π‘ΠΈΠ»Π° АмпСра. Π‘ΠΈΠ»Π° Π›ΠΎΡ€Π΅Π½Ρ†Π°»

Π’Π°Ρ€ΠΈΠ°Π½Ρ‚ 2.

1.Π’ ΠΌΠ°Π³Π½ΠΈΡ‚Π½ΠΎΠΌ ΠΏΠΎΠ»Π΅ находится ΠΏΡ€ΠΎΠ²ΠΎΠ΄Π½ΠΈΠΊ с Ρ‚ΠΎΠΊΠΎΠΌ (рис. 28). Каково Π½Π°ΠΏΡ€Π°Π²Π»Π΅Π½ΠΈΠ΅ силы АмпСра, Π΄Π΅ΠΉΡΡ‚Π²ΡƒΡŽΡ‰Π΅ΠΉ Π½Π° ΠΏΡ€ΠΎΠ²ΠΎΠ΄Π½ΠΈΠΊ?

А.Β ΠžΡ‚ читатСля.Β  Β  Π‘. К Ρ‡ΠΈΡ‚Π°Ρ‚Π΅Π»ΡŽ.Β  Β  Π’.Β Π Π°Π²Π½Π° Π½ΡƒΠ»ΡŽ.

2. ΠŸΡ€ΡΠΌΠΎΠ»ΠΈΠ½Π΅ΠΉΠ½Ρ‹ΠΉ ΠΏΡ€ΠΎΠ²ΠΎΠ΄Π½ΠΈΠΊ Π΄Π»ΠΈΠ½ΠΎΠΉ 5 см находится Π² ΠΎΠ΄Π½ΠΎΡ€ΠΎΠ΄Π½ΠΎΠΌ ΠΌΠ°Π³Π½ΠΈΡ‚Π½ΠΎΠΌ ΠΏΠΎΠ»Π΅ с ΠΈΠ½Π΄ΡƒΠΊΡ†ΠΈΠ΅ΠΉ 5 Π’Π» ΠΈ располоТСн ΠΏΠΎΠ΄ ΡƒΠ³Π»ΠΎΠΌ 30Β° ΠΊ Π²Π΅ΠΊΡ‚ΠΎΡ€Ρƒ ΠΌΠ°Π³Π½ΠΈΡ‚Π½ΠΎΠΉ ΠΈΠ½Π΄ΡƒΠΊΡ†ΠΈΠΈ. Π§Π΅ΠΌΡƒ Ρ€Π°Π²Π½Π° сила, Π΄Π΅ΠΉΡΡ‚Π²ΡƒΡŽΡ‰Π°Ρ Π½Π° ΠΏΡ€ΠΎΠ²ΠΎΠ΄Π½ΠΈΠΊ со стороны ΠΌΠ°Π³Π½ΠΈΡ‚Π½ΠΎΠ³ΠΎ поля, Ссли сила Ρ‚ΠΎΠΊΠ° Π² ΠΏΡ€ΠΎΠ²ΠΎΠ΄Π½ΠΈΠΊΠ΅ 2 А?

3. ΠžΠΏΡ€Π΅Π΄Π΅Π»ΠΈΡ‚Π΅ Π½Π°ΠΏΡ€Π°Π²Π»Π΅Π½ΠΈΠ΅ силы. Рис. β„–2

А β†’ Β  Β  Β  Β  Β  Β  Π‘. ↓ Β  Β  Β  Β  Β Π’. ← Β  Β  Β  Β  Β  Β Π“. ↑

Β  Β  Β  Β  Β  Β  Β  Β  Β  Β  Β  Β  Β  Β  Β  Β  Β  Β  Β  Β  Β  Β  Β  Β  Β  Β  Β  Β  Β  Β  Β  Β  Β  Β  Β  Β  Β  Β  Β  Β  Β  Β  Β  Β  Β  Β  Β  Рис. β„–2

4. Π’ ΠΊΠ°ΠΊΡƒΡŽ сторону отклоняСтся элСктрон ΠΏΠΎΠ΄ дСйствиСм ΠΌΠ°Π³Π½ΠΈΡ‚Π½ΠΎΠ³ΠΎ поля (рис. 30)?

А. Π’Π»Π΅Π²ΠΎ. Π‘. Π’ΠΏΡ€Π°Π²ΠΎ. Π’. Π’Π²Π΅Ρ€Ρ….

5.Β Π’ ΠΌΠ°Π³Π½ΠΈΡ‚Π½ΠΎΠΌ ΠΏΠΎΠ»Π΅ с ΠΈΠ½Π΄ΡƒΠΊΡ†ΠΈΠ΅ΠΉ 5 Π’Π» двиТСтся элСктрон со ΡΠΊΠΎΡ€ΠΎΡΡ‚ΡŒΡŽ 104Β ΠΌ/с, Π½Π°ΠΏΡ€Π°Π²Π»Π΅Π½Π½ΠΎΠΉ пСрпСндикулярно линиям ΠΈΠ½Π΄ΡƒΠΊΡ†ΠΈΠΈ ΠΌΠ°Π³Π½ΠΈΡ‚Π½ΠΎΠ³ΠΎ поля. Π§Π΅ΠΌΡƒ Ρ€Π°Π²Π΅Π½ ΠΌΠΎΠ΄ΡƒΠ»ΡŒ силы, Π΄Π΅ΠΉΡΡ‚Π²ΡƒΡŽΡ‰Π΅ΠΉ Π½Π° элСктрон со стороны ΠΌΠ°Π³Π½ΠΈΡ‚Π½ΠΎΠ³ΠΎ поля?

6. Π’ΠΎ всСх Ρ‡Π΅Ρ‚Ρ‹Ρ€Π΅Ρ… Π²Π°Ρ€ΠΈΠ°Π½Ρ‚Π°Ρ… ΠΎΠΏΡ€Π΅Π΄Π΅Π»ΠΈΡ‚Π΅ Π½Π°ΠΏΡ€Π°Π²Π»Π΅Π½ΠΈΠ΅ силы АмпСра

Β  Β  Β  Β  Β  Β  Β Π³)

A. Π’Π»Π΅Π²ΠΎ. Π‘. Π’ΠΏΡ€Π°Π²ΠΎ. B. Π’Π²Π΅Ρ€Ρ….Β  Π“. ΠžΡ‚ читатСля.Β Β Β Β Β Β Β Β Π”. К Ρ‡ΠΈΡ‚Π°Ρ‚Π΅Π»ΡŽ.Β  Β  Β Π•. Π Π°Π²Π½Π° Π½ΡƒΠ»ΡŽ. Π–) Π²Π½ΠΈΠ·

7. Π’ΠΎ всСх Ρ‡Π΅Ρ‚Ρ‹Ρ€Π΅Ρ… Π²Π°Ρ€ΠΈΠ°Π½Ρ‚Π°Ρ… ΠΎΠΏΡ€Π΅Π΄Π΅Π»ΠΈΡ‚Π΅ Π½Π°ΠΏΡ€Π°Π²Π»Π΅Π½ΠΈΠ΅ силы Π›ΠΎΡ€Π΅Π½Ρ†Π°

A. Π’Π»Π΅Π²ΠΎ. Π‘. Π’ΠΏΡ€Π°Π²ΠΎ. B. Π’Π²Π΅Ρ€Ρ….Β  Π“. ΠžΡ‚ читатСля.Β Β Β Β Β Β Β Β Π”. К Ρ‡ΠΈΡ‚Π°Ρ‚Π΅Π»ΡŽ.Β  Β  Β Π•. Π Π°Π²Π½Π° Π½ΡƒΠ»ΡŽ. Π–) Π²Π½ΠΈΠ·

ΠŸΡ€ΠΈΠ½Ρ†ΠΈΠΏ супСрпозиции элСктричСских ΠΏΠΎΠ»Π΅ΠΉ, ΠΌΠ°Π³Π½ΠΈΡ‚Π½ΠΎΠ΅ ΠΏΠΎΠ»Π΅ ΠΏΡ€ΠΎΠ²ΠΎΠ΄Π½ΠΈΠΊΠ° с Ρ‚ΠΎΠΊΠΎΠΌ, сила АмпСра, сила Π›ΠΎΡ€Π΅Π½Ρ†Π°, ΠΏΡ€Π°Π²ΠΈΠ»ΠΎ Π›Π΅Π½Ρ†Π°

 

Β§ 3. Π­Π»Π΅ΠΊΡ‚Ρ€ΠΎΠ΄ΠΈΠ½Π°ΠΌΠΈΠΊΠ°

 

3.1. ΠžΡΠ½ΠΎΠ²Π½Ρ‹Π΅ понятия ΠΈ Π·Π°ΠΊΠΎΠ½Ρ‹ элСктростатики Π—Π°ΠΊΠΎΠ½ ΠšΡƒΠ»ΠΎΠ½Π°:
сила взаимодСйствия Π΄Π²ΡƒΡ… Ρ‚ΠΎΡ‡Π΅Ρ‡Π½Ρ‹Ρ… Π½Π΅ΠΏΠΎΠ΄Π²ΠΈΠΆΠ½Ρ‹Ρ… зарядов Π² Π²Π°ΠΊΡƒΡƒΠΌΠ΅ прямо ΠΏΡ€ΠΎΠΏΠΎΡ€Ρ†ΠΈΠΎΠ½Π°Π»ΡŒΠ½Π° ΠΏΡ€ΠΎΠΈΠ·Π²Π΅Π΄Π΅Π½ΠΈΡŽ ΠΌΠΎΠ΄ΡƒΠ»Π΅ΠΉ зарядов ΠΈ ΠΎΠ±Ρ€Π°Ρ‚Π½ΠΎ ΠΏΡ€ΠΎΠΏΠΎΡ€Ρ†ΠΈΠΎΠ½Π°Π»ΡŒΠ½Π° ΠΊΠ²Π°Π΄Ρ€Π°Ρ‚Ρƒ расстояния ΠΌΠ΅ΠΆΠ΄Ρƒ Π½ΠΈΠΌΠΈ:

ΠšΠΎΡΡ„Ρ„ΠΈΡ†ΠΈΠ΅Π½Ρ‚ ΠΏΡ€ΠΎΠΏΠΎΡ€Ρ†ΠΈΠΎΠ½Π°Π»ΡŒΠ½ΠΎΡΡ‚ΠΈ Π² этом Π·Π°ΠΊΠΎΠ½Π΅

Π’ БИ коэффициСнт k записываСтся Π² Π²ΠΈΠ΄Π΅

Π³Π΄Π΅ Ξ΅0 = 8, 85 Β· 10βˆ’12 Π€/ΠΌ (элСктричСская постоянная).

 ВочСчными зарядами Π½Π°Π·Ρ‹Π²Π°ΡŽΡ‚ Ρ‚Π°ΠΊΠΈΠ΅ заряды, расстояния ΠΌΠ΅ΠΆΠ΄Ρƒ ΠΊΠΎΡ‚ΠΎΡ€Ρ‹ΠΌΠΈ Π³ΠΎΡ€Π°Π·Π΄ΠΎ большС ΠΈΡ… Ρ€Π°Π·ΠΌΠ΅Ρ€ΠΎΠ².

 ЭлСктричСскиС заряды Π²Π·Π°ΠΈΠΌΠΎΠ΄Π΅ΠΉΡΡ‚Π²ΡƒΡŽΡ‚ ΠΌΠ΅ΠΆΠ΄Ρƒ собой с ΠΏΠΎΠΌΠΎΡ‰ΡŒΡŽ элСктричСского поля. Для качСствСнного описания элСктричСского поля ΠΈΡΠΏΠΎΠ»ΡŒΠ·ΡƒΠ΅Ρ‚ΡΡ силовая характСристика, которая называСтся Β«Π½Π°ΠΏΡ€ΡΠΆΡ‘Π½Π½ΠΎΡΡ‚ΡŒΡŽ элСктричСского поля» (E). ΠΠ°ΠΏΡ€ΡΠΆΡ‘Π½Π½ΠΎΡΡ‚ΡŒ элСктричСского поля Ρ€Π°Π²Π½Π° ΠΎΡ‚Π½ΠΎΡˆΠ΅Π½ΠΈΡŽ силы, Π΄Π΅ΠΉΡΡ‚Π²ΡƒΡŽΡ‰Π΅ΠΉ Π½Π° ΠΏΡ€ΠΎΠ±Π½Ρ‹ΠΉ заряд, ΠΏΠΎΠΌΠ΅Ρ‰Ρ‘Π½Π½Ρ‹ΠΉ Π² Π½Π΅ΠΊΠΎΡ‚ΠΎΡ€ΡƒΡŽ Ρ‚ΠΎΡ‡ΠΊΡƒ поля, ΠΊ Π²Π΅Π»ΠΈΡ‡ΠΈΠ½Π΅ этого заряда:

 НаправлСниС Π²Π΅ΠΊΡ‚ΠΎΡ€Π° напряТённости совпадаСт с Π½Π°ΠΏΡ€Π°Π²Π»Π΅Π½ΠΈΠ΅ΠΌ силы, Π΄Π΅ΠΉΡΡ‚Π²ΡƒΡŽΡ‰Π΅ΠΉ Π½Π° ΠΏΠΎΠ»ΠΎΠΆΠΈΡ‚Π΅Π»ΡŒΠ½Ρ‹ΠΉ ΠΏΡ€ΠΎΠ±Π½Ρ‹ΠΉ заряд. [E]=B/ΠΌ. Из Π·Π°ΠΊΠΎΠ½Π° ΠšΡƒΠ»ΠΎΠ½Π° ΠΈ опрСдСлСния напряТённости поля слСдуСт, Ρ‡Ρ‚ΠΎ Π½Π°ΠΏΡ€ΡΠΆΡ‘Π½Π½ΠΎΡΡ‚ΡŒ поля Ρ‚ΠΎΡ‡Π΅Ρ‡Π½ΠΎΠ³ΠΎ заряда

Π³Π΄Π΅ q β€” заряд, ΡΠΎΠ·Π΄Π°ΡŽΡ‰ΠΈΠΉ ΠΏΠΎΠ»Π΅; r β€” расстояниС ΠΎΡ‚ Ρ‚ΠΎΡ‡ΠΊΠΈ, Π³Π΄Π΅ находится заряд, Π΄ΠΎ Ρ‚ΠΎΡ‡ΠΊΠΈ, Π³Π΄Π΅ создаётся ΠΏΠΎΠ»Π΅.
 Если элСктричСскоС ΠΏΠΎΠ»Π΅ создаётся Π½Π΅ ΠΎΠ΄Π½ΠΈΠΌ, Π° нСсколькими зарядами, Ρ‚ΠΎ для нахоТдСния напряТённости Ρ€Π΅Π·ΡƒΠ»ΡŒΡ‚ΠΈΡ€ΡƒΡŽΡ‰Π΅Π³ΠΎ поля ΠΈΡΠΏΠΎΠ»ΡŒΠ·ΡƒΠ΅Ρ‚ΡΡ ΠΏΡ€ΠΈΠ½Ρ†ΠΈΠΏ супСрпозиции элСктричСских ΠΏΠΎΠ»Π΅ΠΉ: Π½Π°ΠΏΡ€ΡΠΆΡ‘Π½Π½ΠΎΡΡ‚ΡŒ Ρ€Π΅Π·ΡƒΠ»ΡŒΡ‚ΠΈΡ€ΡƒΡŽΡ‰Π΅Π³ΠΎ поля Ρ€Π°Π²Π½Π° Π²Π΅ΠΊΡ‚ΠΎΡ€Π½ΠΎΠΉ суммС напряТённостСй ΠΏΠΎΠ»Π΅ΠΉ, созданных ΠΊΠ°ΠΆΠ΄Ρ‹ΠΌ ΠΈΠ· зарядов β€” источников Π² ΠΎΡ‚Π΄Π΅Π»ΡŒΠ½ΠΎΡΡ‚ΠΈ:

 Работа элСктричСского поля ΠΏΡ€ΠΈ ΠΏΠ΅Ρ€Π΅ΠΌΠ΅Ρ‰Π΅Π½ΠΈΠΈ заряда: Π½Π°ΠΉΠ΄Ρ‘ΠΌ Ρ€Π°Π±ΠΎΡ‚Ρƒ пСрСмСщСния ΠΏΠΎΠ»ΠΎΠΆΠΈΡ‚Π΅Π»ΡŒΠ½ΠΎΠ³ΠΎ заряда силами ΠšΡƒΠ»ΠΎΠ½Π° Π² ΠΎΠ΄Π½ΠΎΡ€ΠΎΠ΄Π½ΠΎΠΌ элСктричСском ΠΏΠΎΠ»Π΅. ΠŸΡƒΡΡ‚ΡŒ ΠΏΠΎΠ»Π΅ ΠΏΠ΅Ρ€Π΅ΠΌΠ΅Ρ‰Π°Π΅Ρ‚ заряд q ΠΈΠ· Ρ‚ΠΎΡ‡ΠΊΠΈ 1 Π² Ρ‚ΠΎΡ‡ΠΊΡƒ 2:


 В элСктричСском ΠΏΠΎΠ»Π΅ Ρ€Π°Π±ΠΎΡ‚Π° Π½Π΅ зависит ΠΎΡ‚ Ρ„ΠΎΡ€ΠΌΡ‹ Ρ‚Ρ€Π°Π΅ΠΊΡ‚ΠΎΡ€ΠΈΠΈ, ΠΏΠΎ ΠΊΠΎΡ‚ΠΎΡ€ΠΎΠΉ пСрСмСщаСтся заряд. Из ΠΌΠ΅Ρ…Π°Π½ΠΈΠΊΠΈ извСстно, Ρ‡Ρ‚ΠΎ Ссли Ρ€Π°Π±ΠΎΡ‚Π° Π½Π΅ зависит ΠΎΡ‚ Ρ„ΠΎΡ€ΠΌΡ‹ Ρ‚Ρ€Π°Π΅ΠΊΡ‚ΠΎΡ€ΠΈΠΈ, Ρ‚ΠΎ ΠΎΠ½Π° Ρ€Π°Π²Π½Π° измСнСнию ΠΏΠΎΡ‚Π΅Π½Ρ†ΠΈΠ°Π»ΡŒΠ½ΠΎΠΉ энСргии с ΠΏΡ€ΠΎΡ‚ΠΈΠ²ΠΎΠΏΠΎΠ»ΠΎΠΆΠ½Ρ‹ΠΌ Π·Π½Π°ΠΊΠΎΠΌ:


ΠžΡ‚ΡΡŽΠ΄Π° слСдуСт, Ρ‡Ρ‚ΠΎ

β€ƒΠŸΠΎΡ‚Π΅Π½Ρ†ΠΈΠ°Π»ΠΎΠΌ элСктричСского поля Π½Π°Π·Ρ‹Π²Π°ΡŽΡ‚ ΠΎΡ‚Π½ΠΎΡˆΠ΅Π½ΠΈΠ΅ ΠΏΠΎΡ‚Π΅Π½Ρ†ΠΈΠ°Π»ΡŒΠ½ΠΎΠΉ энСргии заряда Π² ΠΏΠΎΠ»Π΅ ΠΊ этому заряду:

β€ƒΠ—Π°ΠΏΠΈΡˆΠ΅ΠΌ Ρ€Π°Π±ΠΎΡ‚Ρƒ поля Π² Π²ΠΈΠ΄Π΅

Π—Π΄Π΅ΡΡŒ U = Ο•1 βˆ’ Ο•2 β€” Ρ€Π°Π·Π½ΠΎΡΡ‚ΡŒ ΠΏΠΎΡ‚Π΅Π½Ρ†ΠΈΠ°Π»ΠΎΠ² Π² Π½Π°Ρ‡Π°Π»ΡŒΠ½ΠΎΠΉ ΠΈ ΠΊΠΎΠ½Π΅Ρ‡Π½ΠΎΠΉ Ρ‚ΠΎΡ‡ΠΊΠ°Ρ… Ρ‚Ρ€Π°Π΅ΠΊΡ‚ΠΎΡ€ΠΈΠΈ. Π Π°Π·Π½ΠΎΡΡ‚ΡŒ ΠΏΠΎΡ‚Π΅Π½Ρ†ΠΈΠ°Π»ΠΎΠ² Π½Π°Π·Ρ‹Π²Π°ΡŽΡ‚ Ρ‚Π°ΠΊΠΆΠ΅ напряТСниСм

 Часто наряду с понятиСм Β«Ρ€Π°Π·Π½ΠΎΡΡ‚ΡŒ ΠΏΠΎΡ‚Π΅Π½Ρ†ΠΈΠ°Π»ΠΎΠ²Β» вводят понятиС Β«ΠΏΠΎΡ‚Π΅Π½Ρ†ΠΈΠ°Π» Π½Π΅ΠΊΠΎΡ‚ΠΎΡ€ΠΎΠΉ Ρ‚ΠΎΡ‡ΠΊΠΈ поля». Под ΠΏΠΎΡ‚Π΅Π½Ρ†ΠΈΠ°Π»ΠΎΠΌ Ρ‚ΠΎΡ‡ΠΊΠΈ ΠΏΠΎΠ΄Ρ€Π°Π·ΡƒΠΌΠ΅Π²Π°ΡŽΡ‚ Ρ€Π°Π·Π½ΠΎΡΡ‚ΡŒ ΠΏΠΎΡ‚Π΅Π½Ρ†ΠΈΠ°Π»ΠΎΠ² ΠΌΠ΅ΠΆΠ΄Ρƒ Π΄Π°Π½Π½ΠΎΠΉ Ρ‚ΠΎΡ‡ΠΊΠΎΠΉ ΠΈ Π½Π΅ΠΊΠΎΡ‚ΠΎΡ€ΠΎΠΉ Π·Π°Ρ€Π°Π½Π΅Π΅ Π²Ρ‹Π±Ρ€Π°Π½Π½ΠΎΠΉ Ρ‚ΠΎΡ‡ΠΊΠΎΠΉ поля. Π­Ρ‚Ρƒ Ρ‚ΠΎΡ‡ΠΊΡƒ ΠΌΠΎΠΆΠ½ΠΎ Π²Ρ‹Π±ΠΈΡ€Π°Ρ‚ΡŒ Π² бСсконСчности, Ρ‚ΠΎΠ³Π΄Π° говорят ΠΎ ΠΏΠΎΡ‚Π΅Π½Ρ†ΠΈΠ°Π»Π΅ ΠΎΡ‚Π½ΠΎΡΠΈΡ‚Π΅Π»ΡŒΠ½ΠΎΠΉ бСсконСчности.

β€ƒΠŸΠΎΡ‚Π΅Π½Ρ†ΠΈΠ°Π» поля Ρ‚ΠΎΡ‡Π΅Ρ‡Π½ΠΎΠ³ΠΎ заряда подсчитываСтся ΠΏΠΎ Ρ„ΠΎΡ€ΠΌΡƒΠ»Π΅

β€ƒΠŸΡ€ΠΎΠ΅ΠΊΡ†ΠΈΡ напряТённости элСктричСского поля Π½Π° ΠΊΠ°ΠΊΡƒΡŽ-Π½ΠΈΠ±ΡƒΠ΄ΡŒ ось ΠΈ ΠΏΠΎΡ‚Π΅Π½Ρ†ΠΈΠ°Π» связаны ΡΠΎΠΎΡ‚Π½ΠΎΡˆΠ΅Π½ΠΈΠ΅ΠΌ

 

3.2. Π­Π»Π΅ΠΊΡ‚Ρ€ΠΎΡ‘ΠΌΠΊΠΎΡΡ‚ΡŒ. ΠšΠΎΠ½Π΄Π΅Π½ΡΠ°Ρ‚ΠΎΡ€Ρ‹. ЭнСргия элСктричСского поля β€ƒΠ­Π»Π΅ΠΊΡ‚Ρ€ΠΎΡ‘ΠΌΠΊΠΎΡΡ‚ΡŒΡŽ Ρ‚Π΅Π»Π° Π½Π°Π·Ρ‹Π²Π°ΡŽΡ‚ Π²Π΅Π»ΠΈΡ‡ΠΈΠ½Ρƒ ΠΎΡ‚Π½ΠΎΡˆΠ΅Π½ΠΈΡ

 Ѐормула для подсчёта ёмкости плоского кондСнсатора ΠΈΠΌΠ΅Π΅Ρ‚ Π²ΠΈΠ΄:

Π³Π΄Π΅ S β€” ΠΏΠ»ΠΎΡ‰Π°Π΄ΡŒ ΠΎΠ±ΠΊΠ»Π°Π΄ΠΎΠΊ, d β€” расстояниС ΠΌΠ΅ΠΆΠ΄Ρƒ Π½ΠΈΠΌΠΈ.
β€ƒΠšΠΎΠ½Π΄Π΅Π½ΡΠ°Ρ‚ΠΎΡ€Ρ‹ ΠΌΠΎΠΆΠ½ΠΎ ΡΠΎΠ΅Π΄ΠΈΠ½ΡΡ‚ΡŒ Π² Π±Π°Ρ‚Π°Ρ€Π΅ΠΈ. ΠŸΡ€ΠΈ ΠΏΠ°Ρ€Π°Π»Π»Π΅Π»ΡŒΠ½ΠΎΠΌ соСдинСнии Ρ‘ΠΌΠΊΠΎΡΡ‚ΡŒ Π±Π°Ρ‚Π°Ρ€Π΅ΠΈ C Ρ€Π°Π²Π½Π° суммС ёмкостСй кондСнсаторов:

Разности ΠΏΠΎΡ‚Π΅Π½Ρ†ΠΈΠ°Π»ΠΎΠ² ΠΌΠ΅ΠΆΠ΄Ρƒ ΠΎΠ±ΠΊΠ»Π°Π΄ΠΊΠ°ΠΌΠΈ ΠΎΠ΄ΠΈΠ½Π°ΠΊΠΎΠ²Ρ‹, Π° заряды прямо ΠΏΡ€ΠΎΠΏΠΎΡ€Ρ†ΠΈΠΎΠ½Π°Π»ΡŒΠ½Ρ‹ ёмкостям.
β€ƒΠŸΡ€ΠΈ ΠΏΠΎΡΠ»Π΅Π΄ΠΎΠ²Π°Ρ‚Π΅Π»ΡŒΠ½ΠΎΠΌ соСдинСнии Π²Π΅Π»ΠΈΡ‡ΠΈΠ½Π°, обратная ёмкости Π±Π°Ρ‚Π°Ρ€Π΅ΠΈ, Ρ€Π°Π²Π½Π° суммС ΠΎΠ±Ρ€Π°Ρ‚Π½Ρ‹Ρ… ёмкостСй, входящих Π² Π±Π°Ρ‚Π°Ρ€Π΅ΡŽ:

 Заряды Π½Π° кондСнсаторах ΠΎΠ΄ΠΈΠ½Π°ΠΊΠΎΠ²Ρ‹, Π° разности ΠΏΠΎΡ‚Π΅Π½Ρ†ΠΈΠ°Π»ΠΎΠ² ΠΎΠ±Ρ€Π°Ρ‚Π½ΠΎ ΠΏΡ€ΠΎΠΏΠΎΡ€Ρ†ΠΈΠΎΠ½Π°Π»ΡŒΠ½Ρ‹ ёмкостям.
 ЗаряТСнный кондСнсатор ΠΎΠ±Π»Π°Π΄Π°Π΅Ρ‚ энСргиСй. Π­Π½Π΅Ρ€Π³ΠΈΡŽ заряТСнного кондСнсатора ΠΌΠΎΠΆΠ½ΠΎ ΠΏΠΎΠ΄ΡΡ‡ΠΈΡ‚Π°Ρ‚ΡŒ ΠΏΠΎ любой ΠΈΠ· ΡΠ»Π΅Π΄ΡƒΡŽΡ‰ΠΈΡ… Ρ„ΠΎΡ€ΠΌΡƒΠ»:

 

3.3. ΠžΡΠ½ΠΎΠ²Π½Ρ‹Π΅ понятия ΠΈ Π·Π°ΠΊΠΎΠ½Ρ‹ постоянного Ρ‚ΠΎΠΊΠ°  ЭлСктричСский Ρ‚ΠΎΠΊ β€” Π½Π°ΠΏΡ€Π°Π²Π»Π΅Π½Π½ΠΎΠ΅ Π΄Π²ΠΈΠΆΠ΅Π½ΠΈΠ΅ элСктричСских зарядов. Π’ Ρ€Π°Π·Π½Ρ‹Ρ… вСщСствах носитСлями заряда Π²Ρ‹ΡΡ‚ΡƒΠΏΠ°ΡŽΡ‚ элСмСнтарныС частицы Ρ€Π°Π·Π½ΠΎΠ³ΠΎ Π·Π½Π°ΠΊΠ°. Π—Π° ΠΏΠΎΠ»ΠΎΠΆΠΈΡ‚Π΅Π»ΡŒΠ½ΠΎΠ΅ Π½Π°ΠΏΡ€Π°Π²Π»Π΅Π½ΠΈΠ΅ Ρ‚ΠΎΠΊΠ° принято Π½Π°ΠΏΡ€Π°Π²Π»Π΅Π½ΠΈΠ΅ двиТСния ΠΏΠΎΠ»ΠΎΠΆΠΈΡ‚Π΅Π»ΡŒΠ½Ρ‹Ρ… зарядов. ΠšΠΎΠ»ΠΈΡ‡Π΅ΡΡ‚Π²Π΅Π½Π½ΠΎ элСктричСский Ρ‚ΠΎΠΊ Ρ…Π°Ρ€Π°ΠΊΡ‚Π΅Ρ€ΠΈΠ·ΡƒΡŽΡ‚ Π΅Π³ΠΎ силой. Π­Ρ‚ΠΎ заряд, ΠΏΡ€ΠΎΡˆΠ΅Π΄ΡˆΠΈΠΉ Π·Π° Π΅Π΄ΠΈΠ½ΠΈΡ†Ρƒ Π²Ρ€Π΅ΠΌΠ΅Π½ΠΈ Ρ‡Π΅Ρ€Π΅Π· ΠΏΠΎΠΏΠ΅Ρ€Π΅Ρ‡Π½ΠΎΠ΅ сСчСниС ΠΏΡ€ΠΎΠ²ΠΎΠ΄Π½ΠΈΠΊΠ°:

 Закон Ома для участка Ρ†Π΅ΠΏΠΈ ΠΈΠΌΠ΅Π΅Ρ‚ Π²ΠΈΠ΄:

ΠšΠΎΡΡ„Ρ„ΠΈΡ†ΠΈΠ΅Π½Ρ‚ ΠΏΡ€ΠΎΠΏΠΎΡ€Ρ†ΠΈΠΎΠ½Π°Π»ΡŒΠ½ΠΎΡΡ‚ΠΈ R, Π½Π°Π·Ρ‹Π²Π°Π΅ΠΌΡ‹ΠΉ элСктричСским сопротивлСниСм, являСтся характСристикой ΠΏΡ€ΠΎΠ²ΠΎΠ΄Π½ΠΈΠΊΠ° [R]=Ом. Π‘ΠΎΠΏΡ€ΠΎΡ‚ΠΈΠ²Π»Π΅Π½ΠΈΠ΅ ΠΏΡ€ΠΎΠ²ΠΎΠ΄Π½ΠΈΠΊΠ° зависит ΠΎΡ‚ Π΅Π³ΠΎ Π³Π΅ΠΎΠΌΠ΅Ρ‚Ρ€ΠΈΠΈ ΠΈ свойств ΠΌΠ°Ρ‚Π΅Ρ€ΠΈΠ°Π»Π°:

Π³Π΄Π΅ l β€” Π΄Π»ΠΈΠ½Π° ΠΏΡ€ΠΎΠ²ΠΎΠ΄Π½ΠΈΠΊΠ°, ρ β€” ΡƒΠ΄Π΅Π»ΡŒΠ½ΠΎΠ΅ сопротивлСниС, S β€” ΠΏΠ»ΠΎΡ‰Π°Π΄ΡŒ ΠΏΠΎΠΏΠ΅Ρ€Π΅Ρ‡Π½ΠΎΠ³ΠΎ сСчСния. ρ являСтся характСристикой ΠΌΠ°Ρ‚Π΅Ρ€ΠΈΠ°Π»Π° ΠΈ Π΅Π³ΠΎ состояния. [ρ] = Ом·м.
β€ƒΠŸΡ€ΠΎΠ²ΠΎΠ΄Π½ΠΈΠΊΠΈ ΠΌΠΎΠΆΠ½ΠΎ ΡΠΎΠ΅Π΄ΠΈΠ½ΡΡ‚ΡŒ ΠΏΠΎΡΠ»Π΅Π΄ΠΎΠ²Π°Ρ‚Π΅Π»ΡŒΠ½ΠΎ. Π‘ΠΎΠΏΡ€ΠΎΡ‚ΠΈΠ²Π»Π΅Π½ΠΈΠ΅ Ρ‚Π°ΠΊΠΎΠ³ΠΎ соСдинСния находится ΠΊΠ°ΠΊ сумма сопротивлСний:

β€ƒΠŸΡ€ΠΈ ΠΏΠ°Ρ€Π°Π»Π»Π΅Π»ΡŒΠ½ΠΎΠΌ соСдинСнии Π²Π΅Π»ΠΈΡ‡ΠΈΠ½Π°, обратная ΡΠΎΠΏΡ€ΠΎΡ‚ΠΈΠ²Π»Π΅Π½ΠΈΡŽ, Ρ€Π°Π²Π½Π° суммС ΠΎΠ±Ρ€Π°Ρ‚Π½Ρ‹Ρ… сопротивлСний:

 Для Ρ‚ΠΎΠ³ΠΎ Ρ‡Ρ‚ΠΎΠ±Ρ‹ Π² Ρ†Π΅ΠΏΠΈ Π΄Π»ΠΈΡ‚Π΅Π»ΡŒΠ½ΠΎΠ΅ врСмя ΠΏΡ€ΠΎΡ‚Π΅ΠΊΠ°Π» элСктричСский Ρ‚ΠΎΠΊ, Π² составС Ρ†Π΅ΠΏΠΈ Π΄ΠΎΠ»ΠΆΠ½Ρ‹ ΡΠΎΠ΄Π΅Ρ€ΠΆΠ°Ρ‚ΡŒΡΡ источники Ρ‚ΠΎΠΊΠ°. ΠšΠΎΠ»ΠΈΡ‡Π΅ΡΡ‚Π²Π΅Π½Π½ΠΎ источники Ρ‚ΠΎΠΊΠ° Ρ…Π°Ρ€Π°ΠΊΡ‚Π΅Ρ€ΠΈΠ·ΡƒΡŽΡ‚ ΠΈΡ… элСктродвиТущСй силой (Π­Π”Π‘). Π­Ρ‚ΠΎ ΠΎΡ‚Π½ΠΎΡˆΠ΅Π½ΠΈΠ΅ Ρ€Π°Π±ΠΎΡ‚Ρ‹, ΠΊΠΎΡ‚ΠΎΡ€ΡƒΡŽ ΡΠΎΠ²Π΅Ρ€ΡˆΠ°ΡŽΡ‚ сторонниС силы ΠΏΡ€ΠΈ пСрСносС элСктричСских зарядов ΠΏΠΎ Π·Π°ΠΌΠΊΠ½ΡƒΡ‚ΠΎΠΉ Ρ†Π΅ΠΏΠΈ, ΠΊ Π²Π΅Π»ΠΈΡ‡ΠΈΠ½Π΅ пСрСнСсённого заряда:

 Если ΠΊ Π·Π°ΠΆΠΈΠΌΠ°ΠΌ источника Ρ‚ΠΎΠΊΠ° ΠΏΠΎΠ΄ΠΊΠ»ΡŽΡ‡ΠΈΡ‚ΡŒ Π½Π°Π³Ρ€ΡƒΠ·ΠΎΡ‡Π½ΠΎΠ΅ сопротивлСниС R, Ρ‚ΠΎ Π² ΠΏΠΎΠ»ΡƒΡ‡ΠΈΠ²ΡˆΠ΅ΠΉΡΡ Π·Π°ΠΌΠΊΠ½ΡƒΡ‚ΠΎΠΉ Ρ†Π΅ΠΏΠΈ ΠΏΠΎΡ‚Π΅Ρ‡Ρ‘Ρ‚ Ρ‚ΠΎΠΊ, силу ΠΊΠΎΡ‚ΠΎΡ€ΠΎΠ³ΠΎ ΠΌΠΎΠΆΠ½ΠΎ ΠΏΠΎΠ΄ΡΡ‡ΠΈΡ‚Π°Ρ‚ΡŒ ΠΏΠΎ Ρ„ΠΎΡ€ΠΌΡƒΠ»Π΅

Π­Ρ‚ΠΎ ΡΠΎΠΎΡ‚Π½ΠΎΡˆΠ΅Π½ΠΈΠ΅ Π½Π°Π·Ρ‹Π²Π°ΡŽΡ‚ Π·Π°ΠΊΠΎΠ½ΠΎΠΌ Ома для ΠΏΠΎΠ»Π½ΠΎΠΉ Ρ†Π΅ΠΏΠΈ.

 ЭлСктричСский Ρ‚ΠΎΠΊ, пробСгая ΠΏΠΎ ΠΏΡ€ΠΎΠ²ΠΎΠ΄Π½ΠΈΠΊΠ°ΠΌ, Π½Π°Π³Ρ€Π΅Π²Π°Π΅Ρ‚ ΠΈΡ…, ΡΠΎΠ²Π΅Ρ€ΡˆΠ°Ρ ΠΏΡ€ΠΈ этом Ρ€Π°Π±ΠΎΡ‚Ρƒ

Π³Π΄Π΅ t β€” врСмя, I β€” сила Ρ‚ΠΎΠΊΠ°, U β€” Ρ€Π°Π·Π½ΠΎΡΡ‚ΡŒ ΠΏΠΎΡ‚Π΅Π½Ρ†ΠΈΠ°Π»ΠΎΠ², q β€” ΠΏΡ€ΠΎΡˆΠ΅Π΄ΡˆΠΈΠΉ заряд.

 Закон ДТоуля-Π›Π΅Π½Ρ†Π°:

 

3.4. ΠžΡΠ½ΠΎΠ²Π½Ρ‹Π΅ понятия ΠΈ Π·Π°ΠΊΠΎΠ½Ρ‹ магнитостатики  Π₯арактСристикой ΠΌΠ°Π³Π½ΠΈΡ‚Π½ΠΎΠ³ΠΎ поля являСтся магнитная индукция βž›B. ΠŸΠΎΡΠΊΠΎΠ»ΡŒΠΊΡƒ это Π²Π΅ΠΊΡ‚ΠΎΡ€, Ρ‚ΠΎ слСдуСт ΠΎΠΏΡ€Π΅Π΄Π΅Π»ΠΈΡ‚ΡŒ ΠΈ Π½Π°ΠΏΡ€Π°Π²Π»Π΅Π½ΠΈΠ΅ этого Π²Π΅ΠΊΡ‚ΠΎΡ€Π°, ΠΈ Π΅Π³ΠΎ ΠΌΠΎΠ΄ΡƒΠ»ΡŒ. НаправлСниС Π²Π΅ΠΊΡ‚ΠΎΡ€Π° ΠΌΠ°Π³Π½ΠΈΡ‚Π½ΠΎΠΉ ΠΈΠ½Π΄ΡƒΠΊΡ†ΠΈΠΈ связано с ΠΎΡ€ΠΈΠ΅Π½Ρ‚ΠΈΡ€ΡƒΡŽΡ‰ΠΈΠΌ дСйствиСм ΠΌΠ°Π³Π½ΠΈΡ‚Π½ΠΎΠ³ΠΎ поля Π½Π° ΠΌΠ°Π³Π½ΠΈΡ‚Π½ΡƒΡŽ стрСлку. Π—Π° Π½Π°ΠΏΡ€Π°Π²Π»Π΅Π½ΠΈΠ΅ Π²Π΅ΠΊΡ‚ΠΎΡ€Π° ΠΌΠ°Π³Π½ΠΈΡ‚Π½ΠΎΠΉ ΠΈΠ½Π΄ΡƒΠΊΡ†ΠΈΠΈ принимаСтся Π½Π°ΠΏΡ€Π°Π²Π»Π΅Π½ΠΈΠ΅ ΠΎΡ‚ юТного полюса S ΠΊ сСвСрному N ΠΌΠ°Π³Π½ΠΈΡ‚Π½ΠΎΠΉ стрСлки, свободно ΡƒΡΡ‚Π°Π½Π°Π²Π»ΠΈΠ²Π°ΡŽΡ‰Π΅ΠΉΡΡ Π² ΠΌΠ°Π³Π½ΠΈΡ‚Π½ΠΎΠΌ ΠΏΠΎΠ»Π΅.
 НаправлСниС Π²Π΅ΠΊΡ‚ΠΎΡ€Π° ΠΌΠ°Π³Π½ΠΈΡ‚Π½ΠΎΠΉ ΠΈΠ½Π΄ΡƒΠΊΡ†ΠΈΠΈ прямолинСйного ΠΏΡ€ΠΎΠ²ΠΎΠ΄Π½ΠΈΠΊΠ° с Ρ‚ΠΎΠΊΠ°ΠΌ ΠΌΠΎΠΆΠ½ΠΎ ΠΎΠΏΡ€Π΅Π΄Π΅Π»ΠΈΡ‚ΡŒ с ΠΏΠΎΠΌΠΎΡ‰ΡŒΡŽ ΠΏΡ€Π°Π²ΠΈΠ»Π° Π±ΡƒΡ€Π°Π²Ρ‡ΠΈΠΊΠ°:
Ссли Π½Π°ΠΏΡ€Π°Π²Π»Π΅Π½ΠΈΠ΅ ΠΏΠΎΡΡ‚ΡƒΠΏΠ°Ρ‚Π΅Π»ΡŒΠ½ΠΎΠ³ΠΎ двиТСния Π±ΡƒΡ€Π°Π²Ρ‡ΠΈΠΊΠ° совпадаСт с Π½Π°ΠΏΡ€Π°Π²Π»Π΅Π½ΠΈΠ΅ΠΌ Ρ‚ΠΎΠΊΠ° Π² ΠΏΡ€ΠΎΠ²ΠΎΠ΄Π½ΠΈΠΊΠ΅, Ρ‚ΠΎ Π½Π°ΠΏΡ€Π°Π²Π»Π΅Π½ΠΈΠ΅ вращСния рукоятки Π±ΡƒΡ€Π°Π²Ρ‡ΠΈΠΊΠ° совпадаСт с Π½Π°ΠΏΡ€Π°Π²Π»Π΅Π½ΠΈΠ΅ΠΌ Π²Π΅ΠΊΡ‚ΠΎΡ€Π° ΠΌΠ°Π³Π½ΠΈΡ‚Π½ΠΎΠΉ ΠΈΠ½Π΄ΡƒΠΊΡ†ΠΈΠΈ.

β€ƒΠœΠΎΠ΄ΡƒΠ»Π΅ΠΌ Π²Π΅ΠΊΡ‚ΠΎΡ€Π° ΠΌΠ°Π³Π½ΠΈΡ‚Π½ΠΎΠΉ ΠΈΠ½Π΄ΡƒΠΊΡ†ΠΈΠΈ Π½Π°Π·ΠΎΠ²Ρ‘ΠΌ ΠΎΡ‚Π½ΠΎΡˆΠ΅Π½ΠΈΠ΅ максимальной силы, Π΄Π΅ΠΉΡΡ‚Π²ΡƒΡŽΡ‰Π΅ΠΉ со стороны ΠΌΠ°Π³Π½ΠΈΡ‚Π½ΠΎΠ³ΠΎ поля Π½Π° участок ΠΏΡ€ΠΎΠ²ΠΎΠ΄Π½ΠΈΠΊΠ° с Ρ‚ΠΎΠΊΠΎΠΌ , ΠΊ ΠΏΡ€ΠΎΠΈΠ·Π²Π΅Π΄Π΅Π½ΠΈΡŽ силы Ρ‚ΠΎΠΊΠ° Π½Π° Π΄Π»ΠΈΠ½Ρƒ этого участка:

Π•Π΄ΠΈΠ½ΠΈΡ†Π° ΠΌΠ°Π³Π½ΠΈΡ‚Π½ΠΎΠΉ ΠΈΠ½Π΄ΡƒΠΊΡ†ΠΈΠΈ называСтся тСсла (1 Π’Π»)

β€ƒΠœΠ°Π³Π½ΠΈΡ‚Π½Ρ‹ΠΌ ΠΏΠΎΡ‚ΠΎΠΊΠΎΠΌ Ξ¦ Ρ‡Π΅Ρ€Π΅Π· ΠΏΠΎΠ²Π΅Ρ€Ρ…Π½ΠΎΡΡ‚ΡŒ ΠΊΠΎΠ½Ρ‚ΡƒΡ€Π° ΠΏΠ»ΠΎΡ‰Π°Π΄ΡŒΡŽ S Π½Π°Π·Ρ‹Π²Π°ΡŽΡ‚ Π²Π΅Π»ΠΈΡ‡ΠΈΠ½Ρƒ, Ρ€Π°Π²Π½ΡƒΡŽ ΠΏΡ€ΠΎΠΈΠ·Π²Π΅Π΄Π΅Π½ΠΈΡŽ модуля Π²Π΅ΠΊΡ‚ΠΎΡ€Π° ΠΌΠ°Π³Π½ΠΈΡ‚Π½ΠΎΠΉ ΠΈΠ½Π΄ΡƒΠΊΡ†ΠΈΠΈ Π½Π° ΠΏΠ»ΠΎΡ‰Π°Π΄ΡŒ этой повСрхности ΠΈ Π½Π° косинус ΡƒΠ³Π»Π° ΠΌΠ΅ΠΆΠ΄Ρƒ Π²Π΅ΠΊΡ‚ΠΎΡ€ΠΎΠΌ ΠΌΠ°Π³Π½ΠΈΡ‚Π½ΠΎΠΉ ΠΈΠ½Π΄ΡƒΠΊΡ†ΠΈΠΈ βž›B ΠΈ Π½ΠΎΡ€ΠΌΠ°Π»ΡŒΡŽ ΠΊ повСрхности βž›n:

Π•Π΄ΠΈΠ½ΠΈΡ†Π΅ΠΉ ΠΌΠ°Π³Π½ΠΈΡ‚Π½ΠΎΠ³ΠΎ ΠΏΠΎΡ‚ΠΎΠΊΠ° являСтся Π²Π΅Π±Π΅Ρ€ (1 Π’Π±).
 На ΠΏΡ€ΠΎΠ²ΠΎΠ΄Π½ΠΈΠΊ с Ρ‚ΠΎΠΊΠΎΠΌ, ΠΏΠΎΠΌΠ΅Ρ‰Ρ‘Π½Π½Ρ‹ΠΉ Π² ΠΌΠ°Π³Π½ΠΈΡ‚Π½ΠΎΠ΅ ΠΏΠΎΠ»Π΅, дСйствуСт сила АмпСра

 Закон АмпСра:
Π½Π° ΠΎΡ‚Ρ€Π΅Π·ΠΎΠΊ ΠΏΡ€ΠΎΠ²ΠΎΠ΄Π½ΠΈΠΊΠ° с Ρ‚ΠΎΠΊΠΎΠΌ силой I ΠΈ Π΄Π»ΠΈΠ½ΠΎΠΉ l, ΠΏΠΎΠΌΠ΅Ρ‰Ρ‘Π½Π½Ρ‹ΠΉ Π² ΠΎΠ΄Π½ΠΎΡ€ΠΎΠ΄Π½ΠΎΠ΅ ΠΌΠ°Π³Π½ΠΈΡ‚Π½ΠΎΠ΅ ΠΏΠΎΠ»Π΅ с ΠΈΠ½Π΄ΡƒΠΊΡ†ΠΈΠ΅ΠΉ βž›B , дСйствуСт сила, ΠΌΠΎΠ΄ΡƒΠ»ΡŒ ΠΊΠΎΡ‚ΠΎΡ€ΠΎΠΉ Ρ€Π°Π²Π΅Π½ ΠΏΡ€ΠΎΠΈΠ·Π²Π΅Π΄Π΅Π½ΠΈΡŽ модуля Π²Π΅ΠΊΡ‚ΠΎΡ€Π° ΠΌΠ°Π³Π½ΠΈΡ‚Π½ΠΎΠΉ ΠΈΠ½Π΄ΡƒΠΊΡ†ΠΈΠΈ Π½Π° силу Ρ‚ΠΎΠΊΠ°, Π½Π° Π΄Π»ΠΈΠ½Ρƒ участка ΠΏΡ€ΠΎΠ²ΠΎΠ΄Π½ΠΈΠΊΠ°, находящСгося Π² ΠΌΠ°Π³Π½ΠΈΡ‚Π½ΠΎΠΌ ΠΏΠΎΠ»Π΅, ΠΈ Π½Π° синус ΡƒΠ³Π»Π° ΠΌΠ΅ΠΆΠ΄Ρƒ Π½Π°ΠΏΡ€Π°Π²Π»Π΅Π½ΠΈΠ΅ΠΌ Π²Π΅ΠΊΡ‚ΠΎΡ€Π° βž›B ΠΈ ΠΏΡ€ΠΎΠ²ΠΎΠ΄Π½ΠΈΠΊΠΎΠΌ с Ρ‚ΠΎΠΊΠΎΠΌ:

 НаправлСниС силы АмпСра опрСдСляСтся с ΠΏΠΎΠΌΠΎΡ‰ΡŒΡŽ ΠΏΡ€Π°Π²ΠΈΠ»Π° Π»Π΅Π²ΠΎΠΉ Ρ€ΡƒΠΊΠΈ:
Ссли Π»Π΅Π²ΡƒΡŽ Ρ€ΡƒΠΊΡƒ Ρ€Π°ΡΠΏΠΎΠ»ΠΎΠΆΠΈΡ‚ΡŒ Ρ‚Π°ΠΊ, Ρ‡Ρ‚ΠΎΠ±Ρ‹ пСрпСндикулярная ΠΏΡ€ΠΎΠ²ΠΎΠ΄Π½ΠΈΠΊΡƒ ΡΠΎΡΡ‚Π°Π²Π»ΡΡŽΡ‰Π°Ρ Π²Π΅ΠΊΡ‚ΠΎΡ€Π° ΠΌΠ°Π³Π½ΠΈΡ‚Π½ΠΎΠΉ ΠΈΠ½Π΄ΡƒΠΊΡ†ΠΈΠΈ Π²Ρ…ΠΎΠ΄ΠΈΠ»Π° Π² ладонь, Π° Ρ‡Π΅Ρ‚Ρ‹Ρ€Π΅ вытянутых ΠΏΠ°Π»ΡŒΡ†Π° ΡƒΠΊΠ°Π·Ρ‹Π²Π°Π»ΠΈ Π±Ρ‹ Π½Π°ΠΏΡ€Π°Π²Π»Π΅Π½ΠΈΠ΅ Ρ‚ΠΎΠΊΠ°, Ρ‚ΠΎ ΠΎΡ‚ΠΎΠ³Π½ΡƒΡ‚Ρ‹ΠΉ Π½Π° 90Β° большой ΠΏΠ°Π»Π΅Ρ† ΡƒΠΊΠ°ΠΆΠ΅Ρ‚ Π½Π°ΠΏΡ€Π°Π²Π»Π΅Π½ΠΈΠ΅ силы АмпСра.
 На элСктричСский заряд, двиТущийся Π² ΠΌΠ°Π³Π½ΠΈΡ‚Π½ΠΎΠΌ ΠΏΠΎΠ»Π΅, дСйствуСт сила Π›ΠΎΡ€Π΅Π½Ρ†Π°. ΠœΠΎΠ΄ΡƒΠ»ΡŒ силы Π›ΠΎΡ€Π΅Π½Ρ†Π°, Π΄Π΅ΠΉΡΡ‚Π²ΡƒΡŽΡ‰Π΅ΠΉ Π½Π° ΠΏΠΎΠ»ΠΎΠΆΠΈΡ‚Π΅Π»ΡŒΠ½Ρ‹ΠΉ заряд, Ρ€Π°Π²Π΅Π½ ΠΏΡ€ΠΎΠΈΠ·Π²Π΅Π΄Π΅Π½ΠΈΡŽ модуля заряда Π½Π° ΠΌΠΎΠ΄ΡƒΠ»ΡŒ Π²Π΅ΠΊΡ‚ΠΎΡ€Π° ΠΌΠ°Π³Π½ΠΈΡ‚Π½ΠΎΠΉ ΠΈΠ½Π΄ΡƒΠΊΡ†ΠΈΠΈ ΠΈ Π½Π° синус ΡƒΠ³Π»Π° ΠΌΠ΅ΠΆΠ΄Ρƒ Π²Π΅ΠΊΡ‚ΠΎΡ€ΠΎΠΌ ΠΌΠ°Π³Π½ΠΈΡ‚Π½ΠΎΠΉ ΠΈΠ½Π΄ΡƒΠΊΡ†ΠΈΠΈ ΠΈ Π²Π΅ΠΊΡ‚ΠΎΡ€ΠΎΠΌ скорости двиТущСгося заряда:

 НаправлСниС силы Π›ΠΎΡ€Π΅Π½Ρ†Π° опрСдСляСтся с ΠΏΠΎΠΌΠΎΡ‰ΡŒΡŽ ΠΏΡ€Π°Π²ΠΈΠ»Π° Π»Π΅Π²ΠΎΠΉ Ρ€ΡƒΠΊΠΈ: Ссли Π»Π΅Π²ΡƒΡŽ Ρ€ΡƒΠΊΡƒ Ρ€Π°ΡΠΏΠΎΠ»ΠΎΠΆΠΈΡ‚ΡŒ Ρ‚Π°ΠΊ, Ρ‡Ρ‚ΠΎΠ±Ρ‹ ΡΠΎΡΡ‚Π°Π²Π»ΡΡŽΡ‰Π°Ρ ΠΌΠ°Π³Π½ΠΈΡ‚Π½ΠΎΠΉ ΠΈΠ½Π΄ΡƒΠΊΡ†ΠΈΠΈ, пСрпСндикулярная скорости заряда, Π²Ρ…ΠΎΠ΄ΠΈΠ»Π° Π² ладонь, Π° Ρ‡Π΅Ρ‚Ρ‹Ρ€Π΅ ΠΏΠ°Π»ΡŒΡ†Π° Π±Ρ‹Π»ΠΈ Π½Π°ΠΏΡ€Π°Π²Π»Π΅Π½Ρ‹ ΠΏΠΎ двиТСнию ΠΏΠΎΠ»ΠΎΠΆΠΈΡ‚Π΅Π»ΡŒΠ½ΠΎΠ³ΠΎ заряда, Ρ‚ΠΎ ΠΎΡ‚ΠΎΠ³Π½ΡƒΡ‚Ρ‹ΠΉ Π½Π° 90Β° большой ΠΏΠ°Π»Π΅Ρ† ΠΏΠΎΠΊΠ°ΠΆΠ΅Ρ‚ Π½Π°ΠΏΡ€Π°Π²Π»Π΅Π½ΠΈΠ΅ силы Π›ΠΎΡ€Π΅Π½Ρ†Π°, Π΄Π΅ΠΉΡΡ‚Π²ΡƒΡŽΡ‰Π΅ΠΉ Π½Π° заряд. Для ΠΎΡ‚Ρ€ΠΈΡ†Π°Ρ‚Π΅Π»ΡŒΠ½ΠΎ заряТСнной частицы сила Π›ΠΎΡ€Π΅Π½Ρ†Π° Π½Π°ΠΏΡ€Π°Π²Π»Π΅Π½Π° ΠΏΡ€ΠΎΡ‚ΠΈΠ² направлСния большого ΠΏΠ°Π»ΡŒΡ†Π°.

 

3.5. ΠžΡΠ½ΠΎΠ²Π½Ρ‹Π΅ понятия ΠΈ Π·Π°ΠΊΠΎΠ½Ρ‹ элСктромагнитной ΠΈΠ½Π΄ΡƒΠΊΡ†ΠΈΠΈ  Если Π·Π°ΠΌΠΊΠ½ΡƒΡ‚Ρ‹ΠΉ проводящий ΠΊΠΎΠ½Ρ‚ΡƒΡ€ пронизываСтся ΠΌΠ΅Π½ΡΡŽΡ‰ΠΈΠΌΡΡ ΠΌΠ°Π³Π½ΠΈΡ‚Π½Ρ‹ΠΌ ΠΏΠΎΡ‚ΠΎΠΊΠΎΠΌ, Ρ‚ΠΎ Π² этом ΠΊΠΎΠ½Ρ‚ΡƒΡ€Π΅ Π²ΠΎΠ·Π½ΠΈΠΊΠ°Π΅Ρ‚ Π­Π”Π‘ ΠΈ элСктричСский Ρ‚ΠΎΠΊ. Π­Ρ‚Ρƒ Π­Π”Π‘ Π½Π°Π·Ρ‹Π²Π°ΡŽΡ‚ Π­Π”Π‘ элСктромагнитной ΠΈΠ½Π΄ΡƒΠΊΡ†ΠΈΠΈ, Π° Ρ‚ΠΎΠΊ β€” ΠΈΠ½Π΄ΡƒΠΊΡ†ΠΈΠΎΠ½Π½Ρ‹ΠΌ. Π―Π²Π»Π΅Π½ΠΈΠ΅ ΠΈΡ… возникновСния Π½Π°Π·Ρ‹Π²Π°ΡŽΡ‚ элСктромагнитной ΠΈΠ½Π΄ΡƒΠΊΡ†ΠΈΠ΅ΠΉ. Π­Π”Π‘ ΠΈΠ½Π΄ΡƒΠΊΡ†ΠΈΠΈ ΠΌΠΎΠΆΠ½ΠΎ ΠΏΠΎΠ΄ΡΡ‡ΠΈΡ‚Π°Ρ‚ΡŒ ΠΏΠΎ основному Π·Π°ΠΊΠΎΠ½Ρƒ элСктромагнитной ΠΈΠ½Π΄ΡƒΠΊΡ†ΠΈΠΈ ΠΈΠ»ΠΈ ΠΏΠΎ Π·Π°ΠΊΠΎΠ½Ρƒ ЀарадСя:

Π—Π½Π°ΠΊ Β«βˆ’Β» связан с Π½Π°ΠΏΡ€Π°Π²Π»Π΅Π½ΠΈΠ΅ΠΌ ΠΈΠ½Π΄ΡƒΠΊΡ†ΠΈΠΎΠ½Π½ΠΎΠ³ΠΎ Ρ‚ΠΎΠΊΠ°. Оно опрСдСляСтся ΠΏΠΎ ΠΏΡ€Π°Π²ΠΈΠ»Ρƒ Π›Π΅Π½Ρ†Π°:
ΠΈΠ½Π΄ΡƒΠΊΡ†ΠΈΠΎΠ½Π½Ρ‹ΠΉ Ρ‚ΠΎΠΊ ΠΈΠΌΠ΅Π΅Ρ‚ Ρ‚Π°ΠΊΠΎΠ΅ Π½Π°ΠΏΡ€Π°Π²Π»Π΅Π½ΠΈΠ΅, Ρ‡Ρ‚ΠΎ Π΅Π³ΠΎ дСйствиС противодСйствуСт ΠΏΡ€ΠΈΡ‡ΠΈΠ½Π΅, Π²Ρ‹Π·Π²Π°Π²ΡˆΠ΅ΠΉ появлСниС этого Ρ‚ΠΎΠΊΠ°.
β€ƒΠœΠ°Π³Π½ΠΈΡ‚Π½Ρ‹ΠΉ ΠΏΠΎΡ‚ΠΎΠΊ, ΠΏΡ€ΠΎΠ½ΠΈΠ·Ρ‹Π²Π°ΡŽΡ‰ΠΈΠΉ ΠΊΠΎΠ½Ρ‚ΡƒΡ€, прямо ΠΏΡ€ΠΎΠΏΠΎΡ€Ρ†ΠΈΠΎΠ½Π°Π»Π΅Π½ Ρ‚ΠΎΠΊΡƒ, ΠΏΡ€ΠΎΡ‚Π΅ΠΊΠ°ΡŽΡ‰Π΅ΠΌΡƒ Π² этом ΠΊΠΎΠ½Ρ‚ΡƒΡ€Π΅:

ΠšΠΎΡΡ„Ρ„ΠΈΡ†ΠΈΠ΅Π½Ρ‚ ΠΏΡ€ΠΎΠΏΠΎΡ€Ρ†ΠΈΠΎΠ½Π°Π»ΡŒΠ½ΠΎΡΡ‚ΠΈ L зависит ΠΎΡ‚ Π³Π΅ΠΎΠΌΠ΅Ρ‚Ρ€ΠΈΠΈ ΠΊΠΎΠ½Ρ‚ΡƒΡ€Π° ΠΈ называСтся ΠΈΠ½Π΄ΡƒΠΊΡ‚ΠΈΠ²Π½ΠΎΡΡ‚ΡŒΡŽ, ΠΈΠ»ΠΈ коэффициСнтом самоиндукции этого ΠΊΠΎΠ½Ρ‚ΡƒΡ€Π°. [L] = 1 Π“Π½

β€ƒΠ­Π½Π΅Ρ€Π³ΠΈΡŽ ΠΌΠ°Π³Π½ΠΈΡ‚Π½ΠΎΠ³ΠΎ поля Ρ‚ΠΎΠΊΠ° ΠΌΠΎΠΆΠ½ΠΎ ΠΏΠΎΠ΄ΡΡ‡ΠΈΡ‚Π°Ρ‚ΡŒ ΠΏΠΎ Ρ„ΠΎΡ€ΠΌΡƒΠ»Π΅

Π³Π΄Π΅ L β€” ΠΈΠ½Π΄ΡƒΠΊΡ‚ΠΈΠ²Π½ΠΎΡΡ‚ΡŒ ΠΏΡ€ΠΎΠ²ΠΎΠ΄Π½ΠΈΠΊΠ°, ΡΠΎΠ·Π΄Π°ΡŽΡ‰Π΅Π³ΠΎ ΠΏΠΎΠ»Π΅; I β€” Ρ‚ΠΎΠΊ, Ρ‚Π΅ΠΊΡƒΡ‰ΠΈΠΉ ΠΏΠΎ этому ΠΏΡ€ΠΎΠ²ΠΎΠ΄Π½ΠΈΠΊΡƒ

 

3.6. Π­Π»Π΅ΠΊΡ‚Ρ€ΠΎΠΌΠ°Π³Π½ΠΈΡ‚Π½Ρ‹Π΅ колСбания ΠΈ Π²ΠΎΠ»Π½Ρ‹ β€ƒΠšΠΎΠ»Π΅Π±Π°Ρ‚Π΅Π»ΡŒΠ½Ρ‹ΠΌ ΠΊΠΎΠ½Ρ‚ΡƒΡ€ΠΎΠΌ называСтся элСктричСская Ρ†Π΅ΠΏΡŒ, состоящая ΠΈΠ· ΠΏΠΎΡΠ»Π΅Π΄ΠΎΠ²Π°Ρ‚Π΅Π»ΡŒΠ½ΠΎ соСдинённых кондСнсатора с Ρ‘ΠΌΠΊΠΎΡΡ‚ΡŒΡŽ C ΠΈ ΠΊΠ°Ρ‚ΡƒΡˆΠΊΠΈ с ΠΈΠ½Π΄ΡƒΠΊΡ‚ΠΈΠ²Π½ΠΎΡΡ‚ΡŒΡŽ L (см. рис. 7).

 Для свободных Π½Π΅Π·Π°Ρ‚ΡƒΡ…Π°ΡŽΡ‰ΠΈΡ… ΠΊΠΎΠ»Π΅Π±Π°Π½ΠΈΠΉ Π² ΠΊΠΎΠ½Ρ‚ΡƒΡ€Π΅ цикличСская частота опрСдСляСтся Ρ„ΠΎΡ€ΠΌΡƒΠ»ΠΎΠΉ

β€ƒΠŸΠ΅Ρ€ΠΈΠΎΠ΄ свободных ΠΊΠΎΠ»Π΅Π±Π°Π½ΠΈΠΉ Π² ΠΊΠΎΠ½Ρ‚ΡƒΡ€Π΅ опрСдСляСтся Ρ„ΠΎΡ€ΠΌΡƒΠ»ΠΎΠΉ Вомсона:

 Если Π² LC-ΠΊΠΎΠ½Ρ‚ΡƒΡ€ ΠΏΠΎΡΠ»Π΅Π΄ΠΎΠ²Π°Ρ‚Π΅Π»ΡŒΠ½ΠΎ с L, C ΠΈ R Π²ΠΊΠ»ΡŽΡ‡ΠΈΡ‚ΡŒ источник ΠΏΠ΅Ρ€Π΅ΠΌΠ΅Π½Π½ΠΎΠ³ΠΎ напряТСния, Ρ‚ΠΎ Π² Ρ†Π΅ΠΏΠΈ Π²ΠΎΠ·Π½ΠΈΠΊΠ½ΡƒΡ‚ Π²Ρ‹Π½ΡƒΠΆΠ΄Π΅Π½Π½Ρ‹Π΅ элСктричСскиС колСбания. Π’Π°ΠΊΠΈΠ΅ колСбания принято Π½Π°Π·Ρ‹Π²Π°Ρ‚ΡŒ ΠΏΠ΅Ρ€Π΅ΠΌΠ΅Π½Π½Ρ‹ΠΌ элСктричСским Ρ‚ΠΎΠΊΠΎΠΌ
 В Ρ†Π΅ΠΏΡŒ ΠΏΠ΅Ρ€Π΅ΠΌΠ΅Π½Π½ΠΎΠ³ΠΎ Ρ‚ΠΎΠΊΠ° ΠΌΠΎΠΆΠ½ΠΎ Π²ΠΊΠ»ΡŽΡ‡Π°Ρ‚ΡŒ Ρ‚Ρ€ΠΈ Π²ΠΈΠ΄Π° Π½Π°Π³Ρ€ΡƒΠ·ΠΊΠΈ β€” кондСнсатор, рСзистор ΠΈ ΠΊΠ°Ρ‚ΡƒΡˆΠΊΡƒ индуктивности.

β€ƒΠšΠΎΠ½Π΄Π΅Π½ΡΠ°Ρ‚ΠΎΡ€ ΠΎΠΊΠ°Π·Ρ‹Π²Π°Π΅Ρ‚ ΠΏΠ΅Ρ€Π΅ΠΌΠ΅Π½Π½ΠΎΠΌΡƒ Ρ‚ΠΎΠΊΡƒ сопротивлСниС, ΠΊΠΎΡ‚ΠΎΡ€ΠΎΠ΅ ΠΌΠΎΠΆΠ½ΠΎ ΠΏΠΎΡΡ‡ΠΈΡ‚Π°Ρ‚ΡŒ ΠΏΠΎ Ρ„ΠΎΡ€ΠΌΡƒΠ»Π΅

 Вок, Ρ‚Π΅ΠΊΡƒΡ‰ΠΈΠΉ Ρ‡Π΅Ρ€Π΅Π· кондСнсатор, ΠΏΠΎ Ρ„Π°Π·Π΅ ΠΎΠΏΠ΅Ρ€Π΅ΠΆΠ°Π΅Ρ‚ напряТСниС Π½Π° Ο€/2 ΠΈΠ»ΠΈ Π½Π° Ρ‡Π΅Ρ‚Π²Π΅Ρ€Ρ‚ΡŒ ΠΏΠ΅Ρ€ΠΈΠΎΠ΄Π°, Π° напряТСниС отстаёт ΠΎΡ‚ Ρ‚ΠΎΠΊΠ° Π½Π° Ρ‚Π°ΠΊΠΎΠΉ ΠΆΠ΅ Ρ„Π°Π·ΠΎΠ²Ρ‹ΠΉ ΡƒΠ³ΠΎΠ».

β€ƒΠšΠ°Ρ‚ΡƒΡˆΠΊΠ° индуктивности ΠΎΠΊΠ°Π·Ρ‹Π²Π°Π΅Ρ‚ ΠΏΠ΅Ρ€Π΅ΠΌΠ΅Π½Π½ΠΎΠΌΡƒ Ρ‚ΠΎΠΊΡƒ сопротивлСниС, ΠΊΠΎΡ‚ΠΎΡ€ΠΎΠ΅ ΠΌΠΎΠΆΠ½ΠΎ ΠΏΠΎΡΡ‡ΠΈΡ‚Π°Ρ‚ΡŒ ΠΏΠΎ Ρ„ΠΎΡ€ΠΌΡƒΠ»Π΅

 Вок, Ρ‚Π΅ΠΊΡƒΡ‰ΠΈΠΉ Ρ‡Π΅Ρ€Π΅Π· ΠΊΠ°Ρ‚ΡƒΡˆΠΊΡƒ индуктивности, ΠΏΠΎ Ρ„Π°Π·Π΅ отстаёт ΠΎΡ‚ напряТСния Π½Π° Ο€/2 ΠΈΠ»ΠΈ Π½Π° Ρ‡Π΅Ρ‚Π²Π΅Ρ€Ρ‚ΡŒ ΠΏΠ΅Ρ€ΠΈΠΎΠ΄Π°. НапряТСниС ΠΎΠΏΠ΅Ρ€Π΅ΠΆΠ°Π΅Ρ‚ Ρ‚ΠΎΠΊ Π½Π° Ρ‚Π°ΠΊΠΎΠΉ ΠΆΠ΅ Ρ„Π°Π·ΠΎΠ²Ρ‹ΠΉ ΡƒΠ³ΠΎΠ».

 Врансформатором называСтся устройство, ΠΏΡ€Π΅Π΄Π½Π°Π·Π½Π°Ρ‡Π΅Π½Π½ΠΎΠ΅ для прСобразования ΠΏΠ΅Ρ€Π΅ΠΌΠ΅Π½Π½Ρ‹Ρ… Ρ‚ΠΎΠΊΠΎΠ². Врансформатор состоит ΠΈΠ· Π·Π°ΠΌΠΊΠ½ΡƒΡ‚ΠΎΠ³ΠΎ ΡΡ‚Π°Π»ΡŒΠ½ΠΎΠ³ΠΎ сСрдСчника, Π½Π° ΠΊΠΎΡ‚ΠΎΡ€Ρ‹ΠΉ Π½Π°Π΄Π΅Ρ‚Ρ‹ Π΄Π²Π΅ ΠΊΠ°Ρ‚ΡƒΡˆΠΊΠΈ. ΠšΠ°Ρ‚ΡƒΡˆΠΊΠ°, которая ΠΏΠΎΠ΄ΠΊΠ»ΡŽΡ‡Π°Π΅Ρ‚ΡΡ ΠΊ источнику ΠΏΠ΅Ρ€Π΅ΠΌΠ΅Π½Π½ΠΎΠ³ΠΎ напряТСния, называСтся ΠΏΠ΅Ρ€Π²ΠΈΡ‡Π½ΠΎΠΉ ΠΎΠ±ΠΌΠΎΡ‚ΠΊΠΎΠΉ, Π° ΠΊΠ°Ρ‚ΡƒΡˆΠΊΠ°, которая ΠΏΠΎΠ΄ΠΊΠ»ΡŽΡ‡Π°Π΅Ρ‚ΡΡ ΠΊ ΠΏΠΎΡ‚Ρ€Π΅Π±ΠΈΡ‚Π΅Π»ΡŽ, называСтся Π²Ρ‚ΠΎΡ€ΠΈΡ‡Π½ΠΎΠΉ ΠΎΠ±ΠΌΠΎΡ‚ΠΊΠΎΠΉ. ΠžΡ‚Π½ΠΎΡˆΠ΅Π½ΠΈΠ΅ напряТСния Π½Π° ΠΏΠ΅Ρ€Π²ΠΈΡ‡Π½ΠΎΠΉ ΠΎΠ±ΠΌΠΎΡ‚ΠΊΠ΅ ΠΈ Π²Ρ‚ΠΎΡ€ΠΈΡ‡Π½ΠΎΠΉ ΠΎΠ±ΠΌΠΎΡ‚ΠΊΠ΅ трансформатора Ρ€Π°Π²Π½ΠΎ ΠΎΡ‚Π½ΠΎΡˆΠ΅Π½ΠΈΡŽ числа Π²ΠΈΡ‚ΠΊΠΎΠ² Π² этих ΠΎΠ±ΠΌΠΎΡ‚ΠΊΠ°Ρ…:

Если K > 1, трансформатор ΠΏΠΎΠ½ΠΈΠΆΠ°ΡŽΡ‰ΠΈΠΉ, Ссли K

ЭлСктромагнитная индукция. ΠŸΡ€Π°Π²ΠΈΠ»ΠΎ Π›Π΅Π½Ρ†Π°

Π―Π²Π»Π΅Π½ΠΈΠ΅ элСктромагнитной ΠΈΠ½Π΄ΡƒΠΊΡ†ΠΈΠΈ Π±Ρ‹Π»ΠΎ ΠΎΡ‚ΠΊΡ€Ρ‹Ρ‚ΠΎ Π²Ρ‹Π΄Π°ΡŽΡ‰ΠΈΠΌΡΡ английским Ρ„ΠΈΠ·ΠΈΠΊΠΎΠΌ М.Π€Π°Ρ€Π°Π΄Π΅Π΅ΠΌ Π² 1831Β Π³. Оно Π·Π°ΠΊΠ»ΡŽΡ‡Π°Π΅Ρ‚ΡΡ Π² Π²ΠΎΠ·Π½ΠΈΠΊΠ½ΠΎΠ²Π΅Π½ΠΈΠΈ элСктричСского Ρ‚ΠΎΠΊΠ° Π² Π·Π°ΠΌΠΊΠ½ΡƒΡ‚ΠΎΠΌ проводящСм ΠΊΠΎΠ½Ρ‚ΡƒΡ€Π΅ ΠΏΡ€ΠΈ ΠΈΠ·ΠΌΠ΅Π½Π΅Π½ΠΈΠΈ Π²ΠΎ Π²Ρ€Π΅ΠΌΠ΅Π½ΠΈ ΠΌΠ°Π³Π½ΠΈΡ‚Π½ΠΎΠ³ΠΎ ΠΏΠΎΡ‚ΠΎΠΊΠ°, ΠΏΡ€ΠΎΠ½ΠΈΠ·Ρ‹Π²Π°ΡŽΡ‰Π΅Π³ΠΎ ΠΊΠΎΠ½Ρ‚ΡƒΡ€.

ΠœΠ°Π³Π½ΠΈΡ‚Π½Ρ‹ΠΌ ΠΏΠΎΡ‚ΠΎΠΊΠΎΠΌ Ξ¦ Ρ‡Π΅Ρ€Π΅Π· ΠΏΠ»ΠΎΡ‰Π°Π΄ΡŒ S ΠΊΠΎΠ½Ρ‚ΡƒΡ€Π° Π½Π°Π·Ρ‹Π²Π°ΡŽΡ‚ Π²Π΅Π»ΠΈΡ‡ΠΈΠ½Ρƒ

Ξ¦ = BΒ Β·Β SΒ Β·Β cosΒ Ξ±,

Π³Π΄Π΅ B – ΠΌΠΎΠ΄ΡƒΠ»ΡŒ Π²Π΅ΠΊΡ‚ΠΎΡ€Π° ΠΌΠ°Π³Π½ΠΈΡ‚Π½ΠΎΠΉ ΠΈΠ½Π΄ΡƒΠΊΡ†ΠΈΠΈ, Ξ± – ΡƒΠ³ΠΎΠ» ΠΌΠ΅ΠΆΠ΄Ρƒ Π²Π΅ΠΊΡ‚ΠΎΡ€ΠΎΠΌ Β ΠΈ Π½ΠΎΡ€ΠΌΠ°Π»ΡŒΡŽ Β ΠΊ плоскости ΠΊΠΎΠ½Ρ‚ΡƒΡ€Π° (рис.Β 1.20.1).

Рисунок 1.20.1.

ΠœΠ°Π³Π½ΠΈΡ‚Π½Ρ‹ΠΉ ΠΏΠΎΡ‚ΠΎΠΊ Ρ‡Π΅Ρ€Π΅Π· Π·Π°ΠΌΠΊΠ½ΡƒΡ‚Ρ‹ΠΉ ΠΊΠΎΠ½Ρ‚ΡƒΡ€. НаправлСниС Π½ΠΎΡ€ΠΌΠ°Π»ΠΈ Β Β ΠΈ Π²Ρ‹Π±Ρ€Π°Π½Π½ΠΎΠ΅ ΠΏΠΎΠ»ΠΎΠΆΠΈΡ‚Π΅Π»ΡŒΠ½ΠΎΠ΅ Π½Π°ΠΏΡ€Π°Π²Π»Π΅Π½ΠΈΠ΅ Β Β ΠΎΠ±Ρ…ΠΎΠ΄Π° ΠΊΠΎΠ½Ρ‚ΡƒΡ€Π° связаны ΠΏΡ€Π°Π²ΠΈΠ»ΠΎΠΌ ΠΏΡ€Π°Π²ΠΎΠ³ΠΎ Π±ΡƒΡ€Π°Π²Ρ‡ΠΈΠΊΠ°

ΠžΠΏΡ€Π΅Π΄Π΅Π»Π΅Π½ΠΈΠ΅ ΠΌΠ°Π³Π½ΠΈΡ‚Π½ΠΎΠ³ΠΎ ΠΏΠΎΡ‚ΠΎΠΊΠ° Π½Π΅Ρ‚Ρ€ΡƒΠ΄Π½ΠΎ ΠΎΠ±ΠΎΠ±Ρ‰ΠΈΡ‚ΡŒ Π½Π° случай Π½Π΅ΠΎΠ΄Π½ΠΎΡ€ΠΎΠ΄Π½ΠΎΠ³ΠΎ ΠΌΠ°Π³Π½ΠΈΡ‚Π½ΠΎΠ³ΠΎ поля ΠΈ нСплоского ΠΊΠΎΠ½Ρ‚ΡƒΡ€Π°. Π•Π΄ΠΈΠ½ΠΈΡ†Π° ΠΌΠ°Π³Π½ΠΈΡ‚Π½ΠΎΠ³ΠΎ ΠΏΠΎΡ‚ΠΎΠΊΠ° Π² систСмС БИ называСтся Π’Π΅Π±Π΅Ρ€ΠΎΠΌ (Π’Π±). ΠœΠ°Π³Π½ΠΈΡ‚Π½Ρ‹ΠΉ ΠΏΠΎΡ‚ΠΎΠΊ, Ρ€Π°Π²Π½Ρ‹ΠΉ 1Β Π’Π±, создаСтся ΠΌΠ°Π³Π½ΠΈΡ‚Π½Ρ‹ΠΌ ΠΏΠΎΠ»Π΅ΠΌ с ΠΈΠ½Π΄ΡƒΠΊΡ†ΠΈΠ΅ΠΉ 1Β Π’Π», ΠΏΡ€ΠΎΠ½ΠΈΠ·Ρ‹Π²Π°ΡŽΡ‰ΠΈΠΌ ΠΏΠΎ Π½Π°ΠΏΡ€Π°Π²Π»Π΅Π½ΠΈΡŽ Π½ΠΎΡ€ΠΌΠ°Π»ΠΈ плоский ΠΊΠΎΠ½Ρ‚ΡƒΡ€ ΠΏΠ»ΠΎΡ‰Π°Π΄ΡŒΡŽ 1Β ΠΌ2:

1Β Π’Π±Β =Β 1Β Π’Π»Β Β·Β 1Β ΠΌ2.

Π€Π°Ρ€Π°Π΄Π΅ΠΉ ΡΠΊΡΠΏΠ΅Ρ€ΠΈΠΌΠ΅Π½Ρ‚Π°Π»ΡŒΠ½ΠΎ установил, Ρ‡Ρ‚ΠΎ ΠΏΡ€ΠΈ ΠΈΠ·ΠΌΠ΅Π½Π΅Π½ΠΈΠΈ ΠΌΠ°Π³Π½ΠΈΡ‚Π½ΠΎΠ³ΠΎ ΠΏΠΎΡ‚ΠΎΠΊΠ° Π² проводящСм ΠΊΠΎΠ½Ρ‚ΡƒΡ€Π΅ Π²ΠΎΠ·Π½ΠΈΠΊΠ°Π΅Ρ‚ Π­Π”Π‘ ΠΈΠ½Π΄ΡƒΠΊΡ†ΠΈΠΈ равная скорости измСнСния ΠΌΠ°Π³Π½ΠΈΡ‚Π½ΠΎΠ³ΠΎ ΠΏΠΎΡ‚ΠΎΠΊΠ° Ρ‡Π΅Ρ€Π΅Π· ΠΏΠΎΠ²Π΅Ρ€Ρ…Π½ΠΎΡΡ‚ΡŒ, ΠΎΠ³Ρ€Π°Π½ΠΈΡ‡Π΅Π½Π½ΡƒΡŽ ΠΊΠΎΠ½Ρ‚ΡƒΡ€ΠΎΠΌ, взятой со Π·Π½Π°ΠΊΠΎΠΌ минус:

Π­Ρ‚Π° Ρ„ΠΎΡ€ΠΌΡƒΠ»Π° носит Π½Π°Π·Π²Π°Π½ΠΈΠ΅ Π·Π°ΠΊΠΎΠ½Π° ЀарадСя.

ΠžΠΏΡ‹Ρ‚ ΠΏΠΎΠΊΠ°Π·Ρ‹Π²Π°Π΅Ρ‚, Ρ‡Ρ‚ΠΎ ΠΈΠ½Π΄ΡƒΠΊΡ†ΠΈΠΎΠ½Π½Ρ‹ΠΉ Ρ‚ΠΎΠΊ, Π²ΠΎΠ·Π±ΡƒΠΆΠ΄Π°Π΅ΠΌΡ‹ΠΉ Π² Π·Π°ΠΌΠΊΠ½ΡƒΡ‚ΠΎΠΌ ΠΊΠΎΠ½Ρ‚ΡƒΡ€Π΅ ΠΏΡ€ΠΈ ΠΈΠ·ΠΌΠ΅Π½Π΅Π½ΠΈΠΈ ΠΌΠ°Π³Π½ΠΈΡ‚Π½ΠΎΠ³ΠΎ ΠΏΠΎΡ‚ΠΎΠΊΠ°, всСгда Π½Π°ΠΏΡ€Π°Π²Π»Π΅Π½ Ρ‚Π°ΠΊ, Ρ‡Ρ‚ΠΎ создаваСмоС ΠΈΠΌ ΠΌΠ°Π³Π½ΠΈΡ‚Π½ΠΎΠ΅ ΠΏΠΎΠ»Π΅ прСпятствуСт измСнСнию ΠΌΠ°Π³Π½ΠΈΡ‚Π½ΠΎΠ³ΠΎ ΠΏΠΎΡ‚ΠΎΠΊΠ°, Π²Ρ‹Π·Ρ‹Π²Π°ΡŽΡ‰Π΅Π³ΠΎ ΠΈΠ½Π΄ΡƒΠΊΡ†ΠΈΠΎΠ½Π½Ρ‹ΠΉ Ρ‚ΠΎΠΊ. Π­Ρ‚ΠΎ ΡƒΡ‚Π²Π΅Ρ€ΠΆΠ΄Π΅Π½ΠΈΠ΅, сформулированноС Π²Β 1833Β Π³., называСтся ΠΏΡ€Π°Π²ΠΈΠ»ΠΎΠΌ Π›Π΅Π½Ρ†Π°.

Рис.Β 1.20.2 ΠΈΠ»Π»ΡŽΡΡ‚Ρ€ΠΈΡ€ΡƒΠ΅Ρ‚ ΠΏΡ€Π°Π²ΠΈΠ»ΠΎ Π›Π΅Π½Ρ†Π° Π½Π° ΠΏΡ€ΠΈΠΌΠ΅Ρ€Π΅ Π½Π΅ΠΏΠΎΠ΄Π²ΠΈΠΆΠ½ΠΎΠ³ΠΎ проводящСго ΠΊΠΎΠ½Ρ‚ΡƒΡ€Π°, ΠΊΠΎΡ‚ΠΎΡ€Ρ‹ΠΉ находится Π² ΠΎΠ΄Π½ΠΎΡ€ΠΎΠ΄Π½ΠΎΠΌ ΠΌΠ°Π³Π½ΠΈΡ‚Π½ΠΎΠΌ ΠΏΠΎΠ»Π΅, ΠΌΠΎΠ΄ΡƒΠ»ΡŒ ΠΈΠ½Π΄ΡƒΠΊΡ†ΠΈΠΈ ΠΊΠΎΡ‚ΠΎΡ€ΠΎΠ³ΠΎ увСличиваСтся Π²ΠΎ Π²Ρ€Π΅ΠΌΠ΅Π½ΠΈ.

Рисунок 1.20.2.

Π˜Π»Π»ΡŽΡΡ‚Ρ€Π°Ρ†ΠΈΡ ΠΏΡ€Π°Π²ΠΈΠ»Π° Π›Π΅Π½Ρ†Π°. Π’ этом ΠΏΡ€ΠΈΠΌΠ΅Ρ€Π΅ ,Β  Π° Β . Π˜Π½Π΄ΡƒΠΊΡ†ΠΈΠΎΠ½Π½Ρ‹ΠΉ Ρ‚ΠΎΠΊ IΠΈΠ½Π΄ Ρ‚Π΅Ρ‡Π΅Ρ‚ навстрСчу Π²Ρ‹Π±Ρ€Π°Π½Π½ΠΎΠΌΡƒ ΠΏΠΎΠ»ΠΎΠΆΠΈΡ‚Π΅Π»ΡŒΠ½ΠΎΠΌΡƒ Π½Π°ΠΏΡ€Π°Π²Π»Π΅Π½ΠΈΡŽ Β  ΠΎΠ±Ρ…ΠΎΠ΄Π° ΠΊΠΎΠ½Ρ‚ΡƒΡ€Π°

ΠŸΡ€Π°Π²ΠΈΠ»ΠΎ Π›Π΅Π½Ρ†Π° ΠΎΡ‚Ρ€Π°ΠΆΠ°Π΅Ρ‚ Ρ‚ΠΎΡ‚ ΡΠΊΡΠΏΠ΅Ρ€ΠΈΠΌΠ΅Π½Ρ‚Π°Π»ΡŒΠ½Ρ‹ΠΉ Ρ„Π°ΠΊΡ‚, Ρ‡Ρ‚ΠΎ Β ΠΈ  всСгда ΠΈΠΌΠ΅ΡŽΡ‚ ΠΏΡ€ΠΎΡ‚ΠΈΠ²ΠΎΠΏΠΎΠ»ΠΎΠΆΠ½Ρ‹Π΅ Π·Π½Π°ΠΊΠΈ (Π·Π½Π°ΠΊ «минус» Π² Ρ„ΠΎΡ€ΠΌΡƒΠ»Π΅ ЀарадСя). ΠŸΡ€Π°Π²ΠΈΠ»ΠΎ Π›Π΅Π½Ρ†Π° ΠΈΠΌΠ΅Π΅Ρ‚ Π³Π»ΡƒΠ±ΠΎΠΊΠΈΠΉ физичСский смысл – ΠΎΠ½ΠΎ Π²Ρ‹Ρ€Π°ΠΆΠ°Π΅Ρ‚ Π·Π°ΠΊΠΎΠ½ сохранСния энСргии.

ИзмСнСниС ΠΌΠ°Π³Π½ΠΈΡ‚Π½ΠΎΠ³ΠΎ ΠΏΠΎΡ‚ΠΎΠΊΠ°, ΠΏΡ€ΠΎΠ½ΠΈΠ·Ρ‹Π²Π°ΡŽΡ‰Π΅Π³ΠΎ Π·Π°ΠΌΠΊΠ½ΡƒΡ‚Ρ‹ΠΉ ΠΊΠΎΠ½Ρ‚ΡƒΡ€, ΠΌΠΎΠΆΠ΅Ρ‚ ΠΏΡ€ΠΎΠΈΡΡ…ΠΎΠ΄ΠΈΡ‚ΡŒ ΠΏΠΎ Π΄Π²ΡƒΠΌ ΠΏΡ€ΠΈΡ‡ΠΈΠ½Π°ΠΌ.

1. ΠœΠ°Π³Π½ΠΈΡ‚Π½Ρ‹ΠΉ ΠΏΠΎΡ‚ΠΎΠΊ измСняСтся вслСдствиС пСрСмСщСния ΠΊΠΎΠ½Ρ‚ΡƒΡ€Π° ΠΈΠ»ΠΈ Π΅Π³ΠΎ частСй Π² постоянном Π²ΠΎ Π²Ρ€Π΅ΠΌΠ΅Π½ΠΈ ΠΌΠ°Π³Π½ΠΈΡ‚Π½ΠΎΠΌ ΠΏΠΎΠ»Π΅. Π­Ρ‚ΠΎ случай, ΠΊΠΎΠ³Π΄Π° ΠΏΡ€ΠΎΠ²ΠΎΠ΄Π½ΠΈΠΊΠΈ, Π° вмСстС с Π½ΠΈΠΌΠΈ ΠΈ свободныС носитСли заряда, двиТутся Π² ΠΌΠ°Π³Π½ΠΈΡ‚Π½ΠΎΠΌ ΠΏΠΎΠ»Π΅. Π’ΠΎΠ·Π½ΠΈΠΊΠ½ΠΎΠ²Π΅Π½ΠΈΠ΅ Π­Π”Π‘ ΠΈΠ½Π΄ΡƒΠΊΡ†ΠΈΠΈ ΠΎΠ±ΡŠΡΡΠ½ΡΠ΅Ρ‚ΡΡ дСйствиСм силы Π›ΠΎΡ€Π΅Π½Ρ†Π° Π½Π° свободныС заряды Π² двиТущихся ΠΏΡ€ΠΎΠ²ΠΎΠ΄Π½ΠΈΠΊΠ°Ρ…. Π‘ΠΈΠ»Π° Π›ΠΎΡ€Π΅Π½Ρ†Π° ΠΈΠ³Ρ€Π°Π΅Ρ‚ Π² этом случаС Ρ€ΠΎΠ»ΡŒ стороннСй силы.

Рассмотрим Π² качСствС ΠΏΡ€ΠΈΠΌΠ΅Ρ€Π° Π²ΠΎΠ·Π½ΠΈΠΊΠ½ΠΎΠ²Π΅Π½ΠΈΠ΅ Π­Π”Π‘ ΠΈΠ½Π΄ΡƒΠΊΡ†ΠΈΠΈ Π² ΠΏΡ€ΡΠΌΠΎΡƒΠ³ΠΎΠ»ΡŒΠ½ΠΎΠΌ ΠΊΠΎΠ½Ρ‚ΡƒΡ€Π΅, ΠΏΠΎΠΌΠ΅Ρ‰Π΅Π½Π½ΠΎΠΌ Π² ΠΎΠ΄Π½ΠΎΡ€ΠΎΠ΄Π½ΠΎΠ΅ ΠΌΠ°Π³Π½ΠΈΡ‚Π½ΠΎΠ΅ ΠΏΠΎΠ»Π΅  пСрпСндикулярноС плоскости ΠΊΠΎΠ½Ρ‚ΡƒΡ€Π°. ΠŸΡƒΡΡ‚ΡŒ ΠΎΠ΄Π½Π° ΠΈΠ· сторон ΠΊΠΎΠ½Ρ‚ΡƒΡ€Π° Π΄Π»ΠΈΠ½ΠΎΠΉ l ΡΠΊΠΎΠ»ΡŒΠ·ΠΈΡ‚ со ΡΠΊΠΎΡ€ΠΎΡΡ‚ΡŒΡŽ Β  ΠΏΠΎ Π΄Π²ΡƒΠΌ Π΄Ρ€ΡƒΠ³ΠΈΠΌ сторонам (рис.Β 1.20.3).

Рисунок 1.20.3.

Π’ΠΎΠ·Π½ΠΈΠΊΠ½ΠΎΠ²Π΅Π½ΠΈΠ΅ Π­Π”Π‘ ΠΈΠ½Π΄ΡƒΠΊΡ†ΠΈΠΈ Π² двиТущСмся ΠΏΡ€ΠΎΠ²ΠΎΠ΄Π½ΠΈΠΊΠ΅. Π£ΠΊΠ°Π·Π°Π½Π° ΡΠΎΡΡ‚Π°Π²Π»ΡΡŽΡ‰Π°Ρ силы Π›ΠΎΡ€Π΅Π½Ρ†Π°, Π΄Π΅ΠΉΡΡ‚Π²ΡƒΡŽΡ‰Π΅ΠΉ Π½Π° свободный элСктрон

На свободныС заряды Π½Π° этом участкС ΠΊΠΎΠ½Ρ‚ΡƒΡ€Π° дСйствуСт сила Π›ΠΎΡ€Π΅Π½Ρ†Π°. Одна ΠΈΠ· ΡΠΎΡΡ‚Π°Π²Π»ΡΡŽΡ‰ΠΈΡ… этой силы, связанная с пСрСносной ΡΠΊΠΎΡ€ΠΎΡΡ‚ΡŒ зарядов, Π½Π°ΠΏΡ€Π°Π²Π»Π΅Π½Π° вдоль ΠΏΡ€ΠΎΠ²ΠΎΠ΄Π½ΠΈΠΊΠ°. Π­Ρ‚Π° ΡΠΎΡΡ‚Π°Π²Π»ΡΡŽΡ‰Π°Ρ ΡƒΠΊΠ°Π·Π°Π½Π° Π½Π° рис.Β 1.20.3. Она ΠΈΠ³Ρ€Π°Π΅Ρ‚ Ρ€ΠΎΠ»ΡŒ стороннСй силы. Π•Π΅ ΠΌΠΎΠ΄ΡƒΠ»ΡŒ Ρ€Π°Π²Π΅Π½

Π Π°Π±ΠΎΡ‚Π° силы FΠ› Π½Π° ΠΏΡƒΡ‚ΠΈ l Ρ€Π°Π²Π½Π°

По ΠΎΠΏΡ€Π΅Π΄Π΅Π»Π΅Π½ΠΈΡŽ Π­Π”Π‘

Π’ Π΄Ρ€ΡƒΠ³ΠΈΡ… Π½Π΅ΠΏΠΎΠ΄Π²ΠΈΠΆΠ½Ρ‹Ρ… частях ΠΊΠΎΠ½Ρ‚ΡƒΡ€Π° сторонняя сила Ρ€Π°Π²Π½Π° Π½ΡƒΠ»ΡŽ. Π‘ΠΎΠΎΡ‚Π½ΠΎΡˆΠ΅Π½ΠΈΡŽ для ΠΈΠ½Π΄ ΠΌΠΎΠΆΠ½ΠΎ ΠΏΡ€ΠΈΠ΄Π°Ρ‚ΡŒ ΠΏΡ€ΠΈΠ²Ρ‹Ρ‡Π½Ρ‹ΠΉ Π²ΠΈΠ΄. Π—Π° врСмя Ξ”t ΠΏΠ»ΠΎΡ‰Π°Π΄ΡŒ ΠΊΠΎΠ½Ρ‚ΡƒΡ€Π° измСняСтся Π½Π° Ξ”S = lΟ…Ξ”t. ИзмСнСниС ΠΌΠ°Π³Π½ΠΈΡ‚Π½ΠΎΠ³ΠΎ ΠΏΠΎΡ‚ΠΎΠΊΠ° Π·Π° это врСмя Ρ€Π°Π²Π½ΠΎ ΔΦ = BlΟ…Ξ”t. Π‘Π»Π΅Π΄ΠΎΠ²Π°Ρ‚Π΅Π»ΡŒΠ½ΠΎ,

Для Ρ‚ΠΎΠ³ΠΎ, Ρ‡Ρ‚ΠΎΠ±Ρ‹ ΡƒΡΡ‚Π°Π½ΠΎΠ²ΠΈΡ‚ΡŒ Π·Π½Π°ΠΊ Π² Ρ„ΠΎΡ€ΠΌΡƒΠ»Π΅, ΡΠ²ΡΠ·Ρ‹Π²Π°ΡŽΡ‰Π΅ΠΉ Β ΠΈ Β Π½ΡƒΠΆΠ½ΠΎ Π²Ρ‹Π±Ρ€Π°Ρ‚ΡŒ согласованныС ΠΌΠ΅ΠΆΠ΄Ρƒ собой ΠΏΠΎ ΠΏΡ€Π°Π²ΠΈΠ»Ρƒ ΠΏΡ€Π°Π²ΠΎΠ³ΠΎ Π±ΡƒΡ€Π°Π²Ρ‡ΠΈΠΊΠ° Π½Π°ΠΏΡ€Π°Π²Π»Π΅Π½ΠΈΠ΅ Π½ΠΎΡ€ΠΌΠ°Π»ΠΈ Β ΠΈ ΠΏΠΎΠ»ΠΎΠΆΠΈΡ‚Π΅Π»ΡŒΠ½ΠΎΠ΅ Π½Π°ΠΏΡ€Π°Π²Π»Π΅Π½ΠΈΠ΅ ΠΎΠ±Ρ…ΠΎΠ΄Π° ΠΊΠΎΠ½Ρ‚ΡƒΡ€Π° Β ΠΊΠ°ΠΊ это сдСлано Π½Π° рис.Β 1.20.1 ΠΈ 1.20.2. Если это ΡΠ΄Π΅Π»Π°Ρ‚ΡŒ, Ρ‚ΠΎ Π»Π΅Π³ΠΊΠΎ ΠΏΡ€ΠΈΠΉΡ‚ΠΈ ΠΊ Ρ„ΠΎΡ€ΠΌΡƒΠ»Π΅ ЀарадСя.

Если сопротивлСниС всСй Ρ†Π΅ΠΏΠΈ Ρ€Π°Π²Π½ΠΎ R, Ρ‚ΠΎ ΠΏΠΎ Π½Π΅ΠΉ Π±ΡƒΠ΄Π΅Ρ‚ ΠΏΡ€ΠΎΡ‚Π΅ΠΊΠ°Ρ‚ΡŒ ΠΈΠ½Π΄ΡƒΠΊΡ†ΠΈΠΎΠ½Π½Ρ‹ΠΉ Ρ‚ΠΎΠΊ, Ρ€Π°Π²Π½Ρ‹ΠΉ . Π—Π° врСмя Ξ”t Π½Π° сопротивлСнии R выдСлится Π΄ΠΆΠΎΡƒΠ»Π΅Π²ΠΎ Ρ‚Π΅ΠΏΠ»ΠΎ

Π’ΠΎΠ·Π½ΠΈΠΊΠ°Π΅Ρ‚ вопрос: ΠΎΡ‚ΠΊΡƒΠ΄Π° бСрСтся эта энСргия, вСдь сила Π›ΠΎΡ€Π΅Π½Ρ†Π° Ρ€Π°Π±ΠΎΡ‚Ρ‹ Π½Π΅ ΡΠΎΠ²Π΅Ρ€ΡˆΠ°Π΅Ρ‚! Π­Ρ‚ΠΎΡ‚ парадокс Π²ΠΎΠ·Π½ΠΈΠΊ ΠΏΠΎΡ‚ΠΎΠΌΡƒ, Ρ‡Ρ‚ΠΎ ΠΌΡ‹ ΡƒΡ‡Π»ΠΈ Ρ€Π°Π±ΠΎΡ‚Ρƒ Ρ‚ΠΎΠ»ΡŒΠΊΠΎ ΠΎΠ΄Π½ΠΎΠΉ ΡΠΎΡΡ‚Π°Π²Π»ΡΡŽΡ‰Π΅ΠΉ силы Π›ΠΎΡ€Π΅Π½Ρ†Π°. ΠŸΡ€ΠΈ ΠΏΡ€ΠΎΡ‚Π΅ΠΊΠ°Π½ΠΈΠΈ ΠΈΠ½Π΄ΡƒΠΊΡ†ΠΈΠΎΠ½Π½ΠΎΠ³ΠΎ Ρ‚ΠΎΠΊΠ° ΠΏΠΎ ΠΏΡ€ΠΎΠ²ΠΎΠ΄Π½ΠΈΠΊΡƒ, находящСмуся Π² ΠΌΠ°Π³Π½ΠΈΡ‚Π½ΠΎΠΌ ΠΏΠΎΠ»Π΅, Π½Π° свободныС заряды дСйствуСт Π΅Ρ‰Π΅ ΠΎΠ΄Π½Π° ΡΠΎΡΡ‚Π°Π²Π»ΡΡŽΡ‰Π°Ρ силы Π›ΠΎΡ€Π΅Π½Ρ†Π°, связанная с ΠΎΡ‚Π½ΠΎΡΠΈΡ‚Π΅Π»ΡŒΠ½ΠΎΠΉ ΡΠΊΠΎΡ€ΠΎΡΡ‚ΡŒΡŽ двиТСния зарядов вдоль ΠΏΡ€ΠΎΠ²ΠΎΠ΄Π½ΠΈΠΊΠ°. Π­Ρ‚Π° ΡΠΎΡΡ‚Π°Π²Π»ΡΡŽΡ‰Π°Ρ отвСтствСнна Π·Π° появлСниС силы Π°ΠΌΠΏΠ΅Ρ€Π° Β . Для случая, ΠΈΠ·ΠΎΠ±Ρ€Π°ΠΆΠ΅Π½Π½ΠΎΠ³ΠΎ Π½Π° рис.Β 1.20.3, ΠΌΠΎΠ΄ΡƒΠ»ΡŒ силы АмпСра Ρ€Π°Π²Π΅Π½ FA = IΒ BΒ l. Π‘ΠΈΠ»Π° АмпСра Π½Π°ΠΏΡ€Π°Π²Π»Π΅Π½Π° навстрСчу двиТСнию ΠΏΡ€ΠΎΠ²ΠΎΠ΄Π½ΠΈΠΊΠ°; поэтому ΠΎΠ½Π° ΡΠΎΠ²Π΅Ρ€ΡˆΠ°Π΅Ρ‚ ΠΎΡ‚Ρ€ΠΈΡ†Π°Ρ‚Π΅Π»ΡŒΠ½ΡƒΡŽ ΠΌΠ΅Ρ…Π°Π½ΠΈΡ‡Π΅ΡΠΊΡƒΡŽ Ρ€Π°Π±ΠΎΡ‚Ρƒ. Π—Π° врСмя Ξ”t эта Ρ€Π°Π±ΠΎΡ‚Π° AΠΌΠ΅Ρ… Ρ€Π°Π²Π½Π°

ДвиТущийся Π² ΠΌΠ°Π³Π½ΠΈΡ‚Π½ΠΎΠΌ ΠΏΠΎΠ»Π΅ ΠΏΡ€ΠΎΠ²ΠΎΠ΄Π½ΠΈΠΊ, ΠΏΠΎ ΠΊΠΎΡ‚ΠΎΡ€ΠΎΠΌΡƒ ΠΏΡ€ΠΎΡ‚Π΅ΠΊΠ°Π΅Ρ‚ ΠΈΠ½Π΄ΡƒΠΊΡ†ΠΈΠΎΠ½Π½Ρ‹ΠΉ Ρ‚ΠΎΠΊ, испытываСт ΠΌΠ°Π³Π½ΠΈΡ‚Π½ΠΎΠ΅ Ρ‚ΠΎΡ€ΠΌΠΎΠΆΠ΅Π½ΠΈΠ΅. Полная Ρ€Π°Π±ΠΎΡ‚Π° силы Π›ΠΎΡ€Π΅Π½Ρ†Π° Ρ€Π°Π²Π½Π° Π½ΡƒΠ»ΡŽ. Π”ΠΆΠΎΡƒΠ»Π΅Π²ΠΎ Ρ‚Π΅ΠΏΠ»ΠΎ Π² ΠΊΠΎΠ½Ρ‚ΡƒΡ€Π΅ выдСляСтся Π»ΠΈΠ±ΠΎ Π·Π° счСт Ρ€Π°Π±ΠΎΡ‚Ρ‹ внСшнСй силы, которая ΠΏΠΎΠ΄Π΄Π΅Ρ€ΠΆΠΈΠ²Π°Π΅Ρ‚ ΡΠΊΠΎΡ€ΠΎΡΡ‚ΡŒ ΠΏΡ€ΠΎΠ²ΠΎΠ΄Π½ΠΈΠΊΠ° Π½Π΅ΠΈΠ·ΠΌΠ΅Π½Π½ΠΎΠΉ, Π»ΠΈΠ±ΠΎ Π·Π° счСт ΡƒΠΌΠ΅Π½ΡŒΡˆΠ΅Π½ΠΈΡ кинСтичСской энСргии ΠΏΡ€ΠΎΠ²ΠΎΠ΄Π½ΠΈΠΊΠ°.

2. Вторая ΠΏΡ€ΠΈΡ‡ΠΈΠ½Π° измСнСния ΠΌΠ°Π³Π½ΠΈΡ‚Π½ΠΎΠ³ΠΎ ΠΏΠΎΡ‚ΠΎΠΊΠ°, ΠΏΡ€ΠΎΠ½ΠΈΠ·Ρ‹Π²Π°ΡŽΡ‰Π΅Π³ΠΎ ΠΊΠΎΠ½Ρ‚ΡƒΡ€, – ΠΈΠ·ΠΌΠ΅Π½Π΅Π½ΠΈΠ΅ Π²ΠΎ Π²Ρ€Π΅ΠΌΠ΅Π½ΠΈ ΠΌΠ°Π³Π½ΠΈΡ‚Π½ΠΎΠ³ΠΎ поля ΠΏΡ€ΠΈ Π½Π΅ΠΏΠΎΠ΄Π²ΠΈΠΆΠ½ΠΎΠΌ ΠΊΠΎΠ½Ρ‚ΡƒΡ€Π΅. Π’ этом случаС Π²ΠΎΠ·Π½ΠΈΠΊΠ½ΠΎΠ²Π΅Π½ΠΈΠ΅ Π­Π”Π‘ ΠΈΠ½Π΄ΡƒΠΊΡ†ΠΈΠΈ ΡƒΠΆΠ΅ нСльзя ΠΎΠ±ΡŠΡΡΠ½ΠΈΡ‚ΡŒ дСйствиСм силы Π›ΠΎΡ€Π΅Π½Ρ†Π°. Π­Π»Π΅ΠΊΡ‚Ρ€ΠΎΠ½Ρ‹ Π² Π½Π΅ΠΏΠΎΠ΄Π²ΠΈΠΆΠ½ΠΎΠΌ ΠΏΡ€ΠΎΠ²ΠΎΠ΄Π½ΠΈΠΊΠ΅ ΠΌΠΎΠ³ΡƒΡ‚ ΠΏΡ€ΠΈΠ²ΠΎΠ΄ΠΈΡ‚ΡŒΡΡ Π² Π΄Π²ΠΈΠΆΠ΅Π½ΠΈΠ΅ Ρ‚ΠΎΠ»ΡŒΠΊΠΎ элСктричСским ΠΏΠΎΠ»Π΅ΠΌ. Π­Ρ‚ΠΎ элСктричСскоС ΠΏΠΎΠ»Π΅ пороТдаСтся ΠΈΠ·ΠΌΠ΅Π½ΡΡŽΡ‰ΠΈΠΌΡΡ Π²ΠΎ Π²Ρ€Π΅ΠΌΠ΅Π½ΠΈ ΠΌΠ°Π³Π½ΠΈΡ‚Π½Ρ‹ΠΌ ΠΏΠΎΠ»Π΅ΠΌ. Π Π°Π±ΠΎΡ‚Π° этого поля ΠΏΡ€ΠΈ ΠΏΠ΅Ρ€Π΅ΠΌΠ΅Ρ‰Π΅Π½ΠΈΠΈ Π΅Π΄ΠΈΠ½ΠΈΡ‡Π½ΠΎΠ³ΠΎ ΠΏΠΎΠ»ΠΎΠΆΠΈΡ‚Π΅Π»ΡŒΠ½ΠΎΠ³ΠΎ заряда ΠΏΠΎ Π·Π°ΠΌΠΊΠ½ΡƒΡ‚ΠΎΠΌΡƒ ΠΊΠΎΠ½Ρ‚ΡƒΡ€Ρƒ Ρ€Π°Π²Π½Π° Π­Π”Π‘ ΠΈΠ½Π΄ΡƒΠΊΡ†ΠΈΠΈ Π² Π½Π΅ΠΏΠΎΠ΄Π²ΠΈΠΆΠ½ΠΎΠΌ ΠΏΡ€ΠΎΠ²ΠΎΠ΄Π½ΠΈΠΊΠ΅. Π‘Π»Π΅Π΄ΠΎΠ²Π°Ρ‚Π΅Π»ΡŒΠ½ΠΎ, элСктричСскоС ΠΏΠΎΠ»Π΅, ΠΏΠΎΡ€ΠΎΠΆΠ΄Π΅Π½Π½ΠΎΠ΅ ΠΈΠ·ΠΌΠ΅Π½ΡΡŽΡ‰ΠΈΠΌΡΡ ΠΌΠ°Π³Π½ΠΈΡ‚Π½Ρ‹ΠΌ ΠΏΠΎΠ»Π΅ΠΌ, Π½Π΅ являСтся ΠΏΠΎΡ‚Π΅Π½Ρ†ΠΈΠ°Π»ΡŒΠ½Ρ‹ΠΌ. Π•Π³ΠΎ Π½Π°Π·Ρ‹Π²Π°ΡŽΡ‚ Π²ΠΈΡ…Ρ€Π΅Π²Ρ‹ΠΌ элСктричСским ΠΏΠΎΠ»Π΅ΠΌ. ΠŸΡ€Π΅Π΄ΡΡ‚Π°Π²Π»Π΅Π½ΠΈΠ΅ ΠΎ Π²ΠΈΡ…Ρ€Π΅Π²ΠΎΠΌ элСктричСском ΠΏΠΎΠ»Π΅ Π±Ρ‹Π»ΠΎ Π²Π²Π΅Π΄Π΅Π½ΠΎ Π² Ρ„ΠΈΠ·ΠΈΠΊΡƒ Π²Π΅Π»ΠΈΠΊΠΈΠΌ английским Ρ„ΠΈΠ·ΠΈΠΊΠΎΠΌ ДТСймсом МаксвСллом Π²Β 1861Β Π³.

Π―Π²Π»Π΅Π½ΠΈΠ΅ элСктромагнитной ΠΈΠ½Π΄ΡƒΠΊΡ†ΠΈΠΈ Π² Π½Π΅ΠΏΠΎΠ΄Π²ΠΈΠΆΠ½Ρ‹Ρ… ΠΏΡ€ΠΎΠ²ΠΎΠ΄Π½ΠΈΠΊΠ°Ρ…, Π²ΠΎΠ·Π½ΠΈΠΊΠ°ΡŽΡ‰Π΅Π΅ ΠΏΡ€ΠΈ ΠΈΠ·ΠΌΠ΅Π½Π΅Π½ΠΈΠΈ ΠΎΠΊΡ€ΡƒΠΆΠ°ΡŽΡ‰Π΅Π³ΠΎ ΠΌΠ°Π³Π½ΠΈΡ‚Π½ΠΎΠ³ΠΎ поля, Ρ‚Π°ΠΊΠΆΠ΅ описываСтся Ρ„ΠΎΡ€ΠΌΡƒΠ»ΠΎΠΉ ЀарадСя. Π’Π°ΠΊΠΈΠΌ ΠΎΠ±Ρ€Π°Π·ΠΎΠΌ, явлСния ΠΈΠ½Π΄ΡƒΠΊΡ†ΠΈΠΈ Π² двиТущихся ΠΈ Π½Π΅ΠΏΠΎΠ΄Π²ΠΈΠΆΠ½Ρ‹Ρ… ΠΏΡ€ΠΎΠ²ΠΎΠ΄Π½ΠΈΠΊΠ°Ρ… ΠΏΡ€ΠΎΡ‚Π΅ΠΊΠ°ΡŽΡ‚ ΠΎΠ΄ΠΈΠ½Π°ΠΊΠΎΠ²ΠΎ, Π½ΠΎ физичСская ΠΏΡ€ΠΈΡ‡ΠΈΠ½Π° возникновСния ΠΈΠ½Π΄ΡƒΠΊΡ†ΠΈΠΎΠ½Π½ΠΎΠ³ΠΎ Ρ‚ΠΎΠΊΠ° оказываСтся Π² этих Π΄Π²ΡƒΡ… случаях Ρ€Π°Π·Π»ΠΈΡ‡Π½ΠΎΠΉ: Π² случаС двиТущихся ΠΏΡ€ΠΎΠ²ΠΎΠ΄Π½ΠΈΠΊΠΎΠ² Π­Π”Π‘ ΠΈΠ½Π΄ΡƒΠΊΡ†ΠΈΠΈ обусловлСна силой Π›ΠΎΡ€Π΅Π½Ρ†Π°; Π² случаС Π½Π΅ΠΏΠΎΠ΄Π²ΠΈΠΆΠ½Ρ‹Ρ… ΠΏΡ€ΠΎΠ²ΠΎΠ΄Π½ΠΈΠΊΠΎΠ² Π­Π”Π‘ ΠΈΠ½Π΄ΡƒΠΊΡ†ΠΈΠΈ являСтся слСдствиСм дСйствия Π½Π° свободныС заряды Π²ΠΈΡ…Ρ€Π΅Π²ΠΎΠ³ΠΎ элСктричСского поля, Π²ΠΎΠ·Π½ΠΈΠΊΠ°ΡŽΡ‰Π΅Π³ΠΎ ΠΏΡ€ΠΈ ΠΈΠ·ΠΌΠ΅Π½Π΅Π½ΠΈΠΈ ΠΌΠ°Π³Π½ΠΈΡ‚Π½ΠΎΠ³ΠΎ поля.

МодСль. ЭлСктромагнитная индукция

МодСль. ΠžΠΏΡ‹Ρ‚Ρ‹ ЀарадСя

МодСль. Π“Π΅Π½Π΅Ρ€Π°Ρ‚ΠΎΡ€ ΠΏΠ΅Ρ€Π΅ΠΌΠ΅Π½Π½ΠΎΠ³ΠΎ Ρ‚ΠΎΠΊΠ°

Π‘ΠΈΠ»Π° Π°ΠΌΠΏΠ΅Ρ€Π°, сила Π»ΠΎΡ€Π΅Π½Ρ†Π°.

Π—Π° это Π·Π°Π΄Π°Π½ΠΈΠ΅ Π²Ρ‹ ΠΌΠΎΠΆΠ΅Ρ‚Π΅ ΠΏΠΎΠ»ΡƒΡ‡ΠΈΡ‚ΡŒ 1 Π±Π°Π»Π» Π½Π° Π•Π“Π­ Π² 2020 Π³ΠΎΠ΄Ρƒ

Π—Π°Π΄Π°Π½ΠΈΠ΅ 13 Π•Π“Π­ ΠΏΠΎ Ρ„ΠΈΠ·ΠΈΠΊΠ΅ посвящСно всСм процСссам, Π² ΠΊΠΎΡ‚ΠΎΡ€Ρ‹Ρ… участвуСт элСктричСскоС ΠΈ ΠΌΠ°Π³Π½ΠΈΡ‚Π½ΠΎΠ΅ ΠΏΠΎΠ»Π΅. Π­Ρ‚ΠΎ ΠΎΠ΄ΠΈΠ½ ΠΈΠ· самых ΠΎΠ±ΡˆΠΈΡ€Π½Ρ‹Ρ… вопросов ΠΏΠΎ количСству ΠΎΡ…Π²Π°Ρ‚Ρ‹Π²Π°Π΅ΠΌΡ‹Ρ… ΡƒΡ‡Π΅Π±Π½Ρ‹Ρ… Ρ‚Π΅ΠΌ. Π’Π°ΠΊ, учащСмуся ΠΌΠΎΠΆΠ΅Ρ‚ ΠΏΠΎΠΏΠ°ΡΡ‚ΡŒΡΡ Ρ‚Π΅ΠΌΠ° Β«Π—Π°ΠΊΠΎΠ½ ΠšΡƒΠ»ΠΎΠ½Π°, Π½Π°ΠΏΡ€ΡΠΆΠ΅Π½Π½ΠΎΡΡ‚ΡŒ ΠΈ ΠΏΠΎΡ‚Π΅Π½Ρ†ΠΈΠ°Π» элСктричСского поля», ΠΈ ΠΎΠ½ Π±ΡƒΠ΄Π΅Ρ‚ Π½Π°Ρ…ΠΎΠ΄ΠΈΡ‚ΡŒ Ρ€Π°Π·Π½ΠΎΡΡ‚ΡŒ ΠΏΠΎΡ‚Π΅Π½Ρ†ΠΈΠ°Π»ΠΎΠ² ΠΌΠ΅ΠΆΠ΄Ρƒ Ρ‚ΠΎΡ‡ΠΊΠ°ΠΌΠΈ поля, силу взаимодСйствия ΠΌΠ΅ΠΆΠ΄Ρƒ Ρ‚Π΅Π»Π°ΠΌΠΈ ΠΈΠ»ΠΈ напряТСниС, ΠΏΡ€ΠΈΠ»ΠΎΠΆΠ΅Π½Π½ΠΎΠ΅ ΠΊ ΠΊΠΎΠ½Ρ†Π°ΠΌ ΠΏΡ€ΠΎΠ²ΠΎΠ΄Π½ΠΈΠΊΠ°.

Π’Π΅ΠΌΠ° задания 13 Π•Π“Π­ ΠΏΠΎ Ρ„ΠΈΠ·ΠΈΠΊΠ΅ ΠΌΠΎΠΆΠ΅Ρ‚ ΠΊΠ°ΡΠ°Ρ‚ΡŒΡΡ Ρ‚Π°ΠΊΠΆΠ΅ ΠΌΠ°Π³Π½ΠΈΡ‚Π½ΠΎΠ³ΠΎ ΠΏΠΎΡ‚ΠΎΠΊΠ° ΠΈ ΠΏΠΎΠ΄Ρ€Π°Π·ΡƒΠΌΠ΅Π²Π°Ρ‚ΡŒ вычислСниС модуля Π²Π΅ΠΊΡ‚ΠΎΡ€Π° ΠΈΠ½Π΄ΡƒΠΊΡ†ΠΈΠΈ ΠΌΠ°Π³Π½ΠΈΡ‚Π½ΠΎΠ³ΠΎ поля ΠΈΠ»ΠΈ Π΅Π³ΠΎ Π½Π°ΠΏΡ€Π°Π²Π»Π΅Π½ΠΈΠ΅. Π§Π°ΡΡ‚ΡŒ вопросов посвящСна Π²Ρ‹Ρ‡ΠΈΡΠ»Π΅Π½ΠΈΡŽ силы АмпСра ΠΈ силы Π›ΠΎΡ€Π΅Π½Ρ†Π°.

Π—Π°Π΄Π°Π½ΠΈΠ΅ β„– 13 Π•Π“Π­ ΠΏΠΎ Ρ„ΠΈΠ·ΠΈΠΊΠ΅ ΠΏΠΎΠ΄Ρ€Π°Π·ΡƒΠΌΠ΅Π²Π°Π΅Ρ‚ ΠΊΡ€Π°Ρ‚ΠΊΠΈΠΉ ΠΎΡ‚Π²Π΅Ρ‚ Π½Π° свой вопрос. ΠŸΡ€ΠΈ этом Ρ‡Π°ΡΡ‚ΡŒ Π²Π°Ρ€ΠΈΠ°Π½Ρ‚ΠΎΠ² Ρ‚Ρ€Π΅Π±ΡƒΠ΅Ρ‚ записи числового значСния Π²Π΅Π»ΠΈΡ‡ΠΈΠ½Ρ‹ (с ΠΎΠΊΡ€ΡƒΠ³Π»Π΅Π½ΠΈΠ΅ΠΌ Π΄ΠΎ Π½ΡƒΠΆΠ½Ρ‹Ρ… Π΄ΠΎΠ»Π΅ΠΉ, Ссли ΠΎΡ‚Π²Π΅Ρ‚ΠΎΠΌ являСтся дСсятичная Π΄Ρ€ΠΎΠ±ΡŒ), Π° Π² части ΡƒΡ‡Π΅Π½ΠΈΠΊΡƒ придСтся Π²Ρ‹Π±ΠΈΡ€Π°Ρ‚ΡŒ ΠΈΠ· Ρ‡Π΅Ρ‚Ρ‹Ρ€Π΅Ρ… ΠΏΡ€Π΅Π΄Π»ΠΎΠΆΠ΅Π½Π½Ρ‹Ρ… ΠΎΡ‚Π²Π΅Ρ‚ΠΎΠ² ΠΎΠ΄ΠΈΠ½, ΠΊΠΎΡ‚ΠΎΡ€Ρ‹ΠΉ ΠΎΠ½ считаСт ΠΏΡ€Π°Π²ΠΈΠ»ΡŒΠ½Ρ‹ΠΌ. Π’Π°ΠΊ ΠΊΠ°ΠΊ врСмя прохоТдСния всСго испытания ΠΎΠ³Ρ€Π°Π½ΠΈΡ‡Π΅Π½ΠΎ ΠΎΠΏΡ€Π΅Π΄Π΅Π»Π΅Π½Π½Ρ‹ΠΌ числом ΠΌΠΈΠ½ΡƒΡ‚, Ρ‚ΠΎ ΠΈ Π½Π° Ρ‚Ρ€ΠΈΠ½Π°Π΄Ρ†Π°Ρ‚ΠΎΠΌ вопросС Π½Π°Π΄ΠΎΠ»Π³ΠΎ ΠΎΡΡ‚Π°Π½Π°Π²Π»ΠΈΠ²Π°Ρ‚ΡŒΡΡ Π½Π΅ стоит. Если ΠΎΠ½ Π²Ρ‹Π·Ρ‹Π²Π°Π΅Ρ‚ слоТности, Π»ΡƒΡ‡ΡˆΠ΅ ΠΎΡΡ‚Π°Π²ΠΈΡ‚ΡŒ Π΅Π³ΠΎ Π½Π° самый ΠΊΠΎΠ½Π΅Ρ† экзамСнационного Π²Ρ€Π΅ΠΌΠ΅Π½ΠΈ.

Π—Π°Π΄Π°Π½ΠΈΠ΅ β„–13 Π•Π“Π­ ΠΏΠΎ Ρ„ΠΈΠ·ΠΈΠΊΠ΅ провСряСт Π·Π½Π°Π½ΠΈΠ΅ ΠΏΠΎ Ρ‚Π΅ΠΌΠ΅ Β«Π­Π»Π΅ΠΊΡ‚Ρ€ΠΎΠΌΠ°Π³Π½Π΅Ρ‚ΠΈΠ·ΠΌΒ». Π’ Π·Π°Π΄Π°Ρ‡Π°Ρ… Π΄Π°Π½Π½ΠΎΠ³ΠΎ Ρ‚ΠΈΠΏΠ° Π½Π΅ΠΎΠ±Ρ…ΠΎΠ΄ΠΈΠΌΠΎ Ρ€Π΅ΡˆΠΈΡ‚ΡŒ Π·Π°Π΄Π°Ρ‡ΠΈ, связанныС с элСктричСским ΠΈΠ»ΠΈ ΠΌΠ°Π³Π½ΠΈΡ‚Π½Ρ‹ΠΌ ΠΏΠΎΠ»Π΅ΠΌ.

ВСория ΠΊ заданию β„–13 Π•Π“Π­ ΠΏΠΎ Ρ„ΠΈΠ·ΠΈΠΊΠ΅

ЭлСктричСский заряд

Π’Π΅Π»ΠΈΡ‡ΠΈΠ½Π°, которая опрСдСляСт силу элСктромагнитного дСйствия ΠΈ связываСт силу Π΅Π³ΠΎ с расстояниСм ΠΌΠ΅ΠΆΠ΄Ρƒ Π΄Π΅ΠΉΡΡ‚Π²ΡƒΡŽΡ‰ΠΈΠΌΠΈ Π΄Ρ€ΡƒΠ³ Π½Π° Π΄Ρ€ΡƒΠ³Π° Ρ‚Π΅Π»Π°ΠΌΠΈ, называСтся элСктростатичСским зарядом, ΠΊΠΎΡ‚ΠΎΡ€Ρ‹ΠΉ Ρ…Π°Ρ€Π°ΠΊΡ‚Π΅Ρ€ΠΈΠ·ΡƒΠ΅Ρ‚ ΡΠΏΠΎΡΠΎΠ±Π½ΠΎΡΡ‚ΡŒ Ρ‚Π΅Π»Π° – носитСля заряда β€” ΡΠΎΠ·Π΄Π°Π²Π°Ρ‚ΡŒ элСктромагнитноС ΠΏΠΎΠ»Π΅ ΠΎΠΊΠΎΠ»ΠΎ сСбя, Π° Ρ‚Π°ΠΊΠΆΠ΅ ΠΈΡΠΏΡ‹Ρ‚Ρ‹Π²Π°Ρ‚ΡŒ Π½Π° сСбС воздСйствиС Π²Π½Π΅ΡˆΠ½ΠΈΡ… ΠΏΠΎΠ»Π΅ΠΉ.

Заряды Π±Ρ‹Π²Π°ΡŽΡ‚ Ρ€Π°Π·Π½Ρ‹Ρ… Π·Π½Π°ΠΊΠΎΠ². ΠœΠ΅ΠΆΠ΄ΡƒΠ½Π°Ρ€ΠΎΠ΄Π½ΠΎΠΉ систСмой принято ΡΡ‡ΠΈΡ‚Π°Ρ‚ΡŒ заряд элСктрона ΠΎΡ‚Ρ€ΠΈΡ†Π°Ρ‚Π΅Π»ΡŒΠ½Ρ‹ΠΌ, Π° ΠΏΡ€ΠΈΡ‚ΡΠ³ΠΈΠ²Π°ΡŽΡ‰ΠΈΠΉ этот заряд – ΠΏΠΎΠ»ΠΎΠΆΠΈΡ‚Π΅Π»ΡŒΠ½Ρ‹ΠΌ.

ΠΠ°ΠΏΡ€ΡΠΆΠ΅Π½Π½ΠΎΡΡ‚ΡŒ элСктростатичСского поля являСтся Π²Π΅ΠΊΡ‚ΠΎΡ€Π½ΠΎΠΉ Π²Π΅Π»ΠΈΡ‡ΠΈΠ½ΠΎΠΉ, Π½Π°ΠΏΡ€Π°Π²Π»Π΅Π½Π½ΠΎΠΉ ΠΎΡ‚ ΠΏΠΎΠ»ΠΎΠΆΠΈΡ‚Π΅Π»ΡŒΠ½ΠΎΠ³ΠΎ заряда ΠΊ ΠΎΡ‚Ρ€ΠΈΡ†Π°Ρ‚Π΅Π»ΡŒΠ½ΠΎΠΌΡƒ. Π­Ρ‚ΠΎ силовая характСристика элСктричСского поля.

Π—Π°ΠΊΠΎΠ½ АмпСра

Π—Π°ΠΊΠΎΠ½ АмпСра гласит ΠΎ взаимодСйствии Ρ‚ΠΎΠΊΠΎΠ²: Π² ΠΏΠ°Ρ€Π°Π»Π»Π΅Π»ΡŒΠ½ΠΎ располоТСнных ΠΏΡ€ΠΎΠ²ΠΎΠ΄Π½ΠΈΠΊΠ°Ρ… Ρ‚ΠΎΠΊΠΈ, Ρ‚Π΅ΠΊΡƒΡ‰ΠΈΠ΅ Π² Ρ€Π°Π·Π½Ρ‹Ρ… направлСниях, ΠΎΡ‚Ρ‚Π°Π»ΠΊΠΈΠ²Π°ΡŽΡ‚ΡΡ ΠΎΠ΄ΠΈΠ½ Π΄Ρ€ΡƒΠ³ΠΎΠ³ΠΎ. Если ΠΆΠ΅ Ρ‚ΠΎΠΊΠΈ Π½Π°ΠΏΡ€Π°Π²Π»Π΅Π½Ρ‹ Π² ΠΎΠ΄Π½Ρƒ сторону, ΠΏΡ€ΠΎΠ²ΠΎΠ΄Π½ΠΈΠΊΠΈ ΠΏΡ€ΠΈΡ‚ΡΠ³ΠΈΠ²Π°ΡŽΡ‚ΡΡ.

Π Π°Π·Π±ΠΎΡ€ Ρ‚ΠΈΠΏΠΎΠ²Ρ‹Ρ… Π²Π°Ρ€ΠΈΠ°Π½Ρ‚ΠΎΠ² Π·Π°Π΄Π°Π½ΠΈΠΉ β„–13 Π•Π“Π­ ΠΏΠΎ Ρ„ΠΈΠ·ΠΈΠΊΠ΅

ДСмонстрационный Π²Π°Ρ€ΠΈΠ°Π½Ρ‚ 2018

ΠžΡ‚Ρ€ΠΈΡ†Π°Ρ‚Π΅Π»ΡŒΠ½Ρ‹ΠΉ заряд -q находится Π² ΠΏΠΎΠ»Π΅ Π΄Π²ΡƒΡ… Π½Π΅ΠΏΠΎΠ΄Π²ΠΈΠΆΠ½Ρ‹Ρ… зарядов: ΠΏΠΎΠ»ΠΎΠΆΠΈΡ‚Π΅Π»ΡŒΠ½ΠΎΠ³ΠΎ +Q ΠΈ ΠΎΡ‚Ρ€ΠΈΡ†Π°Ρ‚Π΅Π»ΡŒΠ½ΠΎΠ³ΠΎ -Q (смотри рисунок). ΠšΡƒΠ΄Π° Π½Π°ΠΏΡ€Π°Π²Π»Π΅Π½ΠΎ ΠΎΡ‚Π½ΠΎΡΠΈΡ‚Π΅Π»ΡŒΠ½ΠΎ рисунка (Π²ΠΏΡ€Π°Π²ΠΎ, Π²Π»Π΅Π²ΠΎ, Π²Π²Π΅Ρ€Ρ…, Π²Π½ΠΈΠ·, ΠΊ Π½Π°Π±Π»ΡŽΠ΄Π°Ρ‚Π΅Π»ΡŽ, ΠΎΡ‚ Π½Π°Π±Π»ΡŽΠ΄Π°Ρ‚Π΅Π»Ρ) ускорСниС заряда -q Π² этот ΠΌΠΎΠΌΠ΅Π½Ρ‚ Π²Ρ€Π΅ΠΌΠ΅Π½ΠΈ, Ссли Π½Π° Π½Π΅Π³ΠΎ Π΄Π΅ΠΉΡΡ‚Π²ΡƒΡŽΡ‚ Ρ‚ΠΎΠ»ΡŒΠΊΠΎ заряды +Q ΠΈ -Q ? ΠžΡ‚Π²Π΅Ρ‚ Π·Π°ΠΏΠΈΡˆΠΈΡ‚Π΅ словом (словами).

Алгоритм Ρ€Π΅ΡˆΠ΅Π½ΠΈΡ:
  1. ΠŸΡ€ΠΎΠ²ΠΎΠ΄ΠΈΠΌ Π°Π½Π°Π»ΠΈΠ· рисунка, ΠΏΡ€ΠΈΠ»ΠΎΠΆΠ΅Π½Π½ΠΎΠ³ΠΎ ΠΊ Π·Π°Π΄Π°Ρ‡Π΅.
  2. Π”Π΅Π»Π°Π΅ΠΌ Π²Ρ‹Π²ΠΎΠ΄ ΠΎ Π½Π°ΠΏΡ€Π°Π²Π»Π΅Π½ΠΈΠΈ взаимодСйствия зарядов.
  3. ΠžΠΏΡ€Π΅Π΄Π΅Π»ΡΠ΅ΠΌ Π½Π°ΠΏΡ€Π°Π²Π»Π΅Π½ΠΈΠ΅ ускорСния.
  4. ЗаписываСм ΠΎΡ‚Π²Π΅Ρ‚.
РСшСниС:

1. Из 2-Π³ΠΎ Π·-Π½Π° ΠΡŒΡŽΡ‚ΠΎΠ½Π° слСдуСт, Ρ‡Ρ‚ΠΎ Π½Π°ΠΏΡ€Π°Π²Π»Π΅Π½ΠΈΠ΅ ускорСния Ρ„ΠΈΠ·.Ρ‚Π΅Π»Π° Π² любом случаС совпадаСт с Π½Π°ΠΏΡ€Π°Π²Π»Π΅Π½ΠΈΠ΅ΠΌ Π²Π΅ΠΊΡ‚ΠΎΡ€Π° Ρ€Π°Π²Π½ΠΎΠ΄Π΅ΠΉΡΡ‚Π²ΡƒΡŽΡ‰Π΅ΠΉ силы. ΠŸΠΎΡΡ‚ΠΎΠΌΡƒ, ΡƒΠ·Π½Π°Π² Π½Π°ΠΏΡ€Π°Π²Π»Π΅Π½ΠΈΠ΅ Ρ€Π°Π²Π½ΠΎΠ΄Π΅ΠΉΡΡ‚Π²ΡƒΡŽΡ‰Π΅ΠΉ силы, ΠΏΠΎΠ»ΡƒΡ‡ΠΈΠΌ ΠΎΡ‚Π²Π΅Ρ‚ Π½Π° вопрос Π·Π°Π΄Π°Ρ‡ΠΈ.

На рисункС ΠΈΠ·ΠΎΠ±Ρ€Π°ΠΆΠ΅Π½Ρ‹ Ρ‚Ρ€ΠΈ заряда, ΠΏΡ€ΠΈΡ‡Π΅ΠΌ Π²Π²Π΅Ρ€Ρ…Ρƒ (1) ΠΈ Π²Π½ΠΈΠ·Ρƒ (2) располоТСны заряды Ρ€Π°Π·Π½Ρ‹Ρ… Π·Π½Π°ΠΊΠΎΠ², Π° слСва – заряд, ΠΎΠ΄Π½ΠΎΠΈΠΌΠ΅Π½Π½Ρ‹ΠΉ с Π²Π΅Ρ€Ρ…Π½ΠΈΠΌ:

2. Π Π°Π²Π½ΠΎΠ΄Π΅ΠΉΡΡ‚Π²ΡƒΡŽΡ‰Π°Ρ сил Π±ΡƒΠ΄Π΅Ρ‚ Ρ€Π°Π²Π½ΠΎΠΉ: 𝐹⃗=𝐹⃗ 1 +𝐹⃗ 2 , Π³Π΄Π΅ Π²Π΅ΠΊΡ‚ΠΎΡ€Ρ‹ F 1 ΠΈ F 2 – силы, Π΄Π΅ΠΉΡΡ‚Π²ΡƒΡŽΡ‰ΠΈΠ΅ Π½Π° заряд q со стороны зарядов 1 ΠΈ 2 соотвСтствСнно.

Π˜Π·Π²Π΅ΡΡ‚Π½ΠΎ, Ρ‡Ρ‚ΠΎ заряды, ΠΈΠΌΠ΅ΡŽΡ‰ΠΈΠ΅ ΠΎΠ΄ΠΈΠ½Π°ΠΊΠΎΠ²Ρ‹Π΅ Π·Π½Π°ΠΊΠΈ, ΠΎΡ‚Ρ‚Π°Π»ΠΊΠΈΠ²Π°ΡŽΡ‚ΡΡ, Π° заряды ΠΏΡ€ΠΎΡ‚ΠΈΠ²ΠΎΠΏΠΎΠ»ΠΎΠΆΠ½Ρ‹Ρ… Π·Π½Π°ΠΊΠΎΠ² ΠΏΡ€ΠΈΡ‚ΡΠ³ΠΈΠ²Π°ΡŽΡ‚ΡΡ. Π˜Π·ΠΎΠ±Ρ€Π°ΠΆΠ°Π΅ΠΌ силы взаимодСйствия ΠΌΠ΅ΠΆΠ΄Ρƒ зарядами:

3. ВСкторная сумма сил F1 F2 находится ΠΏΠΎ ΠΏΡ€Π°Π²ΠΈΠ»Ρƒ ΠΏΠ°Ρ€Π°Π»Π»Π΅Π»ΠΎΠ³Ρ€Π°ΠΌΠΌΠ°. ΠŸΡ€ΠΈ этом слСдуСт ΠΈΠΌΠ΅Ρ‚ΡŒ Π² Π²ΠΈΠ΄Ρƒ, Ρ‡Ρ‚ΠΎ Π²Π΅Π»ΠΈΡ‡ΠΈΠ½Ρ‹ сил (Π΄Π»ΠΈΠ½Ρ‹ Π²Π΅ΠΊΡ‚ΠΎΡ€ΠΎΠ²) Π±ΡƒΠ΄ΡƒΡ‚ ΠΎΠ΄ΠΈΠ½Π°ΠΊΠΎΠ²Ρ‹ΠΌΠΈ, ΠΏΠΎΡΠΊΠΎΠ»ΡŒΠΊΡƒ заряды –Q ΠΈ +Q Ρ€Π°Π²Π½Ρ‹ ΠΏΠΎ ΠΌΠΎΠ΄ΡƒΠ»ΡŽ. Π­Ρ‚ΠΎ ΠΎΠ·Π½Π°Ρ‡Π°Π΅Ρ‚, Ρ‡Ρ‚ΠΎ Π²Π΅ΠΊΡ‚ΠΎΡ€Ρ‹ Π½Π°ΠΏΡ€Π°Π²Π»Π΅Π½Ρ‹ симмСтрично ΠΎΡ‚Π½ΠΎΡΠΈΡ‚Π΅Π»ΡŒΠ½ΠΎ Π²Π΅Ρ€Ρ‚ΠΈΠΊΠ°Π»ΡŒΠ½ΠΎΠΉ оси, ΠΊΠ°ΠΊ Π±Ρ‹ Π·Π΅Ρ€ΠΊΠ°Π»ΡŒΠ½ΠΎ ΠΎΡ‚Ρ€Π°ΠΆΠ°ΡΡΡŒ. А ΠΈΡ… Ρ€Π΅Π·ΡƒΠ»ΡŒΡ‚ΠΈΡ€ΡƒΡŽΡ‰Π°Ρ, ΡΠ»Π΅Π΄ΠΎΠ²Π°Ρ‚Π΅Π»ΡŒΠ½ΠΎ, Π½Π°ΠΏΡ€Π°Π²Π»Π΅Π½Π° Π²Π΅Ρ€Ρ‚ΠΈΠΊΠ°Π»ΡŒΠ½ΠΎ Π²Π½ΠΈΠ·, Ρ‚.Π΅. вдоль оси симмСтрии.

ΠžΡ‚Π²Π΅Ρ‚: Π²Π½ΠΈΠ·

ΠŸΠ΅Ρ€Π²Ρ‹ΠΉ Π²Π°Ρ€ΠΈΠ°Π½Ρ‚ задания (Π”Π΅ΠΌΠΈΠ΄ΠΎΠ²Π°, β„–1)

Π’ Ρ‚Ρ€Ρ‘Ρ… Π²Π΅Ρ€ΡˆΠΈΠ½Π°Ρ… Ρ€ΠΎΠΌΠ±Π° располоТСны Ρ‚ΠΎΡ‡Π΅Ρ‡Π½Ρ‹Π΅ заряды +q, -2q ΠΈ +q (q > 0). ΠšΡƒΠ΄Π° Π½Π°ΠΏΡ€Π°Π²Π»Π΅Π½Π° ΠΎΡ‚Π½ΠΎΡΠΈΡ‚Π΅Π»ΡŒΠ½ΠΎ рисунка (Π²Π²Π΅Ρ€Ρ…, Π²Π½ΠΈΠ·, Π²Π»Π΅Π²ΠΎ, Π²ΠΏΡ€Π°Π²ΠΎ, ΠΎΡ‚ Π½Π°Π±Π»ΡŽΠ΄Π°Ρ‚Π΅Π»Ρ, ΠΊ Π½Π°Π±Π»ΡŽΠ΄Π°Ρ‚Π΅Π»ΡŽ) кулоновская сила F, Π΄Π΅ΠΉΡΡ‚Π²ΡƒΡŽΡ‰Π°Ρ Π½Π° ΠΎΡ‚Ρ€ΠΈΡ†Π°Ρ‚Π΅Π»ΡŒΠ½Ρ‹ΠΉ Ρ‚ΠΎΡ‡Π΅Ρ‡Π½Ρ‹ΠΉ заряд -Q, ΠΏΠΎΠΌΠ΅Ρ‰Ρ‘Π½Π½Ρ‹ΠΉ Π² Ρ†Π΅Π½Ρ‚Ρ€ этого Ρ€ΠΎΠΌΠ±Π° (см. рисунок)? ΠžΡ‚Π²Π΅Ρ‚ Π·Π°ΠΏΠΈΡˆΠΈΡ‚Π΅ словом (словами).

Алгоритм Ρ€Π΅ΡˆΠ΅Π½ΠΈΡ:
  1. АнализируСм ΠΏΡ€ΠΈΠ»ΠΎΠΆΠ΅Π½Π½Ρ‹ΠΉ ΠΊ Π·Π°Π΄Π°Ρ‡Π΅ рисунок. ΠžΠΏΡ€Π΅Π΄Π΅Π»ΡΠ΅ΠΌ силы, ΠΊΠΎΡ‚ΠΎΡ€Ρ‹Π΅ Π΄Π΅ΠΉΡΡ‚Π²ΡƒΡŽΡ‚ Π½Π° заряд –Q .
  2. Π˜Π·ΠΎΠ±Ρ€Π°ΠΆΠ°Π΅ΠΌ силы ΠΈ Π½Π°Ρ…ΠΎΠ΄ΠΈΠΌ Ρ€Π°Π²Π½ΠΎΠ΄Π΅ΠΉΡΡ‚Π²ΡƒΡŽΡ‰ΡƒΡŽ.
  3. ЗаписываСм ΠΎΡ‚Π²Π΅Ρ‚.
РСшСниС:

1. На рисункС ΠΏΠΎΠΊΠ°Π·Π°Π½ΠΎ, ΠΊΠ°ΠΊΠΈΠ΅ заряды ΠΈΠΌΠ΅ΡŽΡ‚ ΠΎΠ΄ΠΈΠ½Π°ΠΊΠΎΠ²Ρ‹Π΅ Π·Π½Π°ΠΊΠΈ, Π° ΠΊΠ°ΠΊΠΈΠ΅ Ρ€Π°Π·Π½Ρ‹Π΅. Π‘Π»Π΅Π²Π° ΠΈ справа ΠΎΡ‚ заряда –Q располоТСны заряды ΠΏΠΎΠ»ΠΎΠΆΠΈΡ‚Π΅Π»ΡŒΠ½ΠΎΠ³ΠΎ Π·Π½Π°ΠΊΠ° (+q) , ΠΊΠΎΡ‚ΠΎΡ€Ρ‹Π΅ ΠΏΡ€ΠΈΡ‚ΡΠ³ΠΈΠ²Π°ΡŽΡ‚ заряд –Q, ΠΏΡ€ΠΈΡ‡Π΅ΠΌ с ΠΎΠ΄ΠΈΠ½Π°ΠΊΠΎΠ²ΠΎΠΉ силой. А Π²Π²Π΅Ρ€Ρ…Ρƒ располагаСтся заряд, ΠΎΠ΄Π½ΠΎΠΈΠΌΠ΅Π½Π½Ρ‹ΠΉ с ΠΏΠΎΠΌΠ΅Ρ‰Π΅Π½Π½Ρ‹ΠΌ Π² Ρ†Π΅Π½Ρ‚Ρ€ Ρ€ΠΎΠΌΠ±Π°. Π­Ρ‚ΠΎΡ‚ заряд ΠΎΡ‚Ρ‚Π°Π»ΠΊΠΈΠ²Π°Π΅Ρ‚ –Q .

2. Π˜Π·ΠΎΠ±Ρ€Π°Π·ΠΈΠΌ всС силы, ΠΊΠΎΡ‚ΠΎΡ€Ρ‹Π΅ Π΄Π΅ΠΉΡΡ‚Π²ΡƒΡŽΡ‚ Π½Π° заряд:

ΠŸΠΎΡΠΊΠΎΠ»ΡŒΠΊΡƒ ΠΌΠΎΠ΄ΡƒΠ»ΠΈ зарядов +q ΠΎΠ΄ΠΈΠ½Π°ΠΊΠΎΠ²Ρ‹, силы Π΄Π²ΡƒΡ… зарядов, располоТСнных Π½Π° Π³ΠΎΡ€ΠΈΠ·ΠΎΠ½Ρ‚Π°Π»ΡŒΠ½ΠΎΠΉ прямой (взаимодСйствиС –Q с +q), Ρ€Π°Π²Π½Ρ‹ ΠΌΠ΅ΠΆΠ΄Ρƒ собой, Π½ΠΎ ΠΏΡ€ΠΎΡ‚ΠΈΠ²ΠΎΠΏΠΎΠ»ΠΎΠΆΠ½Ρ‹ ΠΏΠΎ Π½Π°ΠΏΡ€Π°Π²Π»Π΅Π½ΠΈΡŽ. Π­Ρ‚ΠΎ ΠΎΠ·Π½Π°Ρ‡Π°Π΅Ρ‚, Ρ‡Ρ‚ΠΎ Ρ€Π΅Π·ΡƒΠ»ΡŒΡ‚ΠΈΡ€ΡƒΡŽΡ‰Π°Ρ этих Π΄Π²ΡƒΡ… сил Ρ€Π°Π²Π½Π° 0. ΠžΡ‚ΡΡŽΠ΄Π° слСдуСт, Ρ‡Ρ‚ΠΎ Ρ€Π°Π²Π½ΠΎΠ΄Π΅ΠΉΡΡ‚Π²ΡƒΡŽΡ‰Π°Ρ всСх сил совпадаСт с Π½Π°ΠΏΡ€Π°Π²Π»Π΅Π½ΠΈΠ΅ΠΌ Ρ‚Ρ€Π΅Ρ‚ΡŒΠ΅ΠΉ силы – силы взаимодСйствия –Q ΠΈ –2q. Π­Ρ‚ΠΎ Π½Π°ΠΏΡ€Π°Π²Π»Π΅Π½ΠΈΠ΅ – Π²Π΅Ρ€Ρ‚ΠΈΠΊΠ°Π»ΡŒΠ½ΠΎ Π²Π½ΠΈΠ·, Ρ‚.Π΅. вдоль Π²Π΅Ρ€Ρ‚ΠΈΠΊΠ°Π»ΠΈ мСньшСй Π΄ΠΈΠ°Π³ΠΎΠ½Π°Π»ΠΈ Ρ€ΠΎΠΌΠ±Π°.

ΠžΡ‚Π²Π΅Ρ‚: Π²Π½ΠΈΠ·

Π’Ρ‚ΠΎΡ€ΠΎΠΉ Π²Π°Ρ€ΠΈΠ°Π½Ρ‚ задания (Π”Π΅ΠΌΠΈΠ΄ΠΎΠ²Π°, β„–7)

Π’ Π²Π΅Ρ€ΡˆΠΈΠ½Π°Ρ… Ρ€Π°Π²Π½ΠΎΠ±Π΅Π΄Ρ€Π΅Π½Π½ΠΎΠ³ΠΎ Ρ‚Ρ€Π΅ΡƒΠ³ΠΎΠ»ΡŒΠ½ΠΈΠΊΠ° располоТСны Ρ‚ΠΎΡ‡Π΅Ρ‡Π½Ρ‹Π΅ заряды -2q, +q > 0 ΠΈ -2q (см. рисунок). ΠšΡƒΠ΄Π° Π½Π°ΠΏΡ€Π°Π²Π»Π΅Π½ ΠΎΡ‚Π½ΠΎΡΠΈΡ‚Π΅Π»ΡŒΠ½ΠΎ рисунка (Π²Π²Π΅Ρ€Ρ…, Π²Π½ΠΈΠ·, Π²Π»Π΅Π²ΠΎ, Π²ΠΏΡ€Π°Π²ΠΎ, ΠΎΡ‚ Π½Π°Π±Π»ΡŽΠ΄Π°Ρ‚Π΅Π»Ρ, ΠΊ Π½Π°Π±Π»ΡŽΠ΄Π°Ρ‚Π΅Π»ΡŽ) Π²Π΅ΠΊΡ‚ΠΎΡ€ напряТСнности Ρ€Π΅Π·ΡƒΠ»ΡŒΡ‚ΠΈΡ€ΡƒΡŽΡ‰Π΅Π³ΠΎ элСктростатичСского поля Π² Ρ‚ΠΎΡ‡ΠΊΠ΅ О пСрСсСчСния ΠΌΠ΅Π΄ΠΈΠ°Π½ Ρ‚Ρ€Π΅ΡƒΠ³ΠΎΠ»ΡŒΠ½ΠΈΠΊΠ°?

Алгоритм Ρ€Π΅ΡˆΠ΅Π½ΠΈΡ:
  1. РассматриваСм ΠΏΡ€ΠΈΠ»ΠΎΠΆΠ΅Π½Π½Ρ‹ΠΉ ΠΊ Π·Π°Π΄Π°Ρ‡Π΅ рисунок,
  2. Π”Π΅Π»Π°Π΅ΠΌ Π²Ρ‹Π²ΠΎΠ΄ ΠΎΡ‚Π½ΠΎΡΠΈΡ‚Π΅Π»ΡŒΠ½ΠΎ направлСния Π²Π΅ΠΊΡ‚ΠΎΡ€ΠΎΠ² напряТСнности, создаваСмой ΠΊΠ°ΠΆΠ΄Ρ‹ΠΌ зарядом Π² Ρ‚ΠΎΡ‡ΠΊΠ΅ О.
  3. ΠžΠΏΡ€Π΅Π΄Π΅Π»ΡΠ΅ΠΌ, ΠΊΡƒΠ΄Π° Π½Π°ΠΏΡ€Π°Π²Π»Π΅Π½Π° супСрпозиция напряТСнностСй.
  4. ЗаписываСм ΠΎΡ‚Π²Π΅Ρ‚.
РСшСниС:

1. Π˜Π·ΠΎΠ±Ρ€Π°ΠΆΠ΅Π½Π½Ρ‹ΠΉ Π½Π° рисункС Ρ‚Ρ€Π΅ΡƒΠ³ΠΎΠ»ΡŒΠ½ΠΈΠΊ Ρ€Π°Π²Π½ΠΎΠ±Π΅Π΄Ρ€Π΅Π½Π½Ρ‹ΠΉ. O – Ρ‚ΠΎΡ‡ΠΊΠ°, ΠΎΠ΄ΠΈΠ½Π°ΠΊΠΎΠ²ΠΎ удалСнная ΠΎΡ‚ Π²Π΅Ρ€ΡˆΠΈΠ½ основания, ΠΏΠΎΡΠΊΠΎΠ»ΡŒΠΊΡƒ это Ρ‚ΠΎΡ‡ΠΊΠ° пСрСсСчСния ΠΌΠ΅Π΄ΠΈΠ°Π½. Π’ Π²Π΅Ρ€ΡˆΠΈΠ½Π°Ρ… основания ΠΏΠΎΠΌΠ΅Ρ‰Π΅Π½Ρ‹ ΠΎΠ΄ΠΈΠ½Π°ΠΊΠΎΠ²Ρ‹Π΅ заряды -2q.

2. Π’Π΅ΠΊΡ‚ΠΎΡ€ напряТСнности ΠΈΠΌΠ΅Π΅Ρ‚ Π½Π°Ρ‡Π°Π»ΠΎ Ρƒ ΠΏΠΎΠ»ΠΎΠΆΠΈΡ‚Π΅Π»ΡŒΠ½ΠΎΠ³ΠΎ заряда ΠΈ Π½Π°ΠΏΡ€Π°Π²Π»Π΅Π½ ΠΊ ΠΎΡ‚Ρ€ΠΈΡ†Π°Ρ‚Π΅Π»ΡŒΠ½Ρ‹ΠΌ (красныС стрСлки):

ΠŸΠΎΡΠΊΠΎΠ»ΡŒΠΊΡƒ заряды –2q ΠΎΠ΄ΠΈΠ½Π°ΠΊΠΎΠ²Ρ‹ ΠΏΠΎ ΠΌΠΎΠ΄ΡƒΠ»ΡŽ, Ρ‚ΠΎ Π²Π΅Π»ΠΈΡ‡ΠΈΠ½Π° Π²Π΅ΠΊΡ‚ΠΎΡ€ΠΎΠ² Π• ΠΎΠ΄ΠΈΠ½Π°ΠΊΠΎΠ²Π°. Π­Ρ‚ΠΎ ΠΎΠ·Π½Π°Ρ‡Π°Π΅Ρ‚, Ρ‡Ρ‚ΠΎ ΠΈΡ… Ρ€Π΅Π·ΡƒΠ»ΡŒΡ‚ΠΈΡ€ΡƒΡŽΡ‰Π°Ρ (синяя стрСлка) Ρ€Π°Π²Π½ΠΎΡƒΠ΄Π°Π»Π΅Π½Π° ΠΎΡ‚ ΠΊΠ°ΠΆΠ΄ΠΎΠ³ΠΎ ΠΈΠ· Π½ΠΈΡ…, Ρ‚.Π΅. Π±ΡƒΠ΄Π΅Ρ‚ ΠΈΠΌΠ΅Ρ‚ΡŒ Π½Π°ΠΏΡ€Π°Π²Π»Π΅Π½ΠΈΠ΅ Π² ΠΏΡ€Π°Π²ΡƒΡŽ сторону ΠΏΠΎ Π»ΠΈΠ½ΠΈΠΈ ΠΌΠ΅Π΄ΠΈΠ°Π½Ρ‹, ΠΏΡ€ΠΎΠ²Π΅Π΄Π΅Π½Π½ΠΎΠΉ ΠΊ основанию, Π° это – Π½Π°ΠΏΡ€Π°Π²Π»Π΅Π½ΠΈΠ΅ Π²ΠΏΡ€Π°Π²ΠΎ ΠΏΠΎ Π³ΠΎΡ€ΠΈΠ·ΠΎΠ½Ρ‚Π°Π»ΠΈ.

ΠžΡ‚Π²Π΅Ρ‚: Π²ΠΏΡ€Π°Π²ΠΎ

Π’Ρ€Π΅Ρ‚ΠΈΠΉ Π²Π°Ρ€ΠΈΠ°Π½Ρ‚ задания (Π”Π΅ΠΌΠΈΠ΄ΠΎΠ²Π°, β„–25)

Как Π½Π°ΠΏΡ€Π°Π²Π»Π΅Π½Π° (Π²Π²Π΅Ρ€Ρ…, Π²Π½ΠΈΠ·, Π²Π»Π΅Π²ΠΎ, Π²ΠΏΡ€Π°Π²ΠΎ, ΠΎΡ‚ Π½Π°Π±Π»ΡŽΠ΄Π°Ρ‚Π΅Π»Ρ, ΠΊ Π½Π°Π±Π»ΡŽΠ΄Π°Ρ‚Π΅Π»ΡŽ) сила АмпСра, Π΄Π΅ΠΉΡΡ‚Π²ΡƒΡŽΡ‰Π°Ρ Π½Π° ΠΏΡ€ΠΎΠ²ΠΎΠ΄Π½ΠΈΠΊ β„– 3 со стороны Π΄Π²ΡƒΡ… Π΄Ρ€ΡƒΠ³ΠΈΡ… (см. рисунок)? ВсС ΠΏΡ€ΠΎΠ²ΠΎΠ΄Π½ΠΈΠΊΠΈ прямыС, Ρ‚ΠΎΠ½ΠΊΠΈΠ΅, Π΄Π»ΠΈΠ½Π½Ρ‹Π΅, Π»Π΅ΠΆΠ°Ρ‚ Π² ΠΎΠ΄Π½ΠΎΠΉ плоскости ΠΈ ΠΏΠ°Ρ€Π°Π»Π»Π΅Π»ΡŒΠ½Ρ‹ Π΄Ρ€ΡƒΠ³ Π΄Ρ€ΡƒΠ³Ρƒ. Π‘ΠΈΠ»Π° Ρ‚ΠΎΠΊΠ° I Π²ΠΎ всСх ΠΏΡ€ΠΎΠ²ΠΎΠ΄Π½ΠΈΠΊΠ°Ρ… ΠΎΠ΄ΠΈΠ½Π°ΠΊΠΎΠ²Π°. ΠžΡ‚Π²Π΅Ρ‚ Π·Π°ΠΏΠΈΡˆΠΈΡ‚Π΅ словом (словами).Π’Ρ€Π΅Ρ‚ΠΈΠΉ Π²Π°Ρ€ΠΈΠ°Π½Ρ‚ задания (Π”Π΅ΠΌΠΈΠ΄ΠΎΠ²Π°, β„–25)

Алгоритм Ρ€Π΅ΡˆΠ΅Π½ΠΈΡ:
  1. АнализируСм схСму направлСния Ρ‚ΠΎΠΊΠΎΠ² Π² ΠΏΡ€ΠΎΠ²ΠΎΠ΄Π½ΠΈΠΊΠ°Ρ….
  2. Π˜Ρ‰Π΅ΠΌ Π½Π°ΠΏΡ€Π°Π²Π»Π΅Π½ΠΈΠ΅ силы АмпСра, Π΄Π΅ΠΉΡΡ‚Π²ΡƒΡŽΡ‰ΠΈΡ… Π½Π° ΠΏΡ€ΠΎΠ²ΠΎΠ΄Π½ΠΈΠΊ β„–3.
  3. Аналогично ΠΈΡ‰Π΅ΠΌ Π½Π°ΠΏΡ€Π°Π²Π»Π΅Π½ΠΈΠ΅ силы со стороны 1-Π³ΠΎ ΠΏΡ€ΠΎΠ²ΠΎΠ΄Π½ΠΈΠΊΠ°.
  4. ΠžΠΏΡ€Π΅Π΄Π΅Π»ΡΠ΅ΠΌ Ρ€Π΅Π·ΡƒΠ»ΡŒΡ‚ΠΈΡ€ΡƒΡŽΡ‰Π΅Π΅ Π½Π°ΠΏΡ€Π°Π²Π»Π΅Π½ΠΈΠ΅.
  5. ЗаписываСм ΠΎΡ‚Π²Π΅Ρ‚.
РСшСниС:

1. Из Π·-Π½Π° АмпСра слСдуСт, Ρ‡Ρ‚ΠΎ Ρ‚ΠΎΠ½ΠΊΠΈΠ΅ ΠΏΠ°Ρ€Π°Π»Π»Π΅Π»ΡŒΠ½Ρ‹Π΅ ΠΏΡ€ΠΎΠ²ΠΎΠ΄Π½ΠΈΠΊΠΈ с постоянным Ρ‚ΠΎΠΊΠΎΠΌ, двиТущимся Π² ΠΎΠ΄Π½ΠΎΠΌ Π½Π°ΠΏΡ€Π°Π²Π»Π΅Π½ΠΈΠΈ, ΠΏΡ€ΠΈΡ‚ΡΠ³ΠΈΠ²Π°ΡŽΡ‚ΡΡ, Π° с Ρ‚ΠΎΠΊΠ°ΠΌΠΈ, двиТущимися Π² ΠΏΡ€ΠΎΡ‚ΠΈΠ²ΠΎΠΏΠΎΠ»ΠΎΠΆΠ½ΠΎΠΌ Π½Π°ΠΏΡ€Π°Π²Π»Π΅Π½ΠΈΠΈ, ΠΎΡ‚Ρ‚Π°Π»ΠΊΠΈΠ²Π°ΡŽΡ‚ΡΡ. Π­Ρ‚ΠΎ ΠΎΠ·Π½Π°Ρ‡Π°Π΅Ρ‚, Ρ‡Ρ‚ΠΎ ΠΏΡ€ΠΎΠ²ΠΎΠ΄Π½ΠΈΠΊΠΈ β„–2 ΠΈ β„–3 ΠΏΡ€ΠΈΡ‚ΡΠ³ΠΈΠ²Π°ΡŽΡ‚ΡΡ, Π° ΠΏΡ€ΠΎΠ²ΠΎΠ΄Π½ΠΈΠΊΠΈ β„–1 ΠΈ β„–3 ΠΎΡ‚Ρ‚Π°Π»ΠΊΠΈΠ²Π°ΡŽΡ‚ΡΡ.

2. ΠŸΠΎΡΠΊΠΎΠ»ΡŒΠΊΡƒ ΠΏΡ€ΠΎΠ²ΠΎΠ΄Π½ΠΈΠΊΠΈ ΠΏΠ°Ρ€Π°Π»Π»Π΅Π»ΡŒΠ½Ρ‹ ΠΌΠ΅ΠΆΠ΄Ρƒ собой ΠΈ располоТСны ΠΏΠΎ Π³ΠΎΡ€ΠΈΠ·ΠΎΠ½Ρ‚Π°Π»ΠΈ, Ρ‚ΠΎ силы притяТСния ΠΏΡ€ΠΎΠ²ΠΎΠ΄Π½ΠΈΠΊΠΎΠ² Π½Π°ΠΏΡ€Π°Π²Π»Π΅Π½Ρ‹ ΠΏΠΎ Π²Π΅Ρ€Ρ‚ΠΈΠΊΠ°Π»ΠΈ (Ρ‚.Π΅. пСрпСндикулярно). ΠŸΡ€ΠΈ этом сила притяТСния ΠΏΡ€ΠΎΠ²ΠΎΠ΄Π½ΠΈΠΊΠ° β„–3 ΠΊ ΠΏΡ€ΠΎΠ²ΠΎΠ΄Π½ΠΈΠΊΡƒ β„–2 Π½Π°ΠΏΡ€Π°Π²Π»Π΅Π½Π° Π²Π΅Ρ€Ρ‚ΠΈΠΊΠ°Π»ΡŒΠ½ΠΎ Π²Π²Π΅Ρ€Ρ… (красная стрСлка), Π° сила отталкивания ΠΏΡ€ΠΎΠ²ΠΎΠ΄Π½ΠΈΠΊΠ° β„–3 ΠΎΡ‚ ΠΏΡ€ΠΎΠ²ΠΎΠ΄Π½ΠΈΠΊΠ° β„–1 – Π²Π΅Ρ€Ρ‚ΠΈΠΊΠ°Π»ΡŒΠ½ΠΎ Π²Π½ΠΈΠ· (синяя стрСлка).

3. Но ΠΏΠΎΡΠΊΠΎΠ»ΡŒΠΊΡƒ ΠΏΡ€ΠΎΠ²ΠΎΠ΄Π½ΠΈΠΊ β„–2 Π±Π»ΠΈΠΆΠ΅ ΠΊ 3-ΠΌΡƒ, Ρ‡Π΅ΠΌ β„–1, Ρ‚ΠΎ ΠΏΡ€ΠΈ ΠΎΠ΄ΠΈΠ½Π°ΠΊΠΎΠ²Ρ‹Ρ… силах Ρ‚ΠΎΠΊΠΎΠ² воздСйствиС со стороны 2-Π³ΠΎ ΠΏΡ€ΠΎΠ²ΠΎΠ΄Π½ΠΈΠΊΠ° окаТСтся Π±ΠΎΠ»Π΅Π΅ ΡΠΈΠ»ΡŒΠ½Ρ‹ΠΌ, Ρ‚.Π΅. сила притяТСния Π±ΡƒΠ΄Π΅Ρ‚ большСй, Ρ‡Π΅ΠΌ сила отталкивания. Π‘Π»Π΅Π΄ΠΎΠ²Π°Ρ‚Π΅Π»ΡŒΠ½ΠΎ, Ρ€Π΅Π·ΡƒΠ»ΡŒΡ‚ΠΈΡ€ΡƒΡŽΡ‰Π°Ρ Π½Π°ΠΏΡ€Π°Π²Π»Π΅Π½Π° Π²Π΅Ρ€Ρ‚ΠΈΠΊΠ°Π»ΡŒΠ½ΠΎ Π²Π²Π΅Ρ€Ρ….

ΠŸΡ€ΡΠΌΠΎΠ»ΠΈΠ½Π΅ΠΉΠ½Ρ‹ΠΉ ΠΏΡ€ΠΎΠ²ΠΎΠ΄Π½ΠΈΠΊ Π΄Π»ΠΈΠ½ΠΎΠΉ 0,2 ΠΌ находится Π² ΠΎΠ΄Π½ΠΎΡ€ΠΎΠ΄Π½ΠΎΠΌ ΠΌΠ°Π³Π½ΠΈΡ‚Π½ΠΎΠΌ ΠΏΠΎΠ»Π΅ с ΠΈΠ½Π΄ΡƒΠΊΡ†ΠΈΠ΅ΠΉ 4 Π’Π» ΠΈ располоТСн ΠΏΠΎΠ΄ ΡƒΠ³Π»ΠΎΠΌ ΠΊ Π²Π΅ΠΊΡ‚ΠΎΡ€Ρƒ ΠΈΠ½Π΄ΡƒΠΊΡ†ΠΈΠΈ. Π§Π΅ΠΌΡƒ Ρ€Π°Π²Π΅Π½ ΠΌΠΎΠ΄ΡƒΠ»ΡŒ силы, Π΄Π΅ΠΉΡΡ‚Π²ΡƒΡŽΡ‰Π΅ΠΉ Π½Π° ΠΏΡ€ΠΎΠ²ΠΎΠ΄Π½ΠΈΠΊ со стороны ΠΌΠ°Π³Π½ΠΈΡ‚Π½ΠΎΠ³ΠΎ поля ΠΏΡ€ΠΈ силС Ρ‚ΠΎΠΊΠ° Π² Π½Π΅ΠΌ 2 А? (ΠžΡ‚Π²Π΅Ρ‚ Π΄Π°Ρ‚ΡŒ Π² Π½ΡŒΡŽΡ‚ΠΎΠ½Π°Ρ….)

2. ΠŸΡ€ΡΠΌΠΎΠ»ΠΈΠ½Π΅ΠΉΠ½Ρ‹ΠΉ ΠΏΡ€ΠΎΠ²ΠΎΠ΄Π½ΠΈΠΊ Π΄Π»ΠΈΠ½ΠΎΠΉ 0,5 ΠΌ, ΠΏΠΎ ΠΊΠΎΡ‚ΠΎΡ€ΠΎΠΌΡƒ Ρ‚Π΅Ρ‡Π΅Ρ‚ Ρ‚ΠΎΠΊ 6 А, находится Π² ΠΎΠ΄Π½ΠΎΡ€ΠΎΠ΄Π½ΠΎΠΌ ΠΌΠ°Π³Π½ΠΈΡ‚Π½ΠΎΠΌ ΠΏΠΎΠ»Π΅. ΠœΠΎΠ΄ΡƒΠ»ΡŒ Π²Π΅ΠΊΡ‚ΠΎΡ€Π° ΠΌΠ°Π³Π½ΠΈΡ‚Π½ΠΎΠΉ ΠΈΠ½Π΄ΡƒΠΊΡ†ΠΈΠΈ 0,2 Π’Π», ΠΏΡ€ΠΎΠ²ΠΎΠ΄Π½ΠΈΠΊ располоТСн ΠΏΠΎΠ΄ ΡƒΠ³Π»ΠΎΠΌ ΠΊ Π²Π΅ΠΊΡ‚ΠΎΡ€Ρƒ Π’ . Какова сила, Π΄Π΅ΠΉΡΡ‚Π²ΡƒΡŽΡ‰Π°Ρ Π½Π° ΠΏΡ€ΠΎΠ²ΠΎΠ΄Π½ΠΈΠΊ со стороны ΠΌΠ°Π³Π½ΠΈΡ‚Π½ΠΎΠ³ΠΎ поля? (ΠžΡ‚Π²Π΅Ρ‚ Π΄Π°Ρ‚ΡŒ Π² Π½ΡŒΡŽΡ‚ΠΎΠ½Π°Ρ….)

3. ΠŸΡ€ΠΈ силС Ρ‚ΠΎΠΊΠ° Π² ΠΏΡ€ΠΎΠ²ΠΎΠ΄Π½ΠΈΠΊΠ΅ 20 А Π½Π° участок прямого ΠΏΡ€ΠΎΠ²ΠΎΠ΄Π½ΠΈΠΊΠ° Π΄Π»ΠΈΠ½ΠΎΠΉ 50 см Π² ΠΎΠ΄Π½ΠΎΡ€ΠΎΠ΄Π½ΠΎΠΌ ΠΌΠ°Π³Π½ΠΈΡ‚Π½ΠΎΠΌ ΠΏΠΎΠ»Π΅ дСйствуСт сила АмпСра 12 Н. Π’Π΅ΠΊΡ‚ΠΎΡ€ ΠΈΠ½Π΄ΡƒΠΊΡ†ΠΈΠΈ ΠΌΠ°Π³Π½ΠΈΡ‚Π½ΠΎΠ³ΠΎ поля Π½Π°ΠΏΡ€Π°Π²Π»Π΅Π½ ΠΏΠΎΠ΄ ΡƒΠ³Π»ΠΎΠΌ 37Β° ΠΊ ΠΏΡ€ΠΎΠ²ΠΎΠ΄Π½ΠΈΠΊΡƒ ΠžΠΏΡ€Π΅Π΄Π΅Π»ΠΈΡ‚Π΅ ΠΌΠΎΠ΄ΡƒΠ»ΡŒ ΠΈΠ½Π΄ΡƒΠΊΡ†ΠΈΠΈ ΠΌΠ°Π³Π½ΠΈΡ‚Π½ΠΎΠ³ΠΎ поля. ΠžΡ‚Π²Π΅Ρ‚ Π²Ρ‹Ρ€Π°Π·ΠΈΡ‚Π΅ Π² тСслах ΠΈ ΠΎΠΊΡ€ΡƒΠ³Π»ΠΈΡ‚Π΅ Π΄ΠΎ Ρ†Π΅Π»ΠΎΠ³ΠΎ числа.

4. Π”Π°Π½ участок прямого ΠΏΡ€ΠΎΠ²ΠΎΠ΄Π½ΠΈΠΊΠ° Π΄Π»ΠΈΠ½ΠΎΠΉ 50 см Π² ΠΎΠ΄Π½ΠΎΡ€ΠΎΠ΄Π½ΠΎΠΌ ΠΌΠ°Π³Π½ΠΈΡ‚Π½ΠΎΠΌ ΠΏΠΎΠ»Π΅ с ΠΈΠ½Π΄ΡƒΠΊΡ†ΠΈΠ΅ΠΉ 2 Π’Π» ΠΏΡ€ΠΈ силС Ρ‚ΠΎΠΊΠ° Π² ΠΏΡ€ΠΎΠ²ΠΎΠ΄Π½ΠΈΠΊΠ΅ 20 А ΠΈ Π½Π°ΠΏΡ€Π°Π²Π»Π΅Π½ΠΈΠΈ Π²Π΅ΠΊΡ‚ΠΎΡ€Π° ΠΈΠ½Π΄ΡƒΠΊΡ†ΠΈΠΈ ΠΌΠ°Π³Π½ΠΈΡ‚Π½ΠΎΠ³ΠΎ поля ΠΏΠΎΠ΄ ΡƒΠ³Π»ΠΎΠΌ ΠΊ ΠΏΡ€ΠΎΠ²ΠΎΠ΄Π½ΠΈΠΊΡƒ. Какова сила АмпСра, Π΄Π΅ΠΉΡΡ‚Π²ΡƒΡŽΡ‰Π°Ρ Π½Π° этот участок? (ΠžΡ‚Π²Π΅Ρ‚ Π΄Π°Ρ‚ΡŒ Π² Π½ΡŒΡŽΡ‚ΠΎΠ½Π°Ρ….)

5. ΠŸΡ€ΠΎΠ²ΠΎΠ΄Π½ΠΈΠΊ с Ρ‚ΠΎΠΊΠΎΠΌ Π΄Π»ΠΈΠ½ΠΎΠΉ 2 ΠΌ находится Π² ΠΎΠ΄Π½ΠΎΡ€ΠΎΠ΄Π½ΠΎΠΌ ΠΌΠ°Π³Π½ΠΈΡ‚Π½ΠΎΠΌ ΠΏΠΎΠ»Π΅ с ΠΈΠ½Π΄ΡƒΠΊΡ†ΠΈΠ΅ΠΉ ΠŸΡ€ΠΈΡ‡Π΅ΠΌ Π½Π°ΠΏΡ€Π°Π²Π»Π΅Π½ΠΈΠ΅ ΠΌΠ°Π³Π½ΠΈΡ‚Π½ΠΎΠ³ΠΎ поля составляСт 30Β° с Π½Π°ΠΏΡ€Π°Π²Π»Π΅Π½ΠΈΠ΅ΠΌ Ρ‚ΠΎΠΊΠ°. Π§Π΅ΠΌΡƒ Ρ€Π°Π²Π½Π° сила со стороны ΠΌΠ°Π³Π½ΠΈΡ‚Π½ΠΎΠ³ΠΎ поля, Π΄Π΅ΠΉΡΡ‚Π²ΡƒΡŽΡ‰Π°Ρ Π½Π° ΠΏΡ€ΠΎΠ²ΠΎΠ΄Π½ΠΈΠΊ? (ΠžΡ‚Π²Π΅Ρ‚ Π΄Π°Ρ‚ΡŒ Π² Π½ΡŒΡŽΡ‚ΠΎΠ½Π°Ρ….)

6. Π”Π²Π° Π΄Π»ΠΈΠ½Π½Ρ‹Ρ… прямых ΠΏΡ€ΠΎΠ²ΠΎΠ΄Π°, ΠΏΠΎ ΠΊΠΎΡ‚ΠΎΡ€Ρ‹ΠΌ ΠΏΡ€ΠΎΡ‚Π΅ΠΊΠ°ΡŽΡ‚ постоянныС элСктричСскиС Ρ‚ΠΎΠΊΠΈ, располоТСны ΠΏΠ°Ρ€Π°Π»Π»Π΅Π»ΡŒΠ½ΠΎ Π΄Ρ€ΡƒΠ³ Π΄Ρ€ΡƒΠ³Ρƒ. Π’ Ρ‚Π°Π±Π»ΠΈΡ†Π΅ ΠΏΡ€ΠΈΠ²Π΅Π΄Π΅Π½Π° Π·Π°Π²ΠΈΡΠΈΠΌΠΎΡΡ‚ΡŒ модуля силы F ΠΌΠ°Π³Π½ΠΈΡ‚Π½ΠΎΠ³ΠΎ взаимодСйствия этих ΠΏΡ€ΠΎΠ²ΠΎΠ΄ΠΎΠ² ΠΎΡ‚ расстояния r ΠΌΠ΅ΠΆΠ΄Ρƒ Π½ΠΈΠΌΠΈ.

Π§Π΅ΠΌΡƒ Π±ΡƒΠ΄Π΅Ρ‚ Ρ€Π°Π²Π΅Π½ ΠΌΠΎΠ΄ΡƒΠ»ΡŒ силы ΠΌΠ°Π³Π½ΠΈΡ‚Π½ΠΎΠ³ΠΎ взаимодСйствия ΠΌΠ΅ΠΆΠ΄Ρƒ этими ΠΏΡ€ΠΎΠ²ΠΎΠ΄Π°ΠΌΠΈ, Ссли расстояниС ΠΌΠ΅ΠΆΠ΄Ρƒ Π½ΠΈΠΌΠΈ ΡΠ΄Π΅Π»Π°Ρ‚ΡŒ Ρ€Π°Π²Π½Ρ‹ΠΌ 6 ΠΌ, Π½Π΅ мСняя силы Ρ‚Π΅ΠΊΡƒΡ‰ΠΈΡ… Π² ΠΏΡ€ΠΎΠ²ΠΎΠ΄Π°Ρ… Ρ‚ΠΎΠΊΠΎΠ²? (ΠžΡ‚Π²Π΅Ρ‚ Π΄Π°Ρ‚ΡŒ Π² мкН.)

8. ΠŸΡ€ΡΠΌΠΎΠΉ ΠΏΡ€ΠΎΠ²ΠΎΠ΄Π½ΠΈΠΊ Π΄Π»ΠΈΠ½ΠΎΠΉ 50 см Ρ€Π°Π²Π½ΠΎΠΌΠ΅Ρ€Π½ΠΎ ΠΏΠΎΡΡ‚ΡƒΠΏΠ°Ρ‚Π΅Π»ΡŒΠ½ΠΎ двиТСтся Π² ΠΎΠ΄Π½ΠΎΡ€ΠΎΠ΄Π½ΠΎΠΌ постоянном ΠΌΠ°Π³Π½ΠΈΡ‚Π½ΠΎΠΌ ΠΏΠΎΠ»Π΅, Π½Π°ΠΏΡ€Π°Π²Π»Π΅Π½ΠΈΠ΅ ΠΊΠΎΡ‚ΠΎΡ€ΠΎΠ³ΠΎ совпадаСт с Π½Π°ΠΏΡ€Π°Π²Π»Π΅Π½ΠΈΠ΅ΠΌ Π²Π΅Ρ€Ρ‚ΠΈΠΊΠ°Π»ΡŒΠ½ΠΎΠΉ оси Y (Π½Π° рисункС эта ось Π½Π°ΠΏΡ€Π°Π²Π»Π΅Π½Π° Β«Π½Π° нас»). Π‘ΠΊΠΎΡ€ΠΎΡΡ‚ΡŒ ΠΏΡ€ΠΎΠ²ΠΎΠ΄Π½ΠΈΠΊΠ° Π½Π°ΠΏΡ€Π°Π²Π»Π΅Π½Π° пСрпСндикулярно Π΅ΠΌΡƒ, ΠΈ составляСт ΡƒΠ³ΠΎΠ» 30Β° с Π³ΠΎΡ€ΠΈΠ·ΠΎΠ½Ρ‚Π°Π»ΡŒΠ½ΠΎΠΉ осью X , ΠΊΠ°ΠΊ ΠΏΠΎΠΊΠ°Π·Π°Π½ΠΎ Π½Π° рисункС. Π Π°Π·Π½ΠΎΡΡ‚ΡŒ ΠΏΠΎΡ‚Π΅Π½Ρ†ΠΈΠ°Π»ΠΎΠ² ΠΌΠ΅ΠΆΠ΄Ρƒ ΠΊΠΎΠ½Ρ†Π°ΠΌΠΈ ΠΏΡ€ΠΎΠ²ΠΎΠ΄Π½ΠΈΠΊΠ° Ρ€Π°Π²Π½Π° 25 ΠΌΠ’, ΠΌΠΎΠ΄ΡƒΠ»ΡŒ ΠΈΠ½Π΄ΡƒΠΊΡ†ΠΈΠΈ ΠΌΠ°Π³Π½ΠΈΡ‚Π½ΠΎΠ³ΠΎ поля 0,1 Π’Π». ΠžΠΏΡ€Π΅Π΄Π΅Π»ΠΈΡ‚Π΅ ΠΌΠΎΠ΄ΡƒΠ»ΡŒ скорости двиТСния этого ΠΏΡ€ΠΎΠ²ΠΎΠ΄Π½ΠΈΠΊΠ°. (ΠžΡ‚Π²Π΅Ρ‚ Π΄Π°Ρ‚ΡŒ Π² ΠΌΠ΅Ρ‚Ρ€Π°Ρ… Π² сСкунду.)

ΠŸΡ€ΠΈΠΌΠ΅Ρ‡Π°Π½ΠΈΠ΅

9. ΠŸΡ€ΡΠΌΠΎΠΉ ΠΏΡ€ΠΎΠ²ΠΎΠ΄Π½ΠΈΠΊ Π΄Π»ΠΈΠ½ΠΎΠΉ 25 см Ρ€Π°Π²Π½ΠΎΠΌΠ΅Ρ€Π½ΠΎ ΠΏΠΎΡΡ‚ΡƒΠΏΠ°Ρ‚Π΅Π»ΡŒΠ½ΠΎ двиТСтся Π² ΠΎΠ΄Π½ΠΎΡ€ΠΎΠ΄Π½ΠΎΠΌ постоянном ΠΌΠ°Π³Π½ΠΈΡ‚Π½ΠΎΠΌ ΠΏΠΎΠ»Π΅, Π½Π°ΠΏΡ€Π°Π²Π»Π΅Π½ΠΈΠ΅ ΠΊΠΎΡ‚ΠΎΡ€ΠΎΠ³ΠΎ совпадаСт с Π½Π°ΠΏΡ€Π°Π²Π»Π΅Π½ΠΈΠ΅ΠΌ Π²Π΅Ρ€Ρ‚ΠΈΠΊΠ°Π»ΡŒΠ½ΠΎΠΉ оси Y (Π½Π° рисункС эта ось Π½Π°ΠΏΡ€Π°Π²Π»Π΅Π½Π° Β«Π½Π° нас»). Π‘ΠΊΠΎΡ€ΠΎΡΡ‚ΡŒ ΠΏΡ€ΠΎΠ²ΠΎΠ΄Π½ΠΈΠΊΠ° Ρ€Π°Π²Π½Π° 1 ΠΌ/с, Π½Π°ΠΏΡ€Π°Π²Π»Π΅Π½Π° пСрпСндикулярно ΠΏΡ€ΠΎΠ²ΠΎΠ΄Π½ΠΈΠΊΡƒ, ΠΈ составляСт ΡƒΠ³ΠΎΠ» 60Β° с Π³ΠΎΡ€ΠΈΠ·ΠΎΠ½Ρ‚Π°Π»ΡŒΠ½ΠΎΠΉ осью X , ΠΊΠ°ΠΊ ΠΏΠΎΠΊΠ°Π·Π°Π½ΠΎ Π½Π° рисункС. Π Π°Π·Π½ΠΎΡΡ‚ΡŒ ΠΏΠΎΡ‚Π΅Π½Ρ†ΠΈΠ°Π»ΠΎΠ² ΠΌΠ΅ΠΆΠ΄Ρƒ ΠΊΠΎΠ½Ρ†Π°ΠΌΠΈ ΠΏΡ€ΠΎΠ²ΠΎΠ΄Π½ΠΈΠΊΠ° Ρ€Π°Π²Π½Π° 75 ΠΌΠ’. ΠžΠΏΡ€Π΅Π΄Π΅Π»ΠΈΡ‚Π΅ ΠΌΠΎΠ΄ΡƒΠ»ΡŒ ΠΈΠ½Π΄ΡƒΠΊΡ†ΠΈΠΈ ΠΌΠ°Π³Π½ΠΈΡ‚Π½ΠΎΠ³ΠΎ поля. (ΠžΡ‚Π²Π΅Ρ‚ Π΄Π°Ρ‚ΡŒ Π² тСслах.)

ΠŸΡ€ΠΈΠΌΠ΅Ρ‡Π°Π½ΠΈΠ΅ : Π²Π΅ΠΊΡ‚ΠΎΡ€ скорости Π»Π΅ΠΆΠΈΡ‚ Π² плоскости рисунка.

10.

Как Π½Π°ΠΏΡ€Π°Π²Π»Π΅Π½Π° ΠΎΡ‚Π½ΠΎΡΠΈΡ‚Π΅Π»ΡŒΠ½ΠΎ рисунка (Π²ΠΏΡ€Π°Π²ΠΎ, Π²Π»Π΅Π²ΠΎ, Π²Π²Π΅Ρ€Ρ…, Π²Π½ΠΈΠ·, ΠΊ Π½Π°Π±Π»ΡŽΠ΄Π°Ρ‚Π΅Π»ΡŽ, ΠΎΡ‚ Π½Π°Π±Π»ΡŽΠ΄Π°Ρ‚Π΅Π»Ρ) сила АмпСра, Π΄Π΅ΠΉΡΡ‚Π²ΡƒΡŽΡ‰Π°Ρ Π½Π° ΠΏΡ€ΠΎΠ²ΠΎΠ΄Π½ΠΈΠΊ 1 со стороны ΠΏΡ€ΠΎΠ²ΠΎΠ΄Π½ΠΈΠΊΠ° 2 (см. рисунок), Ссли ΠΏΡ€ΠΎΠ²ΠΎΠ΄Π½ΠΈΠΊΠΈ Ρ‚ΠΎΠ½ΠΊΠΈΠ΅, Π΄Π»ΠΈΠ½Π½Ρ‹Π΅, прямыС, ΠΏΠ°Ρ€Π°Π»Π»Π΅Π»ΡŒΠ½Ρ‹ Π΄Ρ€ΡƒΠ³ Π΄Ρ€ΡƒΠ³Ρƒ? (I — сила Ρ‚ΠΎΠΊΠ°.) ΠžΡ‚Π²Π΅Ρ‚ Π·Π°ΠΏΠΈΡˆΠΈΡ‚Π΅ словом (словами).

ΠŸΡ€ΠΈ создании ΠΊΠ°Ρ€Ρ‚ΠΈΠ½Ρ‹ с ΡƒΡ‡Ρ‘Ρ‚ΠΎΠΌ творчСского замысла ΠΈ ΡΡŽΠΆΠ΅Ρ‚Π° ΠΊΠΎΠΌΠΏΠΎΠ·ΠΈΡ†ΠΈΠΈ Ρ…ΡƒΠ΄ΠΎΠΆΠ½ΠΈΠΊ Π²Ρ‹Π±ΠΈΡ€Π°Π΅Ρ‚ ΠΈΠ»ΠΈ Π·Π°Π΄Π°Ρ‘Ρ‚ высоту Ρ‚ΠΎΡ‡ΠΊΠΈ зрСния, Π° Π½Π° Π΅Ρ‘ основС опрСдСляСт ΠΏΠΎΠ»ΠΎΠΆΠ΅Π½ΠΈΠ΅ Π»ΠΈΠ½ΠΈΠΈ Π³ΠΎΡ€ΠΈΠ·ΠΎΠ½Ρ‚Π°.

ΠŸΡ€ΠΈ ΠΈΠ·ΡƒΡ‡Π΅Π½ΠΈΠΈ пСрспСктивы ΠΎΡ‡Π΅Π½ΡŒ часто линию Π³ΠΎΡ€ΠΈΠ·ΠΎΠ½Ρ‚Π° ΡΡ‡ΠΈΡ‚Π°ΡŽΡ‚ Π΄ΡƒΠ³ΠΎΠΎΠ±Ρ€Π°Π·Π½ΠΎΠΉ, учитывая ΡΡ„Π΅Ρ€ΠΈΡ‡Π΅ΡΠΊΡƒΡŽ Ρ„ΠΎΡ€ΠΌΡƒ Π—Π΅ΠΌΠ»ΠΈ. Однако, сравнивая Ρ€Π°Π·ΠΌΠ΅Ρ€Ρ‹ Π—Π΅ΠΌΠ½ΠΎΠ³ΠΎ ΡˆΠ°Ρ€Π° с Π½ΠΈΡ‡Ρ‚ΠΎΠΆΠ½ΠΎ ΠΌΠ°Π»Ρ‹ΠΌ участком Π΅Π³ΠΎ ΠΊΠΎΠ½Ρ‚ΡƒΡ€Π°, ΠΎΡ…Π²Π°Ρ‚Ρ‹Π²Π°Π΅ΠΌΠΎΠ³ΠΎ ΠΏΠΎΠ»Π΅ΠΌ зрСния Ρ‡Π΅Π»ΠΎΠ²Π΅ΠΊΠ°, ΠΊΡ€Π°ΠΉ повСрхности моря(ΠΈΠ»ΠΈ Π—Π΅ΠΌΠ»ΠΈ) воспринимаСтся Π³ΠΎΡ€ΠΈΠ·ΠΎΠ½Ρ‚Π°Π»ΡŒΠ½ΠΎΠΉ Π²ΠΎΠΎΠ±Ρ€Π°ΠΆΠ°Π΅ΠΌΠΎΠΉ Π»ΠΈΠ½ΠΈΠ΅ΠΉ.

Π’ этом Π»Π΅Π³ΠΊΠΎ ΠΌΠΎΠΆΠ½ΠΎ ΡƒΠ±Π΅Π΄ΠΈΡ‚ΡŒΡΡ, Ссли Π²ΡΡ‚Π°Ρ‚ΡŒ Π½Π° ΠΎΡ‚ΠΊΡ€Ρ‹Ρ‚ΠΎΠΉ Ρ€Π°Π²Π½ΠΈΠ½Π½ΠΎΠΉ мСстности ΠΈΠ»ΠΈ Π½Π° Π±Π΅Ρ€Π΅Π³Ρƒ моря ΠΈ Π½Π° ΡƒΡ€ΠΎΠ²Π½Π΅ Π³Π»Π°Π· Π³ΠΎΡ€ΠΈΠ·ΠΎΠ½Ρ‚Π°Π»ΡŒΠ½ΠΎ Ρ€Π°ΡΠΏΠΎΠ»ΠΎΠΆΠΈΡ‚ΡŒ Π»ΠΈΠ½Π΅ΠΉΠΊΡƒ ΠΈΠ»ΠΈ ΠΊΠ°Ρ€Π°Π½Π΄Π°Ρˆ. Π’ΠΎΠ³Π΄Π° Π² ΠΏΡ€Π΅Π΄Π΅Π»Π°Ρ… поля зрСния ΠΊΡ€Π°ΠΉ Π»ΠΈΠ½Π΅ΠΉΠΊΠΈ совпадаСт с ΠΊΡ€Π°Π΅ΠΌ моря ΠΈΠ»ΠΈ Ρ€Π°Π²Π½ΠΈΠ½Π½ΠΎΠΉ мСстности, Ρ‚. Π΅. с Π²ΠΎΠΎΠ±Ρ€Π°ΠΆΠ°Π΅ΠΌΠΎΠΉ Π»ΠΈΠ½ΠΈΠ΅ΠΉ Π³ΠΎΡ€ΠΈΠ·ΠΎΠ½Ρ‚Π°. Π‘Π»Π΅Π΄ΠΎΠ²Π°Ρ‚Π΅Π»ΡŒΠ½ΠΎ, ΠΏΡ€ΠΈ нСбольшом участкС видСния пространства эта ΠΎΠΊΡ€ΡƒΠ³Π»ΠΎΡΡ‚ΡŒ Π—Π΅ΠΌΠ»ΠΈ Π½Π΅ ΠΎΡ‰ΡƒΡ‚ΠΈΠΌΠ° для нашСго зрСния, поэтому Π³ΠΎΡ€ΠΈΠ·ΠΎΠ½Ρ‚ воспринимаСтся Π³ΠΎΡ€ΠΈΠ·ΠΎΠ½Ρ‚Π°Π»ΡŒΠ½ΠΎΠΉ Π»ΠΈΠ½ΠΈΠ΅ΠΉ. ВмСстС с Ρ‚Π΅ΠΌ, Ссли ΠΏΠΎΠ²ΠΎΡ€Π°Ρ‡ΠΈΠ²Π°Ρ‚ΡŒ Π³Π»Π°Π·Π° Π² ΠΎΠ±Π΅ стороны, Π½Π΅ двигая Π³ΠΎΠ»ΠΎΠ²ΠΎΠΉ, Ρ‚ΠΎ ΠΏΡ€ΠΈ большом ΠΎΡ…Π²Π°Ρ‚Π΅ ΠΎΡ‚ΠΊΡ€Ρ‹Ρ‚ΠΎΠ³ΠΎ пространства Π·Ρ€Π΅Π½ΠΈΠ΅ΠΌ Ρ‡Π΅Π»ΠΎΠ²Π΅ΠΊΠ° эта Π½Π΅Π·Π½Π°Ρ‡ΠΈΡ‚Π΅Π»ΡŒΠ½Π°Ρ ΠΎΠΊΡ€ΡƒΠ³Π»ΠΎΡΡ‚ΡŒ Π·Π΅ΠΌΠ½ΠΎΠΉ повСрхности слСгка ощущаСтся.

Π‘Ρ„Π΅Ρ€ΠΈΡ‡Π½ΠΎΡΡ‚ΡŒ Π—Π΅ΠΌΠ»ΠΈ особСнно проявляСтся ΠΏΡ€ΠΈ Π³Π»ΡƒΠ±ΠΈΠ½Π½ΠΎΠΌ ΠΎΡ…Π²Π°Ρ‚Π΅ пространства морских Π΄Π°Π»Π΅ΠΉ. ΠœΡ‹ΡΠ»Π΅Π½Π½ΠΎ прСдставим, Ρ‡Ρ‚ΠΎ ΠΌΡ‹ стоим Π½Π° Π±Π΅Ρ€Π΅Π³Ρƒ моря ΠΈ смотрим вслСд ΡƒΠΏΠ»Ρ‹Π²Π°ΡŽΡ‰Π΅ΠΌΡƒ ΠΏΠ°Ρ€ΠΎΡ…ΠΎΠ΄Ρƒ, фиксируя ΠΏΡ€ΠΈ ΡƒΠ΄Π°Π»Π΅Π½ΠΈΠΈ Π΅Π³ΠΎ Π²ΠΈΠ΄ΠΈΠΌΠΎΡΡ‚ΡŒ.

На схСмС ΠΎΡ‚ΠΌΠ΅Ρ‚ΠΈΠΌ ΠΏΠ΅Ρ€Π²ΠΎΠ½Π°Ρ‡Π°Π»ΡŒΠ½ΠΎΠ΅ ΠΏΠΎΠ»ΠΎΠΆΠ΅Π½ΠΈΠ΅ ΠΏΠ°Ρ€ΠΎΡ…ΠΎΠ΄Π° ΠΏΡ€ΠΈ нСбольшом ΡƒΠ΄Π°Π»Π΅Π½ΠΈΠΈ ΠΈ с Π½Π΅Π·Π½Π°Ρ‡ΠΈΡ‚Π΅Π»ΡŒΠ½Ρ‹ΠΌ ΡƒΠΌΠ΅Π½ΡŒΡˆΠ΅Π½ΠΈΠ΅ΠΌ Π΅Π³ΠΎ Π²Π΅Π»ΠΈΡ‡ΠΈΠ½Ρ‹ (I). Π‘ ΠΏΡ€ΠΈΠ±Π»ΠΈΠΆΠ΅Π½ΠΈΠ΅ΠΌ ΠΊ Π³ΠΎΡ€ΠΈΠ·ΠΎΠ½Ρ‚Ρƒ ΠΏΠ°Ρ€ΠΎΡ…ΠΎΠ΄ с Π±Π΅Ρ€Π΅Π³Π° Π±ΡƒΠ΄Π΅Ρ‚ Π²ΠΈΠ΄Π΅Π½ Π΅Ρ‰Ρ‘ ΠΏΠΎΠ»Π½ΠΎΡΡ‚ΡŒΡŽ, Π½ΠΎ ΠΏΠΎ Ρ€Π°Π·ΠΌΠ΅Ρ€Π°ΠΌ ΠΎΡ‡Π΅Π½ΡŒ нСбольшим (II). Π—Π°Ρ‚Π΅ΠΌ Π±ΡƒΠ΄Π΅Ρ‚ Π²ΠΈΠ΄Π½Π° Π΅Π³ΠΎ вСрхняя палубная Ρ‡Π°ΡΡ‚ΡŒ с дымящСйся Ρ‚Ρ€ΡƒΠ±ΠΎΠΉ (III), ΠΈ, Π½Π°ΠΊΠΎΠ½Π΅Ρ†, ΠΌΠΎΠΆΠ½ΠΎ Π·Π°ΠΌΠ΅Ρ‚ΠΈΡ‚ΡŒ Ρ‚ΠΎΠ»ΡŒΠΊΠΎ ΡˆΠ»Π΅ΠΉΡ„ Π΄Ρ‹ΠΌΠ° (IV).

Π’Π°ΠΊΠΈΠΌ ΠΎΠ±Ρ€Π°Π·ΠΎΠΌ, схСматичный рисунок, сдСланный Π½Π° основС наблюдСний, наглядно ΠΏΠΎΠΊΠ°Π·Ρ‹Π²Π°Π΅Ρ‚ Π½Π° ΡΡƒΡ‰Π΅ΡΡ‚Π²ΡƒΡŽΡ‰ΡƒΡŽ Π² Π΄Π΅ΠΉΡΡ‚Π²ΠΈΡ‚Π΅Π»ΡŒΠ½ΠΎΡΡ‚ΠΈ ΡΡ„Π΅Ρ€ΠΈΡ‡Π΅ΡΠΊΡƒΡŽ Ρ„ΠΎΡ€ΠΌΡƒ повСрхности Π—Π΅ΠΌΠ»ΠΈ.

На основС этого ΠΏΡ€ΠΈΠΌΠ΅Ρ€Π° зафиксируСм наблюдСния зритСля, стоящСго Π½Π° Π±Π΅Ρ€Π΅Π³Ρƒ моря, Π·Π° Π΄Π²ΠΈΠΆΠ΅Π½ΠΈΠ΅ΠΌ Π²Π΅Ρ€Π΅Π½ΠΈΡ†Ρ‹ парусников, ΡƒΠΏΠ»Ρ‹Π²Π°ΡŽΡ‰ΠΈΡ… Π² Π½Π°ΠΏΡ€Π°Π²Π»Π΅Π½ΠΈΠΈ ΠΊ Π³ΠΎΡ€ΠΈΠ·ΠΎΠ½Ρ‚Ρƒ.

Π—Π°ΠΌΠ΅Ρ‚ΠΈΠΌ, Ρ‡Ρ‚ΠΎ Π² Π΄Π΅ΠΉΡΡ‚Π²ΠΈΡ‚Π΅Π»ΡŒΠ½ΠΎΡΡ‚ΠΈ траСктория ΠΈΡ… двиТСния Π·Ρ€ΠΈΡ‚Π΅Π»ΡŒΠ½ΠΎ воспринимаСтся ΠΏΠΎ ΠΊΡ€ΠΈΠ²ΠΎΠΉ с ΡƒΡ‡Ρ‘Ρ‚ΠΎΠΌ сфСричности Π—Π΅ΠΌΠ»ΠΈ. ВмСстС с Ρ‚Π΅ΠΌ ΠΈΠ·ΠΎΠ±Ρ€Π°ΠΆΠ΅Π½ΠΈΠ΅ Π²Π΅Ρ€Π΅Π½ΠΈΡ†Ρ‹ парусников Π½Π° ΠΊΠ°Ρ€Ρ‚ΠΈΠ½Π΅, Π²Ρ‹ΠΏΠΎΠ»Π½Π΅Π½Π½ΠΎΠ΅ ΠΏΠΎ строгим Π·Π°ΠΊΠΎΠ½Π°ΠΌ пСрспСктивы, Π±ΡƒΠ΄Π΅Ρ‚ Π΄Ρ€ΡƒΠ³ΠΈΠΌ. НаправлСниС ΠΏΡƒΡ‚ΠΈ, двиТущихся ΠΊ Π³ΠΎΡ€ΠΈΠ·ΠΎΠ½Ρ‚Ρƒ ΠΎΠ΄ΠΈΠ½ Π·Π° Π΄Ρ€ΡƒΠ³ΠΈΠΌ парусников, тСорСтичСски прСдставляСт собой ΠΏΡ€ΡΠΌΡƒΡŽ линию с ΠΏΡ€Π΅Π΄Π΅Π»ΡŒΠ½ΠΎΠΉ Ρ‚ΠΎΡ‡ΠΊΠΎΠΉ Π½Π° Π³ΠΎΡ€ΠΈΠ·ΠΎΠ½Ρ‚Π΅, Π° ΠΏΡ€ΠΈ ΠΈΡ… построСнии β€” Π΄Π²Π΅ ΠΏΠ°Ρ€Π°Π»Π»Π΅Π»ΡŒΠ½Ρ‹Π΅ прямыС с Ρ‚ΠΎΡ‡ΠΊΠΎΠΉ схода.

Π’Π°ΠΊΠΈΠΌ ΠΎΠ±Ρ€Π°Π·ΠΎΠΌ, ΠΏΡ€ΠΈ Π³Π»ΡƒΠ±ΠΈΠ½Π½ΠΎΠΌ ΠΎΡ…Π²Π°Ρ‚Π΅ Π·Ρ€Π΅Π½ΠΈΠ΅ΠΌ Ρ‡Π΅Π»ΠΎΠ²Π΅ΠΊΠ° ΠΎΡ‚ΠΊΡ€Ρ‹Ρ‚ΠΎΠ³ΠΎ пространства (морских Π΄Π°Π»Π΅ΠΉ) с двиТущимися ΠΎΠ±ΡŠΠ΅ΠΊΡ‚Π°ΠΌΠΈ ΠΎΠΊΡ€ΡƒΠ³Π»ΠΎΡΡ‚ΡŒ Π—Π΅ΠΌΠ»ΠΈ слСгка ощущаСтся. Рассматривая ΡƒΠ΄Π°Π»Ρ‘Π½Π½ΠΎΠ΅ пространство Ρƒ Π³ΠΎΡ€ΠΈΠ·ΠΎΠ½Ρ‚Π°, Π΄ΡƒΠ³ΠΎΠΎΠ±Ρ€Π°Π·Π½ΠΎΡΡ‚ΡŒ ΠΊΠΎΠ½Ρ‚ΡƒΡ€Π° Π·Π΅ΠΌΠ½ΠΎΠ³ΠΎ ΡˆΠ°Ρ€Π° Ρ‚Π°ΠΊΠΆΠ΅ Ρ‡ΡƒΡ‚ΡŒ Π·Π°ΠΌΠ΅Ρ‚Π½Π°. Однако ΠΏΡ€ΠΈ ΠΎΡ…Π²Π°Ρ‚Π΅ ΠΏΠΎΠ»Π΅ΠΌ зрСния нСбольшого участка края Π·Π΅ΠΌΠ»ΠΈ Π³ΠΎΡ€ΠΈΠ·ΠΎΠ½Ρ‚ воспринимаСтся Π³ΠΎΡ€ΠΈΠ·ΠΎΠ½Ρ‚Π°Π»ΡŒΠ½Ρ‹ΠΌ. ЗафиксируСм эти ΠΏΡ€ΠΈΠΌΠ΅Ρ€Ρ‹ Π² памяти ΠΈ Π±ΡƒΠ΄Π΅ΠΌ ΡƒΡ‡ΠΈΡ‚Ρ‹Π²Π°Ρ‚ΡŒ ΠΈΡ… ΠΏΡ€ΠΈ рисовании с Π½Π°Ρ‚ΡƒΡ€Ρ‹ ΠΈ создании ΠΊΠΎΠΌΠΏΠΎΠ·ΠΈΡ†ΠΈΠΉ, связанных с ΠΏΠΎΠ΄ΠΎΠ±Π½Ρ‹ΠΌ ΡΡŽΠΆΠ΅Ρ‚ΠΎΠΌ.

Π‘ ΡƒΡ‡Ρ‘Ρ‚ΠΎΠΌ полоТСния Π½Π° ΠΊΠ°Ρ€Ρ‚ΠΈΠ½Π΅ линия Π³ΠΎΡ€ΠΈΠ·ΠΎΠ½Ρ‚Π° ΠΌΠΎΠΆΠ΅Ρ‚ Π±Ρ‹Ρ‚ΡŒ высокой, срСднСй ΠΈ Π½ΠΈΠ·ΠΊΠΎΠΉ. Если ΠΎΠ½Π° находится Π½Π° ΠΎΠ΄ΠΈΠ½Π°ΠΊΠΎΠ²ΠΎΠΌ расстоянии ΠΎΡ‚ Π²Π΅Ρ€Ρ…Π½Π΅Π³ΠΎ ΠΈ Π½ΠΈΠΆΠ½Π΅Π³ΠΎ рая ΠΊΠ°Ρ€Ρ‚ΠΈΠ½Ρ‹, Ρ‚ΠΎ это срСдний Π³ΠΎΡ€ΠΈΠ·ΠΎΠ½Ρ‚.

ΠŸΡ€ΠΈΠ½ΡΡ‚ΠΎ ΡΡ‡ΠΈΡ‚Π°Ρ‚ΡŒ линию Π³ΠΎΡ€ΠΈΠ·ΠΎΠ½Ρ‚Π° высокой ΠΏΡ€ΠΈ располоТСнии Π΅Ρ‘ Π²Ρ‹ΡˆΠ΅ сСрСдины ΠΊΠ°Ρ€Ρ‚ΠΈΠ½Ρ‹ ΠΈ Π½ΠΈΠ·ΠΊΠΎΠΉ, Ссли ΠΎΠ½Π° Π½ΠΈΠΆΠ΅ сСрСдины. ΠŸΡ€ΠΈ этом Π½Π΅ ΠΈΠΌΠ΅Π΅Ρ‚ значСния, Π² ΠΊΠ°ΠΊΠΎΠΌ мСстС Π²Π΅Ρ€Ρ…Π½Π΅ΠΉ ΠΈ Π½ΠΈΠΆΠ½Π΅ΠΉ ΠΏΠΎΠ»ΠΎΠ²ΠΈΠ½Ρ‹ ΠΊΠ°Ρ€Ρ‚ΠΈΠ½Ρ‹ находится высокий ΠΈ Π½ΠΈΠ·ΠΊΠΈΠΉ Π³ΠΎΡ€ΠΈΠ·ΠΎΠ½Ρ‚. Π—Π°ΠΌΠ΅Ρ‚ΠΈΠΌ, Ρ‡Ρ‚ΠΎ Π² Π΄Π°Π½Π½Ρ‹Ρ… ΠΏΡ€ΠΈΠΌΠ΅Ρ€Π°Ρ… Ρ€Π°Π·Π½ΠΎΠ΅ ΠΏΠΎΠ»ΠΎΠΆΠ΅Π½ΠΈΠ΅ Π»ΠΈΠ½ΠΈΠΈ Π³ΠΎΡ€ΠΈΠ·ΠΎΠ½Ρ‚Π° Π½Π° ΠΊΠ°Ρ€Ρ‚ΠΈΠ½Π΅ связано с ΠΈΠ·ΠΌΠ΅Π½Π΅Π½ΠΈΠ΅ΠΌ высоты Ρ‚ΠΎΡ‡ΠΊΠΈ зрСния (Ρ‚ΠΎ Π΅ΡΡ‚ΡŒ Ρ€ΠΈΡΡƒΡŽΡ‰Π΅Π³ΠΎ) ΠΎΡ‚Π½ΠΎΡΠΈΡ‚Π΅Π»ΡŒΠ½ΠΎ ΠΏΡ€Π΅Π΄ΠΌΠ΅Ρ‚Π½ΠΎΠΉ плоскости. РасстояниС, опрСдСляСмоС ΠΏΠΎΠ»ΠΎΠΆΠ΅Π½ΠΈΠ΅ΠΌ Ρ‚ΠΎΡ‡ΠΊΠΈ зрСния Π΄ΠΎ ΠΊΠ°Ρ€Ρ‚ΠΈΠ½Ρ‹, Π² этом случаС Π½Π΅ мСнялось. ΠŸΠΎΡΡ‚ΠΎΠΌΡƒ Π² ΠΊΠ°ΠΆΠ΄ΠΎΠΉ ΠΈΠ· ΠΊΠ°Ρ€Ρ‚ΠΈΠ½ условно заданная Π²Π΅Π»ΠΈΡ‡ΠΈΠ½Π° ΡˆΠΈΡ€ΠΈΠ½Ρ‹ шоссСйной Π΄ΠΎΡ€ΠΎΠ³ΠΈ, Ρƒ ΠΊΠΎΡ‚ΠΎΡ€ΠΎΠΉ стоит Π·Ρ€ΠΈΡ‚Π΅Π»ΡŒ, одинаковая.

Π‘ ΠΈΠ·ΠΌΠ΅Π½Π΅Π½ΠΈΠ΅ΠΌ высоты полоТСния зритСля ΠΈ Π»ΠΈΠ½ΠΈΠΈ Π³ΠΎΡ€ΠΈΠ·ΠΎΠ½Ρ‚Π°, соотвСтствСнно мСняСтся ΠΈ композиция ΠΊΠ°Ρ€Ρ‚ΠΈΠ½Ρ‹. ΠšΠ°ΠΆΠ΄Ρ‹ΠΉ Ρ€Π°Π·, для Π±ΠΎΠ»Π΅Π΅ ΡƒΠ΄Π°Ρ‡Π½ΠΎΠ³ΠΎ размСщСния изобраТСния Π½Π° листС, стоит Π·Π°Π΄ΡƒΠΌΠ°Ρ‚ΡŒΡΡ, ΠΊΠ°ΠΊΠΎΠ΅ ΠΏΠΎΠ»ΠΎΠΆΠ΅Π½ΠΈΠ΅ Π»ΠΈΠ½ΠΈΠΈ Π³ΠΎΡ€ΠΈΠ·ΠΎΠ½Ρ‚Π° Π±ΡƒΠ΄Π΅Ρ‚ Π² Π΄Π°Π½Π½ΠΎΠΌ случаС Π½Π°ΠΈΠ±ΠΎΠ»Π΅Π΅ ΡƒΠ΄Π°Ρ‡Π½Ρ‹ΠΌ.

Π˜Ρ‚Π°ΠΊ, расстояниС ΠΎΡ‚ основания ΠΊΠ°Ρ€Ρ‚ΠΈΠ½Ρ‹ Π΄ΠΎ Π»ΠΈΠ½ΠΈΠΈ Π³ΠΎΡ€ΠΈΠ·ΠΎΠ½Ρ‚Π° опрСдСляСт высоту Ρ‚ΠΎΡ‡ΠΊΠΈ зрСния, Ρ‚. Π΅. ΠΏΠΎΠ»ΠΎΠΆΠ΅Π½ΠΈΠ΅ зритСля ΠΎΡ‚Π½ΠΎΡΠΈΡ‚Π΅Π»ΡŒΠ½ΠΎ ΠΏΡ€Π΅Π΄ΠΌΠ΅Ρ‚Π½ΠΎΠΉ плоскости. Однако Π² Π½Π΅ΠΊΠΎΡ‚ΠΎΡ€Ρ‹Ρ… случаях ΠΏΡ€ΠΈ ΠΈΠ·ΠΎΠ±Ρ€Π°ΠΆΠ΅Π½ΠΈΠΈ ΠΎΠ΄Π½ΠΎΠ³ΠΎ ΠΈ Ρ‚ΠΎΠ³ΠΎ ΠΆΠ΅ ΡΡŽΠΆΠ΅Ρ‚Π° (ΠΏΠ΅ΠΉΠ·Π°ΠΆΠ°, Π½Π°Ρ‚ΡŽΡ€ΠΌΠΎΡ€Ρ‚Π°, ΠΆΠ°Π½Ρ€ΠΎΠ²ΠΎΠΉ ΠΊΠΎΠΌΠΏΠΎΠ·ΠΈΡ†ΠΈΠΈ) Π½Π° ΠΊΠ°Ρ€Ρ‚ΠΈΠ½Π΅ ΠΌΠΎΠΆΠ½ΠΎ Π·Π°Π΄Π°Ρ‚ΡŒ Ρ€Π°Π·Π½ΠΎΠ΅ ΠΏΠΎΠ»ΠΎΠΆΠ΅Π½ΠΈΠ΅ Π»ΠΈΠ½ΠΈΠΈ Π³ΠΎΡ€ΠΈΠ·ΠΎΠ½Ρ‚Π°, сохранив Π½Π΅ΠΈΠ·ΠΌΠ΅Π½Π½ΠΎΠΉ высоту Ρ‚ΠΎΡ‡ΠΊΠΈ зрСния. Π’ этом случаС измСняСтся расстояниС ΠΎΡ‚ зритСля Π΄ΠΎ ΠΊΠ°Ρ€Ρ‚ΠΈΠ½Ρ‹. Рассмотрим ΠΏΡ€ΠΈΠΌΠ΅Ρ€.

На ΠΊΠ°Ρ€Ρ‚ΠΈΠ½Π΅ ΠΈΠ·ΠΎΠ±Ρ€Π°ΠΆΠ΅Π½Π° ΠΎΠΊΡ€Π°ΠΈΠ½Π° Π³ΠΎΡ€ΠΎΠ΄Π°, которая ΠΎΠ³Ρ€Π°Π½ΠΈΡ‡Π΅Π½Π° трСмя Ρ€Π°ΠΌΠΊΠ°ΠΌΠΈ ΠΎΠ΄ΠΈΠ½Π°ΠΊΠΎΠ²ΠΎΠ³ΠΎ Ρ€Π°Π·ΠΌΠ΅Ρ€Π°. Π’ связи с ΡƒΠ΄Π°Π»Π΅Π½ΠΈΠ΅ΠΌ ΠΊΠ°Ρ€Ρ‚ΠΈΠ½Ρ‹ мСняСтся расстояниС ΠΎΡ‚ Π½Π΅Ρ‘ Π΄ΠΎ зритСля ΠΈ ΠΏΠΎΠ»ΠΎΠΆΠ΅Π½ΠΈΠ΅ Π½Π° Π½Π΅ΠΉ Π»ΠΈΠ½ΠΈΠΈ Π³ΠΎΡ€ΠΈΠ·ΠΎΠ½Ρ‚Π° β€” высокая, срСдняя, низкая.
ВмСстС с Ρ‚Π΅ΠΌ высота Ρ‚ΠΎΡ‡ΠΊΠΈ зрСния ΠΎΡΡ‚Π°Π²Π°Π»Π°ΡΡŒ Π½Π΅ΠΈΠ·ΠΌΠ΅Π½Π½ΠΎΠΉ, Ρ‚Π°ΠΊ ΠΊΠ°ΠΊ Π·Ρ€ΠΈΡ‚Π΅Π»ΡŒ находился Π½Π° ΠΎΠ΄Π½ΠΎΠΌ ΠΈ Ρ‚ΠΎΠΌ ΠΆΠ΅ мСстС.

Рассмотрим Π΄Ρ€ΡƒΠ³ΠΎΠΉ ΠΏΡ€ΠΈΠΌΠ΅Ρ€. На Π΄Π²ΡƒΡ… ΠΊΠ°Ρ€Ρ‚ΠΈΠ½Π°Ρ… ΠΈΠ·ΠΎΠ±Ρ€Π°ΠΆΡ‘Π½ ΠΎΠ΄ΠΈΠ½ ΠΈ Ρ‚ΠΎΡ‚ ΠΆΠ΅ Π½Π°Ρ‚ΡŽΡ€ΠΌΠΎΡ€Ρ‚, ΠΊΠΎΡ‚ΠΎΡ€Ρ‹ΠΉ ΠΎΠ³Ρ€Π°Π½ΠΈΡ‡Π΅Π½ Ρ€Π°ΠΌΠΊΠΎΠΉ ΠΎΠ΄ΠΈΠ½Π°ΠΊΠΎΠ²ΠΎΠ³ΠΎ Ρ€Π°Π·ΠΌΠ΅Ρ€Π°, Π½ΠΎ с Ρ€Π°Π·Π»ΠΈΡ‡Π½Ρ‹ΠΌ Π΅Ρ‘ ΠΏΠΎΠ»ΠΎΠΆΠ΅Π½ΠΈΠ΅ΠΌ.

На Π³ΠΎΡ€ΠΈΠ·ΠΎΠ½Ρ‚Π°Π»ΡŒΠ½ΠΎΠΉ ΠΊΠ°Ρ€Ρ‚ΠΈΠ½Π΅ β€” высокий Π³ΠΎΡ€ΠΈΠ·ΠΎΠ½Ρ‚, ΠΏΠΎΡΠΊΠΎΠ»ΡŒΠΊΡƒ ΠΎΠ½ располоТСн Π±Π»ΠΈΠΆΠ΅ ΠΊ Π²Π΅Ρ€Ρ…Π½Π΅ΠΌΡƒ ΠΊΡ€Π°ΡŽ Ρ€Π°ΠΌΠΊΠΈ, Π° Π½Π° Π²Π΅Ρ€Ρ‚ΠΈΠΊΠ°Π»ΡŒΠ½ΠΎΠΉ β€” срСдний. ВмСстС с Ρ‚Π΅ΠΌ, высота Ρ‚ΠΎΡ‡ΠΊΠΈ зрСния Π½Π΅ мСнялась, Π° ΡΠ»Π΅Π΄ΠΎΠ²Π°Ρ‚Π΅Π»ΡŒΠ½ΠΎ, ΠΈ ΠΏΠΎΠ»ΠΎΠΆΠ΅Π½ΠΈΠ΅ Π»ΠΈΠ½ΠΈΠΈ Π³ΠΎΡ€ΠΈΠ·ΠΎΠ½Ρ‚Π° Π½Π° ΠΊΠ°Ρ€Ρ‚ΠΈΠ½Π΅ ΠΎΡ‚Π½ΠΎΡΠΈΡ‚Π΅Π»ΡŒΠ½ΠΎ ΠΏΡ€Π΅Π΄ΠΌΠ΅Ρ‚ΠΎΠ² Π½Π°Ρ‚ΡŽΡ€ΠΌΠΎΡ€Ρ‚Π°, Ρ‚. ΠΊ. Ρ€ΠΈΡΡƒΡŽΡ‰ΠΈΠΉ находился Π½Π° ΠΎΠ΄Π½ΠΎΠΌ ΠΈ Ρ‚ΠΎΠΌ ΠΆΠ΅ мСстС ΠΈ ΡΠΎΡ…Ρ€Π°Π½ΠΈΠ»ΠΎΡΡŒ Π½Π΅ΠΈΠ·ΠΌΠ΅Π½Π½Ρ‹ΠΌ дистанционноС расстояниС.

ПолоТСниС Π³ΠΎΡ€ΠΈΠ·ΠΎΠ½Ρ‚Π° Π½Π° ΠΊΠ°Ρ€Ρ‚ΠΈΠ½Π΅ ΠΈ высота зритСля ΠΎΡ‚Π½ΠΎΡΠΈΡ‚Π΅Π»ΡŒΠ½ΠΎ ΠΈΠ·ΠΎΠ±Ρ€Π°ΠΆΡ‘Π½Π½Ρ‹Ρ… ΠΏΡ€Π΅Π΄ΠΌΠ΅Ρ‚ΠΎΠ² ΠΈΠ½ΠΎΠ³Π΄Π° ΠΌΠΎΠ³ΡƒΡ‚ Π½Π΅ ΡΠΎΠ²ΠΏΠ°Π΄Π°Ρ‚ΡŒ. Π’ рассмотрСнных Π²Ρ‹ΡˆΠ΅ ΠΏΡ€ΠΈΠΌΠ΅Ρ€Π°Ρ… ΠΎΠΏΡ€Π΅Π΄Π΅Π»Π΅Π½ΠΈΠ΅ высоты Π³ΠΎΡ€ΠΈΠ·ΠΎΠ½Ρ‚Π° Π΄Π°Π½ΠΎ с ΡƒΡ‡Ρ‘Ρ‚ΠΎΠΌ Π΅Ρ‘ полоТСния Π½Π° ΠΊΠ°Ρ€Ρ‚ΠΈΠ½Π΅. На ΠΏΡ€Π°ΠΊΡ‚ΠΈΠΊΠ΅ ΠΏΡ€ΠΈ рисовании с Π½Π°Ρ‚ΡƒΡ€Ρ‹ высоту Π»ΠΈΠ½ΠΈΠΈ Π³ΠΎΡ€ΠΈΠ·ΠΎΠ½Ρ‚Π° ΠΈΠ½ΠΎΠ³Π΄Π° ΠΎΠΏΡ€Π΅Π΄Π΅Π»ΡΡŽΡ‚ ΠΈΠ½Π°Ρ‡Π΅ β€” Π½Π° основС полоТСния зритСля ΠΎΡ‚Π½ΠΎΡΠΈΡ‚Π΅Π»ΡŒΠ½ΠΎ ΠΈΠ·ΠΎΠ±Ρ€Π°ΠΆΠ°Π΅ΠΌΡ‹Ρ… ΠΏΡ€Π΅Π΄ΠΌΠ΅Ρ‚ΠΎΠ². Π’Π°ΠΊ, Π½Π°ΠΏΡ€ΠΈΠΌΠ΅Ρ€ ΠΏΠ΅ΠΉΠ·Π°ΠΆ (см. ΠΈΠ»Π».28) обозрСваСтся с высокого мСста располоТСния зритСля. Однако, выбирая с ΡƒΡ‡Ρ‘Ρ‚ΠΎΠΌ ΠΊΠΎΠΌΠΏΠΎΠ·ΠΈΡ†ΠΈΠΈ ΠΏΠΎΠ»ΠΎΠΆΠ΅Π½ΠΈΠ΅ ΠΊΠ°Ρ€Ρ‚ΠΈΠ½Ρ‹, линия Π³ΠΎΡ€ΠΈΠ·ΠΎΠ½Ρ‚Π° Π½Π° Π½Π΅ΠΉ ΠΌΠΎΠΆΠ΅Ρ‚ Π±Ρ‹Ρ‚ΡŒ срСднСй ΠΈ Π½ΠΈΠ·ΠΊΠΎΠΉ. Π’ этом случаС ΠΈ происходит «нСсовпадСниС! Высокой Ρ‚ΠΎΡ‡ΠΊΠΈ зрСния с ΠΏΠΎΠ»ΠΎΠΆΠ΅Π½ΠΈΠ΅ΠΌ Π³ΠΎΡ€ΠΈΠ·ΠΎΠ½Ρ‚Π° Π½Π° ΠΊΠ°Ρ€Ρ‚ΠΈΠ½Π΅.

ПолСзно Π·Π½Π°Ρ‚ΡŒ, ΠΊΠ°ΠΊ осущСствляСтся поиск ΡƒΠ΄Π°Ρ‡Π½ΠΎΠ³ΠΎ размСщСния ΠΎΠ±ΡŠΠ΅ΠΊΡ‚Π° Π½Π° листС. ΠŸΡ€ΠΈ рисовании с Π½Π°Ρ‚ΡƒΡ€Ρ‹ извСстСн практичСский ΠΏΡ€ΠΈΡ‘ΠΌ использования видоискатСля для опрСдСлСния полоТСния листа Π±ΡƒΠΌΠ°Π³ΠΈ с располоТСниСм Π½Π° Π½Ρ‘ΠΌ ΠΈΠ·ΠΎΠ±Ρ€Π°ΠΆΠ°Π΅ΠΌΡ‹Ρ… ΠΎΠ±ΡŠΠ΅ΠΊΡ‚ΠΎΠ², дистанционного расстояния ΠΈ высоты Π»ΠΈΠ½ΠΈΠΈ Π³ΠΎΡ€ΠΈΠ·ΠΎΠ½Ρ‚Π°. Для этого Π½Π° Π±ΡƒΠΌΠ°Π³Π΅ Π²Ρ‹Ρ€Π΅Π·Π°ΡŽΡ‚ ΠΏΡ€ΡΠΌΠΎΡƒΠ³ΠΎΠ»ΡŒΠ½ΠΈΠΊ, стороны ΠΊΠΎΡ‚ΠΎΡ€ΠΎΠ³ΠΎ ΠΏΡ€ΠΎΠΏΠΎΡ€Ρ†ΠΈΠΎΠ½Π°Π»ΡŒΠ½Ρ‹ листу, Π²Ρ‹Π±Ρ€Π°Π½Π½ΠΎΠΌΡƒ для рисования. Π”Π΅Ρ€ΠΆΠ° Π² Ρ€ΡƒΠΊΠ°Ρ… Π²ΠΈΠ΄ΠΎΠΈΡΠΊΠ°Ρ‚Π΅Π»ΡŒ ΠΈ направляя Π΅Π³ΠΎ Π½Π° ΠΈΠ·ΠΎΠ±Ρ€Π°ΠΆΠ°Π΅ΠΌΡ‹ΠΉ ΠΎΠ±ΡŠΠ΅ΠΊΡ‚, Ρ€ΠΈΡΡƒΡŽΡ‰ΠΈΠΉ фиксируСт Ρ‡Π΅Ρ€Π΅Π· ΠΏΡ€ΡΠΌΠΎΡƒΠ³ΠΎΠ»ΡŒΠ½ΠΎΠ΅ отвСрстиС Π½Π°ΠΈΠ±ΠΎΠ»Π΅Π΅ ΡƒΠ΄Π°Ρ‡Π½ΠΎΠ΅ ΠΊΠΎΠΌΠΏΠΎΠ·ΠΈΡ†ΠΈΠΎΠ½Π½ΠΎΠ΅ Ρ€Π°Π·ΠΌΠ΅Ρ‰Π΅Π½ΠΈΠ΅ ΠΏΡ€Π΅Π΄ΠΌΠ΅Ρ‚ΠΎΠ² ΠΏΡ€ΠΈ Π³ΠΎΡ€ΠΈΠ·ΠΎΠ½Ρ‚Π°Π»ΡŒΠ½ΠΎΠΌ ΠΈΠ»ΠΈ Π²Π΅Ρ€Ρ‚ΠΈΠΊΠ°Π»ΡŒΠ½ΠΎΠΌ ΠΏΠΎΠ»ΠΎΠΆΠ΅Π½ΠΈΠΈ листа с ΡƒΡ‡Ρ‘Ρ‚ΠΎΠΌ высоты Π»ΠΈΠ½ΠΈΠΈ Π³ΠΎΡ€ΠΈΠ·ΠΎΠ½Ρ‚Π°.

ΠŸΡ€ΠΈ рисовании с Π½Π°Ρ‚ΡƒΡ€Ρ‹ Π½ΡƒΠΆΠ½ΠΎ ΡƒΠΌΠ΅Ρ‚ΡŒ ΠΏΡ€Π°Π²ΠΈΠ»ΡŒΠ½ΠΎ ΠΎΠΏΡ€Π΅Π΄Π΅Π»ΡΡ‚ΡŒ линию Π³ΠΎΡ€ΠΈΠ·ΠΎΠ½Ρ‚Π°, ΠΎΡ‚Π½ΠΎΡΠΈΡ‚Π΅Π»ΡŒΠ½ΠΎ ΠΈΠ·ΠΎΠ±Ρ€Π°ΠΆΠ°Π΅ΠΌΡ‹Ρ… ΠΏΡ€Π΅Π΄ΠΌΠ΅Ρ‚ΠΎΠ² ΠΈ Π·Π°Π΄Π°Π²Π°Ρ‚ΡŒ Π΅Ρ‘ ΠΏΠΎΠ»ΠΎΠΆΠ΅Π½ΠΈΠ΅ Π½Π° ΠΊΠ°Ρ€Ρ‚ΠΈΠ½Π΅. На Ρ€Π°Π²Π½ΠΈΠ½Π½ΠΎΠΉ мСстности ΠΏΡ€ΠΈ рисовании ΠΏΠ΅ΠΉΠ·Π°ΠΆΠ° линия Π³ΠΎΡ€ΠΈΠ·ΠΎΠ½Ρ‚Π° Ρ‡Ρ‘Ρ‚ΠΊΠ° Π²ΠΈΠ΄Π½Π°, ΠΊΠ°ΠΊ Π³Ρ€Π°Π½ΠΈΡ†Π° ΠΌΠ΅ΠΆΠ΄Ρƒ Π½Π΅Π±ΠΎΠΌ ΠΈ Π²ΠΈΠ΄ΠΈΠΌΠΎΠΉ Ρ‡Π°ΡΡ‚ΡŒΡŽ Π·Π΅ΠΌΠ»ΠΈ. Если ΠΎΠ½Π° Π·Π°ΠΊΡ€Ρ‹Ρ‚Π° ΠΊΠ°ΠΊΠΈΠΌΠΈ-Π»ΠΈΠ±ΠΎ ΠΎΠ±ΡŠΠ΅ΠΊΡ‚Π°ΠΌΠΈ, Ρ‚ΠΎΠ³Π΄Π° Π΅Ρ‘ находят с ΠΏΠΎΠΌΠΎΡ‰ΡŒΡŽ стакана с Π²ΠΎΠ΄ΠΎΠΉ, поднятого Π½Π° Ρ‚Π°ΠΊΡƒΡŽ высоту, Ρ‡Ρ‚ΠΎΠ±Ρ‹ ΡƒΡ€ΠΎΠ²Π΅Π½ΡŒ Π²ΠΎΠ΄Ρ‹ Π±Ρ‹Π» Π²ΠΈΠ΄Π΅Π½ ΠΊΠ°ΠΊ прямая линия, которая Π·Ρ€ΠΈΡ‚Π΅Π»ΡŒΠ½ΠΎ ΠΈ опрСдСляСт ΠΏΠΎΠ»ΠΎΠΆΠ΅Π½ΠΈΠ΅ Π³ΠΎΡ€ΠΈΠ·ΠΎΠ½Ρ‚Π°. Π­Ρ‚ΠΎΡ‚ ΠΆΠ΅ ΠΏΡ€ΠΈΡ‘ΠΌ ΠΈΡΠΏΠΎΠ»ΡŒΠ·ΡƒΡŽΡ‚ Π² ΠΏΠΎΠΌΠ΅Ρ‰Π΅Π½ΠΈΠΈ ΠΏΡ€ΠΈ рисовании с Π½Π°Ρ‚ΡƒΡ€Ρ‹.

Π˜Ρ‚Π°ΠΊ, ΠΊΠ°ΠΊ ΠΆΠ΅ ΠΏΡ€Π°Π²ΠΈΠ»ΡŒΠ½ΠΎ Π·Π°Π΄Π°Ρ‚ΡŒ Π½Π° ΠΊΠ°Ρ€Ρ‚ΠΈΠ½Π΅ линию Π³ΠΎΡ€ΠΈΠ·ΠΎΠ½Ρ‚Π° ΠΈ ΠΎΡ‚Ρ‡Π΅Π³ΠΎ это зависит? Высоту Π»ΠΈΠ½ΠΈΠΈ Π³ΠΎΡ€ΠΈΠ·ΠΎΠ½Ρ‚Π° Ρ€ΠΈΡΡƒΡŽΡ‰ΠΈΠΉ Π²Ρ‹Π±ΠΈΡ€Π°Π΅Ρ‚ с ΡƒΡ‡Ρ‘Ρ‚ΠΎΠΌ Π·Π°Π΄Π°Ρ‡, ΠΊΠΎΡ‚ΠΎΡ€Ρ‹Π΅ ΠΏΠ΅Ρ€Π΅Π΄ Π½ΠΈΠΌ поставлСны. Π’ зависимости ΠΎΡ‚ Ρ‚ΠΎΠ³ΠΎ, рисуСт Π»ΠΈ ΠΎΠ½ с Π½Π°Ρ‚ΡƒΡ€Ρ‹ ΠΈΠ»ΠΈ ΠΏΠΎ памяти, Ρ€Π°Π±ΠΎΡ‚Π°Π΅Ρ‚ Π»ΠΈ Π½Π°Π΄ созданиСм творчСской ΠΊΠΎΠΌΠΏΠΎΠ·ΠΈΡ†ΠΈΠΈ ΠΈΠ»ΠΈ выполняСт пСрспСктивноС ΠΈΠ·ΠΎΠ±Ρ€Π°ΠΆΠ΅Π½ΠΈΠ΅ ΠΏΠΎ Ρ‡Π΅Ρ€Ρ‚Π΅ΠΆΡƒ ΠΎΠ±ΡŠΠ΅ΠΊΡ‚Π° (ΠΏΠ»Π°Π½Ρƒ ΠΈ фасаду здания, Π½Π°ΠΏΡ€ΠΈΠΌΠ΅Ρ€) β€” Π½Π° основС этого ΠΈ опрСдСляСтся высота Ρ‚ΠΎΡ‡ΠΊΠΈ зрСния ΠΈ ΠΏΠΎΠ»ΠΎΠΆΠ΅Π½ΠΈΠ΅ Π»ΠΈΠ½ΠΈΠΈ Π³ΠΎΡ€ΠΈΠ·ΠΎΠ½Ρ‚Π° Π½Π° ΠΊΠ°Ρ€Ρ‚ΠΈΠ½Π΅.

Высокий Π³ΠΎΡ€ΠΈΠ·ΠΎΠ½Ρ‚, ΠΊΠ°ΠΊ ΠΏΡ€Π°Π²ΠΈΠ»ΠΎ, Π²Ρ‹Π±ΠΈΡ€Π°ΡŽΡ‚ Π² ΠΏΠ΅ΠΉΠ·Π°ΠΆΠ΅ с ΠΈΠ·ΠΎΠ±Ρ€Π°ΠΆΠ΅Π½ΠΈΠ΅ΠΌ просторов бСскрайних стСпСй ΠΈ ΠΏΠΎΠ»Π΅ΠΉ, лСсных массивов ΠΈ Ρ€Π΅Ρ‡Π½Ρ‹Ρ… Π΄Π°Π»Π΅ΠΉ для выявлСния большСй Π³Π»ΡƒΠ±ΠΈΠ½Ρ‹ пространства. Π•Π³ΠΎ ΠΈΡΠΏΠΎΠ»ΡŒΠ·ΡƒΡŽΡ‚ ΠΏΡ€ΠΈ ΠΈΠ·ΠΎΠ±Ρ€Π°ΠΆΠ΅Π½ΠΈΠΈ ΠΏΠ°Π½ΠΎΡ€Π°ΠΌΡ‹ Π³ΠΎΡ€ΠΎΠ΄Π° с высоты Β«ΠΏΡ‚ΠΈΡ‡ΡŒΠ΅Π³ΠΎ ΠΏΠΎΠ»Ρ‘Ρ‚Π°Β» ΠΈΠ»ΠΈ ΡƒΠ»ΠΈΡ†Ρ‹ ΠΏΡ€ΠΈ ΠΎΠ±ΠΎΠ·Ρ€Π΅Π½ΠΈΠΈ Π΅Ρ‘ с высокого мСста полоТСния зритСля.
Низкий Π³ΠΎΡ€ΠΈΠ·ΠΎΠ½Ρ‚ ΠΈΡΠΏΠΎΠ»ΡŒΠ·ΡƒΡŽΡ‚ Π² ΠΏΠ΅ΠΉΠ·Π°ΠΆΠ΅ для ΠΏΠΎΠΊΠ°Π·Π° большого пространства Π½Π΅Π±Π° с Π³Ρ€ΠΎΠ·ΠΎΠ²Ρ‹ΠΌΠΈ Ρ‚ΡƒΡ‡Π°ΠΌΠΈ ΠΈΠ»ΠΈ с освСщёнными солнцСм ΠΎΠ±Π»Π°ΠΊΠ°ΠΌΠΈ.

Π’ сочСтании с ΡƒΠ΄Π»ΠΈΠ½Ρ‘Π½Π½ΠΎΡΡ‚ΡŒΡŽ ΠΊΠ°Ρ€Ρ‚ΠΈΠ½Ρ‹ Π½ΠΈΠ·ΠΊΠΈΠΉ Π³ΠΎΡ€ΠΈΠ·ΠΎΠ½Ρ‚ создаёт Π²ΠΏΠ΅Ρ‡Π°Ρ‚Π»Π΅Π½ΠΈΠ΅ Π΅Ρ‘ панорамности, ΠΎΡ…Π²Π°Ρ‚Π° большого пространства ΠΈ нСпрСрывности двиТСния ΠΎΠ±ΡŠΠ΅ΠΊΡ‚ΠΎΠ² ΠΏΡ€ΠΈ ΠΈΠ·ΠΎΠ±Ρ€Π°ΠΆΠ΅Π½ΠΈΠΈ ΠΊΠ°ΠΊΠΎΠ³ΠΎ-Π»ΠΈΠ±ΠΎ Ρ…Π°Ρ€Π°ΠΊΡ‚Π΅Ρ€Π½ΠΎΠ³ΠΎ ΡΡŽΠΆΠ΅Ρ‚Π° (скачки, Π°Π²Ρ‚ΠΎΡ€Π°Π»Π»ΠΈ, Π±Π΅Π³ΡƒΡ‰ΠΈΠ΅ Π»Ρ‹ΠΆΠ½ΠΈΠΊΠΈ ΠΈ ΠΏΡ€.)

Низкий Π³ΠΎΡ€ΠΈΠ·ΠΎΠ½Ρ‚ Ρ…ΡƒΠ΄ΠΎΠΆΠ½ΠΈΠΊΠΈ ΠΈΡΠΏΠΎΠ»ΡŒΠ·ΡƒΡŽΡ‚ ΠΏΡ€ΠΈ Π²Π΅Ρ€Ρ‚ΠΈΠΊΠ°Π»ΡŒΠ½ΠΎΠΌ располоТСнии ΠΊΠ°Ρ€Ρ‚ΠΈΠ½Ρ‹ для придания ΠΌΠΎΠ½ΡƒΠΌΠ΅Π½Ρ‚Π°Π»ΡŒΠ½ΠΎΡΡ‚ΠΈ высоким ΠΎΠ±ΡŠΠ΅ΠΊΡ‚Π°ΠΌ ΠΈΠ»ΠΈ для ΠΏΠ΅Ρ€Π΅Π΄Π°Ρ‡ΠΈ вСличия пСрсонаТа.

ЭлСктричСская Ρ†Π΅ΠΏΡŒ, состоящая ΠΈΠ· Ρ‡Π΅Ρ‚Ρ‹Ρ€Π΅Ρ… прямолинСйных Π³ΠΎΡ€ΠΈΠ·ΠΎΠ½Ρ‚Π°Π»ΡŒΠ½Ρ‹Ρ… ΠΏΡ€ΠΎΠ²ΠΎΠ΄Π½ΠΈΠΊΠΎΠ² (1-2, 2-3, 3-4, 4-1) ΠΈ источника постоянного Ρ‚ΠΎΠΊΠ°, находится Π² ΠΎΠ΄Π½ΠΎΡ€ΠΎΠ΄Π½ΠΎΠΌ ΠΌΠ°Π³Π½ΠΈΡ‚Π½ΠΎΠΌ ΠΏΠΎΠ»Π΅, Π²Π΅ΠΊΡ‚ΠΎΡ€ ΠΌΠ°Π³Π½ΠΈΡ‚Π½ΠΎΠΉ ΠΈΠ½Π΄ΡƒΠΊΡ†ΠΈΠΈ ΠΊΠΎΡ‚ΠΎΡ€ΠΎΠ³ΠΎ Π’ Π½Π°ΠΏΡ€Π°Π²Π»Π΅Π½ Π²Π΅Ρ€Ρ‚ΠΈΠΊΠ°Π»ΡŒΠ½ΠΎ Π²Π½ΠΈΠ· (см. рисунок, Π²ΠΈΠ΄ свСрху). ΠšΡƒΠ΄Π° Π½Π°ΠΏΡ€Π°Π²Π»Π΅Π½Π° сила АмпСра, Π΄Π΅ΠΉΡΡ‚Π²ΡƒΡŽΡ‰Π°Ρ Π½Π° ΠΏΡ€ΠΎΠ²ΠΎΠ΄Π½ΠΈΠΊ 1-2?

лСктричСская Ρ†Π΅ΠΏΡŒ, состоящая ΠΈΠ· Ρ‡Π΅Ρ‚Ρ‹Ρ€Ρ‘Ρ… прямолинСйных Π³ΠΎΡ€ΠΈΠ·ΠΎΠ½Ρ‚Π°Π»ΡŒΠ½Ρ‹Ρ… ΠΏΡ€ΠΎΠ²ΠΎΠ΄Π½ΠΈΠΊΠΎΠ² (1-2, 2-3, 3-4, 4-1) ΠΈ источника постоянного Ρ‚ΠΎΠΊΠ°, находится Π² ΠΎΠ΄Π½ΠΎΡ€ΠΎΠ΄Π½ΠΎΠΌ ΠΌΠ°Π³Π½ΠΈΡ‚Π½ΠΎΠΌ ΠΏΠΎΠ»Π΅, Π½Π°ΠΏΡ€Π°Π²Π»Π΅Π½Π½ΠΎΠΌ Π²Π΅Ρ€Ρ‚ΠΈΠΊΠ°Π»ΡŒΠ½ΠΎ Π²Π½ΠΈΠ· (см. рисунок, Π²ΠΈΠ΄ свСрху). Как Π½Π°ΠΏΡ€Π°Π²Π»Π΅Π½Π° ΠΎΡ‚Π½ΠΎΡΠΈΡ‚Π΅Π»ΡŒΠ½ΠΎ рисунка (Π²ΠΏΡ€Π°Π²ΠΎ, Π²Π»Π΅Π²ΠΎ, Π²Π²Π΅Ρ€Ρ…, Π²Π½ΠΈΠ·, ΠΊ Π½Π°Π±Π»ΡŽΠ΄Π°Ρ‚Π΅Π»ΡŽ, ΠΎΡ‚ Π½Π°Π±Π»ΡŽΠ΄Π°Ρ‚Π΅Π»Ρ) вызванная этим ΠΏΠΎΠ»Π΅ΠΌ сила АмпСра, Π΄Π΅ΠΉΡΡ‚Π²ΡƒΡŽΡ‰Π°Ρ Π½Π° ΠΏΡ€ΠΎΠ²ΠΎΠ΄Π½ΠΈΠΊ 2-3? ΠžΡ‚Π²Π΅Ρ‚ Π·Π°ΠΏΠΈΡˆΠΈΡ‚Π΅ словом (словами).

ΠŸΡ€ΡΠΌΠΎΠ»ΠΈΠ½Π΅ΠΉΠ½Ρ‹ΠΉ ΠΏΡ€ΠΎΠ²ΠΎΠ΄Π½ΠΈΠΊ Π΄Π»ΠΈΠ½ΠΎΠΉ l = 0,1 ΠΌ, ΠΏΠΎ ΠΊΠΎΡ‚ΠΎΡ€ΠΎΠΌΡƒ Ρ‚Π΅Ρ‡Ρ‘Ρ‚ Ρ‚ΠΎΠΊ I = 2 А, располоТСн Π² ΠΎΠ΄Π½ΠΎΡ€ΠΎΠ΄Π½ΠΎΠΌ ΠΌΠ°Π³Π½ΠΈΡ‚Π½ΠΎΠΌ ΠΏΠΎΠ»Π΅ ΠΏΠΎΠ΄ ΡƒΠ³Π»ΠΎΠΌ 90Β° ΠΊ Π²Π΅ΠΊΡ‚ΠΎΡ€Ρƒ B. Каков ΠΌΠΎΠ΄ΡƒΠ»ΡŒ ΠΈΠ½Π΄ΡƒΠΊΡ†ΠΈΠΈ ΠΌΠ°Π³Π½ΠΈΡ‚Π½ΠΎΠ³ΠΎ поля Π’, Ссли сила, Π΄Π΅ΠΉΡΡ‚Π²ΡƒΡŽΡ‰Π°Ρ Π½Π° ΠΏΡ€ΠΎΠ²ΠΎΠ΄Π½ΠΈΠΊ со стороны ΠΌΠ°Π³Π½ΠΈΡ‚Π½ΠΎΠ³ΠΎ поля, Ρ€Π°Π²Π½Π° 0,2 Н?

4. Π’ ΠΎΠ΄Π½ΠΎΡ€ΠΎΠ΄Π½ΠΎΠΌ ΠΌΠ°Π³Π½ΠΈΡ‚Π½ΠΎΠΌ ΠΏΠΎΠ»Π΅ ΠΏΠΎ Π²Π΅Ρ€Ρ‚ΠΈΠΊΠ°Π»ΡŒΠ½Ρ‹ΠΌ Π½Π°ΠΏΡ€Π°Π²Π»ΡΡŽΡ‰ΠΈΠΌ Π±Π΅Π· трСния ΡΠΊΠΎΠ»ΡŒΠ·ΠΈΡ‚ прямой Π³ΠΎΡ€ΠΈΠ·ΠΎΠ½Ρ‚Π°Π»ΡŒΠ½Ρ‹ΠΉ ΠΏΡ€ΠΎΠ²ΠΎΠ΄Π½ΠΈΠΊ массой 0,2 ΠΊΠ³, ΠΏΠΎ ΠΊΠΎΡ‚ΠΎΡ€ΠΎΠΌΡƒ Ρ‚Π΅Ρ‡Ρ‘Ρ‚ Ρ‚ΠΎΠΊ 2 А. Π’Π΅ΠΊΡ‚ΠΎΡ€ ΠΌΠ°Π³Π½ΠΈΡ‚Π½ΠΎΠΉ ΠΈΠ½Π΄ΡƒΠΊΡ†ΠΈΠΈ Π½Π°ΠΏΡ€Π°Π²Π»Π΅Π½ Π³ΠΎΡ€ΠΈΠ·ΠΎΠ½Ρ‚Π°Π»ΡŒΠ½ΠΎ пСрпСндикулярно ΠΏΡ€ΠΎΠ²ΠΎΠ΄Π½ΠΈΠΊΡƒ (см. рисунок), Π’ = 2 Π’Π». Π§Π΅ΠΌΡƒ Ρ€Π°Π²Π½Π° Π΄Π»ΠΈΠ½Π° ΠΏΡ€ΠΎΠ²ΠΎΠ΄Π½ΠΈΠΊΠ°, Ссли извСстно, Ρ‡Ρ‚ΠΎ ускорСниС ΠΏΡ€ΠΎΠ²ΠΎΠ΄Π½ΠΈΠΊΠ° Π½Π°ΠΏΡ€Π°Π²Π»Π΅Π½ΠΎ Π²Π½ΠΈΠ· ΠΈ Ρ€Π°Π²Π½ΠΎ 2 ΠΌ/с2?

ΠŸΡ€ΡΠΌΠΎΠ»ΠΈΠ½Π΅ΠΉΠ½Ρ‹ΠΉ ΠΏΡ€ΠΎΠ²ΠΎΠ΄Π½ΠΈΠΊ, ΠΏΠΎ ΠΊΠΎΡ‚ΠΎΡ€ΠΎΠΌΡƒ Ρ‚Π΅Ρ‡Ρ‘Ρ‚ Ρ‚ΠΎΠΊ, Ρ€Π°Π²Π½Ρ‹ΠΉ 3 А, располоТСн Π² ΠΎΠ΄Π½ΠΎΡ€ΠΎΠ΄Π½ΠΎΠΌ ΠΌΠ°Π³Π½ΠΈΡ‚Π½ΠΎΠΌ ΠΏΠΎΠ»Π΅ с ΠΈΠ½Π΄ΡƒΠΊΡ†ΠΈΠ΅ΠΉ Π’ = 0,4 Π’Π» ΠΏΠΎΠ΄ ΡƒΠ³Π»ΠΎΠΌ 30Β° ΠΊ Π²Π΅ΠΊΡ‚ΠΎΡ€Ρƒ Π’. ΠœΠΎΠ΄ΡƒΠ»ΡŒ силы, Π΄Π΅ΠΉΡΡ‚Π²ΡƒΡŽΡ‰Π΅ΠΉ Π½Π° ΠΏΡ€ΠΎΠ²ΠΎΠ΄Π½ΠΈΠΊ со стороны ΠΌΠ°Π³Π½ΠΈΡ‚Π½ΠΎΠ³ΠΎ поля, Ρ€Π°Π²Π΅Π½ 0,3 Н. Какова Π΄Π»ΠΈΠ½Π° ΠΏΡ€ΠΎΠ²ΠΎΠ΄Π½ΠΈΠΊΠ°?

Π’ ΠΎΠ΄Π½ΠΎΡ€ΠΎΠ΄Π½ΠΎΠ΅ ΠΌΠ°Π³Π½ΠΈΡ‚Π½ΠΎΠ΅ ΠΏΠΎΠ»Π΅ с ΠΈΠ½Π΄ΡƒΠΊΡ†ΠΈΠ΅ΠΉ 0,085 Π’Π» Π²Π»Π΅Ρ‚Π°Π΅Ρ‚ элСктрон со ΡΠΊΠΎΡ€ΠΎΡΡ‚ΡŒΡŽ 4,6Β·10 7 ΠΌ/с, Π½Π°ΠΏΡ€Π°Π²Π»Π΅Π½Π½ΠΎΠΉ пСрпСндикулярно линиям ΠΈΠ½Π΄ΡƒΠΊΡ†ΠΈΠΈ поля. ΠžΠΏΡ€Π΅Π΄Π΅Π»ΠΈΡ‚Π΅ радиус окруТности, ΠΏΠΎ ΠΊΠΎΡ‚ΠΎΡ€ΠΎΠΉ двиТСтся элСктрон.

Π­Π»Π΅ΠΊΡ‚Ρ€ΠΎΠ½ двиТСтся Π² ΠΎΠ΄Π½ΠΎΡ€ΠΎΠ΄Π½ΠΎΠΌ ΠΌΠ°Π³Π½ΠΈΡ‚Π½ΠΎΠΌ ΠΏΠΎΠ»Π΅ Π² Π²Π°ΠΊΡƒΡƒΠΌΠ΅ пСрпСндикулярно линиям ΠΈΠ½Π΄ΡƒΠΊΡ†ΠΈΠΈ ΠΏΠΎ окруТности радиусом 1 см. ΠžΠΏΡ€Π΅Π΄Π΅Π»ΠΈΡ‚Π΅ ΡΠΊΠΎΡ€ΠΎΡΡ‚ΡŒ двиТСния элСктрона, Ссли магнитная индукция поля 0,2 Π’Π».

Π’ Π·Π°ΡˆΡ‚Ρ€ΠΈΡ…ΠΎΠ²Π°Π½Π½ΠΎΠΉ области Π½Π° рисункС дСйствуСт ΠΎΠ΄Π½ΠΎΡ€ΠΎΠ΄Π½ΠΎΠ΅ ΠΌΠ°Π³Π½ΠΈΡ‚Π½ΠΎΠ΅ ΠΏΠΎΠ»Π΅, Π½Π°ΠΏΡ€Π°Π²Π»Π΅Π½Π½ΠΎΠ΅ пСрпСндикулярно плоскости рисунка, Π’ = 0,1 Π’Π». ΠŸΡ€ΠΎΠ²ΠΎΠ»ΠΎΡ‡Π½ΡƒΡŽ ΠΊΠ²Π°Π΄Ρ€Π°Ρ‚Π½ΡƒΡŽ Ρ€Π°ΠΌΠΊΡƒ сопротивлСниСм R = 10 Ом ΠΈ стороной l = 10 см ΠΏΠ΅Ρ€Π΅ΠΌΠ΅Ρ‰Π°ΡŽΡ‚ Π² плоскости рисунка ΠΏΠΎΡΡ‚ΡƒΠΏΠ°Ρ‚Π΅Π»ΡŒΠ½ΠΎ со ΡΠΊΠΎΡ€ΠΎΡΡ‚ΡŒΡŽ v = 1 ΠΌ/с. Π§Π΅ΠΌΡƒ Ρ€Π°Π²Π΅Π½ ΠΈΠ½Π΄ΡƒΠΊΡ†ΠΈΠΎΠ½Π½Ρ‹ΠΉ Ρ‚ΠΎΠΊ Π² Ρ€Π°ΠΌΠΊΠ΅ Π² состоянии 1?

3} $$ ΠΈ Ρ„ΠΎΡ€ΠΌΡƒΠ»Π° для силы Π›ΠΎΡ€Π΅Π½Ρ†Π° $$ d \ vec {F} _2 = I_2 \ cdot d \ vec {r} _2 \ times \ vec {B} (\ vec {r} _2) $$ Π³Π΄Π΅ $ \ vec {r} _2 $ — Ρ‚ΠΎΡ‡ΠΊΠ° Π½Π° Π²Ρ‚ΠΎΡ€ΠΎΠΌ ΠΏΡ€ΠΎΠ²ΠΎΠ΄Π΅, Π° $ d \ vec {r} _2 $ — ΡΠΎΠΎΡ‚Π²Π΅Ρ‚ΡΡ‚Π²ΡƒΡŽΡ‰ΠΈΠΉ элСмСнт ΠΏΡƒΡ‚ΠΈ. Π˜Π½Ρ‚Π΅Π³Ρ€Π°Π» ΠΏΠΎ путям ΠΏΠΎ Π²Ρ‚ΠΎΡ€ΠΎΠΌΡƒ ΠΏΡ€ΠΎΠ²ΠΎΠ΄Ρƒ Π΄Π°Π΅Ρ‚ Π²Π°ΡˆΡƒ Ρ„ΠΎΡ€ΠΌΡƒΠ»Ρƒ.

Π—Π°ΠΊΠΎΠ½ Π‘ΠΈΠΎ-Π‘Π°Π²Π°Ρ€Π°

Из $ \ def \ div {\ operatorname {div}} \ def \ rot {\ operatorname {rot}} \ def \ grad {\ operatorname {grad}} \ div (\ vec {B}) = 0 $ слСдуСт сущСствованиС Π½Π΅ΠΊΠΎΡ‚ΠΎΡ€ΠΎΠ³ΠΎ Π²Π΅ΠΊΡ‚ΠΎΡ€Π½ΠΎΠ³ΠΎ ΠΏΠΎΡ‚Π΅Π½Ρ†ΠΈΠ°Π»Π° $ \ vec {A} $ с $ \ vec {B} = \ rot \ vec {A} $.ΠŸΠΎΠ΄ΡΡ‚Π°Π²Π»ΡΡ это Π² Π·Π°ΠΊΠΎΠ½ АмпСра (для устойчивого состояния)

$ \ rot (\ vec {H}) = \ vec {S} $

$ \ rot (\ vec {B}) = \ mu_0 \ vec {S} $

Π΄Π°Π΅Ρ‚

$ \ rot (\ rot \ vec {A}) = \ mu_0 \ vec {S} $

По Ρ„ΠΎΡ€ΠΌΡƒΠ»Π΅ $ \ rot \ rot \ vec {A} = \ vec {\ nabla} \ times (\ vec {\ nabla} \ times \ vec {A}) = \ vec {\ nabla} (\ vec {\ Π½Π°Π±Π»Π°} \ cdot \ vec {A}) — (\ vec {\ nabla} \ cdot \ vec {\ nabla}) \ vec {A} = \ grad \ div \ vec {A} — \ Delta \ vec {A} $ ΠΈ ΠΊΠ°Π»ΠΈΠ±Ρ€ΠΎΠ²ΠΎΡ‡Π½ΠΎΠ³ΠΎ условия ΠšΡƒΠ»ΠΎΠ½Π° $ \ div \ vec {A} = 0 $ ΠΏΠΎΠ»ΡƒΡ‡Π°Π΅ΠΌ

$ \ Delta \ vec {A} = — \ mu_0 \ vec {S} $

Для Π·Π°Π΄Π°Ρ‡ΠΈ ΠΎ свободном пространствС это ΡƒΡ€Π°Π²Π½Π΅Π½ΠΈΠ΅ ΠΌΠΎΠΆΠ΅Ρ‚ Π±Ρ‹Ρ‚ΡŒ Ρ€Π΅ΡˆΠ΅Π½ΠΎ с ΠΏΠΎΠΌΠΎΡ‰ΡŒΡŽ Ρ„ΡƒΠ½ΠΊΡ†ΠΈΠΈ Π“Ρ€ΠΈΠ½Π° лапласиана

$$ \ vec {A} (\ vec {r}) = — \ frac {\ mu_0} {4 \ pi} \ int _ {\ vec {r} _1 \ mathbb {R} ^ 3} \ frac {\ vec {S} (\ vec {r_1})} {| \ vec {r} _1- \ vec {r} |} d V_1 $$ Π˜ΡΠΏΠΎΠ»ΡŒΠ·ΡƒΡ $ \ vec {B} = \ rot \ vec {A} $, ΠΏΠΎΠ»ΡƒΡ‡Π°Π΅ΠΌ ΠΏΠ»ΠΎΡ‚Π½ΠΎΡΡ‚ΡŒ ΠΏΠΎΡ‚ΠΎΠΊΠ°

$ \ Displaystyle \ vec {B} (\ vec {r}) = — \ frac {\ mu_0} {4 \ pi} \ int _ {\ vec {r} _1 \ mathbb {R} ^ 3} \ rot _ {\ vec {r} } \ left (\ frac {\ vec {S} (\ vec {r_1})} {| \ vec {r} _1- \ vec {r} |} \ right) d V_1 $

$ \ Displaystyle \ phantom {\ vec {B} (\ vec {r})} = — \ frac {\ mu_0} {4 \ pi} \ int _ {\ vec {r} _1 \ mathbb {R} ^ 3} \ grad _ {\ vec {r}} \ left (\ frac {1} {| \ vec {r} _1- \ vec {r} |} \ right) \ times \ vec {S} (\ vec {r_1}) d V_1 $

$ \ Displaystyle \ phantom {\ vec {B} (\ vec {r})} = \ frac {\ mu_0} {4 \ pi} \ int _ {\ vec {r} _1 \ mathbb {R} ^ 3} \ frac {(\ vec {r} — \ vec {r} _1) \ times \ vec {S} (\ vec {r_1})} {| \ vec {r} — \ vec {r} _1 | ^ 3} d V_1 $

Для ΠΈΠ½Ρ‚Π΅Π³Ρ€Π°Π»Π° ΠΏΠΎ ΠΏΠ»ΠΎΡ‰Π°Π΄ΠΈ ΠΏΠΎΠΏΠ΅Ρ€Π΅Ρ‡Π½ΠΎΠ³ΠΎ сСчСния ΠΏΡ€ΠΎΠ²ΠΎΠ΄Π° измСнСниями $ r_1 $ ΠΏΡ€Π΅Π½Π΅Π±Ρ€Π΅Π³Π°ΡŽΡ‚ ΠΈ $ \ int_ {A _ {\ rm cross}} \ vec {S} d V $ устанавливаСтся Ρ€Π°Π²Π½Ρ‹ΠΌ $ I_1 d \ vec { r} _1 $.3}. $

элСктромагнСтизм — Π‘ΠΈΠ»Π° Π›ΠΎΡ€Π΅Π½Ρ†Π° относится Ρ‚ΠΎΠ»ΡŒΠΊΠΎ ΠΊ силС, описанной Π·Π°ΠΊΠΎΠ½ΠΎΠΌ Π›ΠΎΡ€Π΅Π½Ρ†Π°?

Π˜Ρ‚Π°ΠΊ, ΠΌΠΎΠΉ вопрос, ΠΏΡ€ΠΎΡ‰Π΅ говоря — Ρ‡Ρ‚ΠΎ Ρ‚Π°ΠΊΠΎΠ΅ сила Π›ΠΎΡ€Π΅Π½Ρ†Π°? ΠžΡ‚Π½ΠΎΡΠΈΡ‚ΡΡ Π»ΠΈ это Ρ‚ΠΎΠ»ΡŒΠΊΠΎ ΠΊ силС, описанной Π·Π°ΠΊΠΎΠ½ΠΎΠΌ Π›ΠΎΡ€Π΅Π½Ρ†Π° (F = qvB)? ΠŸΡ€ΠΈΠΌΠ΅Π½ΠΈΠΌΠΎ Π»ΠΈ это Ρ‚ΠΎΠ»ΡŒΠΊΠΎ для ΠΎΠ΄Π½ΠΎΠΉ частицы, Π³Π΄Π΅ сила пСрпСндикулярна Π΄Ρ€ΡƒΠ³ΠΈΠΌ Π²ΠΎΠ²Π»Π΅Ρ‡Π΅Π½Π½Ρ‹ΠΌ Ρ„Π°ΠΊΡ‚ΠΎΡ€Π°ΠΌ?

Π’Π΅Ρ€ΠΌΠΈΠ½ «сила Π›ΠΎΡ€Π΅Π½Ρ†Π°Β» ΠΈΡΠΏΠΎΠ»ΡŒΠ·ΡƒΠ΅Ρ‚ΡΡ Π² Π½Π΅ΡΠΊΠΎΠ»ΡŒΠΊΠΈΡ… Ρ€Π°Π·Π»ΠΈΡ‡Π½Ρ‹Ρ… значСниях. Π”Π°, я знаю.

Π”ΠΎ Π›ΠΎΡ€Π΅Π½Ρ†Π° Π±Ρ‹Π»ΠΈ извСстны ΠΏΡ€Π°Π²ΠΈΠ»ΡŒΠ½Ρ‹Π΅ Ρ„ΠΎΡ€ΠΌΡƒΠ»Ρ‹ для вычислСния силы, Π΄Π΅ΠΉΡΡ‚Π²ΡƒΡŽΡ‰Π΅ΠΉ Π½Π° макроскопичСскиС заряТСнныС ΠΈΠ»ΠΈ токонСсущиС Ρ‚Π΅Π»Π° (АмпСр, Π‘ΠΈΠΎ, Π‘Π°Π²Π°Ρ€, Лаплас ΡƒΠΌΠ΅Π»ΠΈ Π²Ρ‹Ρ‡ΠΈΡΠ»ΡΡ‚ΡŒ силы).Π›ΠΎΡ€Π΅Π½Ρ† сформулировал ΠΌΠΈΠΊΡ€ΠΎΡΠΊΠΎΠΏΠΈΡ‡Π΅ΡΠΊΡƒΡŽ Ρ‚Π΅ΠΎΡ€ΠΈΡŽ заряТСнных частиц, Π² ΠΊΠΎΡ‚ΠΎΡ€ΠΎΠΉ матСрия состоит ΠΈΠ· Ρ‚Π°ΠΊΠΈΡ… частиц. Π’ этой Ρ‚Π΅ΠΎΡ€ΠΈΠΈ сила, Π΄Π΅ΠΉΡΡ‚Π²ΡƒΡŽΡ‰Π°Ρ Π½Π° Π·Π°Ρ€ΡΠΆΠ΅Π½Π½ΡƒΡŽ частицу Π·Π° счСт элСктромагнитного взаимодСйствия, Π²Ρ‹Ρ€Π°ΠΆΠ°Π»Π°ΡΡŒ ΠΊΠ°ΠΊ функция:

1) ΠΏΠ»ΠΎΡ‚Π½ΠΎΡΡ‚ΡŒ заряда $ \ rho $ ΠΈ ΠΏΠ»ΠΎΡ‚Π½ΠΎΡΡ‚ΡŒ ΠΏΠΎΡ‚ΠΎΠΊΠ° элСктричСского Ρ‚ΠΎΠΊΠ° $ \ mathbf j $ (ΠΎΠ±Ρ‹Ρ‡Π½ΠΎ называСмая ΠΏΠ»ΠΎΡ‚Π½ΠΎΡΡ‚ΡŒΡŽ Ρ‚ΠΎΠΊΠ°) ΠΈ

2) Π²Π΅Π»ΠΈΡ‡ΠΈΠ½Ρ‹, ΠΎΠΏΠΈΡΡ‹Π²Π°ΡŽΡ‰ΠΈΠ΅ состояниС элСктромагнитного поля $ \ mathbf d $ ΠΈ $ \ mathbf h $.

Π‘ΠΈΠ»Π° ΠΏΠΎΡΡ‚ΡƒΠ»ΠΈΡ€ΠΎΠ²Π°Π»Π°ΡΡŒ ΠΊΠ°ΠΊ ΠΈΠ½Ρ‚Π΅Π³Ρ€Π°Π»

$$ \ mathbf F = \ int_V \ rho \ mathbf d + \ mathbf j \ times \ mathbf h \, dV $$ Π³Π΄Π΅ ΠΈΠ½Ρ‚Π΅Π³Ρ€ΠΈΡ€ΠΎΠ²Π°Π½ΠΈΠ΅ производится ΠΏΠΎ пространствСнной области, содСрТащСй всС части частицы.

ПозТС, Π² 20-ΠΌ Π²Π΅ΠΊΠ΅, это ΠΎΠ±ΠΎΠ·Π½Π°Ρ‡Π΅Π½ΠΈΠ΅ ΠΏΡ€ΠΈΡˆΠ»ΠΎ Π² ΡƒΠΏΠ°Π΄ΠΎΠΊ, ΠΈ $ \ mathbf d, \ mathbf h $ Π±Ρ‹Π»ΠΈ Π·Π°ΠΌΠ΅Π½Π΅Π½Ρ‹ символами $ \ mathbf e, \ mathbf b $, Π½ΠΎ смысл остался ΠΏΡ€Π΅ΠΆΠ½ΠΈΠΌ. Π­Ρ‚ΠΎ ΠΎΠ΄Π½ΠΎ ΠΈΠ· Π·Π½Π°Ρ‡Π΅Π½ΠΈΠΉ «силы Π›ΠΎΡ€Π΅Π½Ρ†Π°Β».

Π­Ρ‚Π° тСория Π›ΠΎΡ€Π΅Π½Ρ†Π° оказалась вСсьма ΠΏΠΎΠ»Π΅Π·Π½ΠΎΠΉ ΠΈ ΠΏΡ€ΠΈΠ²Π΅Π»Π° ΠΊ ΡƒΡΠΏΠ΅ΡˆΠ½Ρ‹ΠΌ модСлям взаимодСйствия ΠΌΠ°Ρ‚Π΅Ρ€ΠΈΠΈ ΠΈ элСктромагнитного поля (ΠΌΠΎΠ΄Π΅Π»ΠΈ прСломлСния, поглощСния, эффСкта Π—Π΅Π΅ΠΌΠ°Π½Π° ΠΈ Π΄Ρ€.). Но Ρƒ этой «силы Π›ΠΎΡ€Π΅Π½Ρ†Π°Β» Π±Ρ‹Π»Π° сущСствСнная ΠΏΡ€ΠΎΠ±Π»Π΅ΠΌΠ°: ΠΎΠ½Π° Π½Π΅ ΠΌΠΎΠ³Π»Π° ΠΎΠ±ΡŠΡΡΠ½ΠΈΡ‚ΡŒ, ΠΊΠ°ΠΊ ΠΊΡ€ΠΎΡˆΠ΅Ρ‡Π½Ρ‹Π΅ заряТСнныС частицы Π½Π΅Π½ΡƒΠ»Π΅Π²ΠΎΠ³ΠΎ Ρ€Π°Π·ΠΌΠ΅Ρ€Π° ΠΌΠΎΠ³ΡƒΡ‚ ΡƒΠ΄Π΅Ρ€ΠΆΠΈΠ²Π°Ρ‚ΡŒΡΡ вмСстС.И Π΄Π°ΠΆΠ΅ Ссли Π΄ΠΎΠ±Π°Π²ΠΈΡ‚ΡŒ Π½Π΅-ЭМ силы, Ρ‡Ρ‚ΠΎΠ±Ρ‹ ΡƒΠ΄Π΅Ρ€ΠΆΠΈΠ²Π°Ρ‚ΡŒ ΠΈΡ… вмСстС, любая рСлятивистски ΠΏΡ€Π°Π²ΠΈΠ»ΡŒΠ½Π°Ρ модСль Ρ‚Π°ΠΊΠΎΠΉ частицы ΠΊΠΎΠ½Π΅Ρ‡Π½ΠΎΠ³ΠΎ Ρ€Π°Π·ΠΌΠ΅Ρ€Π° ΠΈΠΌΠ΅Π΅Ρ‚ бСсконСчноС количСство стСпСнСй свободы, ΠΈ с Π½Π΅ΠΉ ΠΎΡ‡Π΅Π½ΡŒ Ρ‚Ρ€ΡƒΠ΄Π½ΠΎ Ρ€Π°Π±ΠΎΡ‚Π°Ρ‚ΡŒ.

Π˜Ρ‚Π°ΠΊ, люди ΠΎΡ‡Π΅Π½ΡŒ скоро упростили Ρ‚Π΅ΠΎΡ€ΠΈΡŽ, ΠΏΡ€Π΅Π΄ΠΏΠΎΠ»ΠΎΠΆΠΈΠ², Ρ‡Ρ‚ΠΎ заряТСнныС частицы — это просто Ρ‚ΠΎΡ‡ΠΊΠΈ. Π’ΠΎΠ³Π΄Π° вопрос ΠΎ Ρ‚ΠΎΠΌ, ΠΊΠ°ΠΊ заряд ΠΌΠΎΠΆΠ΅Ρ‚ ΡƒΠ΄Π΅Ρ€ΠΆΠΈΠ²Π°Ρ‚ΡŒΡΡ, ΠΎΡ‚ΠΏΠ°Π΄Π°Π΅Ρ‚ — Π½Π΅Ρ‚ ΠΎΡ‚Ρ‚Π°Π»ΠΊΠΈΠ²Π°ΡŽΡ‰ΠΈΡ… частСй. Богласно этой Ρ‚Π΅ΠΎΡ€ΠΈΠΈ, Ρ„ΠΎΡ€ΠΌΡƒΠ»Π° силы постулируСтся Ρ€Π°Π²Π½ΠΎΠΉ

.

$$ \ mathbf F = q \ mathbf e + q \ mathbf v \ times \ mathbf b $$ Π³Π΄Π΅ $ q $ — заряд частицы, Π° $ \ mathbf e, \ mathbf b $ — внСшнСС элСктричСскоС ΠΈ ΠΌΠ°Π³Π½ΠΈΡ‚Π½ΠΎΠ΅ поля, Π΄Π΅ΠΉΡΡ‚Π²ΡƒΡŽΡ‰ΠΈΠ΅ Π½Π° эту частицу.

НаблюдСния элСктронов Π² ЭМ ΠΏΠΎΠ»Π΅ (экспСримСнты Π”ΠΆ. Π”ΠΆ. Вомсона ΠΈ ΠΌΠ½ΠΎΠ³ΠΈΠ΅ Π±ΠΎΠ»Π΅Π΅ ΠΏΠΎΠ·Π΄Π½ΠΈΠ΅) ΠΏΠΎΠ΄Ρ‚Π²Π΅Ρ€ΠΆΠ΄Π°ΡŽΡ‚, Ρ‡Ρ‚ΠΎ это Π²Ρ‹Ρ€Π°ΠΆΠ΅Π½ΠΈΠ΅ ΠΎΡ‡Π΅Π½ΡŒ Ρ…ΠΎΡ€ΠΎΡˆΠΎ описываСт силу, Π΄Π΅ΠΉΡΡ‚Π²ΡƒΡŽΡ‰ΡƒΡŽ Π½Π° элСктроны Π²ΠΎ внСшнСм ЭМ ΠΏΠΎΠ»Π΅. Π­Ρ‚ΠΎ Π΅Ρ‰Π΅ ΠΎΠ΄Π½ΠΎ Π·Π½Π°Ρ‡Π΅Π½ΠΈΠ΅ Ρ‚Π΅Ρ€ΠΌΠΈΠ½Π° «сила Π›ΠΎΡ€Π΅Π½Ρ†Π°Β».

Иногда люди ΠΈΠΌΠ΅ΡŽΡ‚ Π΄Π΅Π»ΠΎ с частицами, двиТущимися Π² области пространства, Π³Π΄Π΅ элСктричСскоС ΠΏΠΎΠ»Π΅ Π½Π΅Π·Π½Π°Ρ‡ΠΈΡ‚Π΅Π»ΡŒΠ½ΠΎ ΠΈ СдинствСнноС, Ρ‡Ρ‚ΠΎ ΠΈΠΌΠ΅Π΅Ρ‚ Π·Π½Π°Ρ‡Π΅Π½ΠΈΠ΅, — это магнитная сила. Π—Π°Ρ‚Π΅ΠΌ Ρ„ΠΎΡ€ΠΌΡƒΠ»Π° упрощаСтся Π΄ΠΎ

$$ \ mathbf F = q \ mathbf v \ times \ mathbf b.

$

Π­Ρ‚ΠΎ Ρ‚Π°ΠΊΠΆΠ΅ называСтся «силой Π›ΠΎΡ€Π΅Π½Ρ†Π°Β».

Однако, ΠΊ соТалСнию, это Π΅Ρ‰Π΅ Π½Π΅ всС.

Π’ΠΎΠ·Π²Ρ€Π°Ρ‰Π°ΡΡΡŒ ΠΊ макроскопичСским явлСниям, сила Π²Π΅Π»ΠΈΡ‡ΠΈΠ½Ρ‹ $ BIL $, Π΄Π΅ΠΉΡΡ‚Π²ΡƒΡŽΡ‰Π°Ρ Π½Π° ΠΏΡ€ΠΎΠ²ΠΎΠ΄ Π΄Π»ΠΈΠ½ΠΎΠΉ $ L $, нСсущий Ρ‚ΠΎΠΊ $ I $ Π² ΠΌΠ°Π³Π½ΠΈΡ‚Π½ΠΎΠΌ ΠΏΠΎΠ»Π΅ $ B $, Ρ‚Π°ΠΊΠΆΠ΅ называСтся Π² Π½Π΅ΠΊΠΎΡ‚ΠΎΡ€Ρ‹Ρ… ΡƒΡ‡Π΅Π±Π½ΠΈΠΊΠ°Ρ… ΠΈ людьми «силой Π›ΠΎΡ€Π΅Π½Ρ†Π°Β». Π’Π°ΠΊΠΎΠ΅ использованиС каТСтся довольно распространСнным, Π½ΠΎ, Π½Π° ΠΌΠΎΠΉ взгляд, Π½Π΅Π²Π΅Ρ€Π½Ρ‹ΠΌ *; Π›ΡƒΡ‡ΡˆΠ΅Π΅ Π½Π°Π·Π²Π°Π½ΠΈΠ΅ Π±Ρ‹Π»ΠΎ Π±Ρ‹ исходной Ρ‚Π΅Ρ€ΠΌΠΈΠ½ΠΎΠ»ΠΎΠ³ΠΈΠ΅ΠΉ, использованной Π΄ΠΎ Π›ΠΎΡ€Π΅Π½Ρ†Π°: «магнитная сила» ΠΈΠ»ΠΈ «пондСромоторная сила», Ρ‡Ρ‚ΠΎ ΠΏΡ€ΠΈΠΌΠ΅Ρ€Π½ΠΎ ΠΎΠ·Π½Π°Ρ‡Π°Π΅Ρ‚ «сила, Π΄Π΅ΠΉΡΡ‚Π²ΡƒΡŽΡ‰Π°Ρ Π½Π° / двиТущаяся тяТСлая матСрия», Π² ΠΎΡ‚Π»ΠΈΡ‡ΠΈΠ΅ ΠΎΡ‚ «элСктродвиТущСй силы», Ρ‡Ρ‚ΠΎ ΠΏΡ€ΠΈΠΌΠ΅Ρ€Π½ΠΎ ΠΎΠ·Π½Π°Ρ‡Π°Π΅Ρ‚ «сила, ΠΏΠ΅Ρ€Π΅ΠΌΠ΅Ρ‰Π°ΡŽΡ‰Π°Ρ элСктричСство Π² Π΄ΠΈΡ€ΠΈΠΆΠ΅Ρ€ Β».

Когда Π΄Π²Π° ΠΌΠ°Π³Π½ΠΈΡ‚Π° ΠΎΡ‚Ρ‚Π°Π»ΠΊΠΈΠ²Π°ΡŽΡ‚ΡΡ Π΄Ρ€ΡƒΠ³ ΠΎΡ‚ Π΄Ρ€ΡƒΠ³Π°, являСтся Π»ΠΈ Π²ΠΎΠ·Π½ΠΈΠΊΠ°ΡŽΡ‰Π°Ρ сила, которая заставляСт ΠΈΡ… это Π΄Π΅Π»Π°Ρ‚ΡŒ, силой Π›ΠΎΡ€Π΅Π½Ρ†Π°? Если Π½Π΅Ρ‚, Ρ‚ΠΎ Ρ‡Ρ‚ΠΎ это Π·Π° сила ΠΈ ΠΊΠ°ΠΊΠΎΠ΅ ΡƒΡ€Π°Π²Π½Π΅Π½ΠΈΠ΅ слСдуСт ΠΈΡΠΏΠΎΠ»ΡŒΠ·ΠΎΠ²Π°Ρ‚ΡŒ для Π΅Π΅ описания?

Π’ макроскопичСской Ρ‚Π΅ΠΎΡ€ΠΈΠΈ Π½Π΅Ρ‚, ΠΏΠΎΡ‚ΠΎΠΌΡƒ Ρ‡Ρ‚ΠΎ Π½Π΅Ρ‚ заряТСнных частиц. Π‘ΠΈΠ»Π° Π΄ΠΎΠ»ΠΆΠ½Π° Π½Π°Π·Ρ‹Π²Π°Ρ‚ΡŒΡΡ «ΠΌΠ°ΠΊΡ€ΠΎΡΠΊΠΎΠΏΠΈΡ‡Π΅ΡΠΊΠΎΠΉ ΠΌΠ°Π³Π½ΠΈΡ‚Π½ΠΎΠΉ силой» ΠΈ ΠΌΠΎΠΆΠ΅Ρ‚ Π±Ρ‹Ρ‚ΡŒ рассчитана ΠΊΠ°ΠΊ:

1) сила ΠΎΠ΄Π½ΠΎΠ³ΠΎ ΠΌΠ°Π³Π½ΠΈΡ‚Π½ΠΎΠ³ΠΎ диполя Π½Π° Π΄Ρ€ΡƒΠ³ΠΎΠΉ ΠΌΠ°Π³Π½ΠΈΡ‚Π½Ρ‹ΠΉ диполь (кулоновский ΠΏΠΎΠ΄Ρ…ΠΎΠ΄)

2) сила ΠΎΠ΄Π½ΠΎΠ³ΠΎ Ρ‚Π΅Π»Π°, нСсущСго Ρ‚ΠΎΠΊΠΎΠ²ΡƒΡŽ ΠΏΠ΅Ρ‚Π»ΡŽ, Π½Π° Π΄Ρ€ΡƒΠ³ΠΎΠ΅ Ρ‚Π΅Π»ΠΎ, нСсущСС Ρ‚ΠΎΠΊΠΎΠ²ΡƒΡŽ ΠΏΠ΅Ρ‚Π»ΡŽ (ΠΏΠΎΠ΄Ρ…ΠΎΠ΄ АмпСра)

Π’ микроскопичСской Ρ€Π°Π·Ρ€Π°Π±ΠΎΡ‚ΠΊΠ΅ Π²Ρ‚ΠΎΡ€ΠΎΠ³ΠΎ ΠΏΠΎΠ΄Ρ…ΠΎΠ΄Π° сила, Π΄Π΅ΠΉΡΡ‚Π²ΡƒΡŽΡ‰Π°Ρ Π½Π° любоС Ρ‚Π΅Π»ΠΎ с Ρ‚ΠΎΠΊΠΎΠ²ΠΎΠΉ ΠΏΠ΅Ρ‚Π»Π΅ΠΉ, являСтся суммой сил Π›ΠΎΡ€Π΅Π½Ρ†Π° $ q \ mathbf e + q \ mathbf v \ times \ mathbf b $ Π½Π° всСх частицах, ΠΊΠΎΡ‚ΠΎΡ€Ρ‹Π΅ ΡΠ²Π»ΡΡŽΡ‚ΡΡ Ρ‡Π°ΡΡ‚ΡŒΡŽ этого Ρ‚Π΅Π»Π°.

* Π­Ρ‚Π° сила ΠΈ Ρ„ΠΎΡ€ΠΌΡƒΠ»Ρ‹ для Π½Π΅Π΅ Π±Ρ‹Π»ΠΈ извСстны Π΄ΠΎ Ρ‚ΠΎΠ³ΠΎ, ΠΊΠ°ΠΊ Π›ΠΎΡ€Π΅Π½Ρ† Π²Ρ‹ΠΏΠΎΠ»Π½ΠΈΠ» свою Ρ€Π°Π±ΠΎΡ‚Ρƒ. Π’Π°ΠΊΠΆΠ΅ это Π²Ρ‹Ρ€Π°ΠΆΠ΅Π½ΠΈΠ΅ взято ΠΈΠ· макроскопичСской Ρ‚Π΅ΠΎΡ€ΠΈΠΈ, Π³Π΄Π΅ сила дСйствуСт Π½Π° Ρ‚ΡΠΆΠ΅Π»ΡƒΡŽ Ρ‡Π°ΡΡ‚ΡŒ Ρ‚Π΅Π»Π°, Π° Π½Π΅ Ρ‚ΠΎΠ»ΡŒΠΊΠΎ Π½Π° заряТСнныС частицы.

Π‘ΠΈΠ»Π° Π›ΠΎΡ€Π΅Π½Ρ†Π°

Π‘ΠΈΠ»Π° Π›ΠΎΡ€Π΅Π½Ρ†Π°
Π”Π°Π»Π΅Π΅: Π—Π°ΠΊΠΎΠ½ АмпСра Up: НС зависящиС ΠΎΡ‚ Π²Ρ€Π΅ΠΌΠ΅Π½ΠΈ уравнСния МаксвСлла ΠŸΡ€Π΅Π΄Ρ‹Π΄ΡƒΡ‰Π°Ρ: ЭкспСримСнты АмпСра


Π‘ΠΈΠ»Π° Π›ΠΎΡ€Π΅Π½Ρ†Π° ΠŸΠΎΡ‚ΠΎΠΊ элСктричСского Ρ‚ΠΎΠΊΠ° Π²Π½ΠΈΠ· проводящий ΠΏΡ€ΠΎΠ²ΠΎΠ΄ Π² ΠΊΠΎΠ½Π΅Ρ‡Π½ΠΎΠΌ ΠΈΡ‚ΠΎΠ³Π΅ ΠΈΠ·-Π·Π° двиТСния элСктричСски заряТСнныС частицы (Π² Π±ΠΎΠ»ΡŒΡˆΠΈΠ½ΡΡ‚Π²Π΅ случаСв элСктроны) Ρ‡Π΅Ρ€Π΅Π· ΠΏΡ€ΠΎΠ²ΠΎΠ΄ΡΡ‰ΡƒΡŽ срСду.ΠŸΠΎΡΡ‚ΠΎΠΌΡƒ каТСтся Ρ€Π°Π·ΡƒΠΌΠ½Ρ‹ΠΌ, Ρ‡Ρ‚ΠΎ сила, Π΄Π΅ΠΉΡΡ‚Π²ΡƒΡŽΡ‰Π°Ρ Π½Π° ΠΏΡ€ΠΎΠ²ΠΎΠ΄, ΠΊΠΎΠ³Π΄Π° ΠΎΠ½ ΠΏΠΎΠΌΠ΅Ρ‰Π΅Π½ Π² ΠΌΠ°Π³Π½ΠΈΡ‚Π½ΠΎΠ΅ ΠΏΠΎΠ»Π΅, Π΄Π΅ΠΉΡΡ‚Π²ΠΈΡ‚Π΅Π»ΡŒΠ½ΠΎ Ρ€Π°Π²Π½ΠΎΠ΄Π΅ΠΉΡΡ‚Π²ΡƒΡŽΡ‰Π°Ρ сил, Π΄Π΅ΠΉΡΡ‚Π²ΡƒΡŽΡ‰ΠΈΡ… Π½Π° эти двиТущиСся заряды. Позволь Π½Π°ΠΌ ΠŸΡ€Π΅Π΄ΠΏΠΎΠ»ΠΎΠΆΠΈΠΌ, Ρ‡Ρ‚ΠΎ это Ρ‚Π°ΠΊ.

ΠŸΡƒΡΡ‚ΡŒ Π±ΡƒΠ΄Π΅Ρ‚ (равномСрная) ΠΏΠ»ΠΎΡ‰Π°Π΄ΡŒ ΠΏΠΎΠΏΠ΅Ρ€Π΅Ρ‡Π½ΠΎΠ³ΠΎ сСчСния ΠΏΡ€ΠΎΠ²ΠΎΠ΄Π°, ΠΈ ΠΏΡƒΡΡ‚ΡŒ Π±ΡƒΠ΄Π΅Ρ‚ числовая ΠΏΠ»ΠΎΡ‚Π½ΠΎΡΡ‚ΡŒ ΠΌΠΎΠ±ΠΈΠ»ΡŒΠ½Ρ‹Ρ… зарядов Π² ΠΏΡ€ΠΎΠ²ΠΎΠ΄Π½ΠΈΠΊΠ΅. ΠŸΡ€Π΅Π΄ΠΏΠΎΠ»ΠΎΠΆΠΈΠΌ, Ρ‡Ρ‚ΠΎ Ρƒ ΠΌΠΎΠ±ΠΈΠ»ΡŒΠ½Ρ‹Ρ… зарядов Π΅ΡΡ‚ΡŒ заряд ΠΈ ΡΠΊΠΎΡ€ΠΎΡΡ‚ΡŒ. ΠœΡ‹ Π΄ΠΎΠ»ΠΆΠ½Ρ‹ ΠΏΡ€Π΅Π΄ΠΏΠΎΠ»ΠΎΠΆΠΈΡ‚ΡŒ, Ρ‡Ρ‚ΠΎ ΠΏΡ€ΠΎΠ²ΠΎΠ΄Π½ΠΈΠΊ Ρ‚Π°ΠΊΠΆΠ΅ содСрТит стационарныС заряды с зарядовой ΠΈ числовой ΠΏΠ»ΠΎΡ‚Π½ΠΎΡΡ‚ΡŒΡŽ. (скаТСм), Ρ‚Π°ΠΊ Ρ‡Ρ‚ΠΎ чистая ΠΏΠ»ΠΎΡ‚Π½ΠΎΡΡ‚ΡŒ заряда Π² ΠΏΡ€ΠΎΠ²ΠΎΠ΄Π΅ Ρ€Π°Π²Π½Π° Π½ΡƒΠ»ΡŽ.Π£ Π±ΠΎΠ»ΡŒΡˆΠΈΠ½ΡΡ‚Π²Π° Π΄ΠΈΡ€ΠΈΠΆΠ΅Ρ€ΠΎΠ² ΠΏΠΎΠ΄Π²ΠΈΠΆΠ½Ρ‹Π΅ заряды — это элСктроны, Π° стационарныС заряды — это Π°Ρ‚ΠΎΠΌΠ½Ρ‹Π΅ ядра. Π’Π΅Π»ΠΈΡ‡ΠΈΠ½Π° элСктричСского Ρ‚ΠΎΠΊΠ°, ΠΏΡ€ΠΎΡ‚Π΅ΠΊΠ°ΡŽΡ‰Π΅Π³ΠΎ ΠΏΠΎ ΠΏΡ€ΠΎΠ²ΠΎΠ΄Ρƒ, — это просто количСство ΠΊΡƒΠ»ΠΎΠ½ΠΎΠ² Π² сСкунду, ΠΊΠΎΡ‚ΠΎΡ€Ρ‹Π΅ проходят Ρ‡Π΅Ρ€Π΅Π· Π·Π°Π΄Π°Π½Π½ΡƒΡŽ Ρ‚ΠΎΡ‡ΠΊΡƒ. Π—Π° ΠΎΠ΄Π½Ρƒ сСкунду ΠΌΠΎΠ±ΠΈΠ»ΡŒΠ½Ρ‹ΠΉ заряд пСрСмСщаСтся Π½Π° расстояниС, поэтому всС заряды, содСрТащиСся Π² Ρ†ΠΈΠ»ΠΈΠ½Π΄Ρ€ ΠΏΠ»ΠΎΡ‰Π°Π΄ΠΈ ΠΏΠΎΠΏΠ΅Ρ€Π΅Ρ‡Π½ΠΎΠ³ΠΎ сСчСния ΠΈ Π΄Π»ΠΈΠ½Ρ‹ ΠΎΠ±Ρ‚Π΅ΠΊΠ°Π΅Ρ‚ Π·Π°Π΄Π°Π½Π½ΡƒΡŽ Ρ‚ΠΎΡ‡ΠΊΡƒ. Π’Π°ΠΊΠΈΠΌ ΠΎΠ±Ρ€Π°Π·ΠΎΠΌ, Π²Π΅Π»ΠΈΡ‡ΠΈΠ½Π° Ρ‚ΠΎΠΊΠ° составляСт. НаправлСниС Ρ‚ΠΎΠΊ совпадаСт с Π½Π°ΠΏΡ€Π°Π²Π»Π΅Π½ΠΈΠ΅ΠΌ двиТСния зарядов, поэтому Π²Π΅ΠΊΡ‚ΠΎΡ€Π½Ρ‹ΠΉ Ρ‚ΠΎΠΊ .Богласно Ρ„ΠΎΡ€ΠΌΡƒΠ»Π΅. (229) сила Π½Π° Π΅Π΄ΠΈΠ½ΠΈΡ†Ρƒ Π΄Π»ΠΈΠ½Ρ‹, Π΄Π΅ΠΉΡΡ‚Π²ΡƒΡŽΡ‰Π°Ρ Π½Π° ΠΏΡ€ΠΎΠ²ΠΎΠ΄, Ρ€Π°Π²Π½Π°

(232)

Однако Π½Π° Π΅Π΄ΠΈΠ½ΠΈΡ†Π΅ Π΄Π»ΠΈΠ½Ρ‹ ΠΏΡ€ΠΎΠ²ΠΎΠ΄Π° Π΅ΡΡ‚ΡŒ двиТущиСся заряды. Π˜Ρ‚Π°ΠΊ, Ссли ΠΏΡ€Π΅Π΄ΠΏΠΎΠ»ΠΎΠΆΠΈΡ‚ΡŒ Ρ‡Ρ‚ΠΎ Π½Π° ΠΊΠ°ΠΆΠ΄Ρ‹ΠΉ заряд дСйствуСт одинаковая сила ΠΌΠ°Π³Π½ΠΈΡ‚Π½ΠΎΠ³ΠΎ поля (ΠΌΡ‹ ΠΈΠΌΠ΅Π΅ΠΌ Π½Π΅Ρ‚ ΠΏΡ€ΠΈΡ‡ΠΈΠ½ ΠΏΡ€Π΅Π΄ΠΏΠΎΠ»Π°Π³Π°Ρ‚ΡŒ ΠΎΠ±Ρ€Π°Ρ‚Π½ΠΎΠ΅) сила, Π΄Π΅ΠΉΡΡ‚Π²ΡƒΡŽΡ‰Π°Ρ Π½Π° ΠΎΡ‚Π΄Π΅Π»ΡŒΠ½Ρ‹ΠΉ заряд, Ρ€Π°Π²Π½Π°
(233)

ΠœΡ‹ ΠΌΠΎΠΆΠ΅ΠΌ ΠΎΠ±ΡŠΠ΅Π΄ΠΈΠ½ΠΈΡ‚ΡŒ это с Ρ„ΠΎΡ€ΠΌΡƒΠ»ΠΎΠΉ.(169) Π΄Π°Ρ‚ΡŒ силу, Π΄Π΅ΠΉΡΡ‚Π²ΡƒΡŽΡ‰ΡƒΡŽ Π½Π° двиТущийся заряд со ΡΠΊΠΎΡ€ΠΎΡΡ‚ΡŒΡŽ Π² элСктричСском ΠΏΠΎΠ»Π΅ ΠΈ ΠΌΠ°Π³Π½ΠΈΡ‚Π½ΠΎΠΌ ΠΏΠΎΠ»Π΅ :
(234)

Π­Ρ‚ΠΎ называСтся Π·Π°ΠΊΠΎΠ½ силы Π›ΠΎΡ€Π΅Π½Ρ†Π° , Π² Ρ‡Π΅ΡΡ‚ΡŒ голландского Ρ„ΠΈΠ·ΠΈΠΊΠ°. Π₯Π΅Π½Π΄Ρ€ΠΈΠΊ Антун Π›ΠΎΡ€Π΅Π½Ρ†, ΠΊΠΎΡ‚ΠΎΡ€Ρ‹ΠΉ ΠΏΠ΅Ρ€Π²Ρ‹ΠΌ Π΅Π΅ сформулировал. ЭлСктричСский сила, Π΄Π΅ΠΉΡΡ‚Π²ΡƒΡŽΡ‰Π°Ρ Π½Π° Π·Π°Ρ€ΡΠΆΠ΅Π½Π½ΡƒΡŽ частицу, ΠΏΠ°Ρ€Π°Π»Π»Π΅Π»ΡŒΠ½Π° Π»ΠΎΠΊΠ°Π»ΡŒΠ½ΠΎΠΌΡƒ элСктричСскому полю. Однако магнитная сила пСрпСндикулярна ΠΊΠ°ΠΊ мСстному ΠΌΠ°Π³Π½ΠΈΡ‚Π½ΠΎΠΌΡƒ полю. ΠΏΠΎΠ»Π΅ ΠΈ Π½Π°ΠΏΡ€Π°Π²Π»Π΅Π½ΠΈΠ΅ двиТСния частицы.ΠœΠ°Π³Π½ΠΈΡ‚Π½Π°Ρ сила Π½Π΅ дСйствуСт Π½Π° нСподвиТная заряТСнная частица.

Π³. ΡƒΡ€Π°Π²Π½Π΅Π½ΠΈΠ΅ двиТСния свободной частицы заряда ΠΈ ΠΏΠ΅Ρ€Π΅ΠΌΠ΅Ρ‰Π΅Π½ΠΈΠ΅ массы Π² элСктричСском ΠΈ ΠΌΠ°Π³Π½ΠΈΡ‚Π½Ρ‹Π΅ поля

(235)

согласно Π·Π°ΠΊΠΎΠ½Ρƒ силы Π›ΠΎΡ€Π΅Π½Ρ†Π°. Π­Ρ‚ΠΎ ΡƒΡ€Π°Π²Π½Π΅Π½ΠΈΠ΅ двиТСния Π±Ρ‹Π»ΠΎ Π²ΠΏΠ΅Ρ€Π²Ρ‹Π΅ ΠΏΡ€ΠΎΠ²Π΅Ρ€Π΅Π½ΠΎ Π² извСстном экспСримСнтС, ΠΏΡ€ΠΎΠ²Π΅Π΄Π΅Π½Π½ΠΎΠΌ ΠšΠ΅ΠΌΠ±Ρ€ΠΈΠ΄ΠΆΡΠΊΠΈΠΌ Ρ„ΠΈΠ·ΠΈΠΊΠΎΠΌ Π”ΠΆ. Π”ΠΆ. Вомпсон Π² 1897 Π³ΠΎΠ΄Ρƒ. Вомпсон ΠΏΡ€ΠΎΠ²ΠΎΠ΄ΠΈΠ» расслСдованиС ΠΊΠ°Ρ‚ΠΎΠ΄Π½Ρ‹Π΅ Π»ΡƒΡ‡ΠΈ , Ρ‚ΠΎΠ³Π΄Π° таинствСнная Ρ„ΠΎΡ€ΠΌΠ° излучСния, испускаСмого Π½Π°Π³Ρ€Π΅Ρ‚Ρ‹ΠΌ мСталличСский элСмСнт, находящийся ΠΏΠΎΠ΄ большим ΠΎΡ‚Ρ€ΠΈΡ†Π°Ρ‚Π΅Π»ΡŒΠ½Ρ‹ΠΌ напряТСниСм ( i.Π΅. , ΠΊΠ°Ρ‚ΠΎΠ΄) с ΡƒΡ‡Π΅Ρ‚ΠΎΠΌ ΠΊ Π΄Ρ€ΡƒΠ³ΠΎΠΌΡƒ мСталличСскому элСмСнту (, Ρ‚.Π΅. , Π°Π½ΠΎΠ΄) Π² ΠΎΡ‚ΠΊΠ°Ρ‡Π°Π½Π½ΠΎΠΉ Ρ‚Ρ€ΡƒΠ±ΠΊΠ΅. НСмСцкиС Ρ„ΠΈΠ·ΠΈΠΊΠΈ считали, Ρ‡Ρ‚ΠΎ ΠΊΠ°Ρ‚ΠΎΠ΄Π½Ρ‹Π΅ Π»ΡƒΡ‡ΠΈ Ρ„ΠΎΡ€ΠΌΠ° элСктромагнитного излучСния, Π² Ρ‚ΠΎ врСмя ΠΊΠ°ΠΊ британскиС ΠΈ французскиС Ρ„ΠΈΠ·ΠΈΠΊΠΈ ΠΏΠΎΠ΄ΠΎΠ·Ρ€Π΅Π²Π°Π»ΠΈ Ρ‡Ρ‚ΠΎ ΠΎΠ½ΠΈ Π½Π° самом Π΄Π΅Π»Π΅ Π±Ρ‹Π»ΠΈ ΠΏΠΎΡ‚ΠΎΠΊΠΎΠΌ заряТСнных частиц. Вомпсон смог Π΄Π΅ΠΌΠΎΠ½ΡΡ‚Ρ€ΠΈΡ€ΡƒΡŽΡ‚, Ρ‡Ρ‚ΠΎ послСднСС ΠΌΠ½Π΅Π½ΠΈΠ΅ Π±Ρ‹Π»ΠΎ ΠΏΡ€Π°Π²ΠΈΠ»ΡŒΠ½Ρ‹ΠΌ. Π’ экспСримСнтС Вомпсона ΠΊΠ°Ρ‚ΠΎΠ΄Π½Ρ‹Π΅ Π»ΡƒΡ‡ΠΈ ΠΏΡ€ΠΎΡ…ΠΎΠ΄ΠΈΠ»ΠΈ Ρ‡Π΅Ρ€Π΅Π· ΠΎΠ±Π»Π°ΡΡ‚ΡŒ « скрСщСнных » элСктричСских ΠΈ ΠΌΠ°Π³Π½ΠΈΡ‚Π½Ρ‹Ρ… поля (всС Π΅Ρ‰Π΅ Π² Π²Π°ΠΊΡƒΡƒΠΌΠ΅). Поля Π±Ρ‹Π»ΠΈ пСрпСндикулярны исходному Ρ‚Ρ€Π°Π΅ΠΊΡ‚ΠΎΡ€ΠΈΠΈ Π»ΡƒΡ‡Π΅ΠΉ, Π° Ρ‚Π°ΠΊΠΆΠ΅ Π±Ρ‹Π»ΠΈ Π²Π·Π°ΠΈΠΌΠ½ΠΎ пСрпСндикулярны.

Π”Π°Π²Π°ΠΉΡ‚Π΅ ΠΏΡ€ΠΎΠ°Π½Π°Π»ΠΈΠ·ΠΈΡ€ΡƒΠ΅ΠΌ экспСримСнт Вомпсона. ΠŸΡ€Π΅Π΄ΠΏΠΎΠ»ΠΎΠΆΠΈΠΌ, Ρ‡Ρ‚ΠΎ Π»ΡƒΡ‡ΠΈ ΠΈΠ·Π½Π°Ρ‡Π°Π»ΡŒΠ½ΠΎ двиТутся Π² -Π½Π°ΠΏΡ€Π°Π²Π»Π΅Π½ΠΈΠΈ ΠΈ ΠΏΠΎΠ΄Π²Π΅Ρ€ΠΆΠ΅Π½Ρ‹ влиянию ΠΎΠ΄Π½ΠΎΡ€ΠΎΠ΄Π½ΠΎΠ΅ элСктричСскоС ΠΏΠΎΠ»Π΅ Π² -Π½Π°ΠΏΡ€Π°Π²Π»Π΅Π½ΠΈΠΈ ΠΈ ΠΎΠ΄Π½ΠΎΡ€ΠΎΠ΄Π½ΠΎΠ΅ ΠΌΠ°Π³Π½ΠΈΡ‚Π½ΠΎΠ΅ ΠΏΠΎΠ»Π΅. ΠΏΠΎΠ»Π΅ Π² -Π½Π°ΠΏΡ€Π°Π²Π»Π΅Π½ΠΈΠΈ. ΠŸΡ€Π΅Π΄ΠΏΠΎΠ»ΠΎΠΆΠΈΠΌ, ΠΊΠ°ΠΊ это сдСлал Вомпсон, Ρ‡Ρ‚ΠΎ ΠΊΠ°Ρ‚ΠΎΠ΄ Π»ΡƒΡ‡ΠΈ — это ΠΏΠΎΡ‚ΠΎΠΊ частиц массы ΠΈ заряда. Π’ ΡƒΡ€Π°Π²Π½Π΅Π½ΠΈΠ΅ двиТСния частиц Π² -Π½Π°ΠΏΡ€Π°Π²Π»Π΅Π½ΠΈΠΈ ΠΈΠΌΠ΅Π΅Ρ‚ Π²ΠΈΠ΄

(236)

Π³Π΄Π΅ — ΡΠΊΠΎΡ€ΠΎΡΡ‚ΡŒ частиц Π² -Π½Π°ΠΏΡ€Π°Π²Π»Π΅Π½ΠΈΠΈ.Вомпсон Π½Π°Ρ‡Π°Π» свой экспСримСнт с Ρ‚ΠΎΠ»ΡŒΠΊΠΎ Π²ΠΊΠ»ΡŽΡ‡ΠΈΠ² элСктричСскоС ΠΏΠΎΠ»Π΅ Π² своСм Π°ΠΏΠΏΠ°Ρ€Π°Ρ‚Π΅, ΠΈ ΠΈΠ·ΠΌΠ΅Ρ€Π΅Π½ΠΈΠ΅ ΠΎΡ‚ΠΊΠ»ΠΎΠ½Π΅Π½ΠΈΠ΅ Π»ΡƒΡ‡Π° Π² -Π½Π°ΠΏΡ€Π°Π²Π»Π΅Π½ΠΈΠΈ послС Ρ‚ΠΎΠ³ΠΎ, ΠΊΠ°ΠΊ ΠΎΠ½ ΠΏΡ€ΠΎΡˆΠ΅Π» расстояниС Ρ‡Π΅Ρ€Π΅Π· элСктричСскоС ΠΏΠΎΠ»Π΅. Из уравнСния Π²ΠΈΠ΄Π½ΠΎ двиТСния, ΠΊΠΎΡ‚ΠΎΡ€ΠΎΠ΅
(237)

Π³Π΄Π΅ « врСмя ΠΏΠΎΠ»Π΅Ρ‚Π° » Π·Π°ΠΌΠ΅Π½Π΅Π½ΠΎ Π½Π°. Π­Ρ‚Π° Ρ„ΠΎΡ€ΠΌΡƒΠ»Π° Ρ‚ΠΎΠ»ΡŒΠΊΠΎ Π΄Π΅ΠΉΡΡ‚Π²ΠΈΡ‚Π΅Π»ΡŒΠ½ΠΎ, Ссли, Ρ‡Ρ‚ΠΎ прСдполагаСтся. Π—Π°Ρ‚Π΅ΠΌ Вомпсон Π²ΠΊΠ»ΡŽΡ‡ΠΈΠ» ΠΌΠ°Π³Π½ΠΈΡ‚Π½ΠΎΠ΅ ΠΏΠΎΠ»Π΅ Π² Π΅Π³ΠΎ Π°ΠΏΠΏΠ°Ρ€Π°Ρ‚Π΅, ΠΈ ΠΎΡ‚Ρ€Π΅Π³ΡƒΠ»ΠΈΡ€ΠΎΠ²Π°Π» Π΅Π³ΠΎ Ρ‚Π°ΠΊ, Ρ‡Ρ‚ΠΎΠ±Ρ‹ ΠΊΠ°Ρ‚ΠΎΠ΄Π½Ρ‹ΠΉ Π»ΡƒΡ‡ Π±Ρ‹Π» большС Π½Π΅ отклоняСтся.ΠžΡ‚ΡΡƒΡ‚ΡΡ‚Π²ΠΈΠ΅ отклонСния ΠΎΠ·Π½Π°Ρ‡Π°Π΅Ρ‚, Ρ‡Ρ‚ΠΎ Ρ€Π΅Π·ΡƒΠ»ΡŒΡ‚ΠΈΡ€ΡƒΡŽΡ‰Π°Ρ сила, Π΄Π΅ΠΉΡΡ‚Π²ΡƒΡŽΡ‰Π°Ρ Π½Π° частиц Π² -Π½Π°ΠΏΡ€Π°Π²Π»Π΅Π½ΠΈΠΈ Π±Ρ‹Π»ΠΎ Ρ€Π°Π²Π½ΠΎ Π½ΡƒΠ»ΡŽ. Π”Ρ€ΡƒΠ³ΠΈΠΌΠΈ словами, элСктричСскиС ΠΈ ΠΌΠ°Π³Π½ΠΈΡ‚Π½Ρ‹Π΅ силы Ρ‚ΠΎΡ‡Π½ΠΎ сбалансированы. Из уравнСния (236) Ρ‡Ρ‚ΠΎ ΠΏΡ€ΠΈ ΠΏΡ€Π°Π²ΠΈΠ»ΡŒΠ½ΠΎ настроСнной напряТСнности ΠΌΠ°Π³Π½ΠΈΡ‚Π½ΠΎΠ³ΠΎ поля
(238)

Π’Π°ΠΊΠΈΠΌ ΠΎΠ±Ρ€Π°Π·ΠΎΠΌ, уравнСния. (237) ΠΈ (238) ΠΈ ΠΌΠΎΠ³ΡƒΡ‚ Π±Ρ‹Ρ‚ΡŒ ΠΎΠ±ΡŠΠ΅Π΄ΠΈΠ½Π΅Π½Ρ‹ ΠΈ ΠΏΠ΅Ρ€Π΅Π³Ρ€ΡƒΠΏΠΏΠΈΡ€ΠΎΠ²Π°Π½Ρ‹, Ρ‡Ρ‚ΠΎΠ±Ρ‹ ΠΏΠΎΠ»ΡƒΡ‡ΠΈΡ‚ΡŒ ΠΎΡ‚Π½ΠΎΡˆΠ΅Π½ΠΈΠ΅ заряда ΠΊ массС частицы Π² Π΅Π΄ΠΈΠ½ΠΈΡ†Π°Ρ… измСрСния:
(239)

Π˜ΡΠΏΠΎΠ»ΡŒΠ·ΡƒΡ этот ΠΌΠ΅Ρ‚ΠΎΠ΄, Вомпсон сдСлал Π²Ρ‹Π²ΠΎΠ΄, Ρ‡Ρ‚ΠΎ ΠΊΠ°Ρ‚ΠΎΠ΄Π½Ρ‹Π΅ Π»ΡƒΡ‡ΠΈ состоят ΠΈΠ· ΠΎΡ‚Ρ€ΠΈΡ†Π°Ρ‚Π΅Π»ΡŒΠ½ΠΎ заряТСнныС частицы (Π·Π½Π°ΠΊ заряда Π²ΠΈΠ΄Π΅Π½ ΠΈΠ· Π½Π°ΠΏΡ€Π°Π²Π»Π΅Π½ΠΈΠ΅ отклонСния Π² элСктричСском ΠΏΠΎΠ»Π΅) с зарядом ΠΊ массС ΡΠΎΠΎΡ‚Π½ΠΎΡˆΠ΅Π½ΠΈΠ΅ Кл / ΠΊΠ³.Π”Π΅ΡΡΡ‚ΡŒ Π»Π΅Ρ‚ спустя, Π² 1908 Π³ΠΎΠ΄Ρƒ, Π°ΠΌΠ΅Ρ€ΠΈΠΊΠ°Π½Π΅Ρ† Π ΠΎΠ±Π΅Ρ€Ρ‚ МилликСн ΠΏΡ€ΠΎΠ²Π΅Π» свой Π·Π½Π°ΠΌΠ΅Π½ΠΈΡ‚Ρ‹ΠΉ экспСримСнт с « ΠΊΠ°ΠΏΠ»Π΅ΠΉ масла » ΠΈ ΠΎΠ±Π½Π°Ρ€ΡƒΠΆΠΈΠ», Ρ‡Ρ‚ΠΎ ΠΌΠΎΠ±ΠΈΠ»ΡŒΠ½Ρ‹Π΅ элСктричСскиС заряды ΠΊΠ²Π°Π½Ρ‚ΡƒΡŽΡ‚ΡΡ Π² Π΅Π΄ΠΈΠ½ΠΈΡ†Π°Ρ… Π‘. ΠŸΡ€Π΅Π΄ΠΏΠΎΠ»Π°Π³Π°Ρ, Ρ‡Ρ‚ΠΎ ΠΌΠΎΠ±ΠΈΠ»ΡŒΠ½Ρ‹Π΅ элСктричСскиС заряды ΠΈ частицы, ΠΊΠΎΡ‚ΠΎΡ€Ρ‹Π΅ ΡΠΎΡΡ‚Π°Π²Π»ΡΡ‚ΡŒ ΠΊΠ°Ρ‚ΠΎΠ΄Π½Ρ‹Π΅ Π»ΡƒΡ‡ΠΈ ΠΎΠ΄Π½ΠΎ ΠΈ Ρ‚ΠΎ ΠΆΠ΅, ЭкспСримСнты Вомпсона ΠΈ МилликСна ΠΏΡ€Π΅Π΄ΠΏΠΎΠ»Π°Π³Π°ΡŽΡ‚, Ρ‡Ρ‚ΠΎ масса этих частиц ΠΊΠ³. ΠšΠΎΠ½Π΅Ρ‡Π½ΠΎ, это масса элСктрон (соврСмСнноС Π·Π½Π°Ρ‡Π΅Π½ΠΈΠ΅ ΠΊΠ³), ΠΈ C — заряд элСктрона. Π’Π°ΠΊΠΈΠΌ ΠΎΠ±Ρ€Π°Π·ΠΎΠΌ, ΠΊΠ°Ρ‚ΠΎΠ΄Π½Ρ‹Π΅ Π»ΡƒΡ‡ΠΈ, ΠΏΠΎ сути, ΡΠ²Π»ΡΡŽΡ‚ΡΡ ΠΏΠΎΡ‚ΠΎΠΊΠΈ элСктронов, ΠΊΠΎΡ‚ΠΎΡ€Ρ‹Π΅ Π²Ρ‹Π»Π΅Ρ‚Π°ΡŽΡ‚ ΠΈΠ· Π½Π°Π³Ρ€Π΅Ρ‚ΠΎΠ³ΠΎ ΠΊΠ°Ρ‚ΠΎΠ΄Π°, Π° Π·Π°Ρ‚Π΅ΠΌ ускоряСтся ΠΈΠ·-Π·Π° большой Ρ€Π°Π·Π½ΠΈΡ†Ρ‹ напряТСний ΠΌΠ΅ΠΆΠ΄Ρƒ ΠΊΠ°Ρ‚ΠΎΠ΄ΠΎΠΌ ΠΈ Π°Π½ΠΎΠ΄ΠΎΠΌ.

Π’Π΅ΠΏΠ΅Ρ€ΡŒ рассмотрим частицу массы ΠΈ заряда, Π΄Π²ΠΈΠΆΡƒΡ‰ΡƒΡŽΡΡ Π² ΠΎΠ΄Π½ΠΎΡ€ΠΎΠ΄Π½ΠΎΠΉ ΠΌΠ°Π³Π½ΠΈΡ‚Π½ΠΎΠ΅ ΠΏΠΎΠ»Π΅, . Π’ соотвСтствии с Π£Ρ€Π°Π²Π½Π΅Π½ΠΈΠ΅ (235) ΡƒΡ€Π°Π²Π½Π΅Π½ΠΈΠ΅ двиТСния частицы ΠΌΠΎΠΆΠ½ΠΎ Π·Π°ΠΏΠΈΡΠ°Ρ‚ΡŒ:

(240)

Π­Ρ‚ΠΎ сводится ΠΊ

Π—Π΄Π΅ΡΡŒ, называСтся Ρ†ΠΈΠΊΠ»ΠΎΡ‚Ρ€ΠΎΠ½Π½ΠΎΠΉ частотой . ΠŸΡ€ΠΈΠ²Π΅Π΄Π΅Π½Π½Ρ‹Π΅ Π²Ρ‹ΡˆΠ΅ уравнСния ΠΌΠΎΠ³ΡƒΡ‚ Π±Ρ‹Ρ‚ΡŒ Ρ€Π΅ΡˆΠ΅Π½Ρ‹, Ρ‡Ρ‚ΠΎΠ±Ρ‹ Π΄Π°Ρ‚ΡŒ

Π° Ρ‚Π°ΠΊΠΆΠ΅

Богласно этим уравнСниям траСктория частицы прСдставляСт собой ΡΠΏΠΈΡ€Π°Π»ΡŒ ось ΠΊΠΎΡ‚ΠΎΡ€ΠΎΠ³ΠΎ ΠΏΠ°Ρ€Π°Π»Π»Π΅Π»ΡŒΠ½Π° ΠΌΠ°Π³Π½ΠΈΡ‚Π½ΠΎΠΌΡƒ полю.Радиус ΡΠΏΠΈΡ€Π°Π»ΡŒ , Π³Π΄Π΅ постоянная ΡΠΊΠΎΡ€ΠΎΡΡ‚ΡŒ Π² плоскости, пСрпСндикулярной ΠΌΠ°Π³Π½ΠΈΡ‚Π½ΠΎΠΌΡƒ полю. Частица Π΄Ρ€Π΅ΠΉΡ„ΡƒΠ΅Ρ‚ ΠΏΠ°Ρ€Π°Π»Π»Π΅Π»ΡŒΠ½ΠΎ ΠΌΠ°Π³Π½ΠΈΡ‚Π½ΠΎΠΌΡƒ полю с постоянной ΡΠΊΠΎΡ€ΠΎΡΡ‚ΡŒΡŽ,. НаконСц-Ρ‚ΠΎ, частица вращаСтся Π² плоскости, пСрпСндикулярной ΠΌΠ°Π³Π½ΠΈΡ‚Π½ΠΎΠΌΡƒ полю Π½Π° Ρ†ΠΈΠΊΠ»ΠΎΡ‚Ρ€ΠΎΠ½Π΅ частота.

НаконСц, Ссли частица подвСргаСтся Π΄Π΅ΠΉΡΡ‚Π²ΠΈΡŽ силы ΠΈ пСрСмСщаСтся Π½Π° расстояниС Π² ΠΈΠ½Ρ‚Π΅Ρ€Π²Π°Π»Π΅ Π²Ρ€Π΅ΠΌΠ΅Π½ΠΈ, Ρ‚ΠΎ Ρ€Π°Π±ΠΎΡ‚Π°, ΡΠΎΠ²Π΅Ρ€ΡˆΠ°Π΅ΠΌΠ°Ρ Π½Π°Π΄ частицСй сила

(250)

Подвод мощности ΠΊ частицС ΠΎΡ‚ силового поля Ρ€Π°Π²Π΅Π½
(251)

Π³Π΄Π΅ — ΡΠΊΠΎΡ€ΠΎΡΡ‚ΡŒ частицы.Из силы Π›ΠΎΡ€Π΅Π½Ρ†Π° слСдуСт Π·Π°ΠΊΠΎΠ½, ΡƒΡ€Π°Π²Π½Π΅Π½ΠΈΠ΅. (234), Ρ‡Ρ‚ΠΎ ΠΏΠΎΠ΄Π²ΠΎΠ΄ энСргии ΠΊ частицС, двиТущСйся Π² элСктричСском ΠΈ ΠΌΠ°Π³Π½ΠΈΡ‚Π½ΠΎΠΌ поля
(252)

ΠžΠ±Ρ€Π°Ρ‚ΠΈΡ‚Π΅ Π²Π½ΠΈΠΌΠ°Π½ΠΈΠ΅, Ρ‡Ρ‚ΠΎ заряТСнная частица ΠΌΠΎΠΆΠ΅Ρ‚ ΠΏΠΎΠ»ΡƒΡ‡Π°Ρ‚ΡŒ (ΠΈΠ»ΠΈ Ρ‚Π΅Ρ€ΡΡ‚ΡŒ) ΡΠ½Π΅Ρ€Π³ΠΈΡŽ ΠΎΡ‚ элСктричСского ΠΏΠΎΠ»Π΅, Π½ΠΎ Π½Π΅ ΠΎΡ‚ ΠΌΠ°Π³Π½ΠΈΡ‚Π½ΠΎΠ³ΠΎ поля. Π­Ρ‚ΠΎ ΠΏΠΎΡ‚ΠΎΠΌΡƒ, Ρ‡Ρ‚ΠΎ магнитная сила всСгда пСрпСндикулярно Π½Π°ΠΏΡ€Π°Π²Π»Π΅Π½ΠΈΡŽ двиТСния частицы ΠΈ, ΡΠ»Π΅Π΄ΠΎΠ²Π°Ρ‚Π΅Π»ΡŒΠ½ΠΎ, Π΄Π΅Π»Π°Π΅Ρ‚ Π½Π΅Ρ‚ Ρ€Π°Π±ΠΎΡ‚Ρ‹ Π½Π° частицС [см. (250)]. Π’Π°ΠΊ, Π² ускоритСлях частиц ΠΌΠ°Π³Π½ΠΈΡ‚Π½Ρ‹Π΅ поля часто ΠΈΡΠΏΠΎΠ»ΡŒΠ·ΡƒΡŽΡ‚ΡΡ для направлСния двиТСния частиц ( e.Π³Ρ€Π°ΠΌΠΌ. , ΠΏΠΎ ΠΊΡ€ΡƒΠ³Ρƒ), Π½ΠΎ ЀактичСскоС ускорСниС осущСствляСтся элСктричСскими полями.

Π”Π°Π»Π΅Π΅: Π—Π°ΠΊΠΎΠ½ АмпСра Up: НС зависящиС ΠΎΡ‚ Π²Ρ€Π΅ΠΌΠ΅Π½ΠΈ уравнСния МаксвСлла ΠŸΡ€Π΅Π΄Ρ‹Π΄ΡƒΡ‰Π°Ρ: ЭкспСримСнты АмпСра
Π ΠΈΡ‡Π°Ρ€Π΄ Π€ΠΈΡ†ΠΏΠ°Ρ‚Ρ€ΠΈΠΊ 2006-02-02

EEE 403 ЛСкция 7: ΠœΠ°Π³Π½ΠΈΡ‚ΠΎΡΡ‚Π°Ρ‚ΠΈΠΊΠ°: Π·Π°ΠΊΠΎΠ½ силы АмпСра; ΠŸΠ»ΠΎΡ‚Π½ΠΎΡΡ‚ΡŒ ΠΌΠ°Π³Π½ΠΈΡ‚Π½ΠΎΠ³ΠΎ ΠΏΠΎΡ‚ΠΎΠΊΠ°; Π›ΠΎΡ€Π΅Π½Ρ† Ѐорс; Π—Π°ΠΊΠΎΠ½ Π‘ΠΈΠΎ-Π‘Π°Π²Π°Ρ€Ρ‚Π°; ΠŸΡ€ΠΈΠΌΠ΅Π½Π΅Π½ΠΈΠ΅ Π·Π°ΠΊΠΎΠ½Π° АмпСра Π² ΠΈΠ½Ρ‚Π΅Π³Ρ€Π°Π»Π΅.

ΠŸΡ€Π΅Π·Π΅Π½Ρ‚Π°Ρ†ΠΈΡ Π½Π° Ρ‚Π΅ΠΌΡƒ: Β«EEE 403 ЛСкция 7: ΠœΠ°Π³Π½ΠΈΡ‚ΠΎΡΡ‚Π°Ρ‚ΠΈΠΊΠ°: Π—Π°ΠΊΠΎΠ½ силы АмпСра; ΠŸΠ»ΠΎΡ‚Π½ΠΎΡΡ‚ΡŒ ΠΌΠ°Π³Π½ΠΈΡ‚Π½ΠΎΠ³ΠΎ ΠΏΠΎΡ‚ΠΎΠΊΠ°; Π‘ΠΈΠ»Π° Π›ΠΎΡ€Π΅Π½Ρ†Π°; Π—Π°ΠΊΠΎΠ½ Π‘ΠΈΠΎ-Π‘Π°Π²Π°Ρ€Ρ‚Π°; ΠŸΡ€ΠΈΠΌΠ΅Π½Π΅Π½ΠΈΠ΅ Π·Π°ΠΊΠΎΠ½Π° АмпСра Π² ΠΈΠ½Ρ‚Π΅Π³Ρ€Π°Π»Π΅.Β»- стСнограмма ΠΏΡ€Π΅Π·Π΅Π½Ρ‚Π°Ρ†ΠΈΠΈ:

ins [data-ad-slot = «4502451947»] {display: none! important;}} @media (max-width: 800px) {# place_14> ins: not ([data-ad-slot = «4502451947»]) {display: none! important;}} @media (max-width: 800px) {# place_14 {width: 250px;}} @media (max-width: 500 пиксСлСй) {# place_14 {width: 120px;}} ]]>

1 EEE 403 ЛСкция 7: ΠœΠ°Π³Π½ΠΈΡ‚ΠΎΡΡ‚Π°Ρ‚ΠΈΠΊΠ°: Π·Π°ΠΊΠΎΠ½ силы АмпСра; ΠŸΠ»ΠΎΡ‚Π½ΠΎΡΡ‚ΡŒ ΠΌΠ°Π³Π½ΠΈΡ‚Π½ΠΎΠ³ΠΎ ΠΏΠΎΡ‚ΠΎΠΊΠ°; Π›ΠΎΡ€Π΅Π½Ρ† Ѐорс; Π—Π°ΠΊΠΎΠ½ Π‘ΠΈΠΎ-Π‘Π°Π²Π°Ρ€Ρ‚Π°; ΠŸΡ€ΠΈΠΌΠ΅Π½Π΅Π½ΠΈΠ΅ Π·Π°ΠΊΠΎΠ½Π° АмпСра Π² ΠΈΠ½Ρ‚Π΅Π³Ρ€Π°Π»ΡŒΠ½ΠΎΠΉ Ρ„ΠΎΡ€ΠΌΠ΅; Π’Π΅ΠΊΡ‚ΠΎΡ€Π½Ρ‹ΠΉ ΠΌΠ°Π³Π½ΠΈΡ‚Π½Ρ‹ΠΉ ΠΏΠΎΡ‚Π΅Π½Ρ†ΠΈΠ°Π»; ΠœΠ°Π³Π½ΠΈΡ‚Π½Ρ‹ΠΉ диполь; ΠœΠ°Π³Π½ΠΈΡ‚Π½Ρ‹ΠΉ ΠΏΠΎΡ‚ΠΎΠΊ 1

2 ЛСкция 7 Π¦Π΅Π»ΠΈ ΠΠ°Ρ‡Π°Ρ‚ΡŒ ΠΈΠ·ΡƒΡ‡Π΅Π½ΠΈΠ΅ магнитостатики с Π·Π°ΠΊΠΎΠ½Π° силы АмпСра; ΠΏΠ»ΠΎΡ‚Π½ΠΎΡΡ‚ΡŒ ΠΌΠ°Π³Π½ΠΈΡ‚Π½ΠΎΠ³ΠΎ ΠΏΠΎΡ‚ΠΎΠΊΠ°; Π‘ΠΈΠ»Π° Π›ΠΎΡ€Π΅Π½Ρ†Π°; Π—Π°ΠΊΠΎΠ½ Π‘ΠΈΠΎ-Π‘Π°Π²Π°Ρ€Π°; примСнСния Π·Π°ΠΊΠΎΠ½Π° АмпСра Π² ΠΈΠ½Ρ‚Π΅Π³Ρ€Π°Π»ΡŒΠ½ΠΎΠΉ Ρ„ΠΎΡ€ΠΌΠ΅; Π²Π΅ΠΊΡ‚ΠΎΡ€Π½Ρ‹ΠΉ ΠΌΠ°Π³Π½ΠΈΡ‚Π½Ρ‹ΠΉ ΠΏΠΎΡ‚Π΅Π½Ρ†ΠΈΠ°Π»; ΠΌΠ°Π³Π½ΠΈΡ‚Π½Ρ‹ΠΉ диполь; ΠΈ ΠΌΠ°Π³Π½ΠΈΡ‚Π½Ρ‹ΠΉ ΠΏΠΎΡ‚ΠΎΠΊ.2

3 ΠžΠ±Π·ΠΎΡ€ элСктромагнСтизма
Π€ΡƒΠ½Π΄Π°ΠΌΠ΅Π½Ρ‚Π°Π»ΡŒΠ½Ρ‹Π΅ Π·Π°ΠΊΠΎΠ½Ρ‹ классичСской элСктромагнСтизма УравнСния МаксвСлла ЧастныС случаи ЭлСктростатика ΠœΠ°Π³Π½ΠΈΡ‚ΠΎΡΡ‚Π°Ρ‚ΠΈΠΊΠ° ГСомСтричСская ΠΎΠΏΡ‚ΠΈΠΊΠ° Π­Π»Π΅ΠΊΡ‚Ρ€ΠΎΠΌΠ°Π³Π½ΠΈΡ‚Π½Ρ‹Π΅ Π²ΠΎΠ»Π½Ρ‹ Π‘Ρ‚Π°Ρ‚ΠΈΠΊΠ°: тСория Π»ΠΈΠ½ΠΈΠΉ ΠΏΠ΅Ρ€Π΅Π΄Π°Ρ‡ΠΈ Π˜ΡΡ…ΠΎΠ΄Π½Ρ‹Π΅ Π΄Π°Π½Π½Ρ‹Π΅ ΠΈΠ· Π΄Ρ€ΡƒΠ³ΠΈΡ… дисциплин ВСория Ρ†Π΅ΠΏΠ΅ΠΉ Π—Π°ΠΊΠΎΠ½Ρ‹ ΠšΠΈΡ€Ρ…Π³ΠΎΡ„Π° 3

4 ΠœΠ°Π³Π½ΠΈΡ‚ΠΎΡΡ‚Π°Ρ‚ΠΈΠΊΠ° ΠœΠ°Π³Π½ΠΈΡ‚ΠΎΡΡ‚Π°Ρ‚ΠΈΠΊΠ° — это Ρ€Π°Π·Π΄Π΅Π» элСктромагнСтизма, ΠΈΠΌΠ΅ΡŽΡ‰ΠΈΠΉ Π΄Π΅Π»ΠΎ с эффСктами элСктричСских зарядов Π² ΡƒΡΡ‚Π°Π½ΠΎΠ²ΠΈΠ²ΡˆΠ΅ΠΌΡΡ Π΄Π²ΠΈΠΆΠ΅Π½ΠΈΠΈ (Ρ‚.Π΅, постоянный Ρ‚ΠΎΠΊ ΠΈΠ»ΠΈ постоянный Ρ‚ΠΎΠΊ). Основной Π·Π°ΠΊΠΎΠ½ магнитостатики — это Π·Π°ΠΊΠΎΠ½ силы АмпСра. Π—Π°ΠΊΠΎΠ½ силы АмпСра Π°Π½Π°Π»ΠΎΠ³ΠΈΡ‡Π΅Π½ Π·Π°ΠΊΠΎΠ½Ρƒ ΠšΡƒΠ»ΠΎΠ½Π° Π² элСктростатикС. 4

5 ΠœΠ°Π³Π½ΠΈΡ‚ΠΎΡΡ‚Π°Ρ‚ΠΈΠΊΠ° (ΠΏΡ€ΠΎΠ΄ΠΎΠ»ΠΆΠ΅Π½ΠΈΠ΅)
Π’ магнитостатикС ΠΌΠ°Π³Π½ΠΈΡ‚Π½ΠΎΠ΅ ΠΏΠΎΠ»Π΅ создаСтся постоянными Ρ‚ΠΎΠΊΠ°ΠΌΠΈ. ΠœΠ°Π³Π½ΠΈΡ‚ΠΎΡΡ‚Π°Ρ‚ΠΈΡ‡Π΅ΡΠΊΠΎΠ΅ ΠΏΠΎΠ»Π΅ Π½Π΅ допускаСт ΠΈΠ½Π΄ΡƒΠΊΡ‚ΠΈΠ²Π½ΠΎΠΉ связи ΠΌΠ΅ΠΆΠ΄Ρƒ цСпями связи ΠΌΠ΅ΠΆΠ΄Ρƒ элСктричСским ΠΈ ΠΌΠ°Π³Π½ΠΈΡ‚Π½Ρ‹ΠΌ полями 5

6 Π—Π°ΠΊΠΎΠ½ силы АмпСра Π—Π°ΠΊΠΎΠ½ силы АмпСра — это Β«Π·Π°ΠΊΠΎΠ½ дСйствия» ΠΌΠ΅ΠΆΠ΄Ρƒ Ρ‚ΠΎΠΊΠΎΠ²Π΅Π΄ΡƒΡ‰ΠΈΠΌΠΈ цСпями.Π—Π°ΠΊΠΎΠ½ силы АмпСра опрСдСляСт ΠΌΠ°Π³Π½ΠΈΡ‚Π½ΡƒΡŽ силу ΠΌΠ΅ΠΆΠ΄Ρƒ двумя ΠΊΠΎΠ½Ρ‚ΡƒΡ€Π°ΠΌΠΈ, нСсущими Ρ‚ΠΎΠΊ, Π² пустой всСлСнной. Π—Π°ΠΊΠΎΠ½ силы АмпСра Π²ΠΊΠ»ΡŽΡ‡Π°Π΅Ρ‚ Π·Π°ΠΌΠΊΠ½ΡƒΡ‚Ρ‹Π΅ Ρ†Π΅ΠΏΠΈ, ΠΏΠΎΡΠΊΠΎΠ»ΡŒΠΊΡƒ Ρ‚ΠΎΠΊ Π΄ΠΎΠ»ΠΆΠ΅Π½ Ρ‚Π΅Ρ‡ΡŒ ΠΏΠΎ Π·Π°ΠΌΠΊΠ½ΡƒΡ‚Ρ‹ΠΌ ΠΊΠΎΠ½Ρ‚ΡƒΡ€Π°ΠΌ. 6

7 Π—Π°ΠΊΠΎΠ½ силы АмпСра (ΠΏΡ€ΠΎΠ΄ΠΎΠ»ΠΆΠ΅Π½ΠΈΠ΅)
Π­ΠΊΡΠΏΠ΅Ρ€ΠΈΠΌΠ΅Π½Ρ‚Π°Π»ΡŒΠ½Ρ‹Π΅ Ρ„Π°ΠΊΡ‚Ρ‹: ΠΏΡ€ΠΈΡ‚ΡΠ³ΠΈΠ²Π°ΡŽΡ‚ΡΡ Π΄Π²Π° ΠΏΠ°Ρ€Π°Π»Π»Π΅Π»ΡŒΠ½Ρ‹Ρ… ΠΏΡ€ΠΎΠ²ΠΎΠ΄Π°, ΠΏΠΎ ΠΊΠΎΡ‚ΠΎΡ€Ρ‹ΠΌ Ρ‚Π΅Ρ‡Π΅Ρ‚ Ρ‚ΠΎΠΊ Π² ΠΎΠ΄Π½ΠΎΠΌ Π½Π°ΠΏΡ€Π°Π²Π»Π΅Π½ΠΈΠΈ. Π”Π²Π° ΠΏΠ°Ρ€Π°Π»Π»Π΅Π»ΡŒΠ½Ρ‹Ρ… ΠΏΡ€ΠΎΠ²ΠΎΠ΄Π°, ΠΏΠΎ ΠΊΠΎΡ‚ΠΎΡ€Ρ‹ΠΌ Ρ‚Π΅Ρ‡Π΅Ρ‚ Ρ‚ΠΎΠΊ Π² ΠΏΡ€ΠΎΡ‚ΠΈΠ²ΠΎΠΏΠΎΠ»ΠΎΠΆΠ½Ρ‹Ρ… направлСниях, ΠΎΡ‚Ρ‚Π°Π»ΠΊΠΈΠ²Π°ΡŽΡ‚ΡΡ.ο‚΄ I1 I2 F12 F21 ο‚΄ ο‚· I1 I2 F12 F21 7

8 Π—Π°ΠΊΠΎΠ½ силы АмпСра (ΠΏΡ€ΠΎΠ΄ΠΎΠ»ΠΆΠ΅Π½ΠΈΠ΅)
Π­ΠΊΡΠΏΠ΅Ρ€ΠΈΠΌΠ΅Π½Ρ‚Π°Π»ΡŒΠ½Ρ‹Π΅ Ρ„Π°ΠΊΡ‚Ρ‹: ΠšΠΎΡ€ΠΎΡ‚ΠΊΠΈΠΉ Ρ‚ΠΎΠΊΠΎΠ²Π΅Π΄ΡƒΡ‰ΠΈΠΉ ΠΏΡ€ΠΎΠ²ΠΎΠ΄, ΠΎΡ€ΠΈΠ΅Π½Ρ‚ΠΈΡ€ΠΎΠ²Π°Π½Π½Ρ‹ΠΉ пСрпСндикулярно Π΄Π»ΠΈΠ½Π½ΠΎΠΌΡƒ Ρ‚ΠΎΠΊΠΎΠ²Π΅Π΄ΡƒΡ‰Π΅ΠΌΡƒ ΠΏΡ€ΠΎΠ²ΠΎΠ΄Ρƒ, Π½Π΅ испытываСт силы. F12 = 0 ο‚΄ I2 I1 8

9 Π—Π°ΠΊΠΎΠ½ силы АмпСра (ΠΏΡ€ΠΎΠ΄ΠΎΠ»ΠΆΠ΅Π½ΠΈΠ΅)
Π­ΠΊΡΠΏΠ΅Ρ€ΠΈΠΌΠ΅Π½Ρ‚Π°Π»ΡŒΠ½Ρ‹Π΅ Ρ„Π°ΠΊΡ‚Ρ‹: Π²Π΅Π»ΠΈΡ‡ΠΈΠ½Π° силы ΠΎΠ±Ρ€Π°Ρ‚Π½ΠΎ ΠΏΡ€ΠΎΠΏΠΎΡ€Ρ†ΠΈΠΎΠ½Π°Π»ΡŒΠ½Π° ΠΊΠ²Π°Π΄Ρ€Π°Ρ‚Ρƒ расстояния.Π’Π΅Π»ΠΈΡ‡ΠΈΠ½Π° силы ΠΏΡ€ΠΎΠΏΠΎΡ€Ρ†ΠΈΠΎΠ½Π°Π»ΡŒΠ½Π° ΠΏΡ€ΠΎΠΈΠ·Π²Π΅Π΄Π΅Π½ΠΈΡŽ Ρ‚ΠΎΠΊΠΎΠ², пСрСносимых двумя ΠΏΡ€ΠΎΠ²ΠΎΠ΄Π°ΠΌΠΈ. 9

10 Π—Π°ΠΊΠΎΠ½ силы АмпСра (ΠΏΡ€ΠΎΠ΄ΠΎΠ»ΠΆΠ΅Π½ΠΈΠ΅)
НаправлСниС силы, установлСнноС ΡΠΊΡΠΏΠ΅Ρ€ΠΈΠΌΠ΅Π½Ρ‚Π°Π»ΡŒΠ½Ρ‹ΠΌΠΈ Ρ„Π°ΠΊΡ‚Π°ΠΌΠΈ, ΠΌΠΎΠΆΠ΅Ρ‚ Π±Ρ‹Ρ‚ΡŒ матСматичСски прСдставлСно Π΅Π΄ΠΈΠ½ΠΈΡ‡Π½Ρ‹ΠΌ Π²Π΅ΠΊΡ‚ΠΎΡ€ΠΎΠΌ Π² Π½Π°ΠΏΡ€Π°Π²Π»Π΅Π½ΠΈΠΈ Ρ‚ΠΎΠΊΠ° I1 Π΅Π΄ΠΈΠ½ΠΈΡ‡Π½Ρ‹ΠΉ Π²Π΅ΠΊΡ‚ΠΎΡ€ Π² Π½Π°ΠΏΡ€Π°Π²Π»Π΅Π½ΠΈΠΈ Ρ‚ΠΎΠΊΠ° I2 Π΅Π΄ΠΈΠ½ΠΈΡ‡Π½Ρ‹ΠΉ Π²Π΅ΠΊΡ‚ΠΎΡ€ Π² Π½Π°ΠΏΡ€Π°Π²Π»Π΅Π½ΠΈΠΈ силы Π½Π° I2 ΠΈΠ·-Π·Π° ΠΊ Π΅Π΄ΠΈΠ½ΠΈΡ‡Π½ΠΎΠΌΡƒ Π²Π΅ΠΊΡ‚ΠΎΡ€Ρƒ I1 Π² Π½Π°ΠΏΡ€Π°Π²Π»Π΅Π½ΠΈΠΈ I2 ΠΎΡ‚ I1 10

11 Π—Π°ΠΊΠΎΠ½ силы АмпСра (ΠΏΡ€ΠΎΠ΄ΠΎΠ»ΠΆΠ΅Π½ΠΈΠ΅)
Π‘ΠΈΠ»Π°, Π΄Π΅ΠΉΡΡ‚Π²ΡƒΡŽΡ‰Π°Ρ Π½Π° Ρ‚ΠΎΠΊΠΎΠ²Ρ‹ΠΉ элСмСнт I2 dl2 Ρ‚ΠΎΠΊΠΎΠ²Ρ‹ΠΌ элСмСнтом I1 dl1, опрСдСляСтся ΠΊΠ°ΠΊ ΠΏΡ€ΠΎΠ½ΠΈΡ†Π°Π΅ΠΌΠΎΡΡ‚ΡŒ свободного пространства m0 = 4p ο‚΄ 10-7 Π€ / ΠΌ 11

12 Π—Π°ΠΊΠΎΠ½ силы АмпСра (ΠΏΡ€ΠΎΠ΄ΠΎΠ»ΠΆΠ΅Π½ΠΈΠ΅)
Полная сила, Π΄Π΅ΠΉΡΡ‚Π²ΡƒΡŽΡ‰Π°Ρ Π½Π° Ρ†Π΅ΠΏΡŒ C2, ΠΈΠΌΠ΅ΡŽΡ‰ΡƒΡŽ Ρ‚ΠΎΠΊ I2, Ρ†Π΅ΠΏΡŒΡŽ C1, ΠΈΠΌΠ΅ΡŽΡ‰Π΅ΠΉ Ρ‚ΠΎΠΊ I1, Ρ€Π°Π²Π½Π° 12

13 Π—Π°ΠΊΠΎΠ½ силы АмпСра (ΠΏΡ€ΠΎΠ΄ΠΎΠ»ΠΆΠ΅Π½ΠΈΠ΅)
Π‘ΠΈΠ»Π° Π½Π° C1, создаваСмая C2, Ρ€Π°Π²Π½Π° ΠΏΠΎ Π²Π΅Π»ΠΈΡ‡ΠΈΠ½Π΅, Π½ΠΎ ΠΏΡ€ΠΎΡ‚ΠΈΠ²ΠΎΠΏΠΎΠ»ΠΎΠΆΠ½Π° ΠΏΠΎ Π½Π°ΠΏΡ€Π°Π²Π»Π΅Π½ΠΈΡŽ силС, Π΄Π΅ΠΉΡΡ‚Π²ΡƒΡŽΡ‰Π΅ΠΉ Π½Π° C2 ΠΈΠ·-Π·Π° C1.13

14 ΠŸΠ»ΠΎΡ‚Π½ΠΎΡΡ‚ΡŒ ΠΌΠ°Π³Π½ΠΈΡ‚Π½ΠΎΠ³ΠΎ ΠΏΠΎΡ‚ΠΎΠΊΠ° Π‘ΠΈΠ»ΠΎΠ²ΠΎΠΉ Π·Π°ΠΊΠΎΠ½ АмпСра описываСт «дСйствиС Π½Π° расстоянии», Π°Π½Π°Π»ΠΎΠ³ΠΈΡ‡Π½ΠΎΠ΅ Π·Π°ΠΊΠΎΠ½Ρƒ ΠšΡƒΠ»ΠΎΠ½Π°. Π’ Π·Π°ΠΊΠΎΠ½Π΅ ΠšΡƒΠ»ΠΎΠ½Π° Π±Ρ‹Π»ΠΎ ΠΏΠΎΠ»Π΅Π·Π½ΠΎ ввСсти понятиС элСктричСского поля для описания взаимодСйствия ΠΌΠ΅ΠΆΠ΄Ρƒ зарядами. Π’ Π·Π°ΠΊΠΎΠ½Π΅ АмпСра ΠΌΡ‹ ΠΌΠΎΠΆΠ΅ΠΌ ΠΎΠΏΡ€Π΅Π΄Π΅Π»ΠΈΡ‚ΡŒ ΡΠΎΠΎΡ‚Π²Π΅Ρ‚ΡΡ‚Π²ΡƒΡŽΡ‰Π΅Π΅ ΠΏΠΎΠ»Π΅, ΠΊΠΎΡ‚ΠΎΡ€ΠΎΠ΅ ΠΌΠΎΠΆΠ½ΠΎ Ρ€Π°ΡΡΠΌΠ°Ρ‚Ρ€ΠΈΠ²Π°Ρ‚ΡŒ ΠΊΠ°ΠΊ срСдство, с ΠΏΠΎΠΌΠΎΡ‰ΡŒΡŽ ΠΊΠΎΡ‚ΠΎΡ€ΠΎΠ³ΠΎ Ρ‚ΠΎΠΊΠΈ Π΄Π΅ΠΉΡΡ‚Π²ΡƒΡŽΡ‚ Π΄Ρ€ΡƒΠ³ Π½Π° Π΄Ρ€ΡƒΠ³Π°. 14

15 ΠŸΠ»ΠΎΡ‚Π½ΠΎΡΡ‚ΡŒ ΠΌΠ°Π³Π½ΠΈΡ‚Π½ΠΎΠ³ΠΎ ΠΏΠΎΡ‚ΠΎΠΊΠ° (ΠΏΡ€ΠΎΠ΄ΠΎΠ»ΠΆΠ΅Π½ΠΈΠ΅)
ΠŸΠ»ΠΎΡ‚Π½ΠΎΡΡ‚ΡŒ ΠΌΠ°Π³Π½ΠΈΡ‚Π½ΠΎΠ³ΠΎ ΠΏΠΎΡ‚ΠΎΠΊΠ° ΠΌΠΎΠΆΠ½ΠΎ ввСсти, написав 15

16 ΠŸΠ»ΠΎΡ‚Π½ΠΎΡΡ‚ΡŒ ΠΌΠ°Π³Π½ΠΈΡ‚Π½ΠΎΠ³ΠΎ ΠΏΠΎΡ‚ΠΎΠΊΠ° (ΠΏΡ€ΠΎΠ΄ΠΎΠ»ΠΆΠ΅Π½ΠΈΠ΅)
Π³Π΄Π΅ ΠΏΠ»ΠΎΡ‚Π½ΠΎΡΡ‚ΡŒ ΠΌΠ°Π³Π½ΠΈΡ‚Π½ΠΎΠ³ΠΎ ΠΏΠΎΡ‚ΠΎΠΊΠ° Π² мСстС располоТСния dl2 ΠΈΠ·-Π·Π° Ρ‚ΠΎΠΊΠ° I1 Π² C1 16

17 ΠŸΠ»ΠΎΡ‚Π½ΠΎΡΡ‚ΡŒ ΠΌΠ°Π³Π½ΠΈΡ‚Π½ΠΎΠ³ΠΎ ΠΏΠΎΡ‚ΠΎΠΊΠ° (ΠΏΡ€ΠΎΠ΄ΠΎΠ»ΠΆΠ΅Π½ΠΈΠ΅)
ΠŸΡ€Π΅Π΄ΠΏΠΎΠ»ΠΎΠΆΠΈΠΌ, Ρ‡Ρ‚ΠΎ бСсконСчно ΠΌΠ°Π»Ρ‹ΠΉ Ρ‚ΠΎΠΊΠΎΠ²Ρ‹ΠΉ элСмСнт Idl ΠΏΠΎΠ³Ρ€ΡƒΠΆΠ΅Π½ Π² ΠΎΠ±Π»Π°ΡΡ‚ΡŒ плотности ΠΌΠ°Π³Π½ΠΈΡ‚Π½ΠΎΠ³ΠΎ ΠΏΠΎΡ‚ΠΎΠΊΠ° B.Π’Π΅ΠΊΡƒΡ‰ΠΈΠΉ элСмСнт испытываСт силу dF, Ρ€Π°Π²Π½ΡƒΡŽ 17

18 ΠŸΠ»ΠΎΡ‚Π½ΠΎΡΡ‚ΡŒ ΠΌΠ°Π³Π½ΠΈΡ‚Π½ΠΎΠ³ΠΎ ΠΏΠΎΡ‚ΠΎΠΊΠ° (ΠΏΡ€ΠΎΠ΄ΠΎΠ»ΠΆΠ΅Π½ΠΈΠ΅)
Полная сила, прилоТСнная ΠΊ Ρ†Π΅ΠΏΠΈ C, ΠΏΠΎ ΠΊΠΎΡ‚ΠΎΡ€ΠΎΠΉ ΠΏΡ€ΠΎΡ…ΠΎΠ΄ΠΈΡ‚ Ρ‚ΠΎΠΊ I, которая ΠΏΠΎΠ³Ρ€ΡƒΠΆΠ΅Π½Π° Π² ΠΌΠ°Π³Π½ΠΈΡ‚Π½ΡƒΡŽ ΠΈΠ½Π΄ΡƒΠΊΡ†ΠΈΡŽ B, Ρ€Π°Π²Π½Π° 18

19 Π‘ΠΈΠ»Π°, Π΄Π΅ΠΉΡΡ‚Π²ΡƒΡŽΡ‰Π°Ρ Π½Π° двиТущийся заряд
ДвиТущийся Ρ‚ΠΎΡ‡Π΅Ρ‡Π½Ρ‹ΠΉ заряд, ΠΏΠΎΠΌΠ΅Ρ‰Π΅Π½Π½Ρ‹ΠΉ Π² ΠΌΠ°Π³Π½ΠΈΡ‚Π½ΠΎΠ΅ ΠΏΠΎΠ»Π΅, испытываСт силу, ΠΎΠΏΡ€Π΅Π΄Π΅Π»ΡΠ΅ΠΌΡƒΡŽ Π²Π΅Π»ΠΈΡ‡ΠΈΠ½ΠΎΠΉ Π‘ΠΈΠ»Π°, испытываСмая Ρ‚ΠΎΡ‡Π΅Ρ‡Π½Ρ‹ΠΌ зарядом, Π½Π°ΠΏΡ€Π°Π²Π»Π΅Π½Π° β€‹β€‹Π²Π½ΡƒΡ‚Ρ€ΡŒ Π±ΡƒΠΌΠ°Π³ΠΈ.B v Q 19

20 Π‘ΠΈΠ»Π° Π›ΠΎΡ€Π΅Π½Ρ†Π°. Если Ρ‚ΠΎΡ‡Π΅Ρ‡Π½Ρ‹ΠΉ заряд двиТСтся Π² области, Π³Π΄Π΅ ΡΡƒΡ‰Π΅ΡΡ‚Π²ΡƒΡŽΡ‚ ΠΊΠ°ΠΊ элСктричСскиС, Ρ‚Π°ΠΊ ΠΈ ΠΌΠ°Π³Π½ΠΈΡ‚Π½Ρ‹Π΅ поля, Ρ‚ΠΎ ΠΎΠ½ испытываСт ΠΎΠ±Ρ‰ΡƒΡŽ силу, Π·Π°Π΄Π°Π½Π½ΡƒΡŽ ΡƒΡ€Π°Π²Π½Π΅Π½ΠΈΠ΅ΠΌ силы Π›ΠΎΡ€Π΅Π½Ρ†Π°. Π£Ρ€Π°Π²Π½Π΅Π½ΠΈΠ΅ силы Π›ΠΎΡ€Π΅Π½Ρ†Π° ΠΏΠΎΠ»Π΅Π·Π½ΠΎ для опрСдСлСния уравнСния двиТСния элСктронов Π² элСктромагнитных ΠΎΡ‚ΠΊΠ»ΠΎΠ½ΡΡŽΡ‰ΠΈΡ… систСмах, Ρ‚Π°ΠΊΠΈΡ… ΠΊΠ°ΠΊ Π­Π›Π’. 20

21 Π³ΠΎΠ΄ Π—Π°ΠΊΠΎΠ½ Π‘ΠΈΠΎ-Π‘Π°Π²Π°Ρ€Π° Π—Π°ΠΊΠΎΠ½ Π‘ΠΈΠΎ-Π‘Π°Π²Π°Ρ€Π° Π΄Π°Π΅Ρ‚ Π½Π°ΠΌ B-ΠΏΠΎΠ»Π΅, Π²ΠΎΠ·Π½ΠΈΠΊΠ°ΡŽΡ‰Π΅Π΅ Π² ΡƒΠΊΠ°Π·Π°Π½Π½ΠΎΠΉ Ρ‚ΠΎΡ‡ΠΊΠ΅ P ΠΈΠ· Π΄Π°Π½Π½ΠΎΠ³ΠΎ распрСдСлСния Ρ‚ΠΎΠΊΠ°.Π­Ρ‚ΠΎ основной Π·Π°ΠΊΠΎΠ½ магнитостатики. 21 Π³ΠΎΠ΄

22 Π—Π°ΠΊΠΎΠ½ Π‘ΠΈΠΎ-Π‘Π°Π²Π°Ρ€Π° (ΠΏΡ€ΠΎΠ΄ΠΎΠ»ΠΆΠ΅Π½ΠΈΠ΅)
Π’ΠΊΠ»Π°Π΄ Π² B-ΠΏΠΎΠ»Π΅ Π² Ρ‚ΠΎΡ‡ΠΊΠ΅ P ΠΎΡ‚ Π΄ΠΈΡ„Ρ„Π΅Ρ€Π΅Π½Ρ†ΠΈΠ°Π»ΡŒΠ½ΠΎΠ³ΠΎ Ρ‚ΠΎΠΊΠΎΠ²ΠΎΠ³ΠΎ элСмСнта Idl ’равСн 22

23 Π—Π°ΠΊΠΎΠ½ Π‘ΠΈΠΎ-Π‘Π°Π²Π°Ρ€Π° (ΠΏΡ€ΠΎΠ΄ΠΎΠ»ΠΆΠ΅Π½ΠΈΠ΅)
23

24 Π—Π°ΠΊΠΎΠ½ Π‘ΠΈΠΎ-Π‘Π°Π²Π°Ρ€Π° (ΠΏΡ€ΠΎΠ΄ΠΎΠ»ΠΆΠ΅Π½ΠΈΠ΅)
ΠŸΠΎΠ»Π½Ρ‹ΠΉ ΠΌΠ°Π³Π½ΠΈΡ‚Π½Ρ‹ΠΉ ΠΏΠΎΡ‚ΠΎΠΊ Π² Ρ‚ΠΎΡ‡ΠΊΠ΅ P, создаваСмый всСй Ρ†Π΅ΠΏΡŒΡŽ C, Ρ€Π°Π²Π΅Π½ 24

25 Π’ΠΈΠΏΡ‹ распрСдСлСния Ρ‚ΠΎΠΊΠ°
ЛинСйная ΠΏΠ»ΠΎΡ‚Π½ΠΎΡΡ‚ΡŒ Ρ‚ΠΎΠΊΠ° (Ρ‚ΠΎΠΊ) — Π²ΠΎΠ·Π½ΠΈΠΊΠ°Π΅Ρ‚ для бСсконСчно Ρ‚ΠΎΠ½ΠΊΠΈΡ… Π½ΠΈΡ‚Π΅Π²ΠΈΠ΄Π½Ρ‹Ρ… Ρ‚Π΅Π» (Ρ‚.Π΅.Π΅. ΠΏΡ€ΠΎΠ²ΠΎΠ»ΠΎΠΊΠΈ ΠΏΡ€Π΅Π½Π΅Π±Ρ€Π΅ΠΆΠΈΠΌΠΎ ΠΌΠ°Π»ΠΎΠ³ΠΎ Π΄ΠΈΠ°ΠΌΠ΅Ρ‚Ρ€Π°). ΠŸΠ»ΠΎΡ‚Π½ΠΎΡΡ‚ΡŒ повСрхностного Ρ‚ΠΎΠΊΠ° (Ρ‚ΠΎΠΊ Π½Π° Π΅Π΄ΠΈΠ½ΠΈΡ†Ρƒ ΡˆΠΈΡ€ΠΈΠ½Ρ‹) — Π²ΠΎΠ·Π½ΠΈΠΊΠ°Π΅Ρ‚, ΠΊΠΎΠ³Π΄Π° Ρ‚Π΅Π»ΠΎ идСально проводящСС. ОбъСмная ΠΏΠ»ΠΎΡ‚Π½ΠΎΡΡ‚ΡŒ Ρ‚ΠΎΠΊΠ° (Ρ‚ΠΎΠΊ Π½Π° Π΅Π΄ΠΈΠ½ΠΈΡ†Ρƒ ΠΏΠ»ΠΎΡ‰Π°Π΄ΠΈ ΠΏΠΎΠΏΠ΅Ρ€Π΅Ρ‡Π½ΠΎΠ³ΠΎ сСчСния) — самый ΠΎΠ±Ρ‰ΠΈΠΉ. 25

26 Π—Π°ΠΊΠΎΠ½ Π‘ΠΈΠΎ-Π‘Π°Π²Π°Ρ€Π° (ΠΏΡ€ΠΎΠ΄ΠΎΠ»ΠΆΠ΅Π½ΠΈΠ΅)
Для повСрхностного распрСдСлСния Ρ‚ΠΎΠΊΠ° Π·Π°ΠΊΠΎΠ½ B-S ΠΏΡ€ΠΈΠ½ΠΈΠΌΠ°Π΅Ρ‚ Π²ΠΈΠ΄ Для объСмного распрСдСлСния Ρ‚ΠΎΠΊΠ° Π·Π°ΠΊΠΎΠ½ B-S ΠΏΡ€ΠΈΠ½ΠΈΠΌΠ°Π΅Ρ‚ Π²ΠΈΠ΄ 26

27 Циркулярный Π·Π°ΠΊΠΎΠ½ АмпСра Π² ΠΈΠ½Ρ‚Π΅Π³Ρ€Π°Π»ΡŒΠ½ΠΎΠΉ Ρ„ΠΎΡ€ΠΌΠ΅ Циркуляционный Π·Π°ΠΊΠΎΠ½
АмпСра Π² ΠΈΠ½Ρ‚Π΅Π³Ρ€Π°Π»ΡŒΠ½ΠΎΠΉ Ρ„ΠΎΡ€ΠΌΠ΅ гласит, Ρ‡Ρ‚ΠΎ «циркуляция плотности ΠΌΠ°Π³Π½ΠΈΡ‚Π½ΠΎΠ³ΠΎ ΠΏΠΎΡ‚ΠΎΠΊΠ° Π² свободном пространствС ΠΏΡ€ΠΎΠΏΠΎΡ€Ρ†ΠΈΠΎΠ½Π°Π»ΡŒΠ½Π° ΠΏΠΎΠ»Π½ΠΎΠΌΡƒ Ρ‚ΠΎΠΊΡƒ Ρ‡Π΅Ρ€Π΅Π· ΠΏΠΎΠ²Π΅Ρ€Ρ…Π½ΠΎΡΡ‚ΡŒ, ΠΎΠ³Ρ€Π°Π½ΠΈΡ‡ΠΈΠ²Π°ΡŽΡ‰ΡƒΡŽ ΠΏΡƒΡ‚ΡŒ, ΠΏΠΎ ΠΊΠΎΡ‚ΠΎΡ€ΠΎΠΌΡƒ вычисляСтся циркуляция.”27

28 Π³ΠΎΠ΄ Циркулярный Π·Π°ΠΊΠΎΠ½ АмпСра Π² ΠΈΠ½Ρ‚Π΅Π³Ρ€Π°Π»ΡŒΠ½ΠΎΠΉ Ρ„ΠΎΡ€ΠΌΠ΅ (ΠΏΡ€ΠΎΠ΄ΠΎΠ»ΠΆΠ΅Π½ΠΈΠ΅)
По соглашСнию считаСтся, Ρ‡Ρ‚ΠΎ dS соотвСтствуСт Π½Π°ΠΏΡ€Π°Π²Π»Π΅Π½ΠΈΡŽ, ΠΎΠΏΡ€Π΅Π΄Π΅Π»Π΅Π½Π½ΠΎΠΌΡƒ ΠΏΡ€Π°Π²ΠΈΠ»ΠΎΠΌ ΠΏΡ€Π°Π²ΠΎΠΉ Ρ€ΡƒΠΊΠΈ, примСняСмым ΠΊ dl. S dl dS ΠŸΠΎΡΠΊΠΎΠ»ΡŒΠΊΡƒ объСмная ΠΏΠ»ΠΎΡ‚Π½ΠΎΡΡ‚ΡŒ Ρ‚ΠΎΠΊΠ° являСтся Π½Π°ΠΈΠ±ΠΎΠ»Π΅Π΅ ΠΎΠ±Ρ‰Π΅ΠΉ, ΠΌΡ‹ ΠΌΠΎΠΆΠ΅ΠΌ Π·Π°ΠΏΠΈΡΠ°Ρ‚ΡŒ Iencl Ρ‚Π°ΠΊΠΈΠΌ ΠΎΠ±Ρ€Π°Π·ΠΎΠΌ. 28 Π³ΠΎΠ΄

29 Π—Π°ΠΊΠΎΠ½ АмпСра ΠΈ Π·Π°ΠΊΠΎΠ½ Гаусса
Как Π·Π°ΠΊΠΎΠ½ Гаусса слСдуСт ΠΈΠ· Π·Π°ΠΊΠΎΠ½Π° ΠšΡƒΠ»ΠΎΠ½Π°, Ρ‚Π°ΠΊ ΠΈ Π·Π°ΠΊΠΎΠ½ АмпСра слСдуСт ΠΈΠ· Π·Π°ΠΊΠΎΠ½Π° силы АмпСра.Как Π·Π°ΠΊΠΎΠ½ Гаусса ΠΌΠΎΠΆΠ½ΠΎ ΠΈΡΠΏΠΎΠ»ΡŒΠ·ΠΎΠ²Π°Ρ‚ΡŒ для получСния элСктростатичСского поля ΠΈΠ· симмСтричных распрСдСлСний заряда, Ρ‚Π°ΠΊ ΠΈ Π·Π°ΠΊΠΎΠ½ АмпСра ΠΌΠΎΠΆΠ½ΠΎ ΠΈΡΠΏΠΎΠ»ΡŒΠ·ΠΎΠ²Π°Ρ‚ΡŒ для получСния магнитостатичСского поля ΠΈΠ· симмСтричных распрСдСлСний Ρ‚ΠΎΠΊΠ°. 29

30 ΠŸΡ€ΠΈΠΌΠ΅Π½Π΅Π½ΠΈΠ΅ Π·Π°ΠΊΠΎΠ½Π° АмпСра
Π—Π°ΠΊΠΎΠ½ АмпСра Π² ΠΈΠ½Ρ‚Π΅Π³Ρ€Π°Π»ΡŒΠ½ΠΎΠΉ Ρ„ΠΎΡ€ΠΌΠ΅ прСдставляСт собой ΠΈΠ½Ρ‚Π΅Π³Ρ€Π°Π»ΡŒΠ½ΠΎΠ΅ ΡƒΡ€Π°Π²Π½Π΅Π½ΠΈΠ΅ для нСизвСстной плотности ΠΌΠ°Π³Π½ΠΈΡ‚Π½ΠΎΠ³ΠΎ ΠΏΠΎΡ‚ΠΎΠΊΠ°, Π²ΠΎΠ·Π½ΠΈΠΊΠ°ΡŽΡ‰Π΅ΠΉ Π² Ρ€Π΅Π·ΡƒΠ»ΡŒΡ‚Π°Ρ‚Π΅ Π·Π°Π΄Π°Π½Π½ΠΎΠ³ΠΎ распрСдСлСния Ρ‚ΠΎΠΊΠ°. извСстно нСизвСстно 30

31 Π³ΠΎΠ΄ ΠŸΡ€ΠΈΠ»ΠΎΠΆΠ΅Π½ΠΈΡ Π·Π°ΠΊΠΎΠ½Π° АмпСра (ΠΏΡ€ΠΎΠ΄ΠΎΠ»ΠΆΠ΅Π½ΠΈΠ΅)
Π’ ΠΎΠ±Ρ‰Π΅ΠΌ, Ρ€Π΅ΡˆΠ΅Π½ΠΈΡ ΠΈΠ½Ρ‚Π΅Π³Ρ€Π°Π»ΡŒΠ½Ρ‹Ρ… ΡƒΡ€Π°Π²Π½Π΅Π½ΠΈΠΉ Π΄ΠΎΠ»ΠΆΠ½Ρ‹ Π±Ρ‹Ρ‚ΡŒ ΠΏΠΎΠ»ΡƒΡ‡Π΅Π½Ρ‹ с использованиСм числСнных ΠΌΠ΅Ρ‚ΠΎΠ΄ΠΎΠ².Однако для Π½Π΅ΠΊΠΎΡ‚ΠΎΡ€Ρ‹Ρ… симмСтричных распрСдСлСний Ρ‚ΠΎΠΊΠ° ΠΌΠΎΠ³ΡƒΡ‚ Π±Ρ‹Ρ‚ΡŒ ΠΏΠΎΠ»ΡƒΡ‡Π΅Π½Ρ‹ Ρ€Π΅ΡˆΠ΅Π½ΠΈΡ Π² Π·Π°ΠΌΠΊΠ½ΡƒΡ‚ΠΎΠΉ Ρ„ΠΎΡ€ΠΌΠ΅ Π·Π°ΠΊΠΎΠ½Π° АмпСра. 31 Π³ΠΎΠ΄

32 ΠŸΡ€ΠΈΠ»ΠΎΠΆΠ΅Π½ΠΈΡ Π·Π°ΠΊΠΎΠ½Π° АмпСра (ΠΏΡ€ΠΎΠ΄ΠΎΠ»ΠΆΠ΅Π½ΠΈΠ΅)
РСшСниС Π² Π·Π°ΠΊΡ€Ρ‹Ρ‚ΠΎΠΉ Ρ„ΠΎΡ€ΠΌΠ΅ Π·Π°ΠΊΠΎΠ½Π° АмпСра зависит ΠΎΡ‚ нашСй способности ΠΏΠΎΡΡ‚Ρ€ΠΎΠΈΡ‚ΡŒ подходящСС сСмСйство ΠΏΡƒΡ‚Π΅ΠΉ АмпСра. АмпСровский ΠΏΡƒΡ‚ΡŒ — это Π·Π°ΠΌΠΊΠ½ΡƒΡ‚Ρ‹ΠΉ ΠΊΠΎΠ½Ρ‚ΡƒΡ€, ΠΊ ΠΊΠΎΡ‚ΠΎΡ€ΠΎΠΌΡƒ ΠΏΠ»ΠΎΡ‚Π½ΠΎΡΡ‚ΡŒ ΠΌΠ°Π³Π½ΠΈΡ‚Π½ΠΎΠ³ΠΎ ΠΏΠΎΡ‚ΠΎΠΊΠ° являСтся Ρ‚Π°Π½Π³Π΅Π½Ρ†ΠΈΠ°Π»ΡŒΠ½ΠΎΠΉ ΠΈ Π½Π° ΠΊΠΎΡ‚ΠΎΡ€ΠΎΠΉ Ρ€Π°Π²Π½Π° постоянному Π·Π½Π°Ρ‡Π΅Π½ΠΈΡŽ.32

33 ΠŸΠ»ΠΎΡ‚Π½ΠΎΡΡ‚ΡŒ ΠΌΠ°Π³Π½ΠΈΡ‚Π½ΠΎΠ³ΠΎ ΠΏΠΎΡ‚ΠΎΠΊΠ° бСсконСчного Π»ΠΈΠ½Π΅ΠΉΠ½ΠΎΠ³ΠΎ Ρ‚ΠΎΠΊΠ° с использованиСм Π·Π°ΠΊΠΎΠ½Π° АмпСра
Рассмотрим бСсконСчный Π»ΠΈΠ½Π΅ΠΉΠ½Ρ‹ΠΉ Ρ‚ΠΎΠΊ вдоль оси z, нСсущий Ρ‚ΠΎΠΊ Π² + z-Π½Π°ΠΏΡ€Π°Π²Π»Π΅Π½ΠΈΠΈ: I 33

34 ΠŸΠ»ΠΎΡ‚Π½ΠΎΡΡ‚ΡŒ ΠΌΠ°Π³Π½ΠΈΡ‚Π½ΠΎΠ³ΠΎ ΠΏΠΎΡ‚ΠΎΠΊΠ° бСсконСчного Π»ΠΈΠ½Π΅ΠΉΠ½ΠΎΠ³ΠΎ Ρ‚ΠΎΠΊΠ° с использованиСм Π·Π°ΠΊΠΎΠ½Π° АмпСра (ΠΏΡ€ΠΎΠ΄ΠΎΠ»ΠΆΠ΅Π½ΠΈΠ΅)
(1) ΠŸΡ€Π΅Π΄ΠΏΠΎΠ»ΠΎΠΆΠΈΠΌ, исходя ΠΈΠ· симмСтрии ΠΈ ΠΏΡ€Π°Π²ΠΈΠ»Π° ΠΏΡ€Π°Π²ΠΎΠΉ Ρ€ΡƒΠΊΠΈ, Ρ„ΠΎΡ€ΠΌΡƒ поля (2) ΠŸΠΎΡΡ‚Ρ€ΠΎΠΉΡ‚Π΅ сСмСйство окруТностСй ампСровских ΠΏΡƒΡ‚Π΅ΠΉ радиуса r, Π³Π΄Π΅ 34

35 Π³ΠΎΠ΄ ΠŸΠ»ΠΎΡ‚Π½ΠΎΡΡ‚ΡŒ ΠΌΠ°Π³Π½ΠΈΡ‚Π½ΠΎΠ³ΠΎ ΠΏΠΎΡ‚ΠΎΠΊΠ° бСсконСчного Π»ΠΈΠ½Π΅ΠΉΠ½ΠΎΠ³ΠΎ Ρ‚ΠΎΠΊΠ° с использованиСм Π·Π°ΠΊΠΎΠ½Π° АмпСра (ΠΏΡ€ΠΎΠ΄ΠΎΠ»ΠΆΠ΅Π½ΠΈΠ΅)
(3) ΠžΡ†Π΅Π½ΠΈΡ‚Π΅ ΠΏΠΎΠ»Π½Ρ‹ΠΉ Ρ‚ΠΎΠΊ, проходящий Ρ‡Π΅Ρ€Π΅Π· ΠΏΠΎΠ²Π΅Ρ€Ρ…Π½ΠΎΡΡ‚ΡŒ, ΠΎΠ³Ρ€Π°Π½ΠΈΡ‡Π΅Π½Π½ΡƒΡŽ ампСровским ΠΏΡƒΡ‚Π΅ΠΌ 35

36 ΠŸΠ»ΠΎΡ‚Π½ΠΎΡΡ‚ΡŒ ΠΌΠ°Π³Π½ΠΈΡ‚Π½ΠΎΠ³ΠΎ ΠΏΠΎΡ‚ΠΎΠΊΠ° бСсконСчного Π»ΠΈΠ½Π΅ΠΉΠ½ΠΎΠ³ΠΎ Ρ‚ΠΎΠΊΠ° с использованиСм Π·Π°ΠΊΠΎΠ½Π° АмпСра (ΠΏΡ€ΠΎΠ΄ΠΎΠ»ΠΆΠ΅Π½ΠΈΠ΅)
ΠŸΡƒΡ‚ΡŒ ΠΏΠΎ Π°ΠΌΠΏΠ΅Ρ€Ρƒ 36

37 (4) Для ΠΊΠ°ΠΆΠ΄ΠΎΠ³ΠΎ ампСровского ΠΏΡƒΡ‚ΠΈ ΠΎΡ†Π΅Π½ΠΈΡ‚Π΅ ΠΈΠ½Ρ‚Π΅Π³Ρ€Π°Π»
ΠŸΠ»ΠΎΡ‚Π½ΠΎΡΡ‚ΡŒ ΠΌΠ°Π³Π½ΠΈΡ‚Π½ΠΎΠ³ΠΎ ΠΏΠΎΡ‚ΠΎΠΊΠ° бСсконСчного Π»ΠΈΠ½Π΅ΠΉΠ½ΠΎΠ³ΠΎ Ρ‚ΠΎΠΊΠ°, ΠΈΡΠΏΠΎΠ»ΡŒΠ·ΡƒΡ Π·Π°ΠΊΠΎΠ½ АмпСра (ΠΏΡ€ΠΎΠ΄ΠΎΠ»ΠΆΠ΅Π½ΠΈΠ΅) (4) Для ΠΊΠ°ΠΆΠ΄ΠΎΠ³ΠΎ ампСровского ΠΏΡƒΡ‚ΠΈ ΠΎΡ†Π΅Π½ΠΈΡ‚Π΅ ΠΈΠ½Ρ‚Π΅Π³Ρ€Π°Π»ΡŒΠ½ΡƒΡŽ Π΄Π»ΠΈΠ½Ρƒ ампСровского ΠΏΡƒΡ‚ΠΈ.Π²Π΅Π»ΠΈΡ‡ΠΈΠ½Π° B Π½Π° ампСровской трассС. 37

38 (5) Π Π΅ΡˆΠΈΡ‚Π΅ ΠΎΡ‚Π½ΠΎΡΠΈΡ‚Π΅Π»ΡŒΠ½ΠΎ B Π½Π° ΠΊΠ°ΠΆΠ΄ΠΎΠΌ ампСровском ΠΏΡƒΡ‚ΠΈ
ΠŸΠ»ΠΎΡ‚Π½ΠΎΡΡ‚ΡŒ ΠΌΠ°Π³Π½ΠΈΡ‚Π½ΠΎΠ³ΠΎ ΠΏΠΎΡ‚ΠΎΠΊΠ° бСсконСчного Π»ΠΈΠ½Π΅ΠΉΠ½ΠΎΠ³ΠΎ Ρ‚ΠΎΠΊΠ°, ΠΈΡΠΏΠΎΠ»ΡŒΠ·ΡƒΡ Π·Π°ΠΊΠΎΠ½ АмпСра (ΠΏΡ€ΠΎΠ΄ΠΎΠ»ΠΆΠ΅Π½ΠΈΠ΅) (5) Π Π΅ΡˆΠΈΡ‚Π΅ ΠΎΡ‚Π½ΠΎΡΠΈΡ‚Π΅Π»ΡŒΠ½ΠΎ B Π½Π° ΠΊΠ°ΠΆΠ΄ΠΎΠΌ ампСровском ΠΏΡƒΡ‚ΠΈ 38

39 ΠŸΡ€ΠΈΠΌΠ΅Π½Π΅Π½ΠΈΠ΅ Ρ‚Π΅ΠΎΡ€Π΅ΠΌΡ‹ Бтокса ΠΊ Π·Π°ΠΊΠΎΠ½Ρƒ АмпСра
οƒž ΠŸΠΎΡΠΊΠΎΠ»ΡŒΠΊΡƒ сказанноС Π²Ρ‹ΡˆΠ΅ Π΄ΠΎΠ»ΠΆΠ½ΠΎ Π²Ρ‹ΠΏΠΎΠ»Π½ΡΡ‚ΡŒΡΡ для любой повСрхности S, ΠΌΡ‹ Π΄ΠΎΠ»ΠΆΠ½Ρ‹ ΠΈΠΌΠ΅Ρ‚ΡŒ Π΄ΠΈΡ„Ρ„Π΅Ρ€Π΅Π½Ρ†ΠΈΠ°Π»ΡŒΠ½ΡƒΡŽ Ρ„ΠΎΡ€ΠΌΡƒ Π·Π°ΠΊΠΎΠ½Π° АмпСра 39

40 Π—Π°ΠΊΠΎΠ½ АмпСра Π² Π΄ΠΈΡ„Ρ„Π΅Ρ€Π΅Π½Ρ†ΠΈΠ°Π»ΡŒΠ½ΠΎΠΉ Ρ„ΠΎΡ€ΠΌΠ΅
Π—Π°ΠΊΠΎΠ½ АмпСра Π² Π΄ΠΈΡ„Ρ„Π΅Ρ€Π΅Π½Ρ†ΠΈΠ°Π»ΡŒΠ½ΠΎΠΉ Ρ„ΠΎΡ€ΠΌΠ΅ ΠΏΠΎΠ΄Ρ€Π°Π·ΡƒΠΌΠ΅Π²Π°Π΅Ρ‚, Ρ‡Ρ‚ΠΎ B-ΠΏΠΎΠ»Π΅ являСтся консСрвативным Π·Π° ΠΏΡ€Π΅Π΄Π΅Π»Π°ΠΌΠΈ Ρ€Π΅Π³ΠΈΠΎΠ½ΠΎΠ², Π³Π΄Π΅ ΠΏΡ€ΠΎΡ‚Π΅ΠΊΠ°Π΅Ρ‚ Ρ‚ΠΎΠΊ.40

41 Π³ΠΎΠ΄ Π€ΡƒΠ½Π΄Π°ΠΌΠ΅Π½Ρ‚Π°Π»ΡŒΠ½Ρ‹Π΅ постулаты магнитостатики
Π—Π°ΠΊΠΎΠ½ АмпСра Π² Π΄ΠΈΡ„Ρ„Π΅Ρ€Π΅Π½Ρ†ΠΈΠ°Π»ΡŒΠ½ΠΎΠΉ Ρ„ΠΎΡ€ΠΌΠ΅ ΠžΡ‚ΡΡƒΡ‚ΡΡ‚Π²ΠΈΠ΅ ΠΈΠ·ΠΎΠ»ΠΈΡ€ΠΎΠ²Π°Π½Π½Ρ‹Ρ… ΠΌΠ°Π³Π½ΠΈΡ‚Π½Ρ‹Ρ… зарядов B являСтся ΡΠΎΠ»Π΅Π½ΠΎΠΈΠ΄Π°Π»ΡŒΠ½Ρ‹ΠΌ 41

42 Π’Π΅ΠΊΡ‚ΠΎΡ€Π½Ρ‹ΠΉ ΠΌΠ°Π³Π½ΠΈΡ‚Π½Ρ‹ΠΉ ΠΏΠΎΡ‚Π΅Π½Ρ†ΠΈΠ°Π»
ВоТдСство Π²Π΅ΠΊΡ‚ΠΎΡ€Π°: «ДивСргСнция Ρ€ΠΎΡ‚ΠΎΡ€Π° любого Π²Π΅ΠΊΡ‚ΠΎΡ€Π½ΠΎΠ³ΠΎ поля тоТдСствСнно Ρ€Π°Π²Π½Π° Π½ΡƒΠ»ΡŽΒ». БлСдствиС: «Если дивСргСнция Π²Π΅ΠΊΡ‚ΠΎΡ€Π½ΠΎΠ³ΠΎ поля тоТдСствСнно Ρ€Π°Π²Π½Π° Π½ΡƒΠ»ΡŽ, Ρ‚ΠΎ это Π²Π΅ΠΊΡ‚ΠΎΡ€Π½ΠΎΠ΅ ΠΏΠΎΠ»Π΅ ΠΌΠΎΠΆΠ½ΠΎ Π·Π°ΠΏΠΈΡΠ°Ρ‚ΡŒ ΠΊΠ°ΠΊ Ρ€ΠΎΡ‚ΠΎΡ€ Π½Π΅ΠΊΠΎΡ‚ΠΎΡ€ΠΎΠ³ΠΎ Π²Π΅ΠΊΡ‚ΠΎΡ€Π½ΠΎΠ³ΠΎ ΠΏΠΎΡ‚Π΅Π½Ρ†ΠΈΠ°Π»ΡŒΠ½ΠΎΠ³ΠΎ поля.”42

43 Π³ΠΎΠ΄ Π’Π΅ΠΊΡ‚ΠΎΡ€Π½Ρ‹ΠΉ ΠΌΠ°Π³Π½ΠΈΡ‚Π½Ρ‹ΠΉ ΠΏΠΎΡ‚Π΅Π½Ρ†ΠΈΠ°Π» (ΠΏΡ€ΠΎΠ΄ΠΎΠ»ΠΆΠ΅Π½ΠΈΠ΅)
ΠŸΠΎΡΠΊΠΎΠ»ΡŒΠΊΡƒ ΠΏΠ»ΠΎΡ‚Π½ΠΎΡΡ‚ΡŒ ΠΌΠ°Π³Π½ΠΈΡ‚Π½ΠΎΠ³ΠΎ ΠΏΠΎΡ‚ΠΎΠΊΠ° являСтся солСноидальной, Π΅Π΅ ΠΌΠΎΠΆΠ½ΠΎ Π·Π°ΠΏΠΈΡΠ°Ρ‚ΡŒ ΠΊΠ°ΠΊ Ρ€ΠΎΡ‚ΠΎΡ€ Π²Π΅ΠΊΡ‚ΠΎΡ€Π½ΠΎΠ³ΠΎ поля, Π½Π°Π·Ρ‹Π²Π°Π΅ΠΌΠΎΠ³ΠΎ Π²Π΅ΠΊΡ‚ΠΎΡ€Π½Ρ‹ΠΌ ΠΌΠ°Π³Π½ΠΈΡ‚Π½Ρ‹ΠΌ ΠΏΠΎΡ‚Π΅Π½Ρ†ΠΈΠ°Π»ΠΎΠΌ. 43 Π³ΠΎΠ΄

44 Π³ΠΎΠ΄ Π’Π΅ΠΊΡ‚ΠΎΡ€Π½Ρ‹ΠΉ ΠΌΠ°Π³Π½ΠΈΡ‚Π½Ρ‹ΠΉ ΠΏΠΎΡ‚Π΅Π½Ρ†ΠΈΠ°Π» (ΠΏΡ€ΠΎΠ΄ΠΎΠ»ΠΆΠ΅Π½ΠΈΠ΅)
ΠžΠ±Ρ‰Π°Ρ Ρ„ΠΎΡ€ΠΌΠ° Π·Π°ΠΊΠΎΠ½Π° B-S: ΠžΠ±Ρ€Π°Ρ‚ΠΈΡ‚Π΅ Π²Π½ΠΈΠΌΠ°Π½ΠΈΠ΅, Ρ‡Ρ‚ΠΎ 44

45 Π’Π΅ΠΊΡ‚ΠΎΡ€Π½Ρ‹ΠΉ ΠΌΠ°Π³Π½ΠΈΡ‚Π½Ρ‹ΠΉ ΠΏΠΎΡ‚Π΅Π½Ρ†ΠΈΠ°Π» (ΠΏΡ€ΠΎΠ΄ΠΎΠ»ΠΆΠ΅Π½ΠΈΠ΅)
ΠšΡ€ΠΎΠΌΠ΅ Ρ‚ΠΎΠ³ΠΎ, ΠΎΠ±Ρ€Π°Ρ‚ΠΈΡ‚Π΅ Π²Π½ΠΈΠΌΠ°Π½ΠΈΠ΅, Ρ‡Ρ‚ΠΎ ΠΎΠΏΠ΅Ρ€Π°Ρ‚ΠΎΡ€ del Ρ€Π°Π±ΠΎΡ‚Π°Π΅Ρ‚ Ρ‚ΠΎΠ»ΡŒΠΊΠΎ с ΠΊΠΎΠΎΡ€Π΄ΠΈΠ½Π°Ρ‚Π°ΠΌΠΈ Π±Π΅Π· ΡˆΡ‚Ρ€ΠΈΡ…Π°, Ρ‚Π°ΠΊ Ρ‡Ρ‚ΠΎ 45

46 Π’Π΅ΠΊΡ‚ΠΎΡ€Π½Ρ‹ΠΉ ΠΌΠ°Π³Π½ΠΈΡ‚Π½Ρ‹ΠΉ ΠΏΠΎΡ‚Π΅Π½Ρ†ΠΈΠ°Π» (ΠΏΡ€ΠΎΠ΄ΠΎΠ»ΠΆΠ΅Π½ΠΈΠ΅)
Π‘Π»Π΅Π΄ΠΎΠ²Π°Ρ‚Π΅Π»ΡŒΠ½ΠΎ, ΠΌΡ‹ ΠΈΠΌΠ΅Π΅ΠΌ 46

47 Π’Π΅ΠΊΡ‚ΠΎΡ€Π½Ρ‹ΠΉ ΠΌΠ°Π³Π½ΠΈΡ‚Π½Ρ‹ΠΉ ΠΏΠΎΡ‚Π΅Π½Ρ†ΠΈΠ°Π» (ΠΏΡ€ΠΎΠ΄ΠΎΠ»ΠΆΠ΅Π½ΠΈΠ΅)
Для повСрхностного распрСдСлСния Ρ‚ΠΎΠΊΠ° Π²Π΅ΠΊΡ‚ΠΎΡ€Π½Ρ‹ΠΉ ΠΌΠ°Π³Π½ΠΈΡ‚Π½Ρ‹ΠΉ ΠΏΠΎΡ‚Π΅Π½Ρ†ΠΈΠ°Π» опрСдСляСтся ΠΊΠ°ΠΊ Для Π»ΠΈΠ½Π΅ΠΉΠ½ΠΎΠ³ΠΎ Ρ‚ΠΎΠΊΠ° Π²Π΅ΠΊΡ‚ΠΎΡ€Π½Ρ‹ΠΉ ΠΌΠ°Π³Π½ΠΈΡ‚Π½Ρ‹ΠΉ ΠΏΠΎΡ‚Π΅Π½Ρ†ΠΈΠ°Π» опрСдСляСтся ΠΊΠ°ΠΊ 47

48 Π’Π΅ΠΊΡ‚ΠΎΡ€Π½Ρ‹ΠΉ ΠΌΠ°Π³Π½ΠΈΡ‚Π½Ρ‹ΠΉ ΠΏΠΎΡ‚Π΅Π½Ρ†ΠΈΠ°Π» (ΠΏΡ€ΠΎΠ΄ΠΎΠ»ΠΆΠ΅Π½ΠΈΠ΅)
Π’ Π½Π΅ΠΊΠΎΡ‚ΠΎΡ€Ρ‹Ρ… случаях ΠΏΡ€ΠΎΡ‰Π΅ ΠΎΡ†Π΅Π½ΠΈΡ‚ΡŒ Π²Π΅ΠΊΡ‚ΠΎΡ€Π½Ρ‹ΠΉ ΠΌΠ°Π³Π½ΠΈΡ‚Π½Ρ‹ΠΉ ΠΏΠΎΡ‚Π΅Π½Ρ†ΠΈΠ°Π» ΠΈ Π·Π°Ρ‚Π΅ΠΌ ΠΈΡΠΏΠΎΠ»ΡŒΠ·ΠΎΠ²Π°Ρ‚ΡŒ B =  A, Ρ‡Π΅ΠΌ ΠΈΡΠΏΠΎΠ»ΡŒΠ·ΠΎΠ²Π°Ρ‚ΡŒ Π·Π°ΠΊΠΎΠ½ B-S для нСпосрСдствСнного опрСдСлСния B.Π’ Π½Π΅ΠΊΠΎΡ‚ΠΎΡ€ΠΎΠΌ смыслС Π²Π΅ΠΊΡ‚ΠΎΡ€Π½Ρ‹ΠΉ ΠΌΠ°Π³Π½ΠΈΡ‚Π½Ρ‹ΠΉ ΠΏΠΎΡ‚Π΅Π½Ρ†ΠΈΠ°Π» A Π°Π½Π°Π»ΠΎΠ³ΠΈΡ‡Π΅Π½ скалярному элСктричСскому ΠΏΠΎΡ‚Π΅Π½Ρ†ΠΈΠ°Π»Ρƒ V. 48

49 Π’Π΅ΠΊΡ‚ΠΎΡ€Π½Ρ‹ΠΉ ΠΌΠ°Π³Π½ΠΈΡ‚Π½Ρ‹ΠΉ ΠΏΠΎΡ‚Π΅Π½Ρ†ΠΈΠ°Π» (ΠΏΡ€ΠΎΠ΄ΠΎΠ»ΠΆΠ΅Π½ΠΈΠ΅)
Π’ классичСской Ρ„ΠΈΠ·ΠΈΠΊΠ΅ Π²Π΅ΠΊΡ‚ΠΎΡ€Π½Ρ‹ΠΉ ΠΌΠ°Π³Π½ΠΈΡ‚Π½Ρ‹ΠΉ ΠΏΠΎΡ‚Π΅Π½Ρ†ΠΈΠ°Π» рассматриваСтся ΠΊΠ°ΠΊ Π²ΡΠΏΠΎΠΌΠΎΠ³Π°Ρ‚Π΅Π»ΡŒΠ½Π°Ρ функция, Π½Π΅ ΠΈΠΌΠ΅ΡŽΡ‰Π°Ρ физичСского смысла. Однако Π² ΠΊΠ²Π°Π½Ρ‚ΠΎΠ²ΠΎΠΉ ΠΌΠ΅Ρ…Π°Π½ΠΈΠΊΠ΅ Π΅ΡΡ‚ΡŒ явлСния, ΠΊΠΎΡ‚ΠΎΡ€Ρ‹Π΅ ΠΏΡ€Π΅Π΄ΠΏΠΎΠ»Π°Π³Π°ΡŽΡ‚, Ρ‡Ρ‚ΠΎ Π²Π΅ΠΊΡ‚ΠΎΡ€Π½Ρ‹ΠΉ ΠΌΠ°Π³Π½ΠΈΡ‚Π½Ρ‹ΠΉ ΠΏΠΎΡ‚Π΅Π½Ρ†ΠΈΠ°Π» являСтся Ρ€Π΅Π°Π»ΡŒΠ½Ρ‹ΠΌ (Ρ‚ΠΎ Π΅ΡΡ‚ΡŒ ΠΈΠ·ΠΌΠ΅Ρ€ΠΈΠΌΡ‹ΠΌ) ΠΏΠΎΠ»Π΅ΠΌ.49

50 ΠœΠ°Π³Π½ΠΈΡ‚Π½Ρ‹ΠΉ диполь ΠœΠ°Π³Π½ΠΈΡ‚Π½Ρ‹ΠΉ диполь прСдставляСт собой Π½Π΅Π±ΠΎΠ»ΡŒΡˆΡƒΡŽ Ρ‚ΠΎΠΊΠΎΠ²Π΅Π΄ΡƒΡ‰ΡƒΡŽ ΠΏΠ΅Ρ‚Π»ΡŽ. Π’ΠΎΡ‡Π΅Ρ‡Π½Ρ‹ΠΉ заряд (зарядовый монополь) — ΠΏΡ€ΠΎΡΡ‚Π΅ΠΉΡˆΠΈΠΉ источник элСктростатичСского поля. ΠœΠ°Π³Π½ΠΈΡ‚Π½Ρ‹ΠΉ диполь — ΠΏΡ€ΠΎΡΡ‚Π΅ΠΉΡˆΠΈΠΉ источник магнитостатичСского поля. ΠœΠ°Π³Π½ΠΈΡ‚Π½ΠΎΠ³ΠΎ монополя Π½Π΅ сущСствуСт (ΠΏΠΎ ΠΊΡ€Π°ΠΉΠ½Π΅ΠΉ ΠΌΠ΅Ρ€Π΅, Π² Ρ‚ΠΎΠΌ, Ρ‡Ρ‚ΠΎ касаСтся классичСской Ρ„ΠΈΠ·ΠΈΠΊΠΈ). 50

51 ΠœΠ°Π³Π½ΠΈΡ‚Π½Ρ‹ΠΉ диполь (ΠΏΡ€ΠΎΠ΄ΠΎΠ»ΠΆΠ΅Π½ΠΈΠ΅)
ΠœΠ°Π³Π½ΠΈΡ‚Π½Ρ‹ΠΉ диполь Π°Π½Π°Π»ΠΎΠ³ΠΈΡ‡Π΅Π½ элСктричСскому диполю.Подобно Ρ‚ΠΎΠΌΡƒ, ΠΊΠ°ΠΊ элСктричСский диполь ΠΏΠΎΠ»Π΅Π·Π΅Π½, помогая Π½Π°ΠΌ ΠΏΠΎΠ½ΡΡ‚ΡŒ ΠΏΠΎΠ²Π΅Π΄Π΅Π½ΠΈΠ΅ диэлСктричСских ΠΌΠ°Ρ‚Π΅Ρ€ΠΈΠ°Π»ΠΎΠ², Ρ‚Π°ΠΊ ΠΈ ΠΌΠ°Π³Π½ΠΈΡ‚Π½Ρ‹ΠΉ диполь ΠΏΠΎΠ»Π΅Π·Π΅Π½, помогая Π½Π°ΠΌ ΠΏΠΎΠ½ΡΡ‚ΡŒ ΠΏΠΎΠ²Π΅Π΄Π΅Π½ΠΈΠ΅ ΠΌΠ°Π³Π½ΠΈΡ‚Π½Ρ‹Ρ… ΠΌΠ°Ρ‚Π΅Ρ€ΠΈΠ°Π»ΠΎΠ². 51

52 ΠœΠ°Π³Π½ΠΈΡ‚Π½Ρ‹ΠΉ диполь (ΠΏΡ€ΠΎΠ΄ΠΎΠ»ΠΆΠ΅Π½ΠΈΠ΅)
Рассмотрим Π½Π΅Π±ΠΎΠ»ΡŒΡˆΡƒΡŽ ΠΊΡ€ΡƒΠ³ΠΎΠ²ΡƒΡŽ ΠΏΠ΅Ρ‚Π»ΡŽ радиуса b, ΠΏΠΎ ΠΊΠΎΡ‚ΠΎΡ€ΠΎΠΉ ΠΏΡ€ΠΎΡ…ΠΎΠ΄ΠΈΡ‚ постоянный Ρ‚ΠΎΠΊ I. ΠŸΡ€Π΅Π΄ΠΏΠΎΠ»ΠΎΠΆΠΈΠΌ, Ρ‡Ρ‚ΠΎ радиус ΠΏΡ€ΠΎΠ²ΠΎΠ»ΠΎΠΊΠΈ ΠΈΠΌΠ΅Π΅Ρ‚ Π½Π΅Π·Π½Π°Ρ‡ΠΈΡ‚Π΅Π»ΡŒΠ½ΠΎΠ΅ ΠΏΠΎΠΏΠ΅Ρ€Π΅Ρ‡Π½ΠΎΠ΅ сСчСниС. Ρ… Π³ Π± 52

53 ΠœΠ°Π³Π½ΠΈΡ‚Π½Ρ‹ΠΉ диполь (ΠΏΡ€ΠΎΠ΄ΠΎΠ»ΠΆΠ΅Π½ΠΈΠ΅)
Π’Π΅ΠΊΡ‚ΠΎΡ€Π½Ρ‹ΠΉ ΠΌΠ°Π³Π½ΠΈΡ‚Π½Ρ‹ΠΉ ΠΏΠΎΡ‚Π΅Π½Ρ†ΠΈΠ°Π» оцСниваСтся для R >> b ΠΊΠ°ΠΊ 53

54 ΠœΠ°Π³Π½ΠΈΡ‚Π½Ρ‹ΠΉ диполь (ΠΏΡ€ΠΎΠ΄ΠΎΠ»ΠΆΠ΅Π½ΠΈΠ΅)
ΠŸΠ»ΠΎΡ‚Π½ΠΎΡΡ‚ΡŒ ΠΌΠ°Π³Π½ΠΈΡ‚Π½ΠΎΠ³ΠΎ ΠΏΠΎΡ‚ΠΎΠΊΠ° оцСниваСтся для R >> b ΠΊΠ°ΠΊ 54

55 ΠœΠ°Π³Π½ΠΈΡ‚Π½Ρ‹ΠΉ диполь (ΠΏΡ€ΠΎΠ΄ΠΎΠ»ΠΆΠ΅Π½ΠΈΠ΅)
Π’Ρ‹Π·ΠΎΠ² элСктричСского диполя ЭлСктричСскоС ΠΏΠΎΠ»Π΅, обусловлСнноС Π΄ΠΈΠΏΠΎΠ»Π΅ΠΌ элСктричСского заряда, ΠΈ ΠΌΠ°Π³Π½ΠΈΡ‚Π½ΠΎΠ΅ ΠΏΠΎΠ»Π΅, обусловлСнноС ΠΌΠ°Π³Π½ΠΈΡ‚Π½Ρ‹ΠΌ Π΄ΠΈΠΏΠΎΠ»Π΅ΠΌ, ΡΠ²Π»ΡΡŽΡ‚ΡΡ Π΄Π²ΠΎΠΉΠ½Ρ‹ΠΌΠΈ Π²Π΅Π»ΠΈΡ‡ΠΈΠ½Π°ΠΌΠΈ.55

56 ΠœΠ°Π³Π½ΠΈΡ‚Π½Ρ‹ΠΉ Π΄ΠΈΠΏΠΎΠ»ΡŒΠ½Ρ‹ΠΉ ΠΌΠΎΠΌΠ΅Π½Ρ‚
ΠœΠ°Π³Π½ΠΈΡ‚Π½Ρ‹ΠΉ Π΄ΠΈΠΏΠΎΠ»ΡŒΠ½Ρ‹ΠΉ ΠΌΠΎΠΌΠ΅Π½Ρ‚ ΠΌΠΎΠΆΠ½ΠΎ ΠΎΠΏΡ€Π΅Π΄Π΅Π»ΠΈΡ‚ΡŒ ΠΊΠ°ΠΊ Π’Π΅Π»ΠΈΡ‡ΠΈΠ½Π° дипольного ΠΌΠΎΠΌΠ΅Π½Ρ‚Π°, ΡΠ²Π»ΡΡŽΡ‰Π°ΡΡΡ ΠΏΡ€ΠΎΠΈΠ·Π²Π΅Π΄Π΅Π½ΠΈΠ΅ΠΌ силы Ρ‚ΠΎΠΊΠ° ΠΈ ΠΏΠ»ΠΎΡ‰Π°Π΄ΠΈ ΠΊΠΎΠ½Ρ‚ΡƒΡ€Π°. НаправлСниС дипольного ΠΌΠΎΠΌΠ΅Π½Ρ‚Π° опрСдСляСтся Π½Π°ΠΏΡ€Π°Π²Π»Π΅Π½ΠΈΠ΅ΠΌ Ρ‚ΠΎΠΊΠ° ΠΏΠΎ ΠΏΡ€Π°Π²ΠΈΠ»Ρƒ ΠΏΡ€Π°Π²ΠΎΠΉ Ρ€ΡƒΠΊΠΈ. 56

57 Π³ΠΎΠ΄ ΠœΠ°Π³Π½ΠΈΡ‚Π½Ρ‹ΠΉ Π΄ΠΈΠΏΠΎΠ»ΡŒΠ½Ρ‹ΠΉ ΠΌΠΎΠΌΠ΅Π½Ρ‚ (ΠΏΡ€ΠΎΠ΄ΠΎΠ»ΠΆΠ΅Π½ΠΈΠ΅)
ΠœΡ‹ ΠΌΠΎΠΆΠ΅ΠΌ Π·Π°ΠΏΠΈΡΠ°Ρ‚ΡŒ Π²Π΅ΠΊΡ‚ΠΎΡ€Π½Ρ‹ΠΉ ΠΌΠ°Π³Π½ΠΈΡ‚Π½Ρ‹ΠΉ ΠΏΠΎΡ‚Π΅Π½Ρ†ΠΈΠ°Π» Ρ‡Π΅Ρ€Π΅Π· ΠΌΠ°Π³Π½ΠΈΡ‚Π½Ρ‹ΠΉ Π΄ΠΈΠΏΠΎΠ»ΡŒΠ½Ρ‹ΠΉ ΠΌΠΎΠΌΠ΅Π½Ρ‚ ΠΊΠ°ΠΊ ΠœΡ‹ ΠΌΠΎΠΆΠ΅ΠΌ Π·Π°ΠΏΠΈΡΠ°Ρ‚ΡŒ ΠΏΠΎΠ»Π΅ B Ρ‡Π΅Ρ€Π΅Π· ΠΌΠ°Π³Π½ΠΈΡ‚Π½Ρ‹ΠΉ Π΄ΠΈΠΏΠΎΠ»ΡŒΠ½Ρ‹ΠΉ ΠΌΠΎΠΌΠ΅Π½Ρ‚ ΠΊΠ°ΠΊ 57

58 ДивСргСнция B-поля. B-ΠΏΠΎΠ»Π΅ солСноидально, Ρ‚.Π΅.Π΅. дивСргСнция B-поля тоТдСствСнно Ρ€Π°Π²Π½Π° Π½ΡƒΠ»ΡŽ: ЀизичСски это ΠΎΠ·Π½Π°Ρ‡Π°Π΅Ρ‚, Ρ‡Ρ‚ΠΎ ΠΌΠ°Π³Π½ΠΈΡ‚Π½Ρ‹Ρ… зарядов (ΠΌΠΎΠ½ΠΎΠΏΠΎΠ»Π΅ΠΉ) Π½Π΅ сущСствуСт. ΠœΠ°Π³Π½ΠΈΡ‚Π½Ρ‹ΠΉ заряд ΠΌΠΎΠΆΠ½ΠΎ Ρ€Π°ΡΡΠΌΠ°Ρ‚Ρ€ΠΈΠ²Π°Ρ‚ΡŒ ΠΊΠ°ΠΊ ΠΈΠ·ΠΎΠ»ΠΈΡ€ΠΎΠ²Π°Π½Π½Ρ‹ΠΉ ΠΌΠ°Π³Π½ΠΈΡ‚Π½Ρ‹ΠΉ полюс. 58

59 ДивСргСнция B-поля (ΠΏΡ€ΠΎΠ΄ΠΎΠ»ΠΆΠ΅Π½ΠΈΠ΅)
НСзависимо ΠΎΡ‚ Ρ‚ΠΎΠ³ΠΎ, насколько ΠΌΠ°Π»ΠΎΠ΅ ΠΌΠ°Π³Π½ΠΈΡ‚Π½ΠΎΠ΅ ΠΏΠΎΠ»Π΅ Ρ€Π°Π·Π΄Π΅Π»Π΅Π½ΠΎ, ΠΎΠ½ΠΎ всСгда ΠΈΠΌΠ΅Π΅Ρ‚ сСвСрный полюс ΠΈ ΡŽΠΆΠ½Ρ‹ΠΉ полюс. Π­Π»Π΅ΠΌΠ΅Π½Ρ‚Π°Ρ€Π½Ρ‹ΠΉ источник ΠΌΠ°Π³Π½ΠΈΡ‚Π½ΠΎΠ³ΠΎ поля — ΠΌΠ°Π³Π½ΠΈΡ‚Π½Ρ‹ΠΉ диполь.Н Π‘ И Н Π‘ 59

60 ΠœΠ°Π³Π½ΠΈΡ‚Π½Ρ‹ΠΉ ΠΏΠΎΡ‚ΠΎΠΊ ΠœΠ°Π³Π½ΠΈΡ‚Π½Ρ‹ΠΉ ΠΏΠΎΡ‚ΠΎΠΊ, ΠΏΠ΅Ρ€Π΅ΡΠ΅ΠΊΠ°ΡŽΡ‰ΠΈΠΉ ΠΎΡ‚ΠΊΡ€Ρ‹Ρ‚ΡƒΡŽ ΠΏΠΎΠ²Π΅Ρ€Ρ…Π½ΠΎΡΡ‚ΡŒ S, Ρ€Π°Π²Π΅Π½
Π’Ρ‚-Π’Π± / ΠΌ2 60

61 ΠœΠ°Π³Π½ΠΈΡ‚Π½Ρ‹ΠΉ ΠΏΠΎΡ‚ΠΎΠΊ (ΠΏΡ€ΠΎΠ΄ΠΎΠ»ΠΆΠ΅Π½ΠΈΠ΅)
Π˜ΡΡ…ΠΎΠ΄Ρ ΠΈΠ· Ρ‚Π΅ΠΎΡ€Π΅ΠΌΡ‹ ΠΎ расходимости, ΠΌΡ‹ ΠΈΠΌΠ΅Π΅ΠΌ Π‘Π»Π΅Π΄ΠΎΠ²Π°Ρ‚Π΅Π»ΡŒΠ½ΠΎ, чистый ΠΌΠ°Π³Π½ΠΈΡ‚Π½Ρ‹ΠΉ ΠΏΠΎΡ‚ΠΎΠΊ, ΠΏΠΎΠΊΠΈΠ΄Π°ΡŽΡ‰ΠΈΠΉ Π»ΡŽΠ±ΡƒΡŽ Π·Π°ΠΌΠΊΠ½ΡƒΡ‚ΡƒΡŽ ΠΏΠΎΠ²Π΅Ρ€Ρ…Π½ΠΎΡΡ‚ΡŒ, Ρ€Π°Π²Π΅Π½ Π½ΡƒΠ»ΡŽ. Π­Ρ‚ΠΎ Π΅Ρ‰Π΅ ΠΎΠ΄Π½ΠΎ проявлСниС отсутствия ΠΌΠ°Π³Π½ΠΈΡ‚Π½Ρ‹Ρ… зарядов.61

62 ΠœΠ°Π³Π½ΠΈΡ‚Π½Ρ‹ΠΉ ΠΏΠΎΡ‚ΠΎΠΊ ΠΈ Π²Π΅ΠΊΡ‚ΠΎΡ€Π½Ρ‹ΠΉ ΠΌΠ°Π³Π½ΠΈΡ‚Π½Ρ‹ΠΉ ΠΏΠΎΡ‚Π΅Π½Ρ†ΠΈΠ°Π»
ΠœΠ°Π³Π½ΠΈΡ‚Π½Ρ‹ΠΉ ΠΏΠΎΡ‚ΠΎΠΊ Ρ‡Π΅Ρ€Π΅Π· ΠΎΡ‚ΠΊΡ€Ρ‹Ρ‚ΡƒΡŽ ΠΏΠΎΠ²Π΅Ρ€Ρ…Π½ΠΎΡΡ‚ΡŒ ΠΌΠΎΠΆΠ½ΠΎ ΠΎΡ†Π΅Π½ΠΈΡ‚ΡŒ с ΠΏΠΎΠΌΠΎΡ‰ΡŒΡŽ Π²Π΅ΠΊΡ‚ΠΎΡ€Π½ΠΎΠ³ΠΎ ΠΌΠ°Π³Π½ΠΈΡ‚Π½ΠΎΠ³ΠΎ ΠΏΠΎΡ‚Π΅Π½Ρ†ΠΈΠ°Π»Π°, ΠΈΡΠΏΠΎΠ»ΡŒΠ·ΡƒΡ Ρ‚Π΅ΠΎΡ€Π΅ΠΌΡƒ Бтокса: 62

63 ЭлСктромагнитная сила
ЭлСктромагнитная сила задаСтся ΡƒΡ€Π°Π²Π½Π΅Π½ΠΈΠ΅ΠΌ силы Π›ΠΎΡ€Π΅Π½Ρ†Π° (ΠΏΠΎ голландскому Ρ„ΠΈΠ·ΠΈΠΊΡƒ Π₯Π΅Π½Π΄Ρ€ΠΈΠΊΡƒ Антуну Π›ΠΎΡ€Π΅Π½Ρ‚Ρ†Ρƒ (1853-1928)). Π£Ρ€Π°Π²Π½Π΅Π½ΠΈΠ΅ силы Π›ΠΎΡ€Π΅Π½Ρ†Π° вСсьма ΠΏΠΎΠ»Π΅Π·Π½ΠΎ для опрСдСлСния ΠΏΡƒΡ‚Π΅ΠΉ, ΠΏΠΎ ΠΊΠΎΡ‚ΠΎΡ€Ρ‹ΠΌ заряТСнныС частицы Π±ΡƒΠ΄ΡƒΡ‚ Π΄Π²ΠΈΠ³Π°Ρ‚ΡŒΡΡ Ρ‡Π΅Ρ€Π΅Π· элСктричСскиС ΠΈ ΠΌΠ°Π³Π½ΠΈΡ‚Π½Ρ‹Π΅ поля. .Если ΠΌΡ‹ Ρ‚Π°ΠΊΠΆΠ΅ Π·Π½Π°Π΅ΠΌ массу частицы, m, сила связана с ускорСниСм ΡƒΡ€Π°Π²Π½Π΅Π½ΠΈΠ΅ΠΌ. ΠŸΠ΅Ρ€Π²Ρ‹ΠΉ Ρ‡Π»Π΅Π½ Π² ΡƒΡ€Π°Π²Π½Π΅Π½ΠΈΠΈ силы Π›ΠΎΡ€Π΅Π½Ρ†Π° прСдставляСт ΡΠ»Π΅ΠΊΡ‚Ρ€ΠΈΡ‡Π΅ΡΠΊΡƒΡŽ силу Fe, Π΄Π΅ΠΉΡΡ‚Π²ΡƒΡŽΡ‰ΡƒΡŽ Π½Π° заряд q Π² элСктричСском ΠΏΠΎΠ»Π΅, Ρ€Π°Π²Π½ΠΎΠ΅. ЭлСктричСская сила Π½Π°ΠΏΡ€Π°Π²Π»Π΅Π½Π° ​​в Π½Π°ΠΏΡ€Π°Π²Π»Π΅Π½ΠΈΠΈ элСктричСского поля.

64 ΠœΠ°Π³Π½ΠΈΡ‚Π½Π°Ρ сила Π’Ρ‚ΠΎΡ€ΠΎΠΉ Ρ‡Π»Π΅Π½ Π² ΡƒΡ€Π°Π²Π½Π΅Π½ΠΈΠΈ силы Π›ΠΎΡ€Π΅Π½Ρ†Π° прСдставляСт ΠΌΠ°Π³Π½ΠΈΡ‚Π½ΡƒΡŽ силу Fm (N) Π½Π° двиТущийся заряд q (C), Π³Π΄Π΅ ΡΠΊΠΎΡ€ΠΎΡΡ‚ΡŒ заряда Ρ€Π°Π²Π½Π° u (ΠΌ / сСк) Π² ΠΏΠΎΠ»Π΅ с ΠΏΠ»ΠΎΡ‚Π½ΠΎΡΡ‚ΡŒΡŽ ΠΌΠ°Π³Π½ΠΈΡ‚Π½ΠΎΠ³ΠΎ ΠΏΠΎΡ‚ΠΎΠΊΠ° B ( Π’Ρ‚ / ΠΌ2).Π•Π΄ΠΈΠ½ΠΈΡ†Ρ‹ ΠΏΠΎΠ΄Ρ‚Π²Π΅Ρ€ΠΆΠ΄Π°ΡŽΡ‚ΡΡ эквивалСнтностями Wb = (V) (sec) ΠΈ J = (N) (m) = (C) (V). ΠœΠ°Π³Π½ΠΈΡ‚Π½Π°Ρ сила находится ΠΏΠΎΠ΄ прямым ΡƒΠ³Π»ΠΎΠΌ ΠΊ ​​магнитному полю. ΠœΠ°Π³Π½ΠΈΡ‚Π½Π°Ρ сила Ρ‚Ρ€Π΅Π±ΡƒΠ΅Ρ‚, Ρ‡Ρ‚ΠΎΠ±Ρ‹ заряТСнная частица Π½Π°Ρ…ΠΎΠ΄ΠΈΠ»Π°ΡΡŒ Π² Π΄Π²ΠΈΠΆΠ΅Π½ΠΈΠΈ. Π‘Π»Π΅Π΄ΡƒΠ΅Ρ‚ ΠΎΡ‚ΠΌΠ΅Ρ‚ΠΈΡ‚ΡŒ, Ρ‡Ρ‚ΠΎ, ΠΏΠΎΡΠΊΠΎΠ»ΡŒΠΊΡƒ магнитная сила дСйствуСт Π² Π½Π°ΠΏΡ€Π°Π²Π»Π΅Π½ΠΈΠΈ, пСрпСндикулярном скорости частицы, ускорСниС пСрпСндикулярно скорости, ΠΈ Π²Π΅Π»ΠΈΡ‡ΠΈΠ½Π° Π²Π΅ΠΊΡ‚ΠΎΡ€Π° скорости Π½Π΅ измСняСтся. ΠŸΠΎΡΠΊΠΎΠ»ΡŒΠΊΡƒ магнитная сила находится ΠΏΠΎΠ΄ прямым ΡƒΠ³Π»ΠΎΠΌ ΠΊ ​​магнитному полю, Ρ€Π°Π±ΠΎΡ‚Π°, ΡΠΎΠ²Π΅Ρ€ΡˆΠ°Π΅ΠΌΠ°Ρ ΠΌΠ°Π³Π½ΠΈΡ‚Π½Ρ‹ΠΌ ΠΏΠΎΠ»Π΅ΠΌ, опрСдСляСтся Π²Ρ‹Ρ€Π°ΠΆΠ΅Π½ΠΈΠ΅ΠΌ

65 ΠœΠ°Π³Π½ΠΈΡ‚Π½Π°Ρ сила D3.10. Π’ ΠΎΠΏΡ€Π΅Π΄Π΅Π»Π΅Π½Π½Ρ‹ΠΉ ΠΌΠΎΠΌΠ΅Π½Ρ‚ Π²Ρ€Π΅ΠΌΠ΅Π½ΠΈ Π² области пространства, Π³Π΄Π΅ E = 0 ΠΈ B = 3ay Wb / m2, 2-килограммовая частица заряда 1 C двиТСтся со ΡΠΊΠΎΡ€ΠΎΡΡ‚ΡŒΡŽ 2ax ΠΌ / сСк. Каково ускорСниС частицы Π·Π° счСт ΠΌΠ°Π³Π½ΠΈΡ‚Π½ΠΎΠ³ΠΎ поля? Π”Π°Π½ΠΎ: q = 1 нКл, m = 2 ΠΊΠ³, u = 2 ax (ΠΌ / сСк), E = 0, B = 3 ay Wb / ΠΌ2. Π£Ρ€Π°Π²Π½Π΅Π½ΠΈΠ΅ силы Π›ΠΎΡ€Π΅Π½Ρ†Π° Π²ΠΎ Π²Ρ‚ΠΎΡ€ΠΎΠΌ Π·Π°ΠΊΠΎΠ½Π΅ ΠΡŒΡŽΡ‚ΠΎΠ½Π° Для расчСта Π΅Π΄ΠΈΠ½ΠΈΡ†: P3.33: Заряд 10. нКл со ΡΠΊΠΎΡ€ΠΎΡΡ‚ΡŒΡŽ 100 ΠΌ / сСк Π² Π½Π°ΠΏΡ€Π°Π²Π»Π΅Π½ΠΈΠΈ z Π²Ρ…ΠΎΠ΄ΠΈΡ‚ Π² ΠΎΠ±Π»Π°ΡΡ‚ΡŒ, Π³Π΄Π΅ Π½Π°ΠΏΡ€ΡΠΆΠ΅Π½Π½ΠΎΡΡ‚ΡŒ элСктричСского поля Ρ€Π°Π²Π½Π° 800. Π’ / ΠΌ ось ΠΈ ΠΏΠ»ΠΎΡ‚Π½ΠΎΡΡ‚ΡŒ ΠΌΠ°Π³Π½ΠΈΡ‚Π½ΠΎΠ³ΠΎ ΠΏΠΎΡ‚ΠΎΠΊΠ° 12.0 Π’Ρ‚ / ΠΌ2 Π³ΠΎΠ΄. ΠžΠΏΡ€Π΅Π΄Π΅Π»ΠΈΡ‚Π΅ Π²Π΅ΠΊΡ‚ΠΎΡ€ силы, Π΄Π΅ΠΉΡΡ‚Π²ΡƒΡŽΡ‰Π΅ΠΉ Π½Π° заряд. Π”Π°Π½ΠΎ: q = 10 нКл, u = 100 Π°Π·ΠΈΠΌΡƒΡ‚ΠΎΠ² (ΠΌ / сСк), E = 800 ax V / m, B = 12.0 ay Wb / m2.

66 ΠœΠ°Π³Π½ΠΈΡ‚Π½Π°Ρ сила Π½Π° элСмСнтС Ρ‚ΠΎΠΊΠ°
Рассмотрим линию, ΠΏΡ€ΠΎΠ²ΠΎΠ΄ΡΡ‰ΡƒΡŽ Ρ‚ΠΎΠΊ Π² присутствии ΠΌΠ°Π³Π½ΠΈΡ‚Π½ΠΎΠ³ΠΎ поля. ΠœΡ‹ Ρ…ΠΎΡ‚ΠΈΠΌ Π½Π°ΠΉΡ‚ΠΈ Ρ€Π΅Π·ΡƒΠ»ΡŒΡ‚ΠΈΡ€ΡƒΡŽΡ‰ΡƒΡŽ силу Π½Π° Π»ΠΈΠ½ΠΈΠΈ. ΠœΡ‹ ΠΌΠΎΠΆΠ΅ΠΌ ΠΏΠΎΡΠΌΠΎΡ‚Ρ€Π΅Ρ‚ΡŒ Π½Π° нСбольшой Π΄ΠΈΡ„Ρ„Π΅Ρ€Π΅Π½Ρ†ΠΈΠ°Π»ΡŒΠ½Ρ‹ΠΉ сСгмСнт dQ заряда, двиТущСгося со ΡΠΊΠΎΡ€ΠΎΡΡ‚ΡŒΡŽ u, ΠΈ ΠΌΠΎΠΆΠ΅ΠΌ Π²Ρ‹Ρ‡ΠΈΡΠ»ΠΈΡ‚ΡŒ Π΄ΠΈΡ„Ρ„Π΅Ρ€Π΅Π½Ρ†ΠΈΠ°Π»ΡŒΠ½ΡƒΡŽ силу, Π΄Π΅ΠΉΡΡ‚Π²ΡƒΡŽΡ‰ΡƒΡŽ Π½Π° этот заряд, исходя ΠΈΠ· скорости. Π‘ΠΊΠΎΡ€ΠΎΡΡ‚ΡŒ Ρ‚Π°ΠΊΠΆΠ΅ ΠΌΠΎΠΆΠ½ΠΎ Π·Π°ΠΏΠΈΡΠ°Ρ‚ΡŒ ΠΎΡ‚Ρ€Π΅Π·ΠΊΠΎΠΌ Π‘Π»Π΅Π΄ΠΎΠ²Π°Ρ‚Π΅Π»ΡŒΠ½ΠΎ, ΠΏΠΎΡΠΊΠΎΠ»ΡŒΠΊΡƒ dQ / dt (Π² Кл / сСк) соотвСтствуСт Ρ‚ΠΎΠΊ I Π² Π»ΠΈΠ½ΠΈΠΈ, ΠΊΠΎΡ‚ΠΎΡ€Ρ‹ΠΉ ΠΌΡ‹ ΠΈΠΌΠ΅Π΅ΠΌ (часто Π½Π°Π·Ρ‹Π²Π°Π΅ΠΌΠΎΠ΅ ΡƒΡ€Π°Π²Π½Π΅Π½ΠΈΠ΅ΠΌ двигатСля), ΠΌΡ‹ ΠΌΠΎΠΆΠ΅ΠΌ ΠΈΡΠΏΠΎΠ»ΡŒΠ·ΠΎΠ²Π°Ρ‚ΡŒ, Ρ‡Ρ‚ΠΎΠ±Ρ‹ Π½Π°ΠΉΡ‚ΠΈ силу ΠΈΠ· Π½Π°Π±ΠΎΡ€Π° элСмСнтов Ρ‚ΠΎΠΊΠ°, ΠΈΡΠΏΠΎΠ»ΡŒΠ·ΡƒΡ ΠΈΠ½Ρ‚Π΅Π³Ρ€Π°Π»

67 ΠœΠ°Π³Π½ΠΈΡ‚Π½Π°Ρ сила — элСмСнт бСсконСчного Ρ‚ΠΎΠΊΠ°
Π”Π°Π²Π°ΠΉΡ‚Π΅ рассмотрим линию Ρ‚ΠΎΠΊΠ° I Π² Π½Π°ΠΏΡ€Π°Π²Π»Π΅Π½ΠΈΠΈ + az Π½Π° оси z.Для элСмСнта Ρ‚ΠΎΠΊΠ° IdLa ΠΌΡ‹ ΠΈΠΌΠ΅Π΅ΠΌ ΠŸΠ»ΠΎΡ‚Π½ΠΎΡΡ‚ΡŒ ΠΌΠ°Π³Π½ΠΈΡ‚Π½ΠΎΠ³ΠΎ ΠΏΠΎΡ‚ΠΎΠΊΠ° B1 для Π»ΠΈΠ½ΠΈΠΈ Ρ‚ΠΎΠΊΠ° бСсконСчной Π΄Π»ΠΈΠ½Ρ‹: ΠœΡ‹ Π·Π½Π°Π΅ΠΌ, Ρ‡Ρ‚ΠΎ этот элСмСнт создаСт ΠΌΠ°Π³Π½ΠΈΡ‚Π½ΠΎΠ΅ ΠΏΠΎΠ»Π΅, Π½ΠΎ ΠΏΠΎΠ»Π΅ Π½Π΅ ΠΌΠΎΠΆΠ΅Ρ‚ ΠΎΠΊΠ°Π·Ρ‹Π²Π°Ρ‚ΡŒ ΠΌΠ°Π³Π½ΠΈΡ‚Π½ΡƒΡŽ силу Π½Π° элСмСнт, ΡΠΎΠ·Π΄Π°ΡŽΡ‰ΠΈΠΉ Π΅Π³ΠΎ. Π’ качСствС Π°Π½Π°Π»ΠΎΠ³ΠΈΠΈ рассмотрим, Ρ‡Ρ‚ΠΎ элСктричСскоС ΠΏΠΎΠ»Π΅ Ρ‚ΠΎΡ‡Π΅Ρ‡Π½ΠΎΠ³ΠΎ заряда Π½Π΅ ΠΌΠΎΠΆΠ΅Ρ‚ ΠΎΠΊΠ°Π·Ρ‹Π²Π°Ρ‚ΡŒ Π½Π° сСбя ΡΠ»Π΅ΠΊΡ‚Ρ€ΠΈΡ‡Π΅ΡΠΊΡƒΡŽ силу. Как насчСт поля ΠΈΠ· Π²Ρ‚ΠΎΡ€ΠΎΠ³ΠΎ Ρ‚Π΅ΠΊΡƒΡ‰Π΅Π³ΠΎ элСмСнта IdLb Π² этой строкС? Из Π·Π°ΠΊΠΎΠ½Π° Π‘ΠΈΠΎ-Π‘Π°Π²Π°Ρ€Π° ΠΌΡ‹ Π²ΠΈΠ΄ΠΈΠΌ, Ρ‡Ρ‚ΠΎ пСрСкрСстноС ΠΏΡ€ΠΎΠΈΠ·Π²Π΅Π΄Π΅Π½ΠΈΠ΅ Π² этом ΠΊΠΎΠ½ΠΊΡ€Π΅Ρ‚Π½ΠΎΠΌ случаС Π±ΡƒΠ΄Π΅Ρ‚ Ρ€Π°Π²Π½ΠΎ Π½ΡƒΠ»ΡŽ, ΠΏΠΎΡΠΊΠΎΠ»ΡŒΠΊΡƒ IdL ΠΈ aR Π±ΡƒΠ΄ΡƒΡ‚ Π² ΠΎΠ΄Π½ΠΎΠΌ Π½Π°ΠΏΡ€Π°Π²Π»Π΅Π½ΠΈΠΈ.Π˜Ρ‚Π°ΠΊ, ΠΌΡ‹ ΠΌΠΎΠΆΠ΅ΠΌ ΡΠΊΠ°Π·Π°Ρ‚ΡŒ, Ρ‡Ρ‚ΠΎ прямая линия Ρ‚ΠΎΠΊΠ° Π½Π΅ ΠΎΠΊΠ°Π·Ρ‹Π²Π°Π΅Ρ‚ Π½Π° сСбя ΠΌΠ°Π³Π½ΠΈΡ‚Π½ΠΎΠΉ силы.

68 ΠœΠ°Π³Π½ΠΈΡ‚Π½Π°Ρ сила — Π΄Π²Π° элСмСнта Ρ‚ΠΎΠΊΠ°
Π’Π΅ΠΏΠ΅Ρ€ΡŒ Π΄Π°Π²Π°ΠΉΡ‚Π΅ рассмотрим Π²Ρ‚ΠΎΡ€ΡƒΡŽ линию Ρ‚ΠΎΠΊΠ°, ΠΏΠ°Ρ€Π°Π»Π»Π΅Π»ΡŒΠ½ΡƒΡŽ ΠΏΠ΅Ρ€Π²ΠΎΠΉ. Π‘ΠΈΠ»Π° dF12 ΠΎΡ‚ ΠΌΠ°Π³Π½ΠΈΡ‚Π½ΠΎΠ³ΠΎ поля Π»ΠΈΠ½ΠΈΠΈ 1, Π΄Π΅ΠΉΡΡ‚Π²ΡƒΡŽΡ‰Π°Ρ Π½Π° Π΄ΠΈΡ„Ρ„Π΅Ρ€Π΅Π½Ρ†ΠΈΠ°Π»ΡŒΠ½Ρ‹ΠΉ участок Π»ΠΈΠ½ΠΈΠΈ 2, Ρ€Π°Π²Π½Π° a = -ax ρ = y. ΠŸΠ»ΠΎΡ‚Π½ΠΎΡΡ‚ΡŒ ΠΌΠ°Π³Π½ΠΈΡ‚Π½ΠΎΠ³ΠΎ ΠΏΠΎΡ‚ΠΎΠΊΠ° B1 для Π»ΠΈΠ½ΠΈΠΈ Ρ‚ΠΎΠΊΠ° бСсконСчной Π΄Π»ΠΈΠ½Ρ‹ вызываСтся ΠΈΠ· уравнСния, ΠΊΠΎΡ‚ΠΎΡ€ΠΎΠ΅ Π΄ΠΎΠ»ΠΆΠ½ΠΎ Π±Ρ‹Ρ‚ΡŒ. ΠΌΡ‹ Π²ΠΈΠ΄ΠΈΠΌ, Ρ‡Ρ‚ΠΎ ρ = y ΠΈ a = -ax.ΠŸΠΎΠ΄ΡΡ‚Π°Π²Π»ΡΡ это Π² ΠΏΡ€ΠΈΠ²Π΅Π΄Π΅Π½Π½ΠΎΠ΅ Π²Ρ‹ΡˆΠ΅ ΡƒΡ€Π°Π²Π½Π΅Π½ΠΈΠ΅ ΠΈ учитывая, Ρ‡Ρ‚ΠΎ dL2 = dzaz, ΠΌΡ‹ ΠΈΠΌΠ΅Π΅ΠΌ

69 ΠœΠ°Π³Π½ΠΈΡ‚Π½Π°Ρ сила Π½Π° элСмСнтС Ρ‚ΠΎΠΊΠ°
Π§Ρ‚ΠΎΠ±Ρ‹ Π½Π°ΠΉΡ‚ΠΈ ΠΏΠΎΠ»Π½ΡƒΡŽ силу Π½Π° Π΄Π»ΠΈΠ½Π΅ L Π»ΠΈΠ½ΠΈΠΈ 2 ΠΈΠ· поля Π»ΠΈΠ½ΠΈΠΈ 1, ΠΌΡ‹ Π΄ΠΎΠ»ΠΆΠ½Ρ‹ ΠΈΠ½Ρ‚Π΅Π³Ρ€ΠΈΡ€ΠΎΠ²Π°Ρ‚ΡŒ dF12 ΠΎΡ‚ + L Π΄ΠΎ 0. ΠœΡ‹ ΠΈΠ½Ρ‚Π΅Π³Ρ€ΠΈΡ€ΡƒΠ΅ΠΌ Π² этом Π½Π°ΠΏΡ€Π°Π²Π»Π΅Π½ΠΈΠΈ, Ρ‡Ρ‚ΠΎΠ±Ρ‹ ΡƒΡ‡Π΅ΡΡ‚ΡŒ Π½Π°ΠΏΡ€Π°Π²Π»Π΅Π½ΠΈΠ΅ элСктричСский Ρ‚ΠΎΠΊ. a = -ax ρ = y Π­Ρ‚ΠΎ Π΄Π°Π΅Ρ‚ Π½Π°ΠΌ силу отталкивания. Если Π±Ρ‹ ΠΌΡ‹ вмСсто этого искали F21, ΠΌΠ°Π³Π½ΠΈΡ‚Π½ΡƒΡŽ силу, Π΄Π΅ΠΉΡΡ‚Π²ΡƒΡŽΡ‰ΡƒΡŽ Π½Π° линию 1 ΠΈΠ· поля Π»ΠΈΠ½ΠΈΠΈ 2, ΠΌΡ‹ нашли Π±Ρ‹ F21 = -F12.Π’Ρ‹Π²ΠΎΠ΄: 1) Π”Π²Π΅ ΠΏΠ°Ρ€Π°Π»Π»Π΅Π»ΡŒΠ½Ρ‹Π΅ Π»ΠΈΠ½ΠΈΠΈ с Ρ‚ΠΎΠΊΠΎΠΌ Π² ΠΏΡ€ΠΎΡ‚ΠΈΠ²ΠΎΠΏΠΎΠ»ΠΎΠΆΠ½Ρ‹Ρ… направлСниях ΠΈΡΠΏΡ‹Ρ‚Ρ‹Π²Π°ΡŽΡ‚ силу отталкивания. 2) Для ΠΏΠ°Ρ€Ρ‹ ΠΏΠ°Ρ€Π°Π»Π»Π΅Π»ΡŒΠ½Ρ‹Ρ… Π»ΠΈΠ½ΠΈΠΉ с Ρ‚ΠΎΠΊΠΎΠΌ Π² ΠΎΠ΄Π½ΠΎΠΌ Π½Π°ΠΏΡ€Π°Π²Π»Π΅Π½ΠΈΠΈ Π²ΠΎΠ·Π½ΠΈΠΊΠ½Π΅Ρ‚ сила притяТСния.

70 ΠœΠ°Π³Π½ΠΈΡ‚Π½Π°Ρ сила Π½Π° Ρ‚Π΅ΠΊΡƒΡ‰Π΅ΠΌ элСмСнтС
Π’ Π±ΠΎΠ»Π΅Π΅ ΠΎΠ±Ρ‰Π΅ΠΌ случаС, ΠΊΠΎΠ³Π΄Π° Π΄Π²Π΅ Π»ΠΈΠ½ΠΈΠΈ Π½Π΅ ΠΏΠ°Ρ€Π°Π»Π»Π΅Π»ΡŒΠ½Ρ‹ ΠΈΠ»ΠΈ Π½Π΅ прямыС, ΠΌΡ‹ ΠΌΠΎΠ³Π»ΠΈ Π±Ρ‹ ΠΈΡΠΏΠΎΠ»ΡŒΠ·ΠΎΠ²Π°Ρ‚ΡŒ Π·Π°ΠΊΠΎΠ½ Π‘ΠΈΠΎ-Π‘Π°Π²Π°Ρ€Π°, Ρ‡Ρ‚ΠΎΠ±Ρ‹ Π½Π°ΠΉΡ‚ΠΈ B1 ΠΈ ΠΏΡ€ΠΈΠΉΡ‚ΠΈ ΠΊ этому ΡƒΡ€Π°Π²Π½Π΅Π½ΠΈΡŽ, извСстному ΠΊΠ°ΠΊ Π·Π°ΠΊΠΎΠ½ силы АмпСра. ΠΌΠ΅ΠΆΠ΄Ρƒ ΠΏΠ°Ρ€ΠΎΠΉ Ρ‚ΠΎΠΊΠΎΠ²Π΅Π΄ΡƒΡ‰ΠΈΡ… Ρ†Π΅ΠΏΠ΅ΠΉ ΠΈ Π°Π½Π°Π»ΠΎΠ³ΠΈΡ‡Π΅Π½ Π·Π°ΠΊΠΎΠ½Ρƒ силы ΠšΡƒΠ»ΠΎΠ½Π° ΠΌΠ΅ΠΆΠ΄Ρƒ ΠΏΠ°Ρ€ΠΎΠΉ зарядов.

71 ΠœΠ°Π³Π½ΠΈΡ‚Π½Π°Ρ сила D3.11: КаТдая ΠΏΠ°Ρ€Π° ΠΏΠ°Ρ€Π°Π»Π»Π΅Π»ΡŒΠ½Ρ‹Ρ… Π»ΠΈΠ½ΠΈΠΉ бСсконСчной Π΄Π»ΠΈΠ½Ρ‹ пСрСносит Ρ‚ΠΎΠΊ I = 2A Π² ΠΎΠ΄Π½ΠΎΠΌ ΠΈ Ρ‚ΠΎΠΌ ΠΆΠ΅ Π½Π°ΠΏΡ€Π°Π²Π»Π΅Π½ΠΈΠΈ. ΠžΠΏΡ€Π΅Π΄Π΅Π»ΠΈΡ‚Π΅ Π²Π΅Π»ΠΈΡ‡ΠΈΠ½Ρƒ силы Π½Π° Π΅Π΄ΠΈΠ½ΠΈΡ†Ρƒ Π΄Π»ΠΈΠ½Ρ‹ ΠΌΠ΅ΠΆΠ΄Ρƒ двумя линиями, Ссли расстояниС ΠΌΠ΅ΠΆΠ΄Ρƒ Π½ΠΈΠΌΠΈ составляСт (Π°) 10 см, (Π±) 100 см. ΠžΡ‚Ρ‚Π°Π»ΠΊΠΈΠ²Π°ΡŽΡ‰Π°Ρ ΠΈΠ»ΠΈ ΠΏΡ€ΠΈΡ‚ΡΠ³Π°Ρ‚Π΅Π»ΡŒΠ½Π°Ρ сила? (ΠžΡ‚Π²Π΅Ρ‚: (a) 8 мН / ΠΌ, (b) 0,8 мН / ΠΌ, притяТСниС) ΠœΠ°Π³Π½ΠΈΡ‚Π½Π°Ρ сила ΠΌΠ΅ΠΆΠ΄Ρƒ двумя Ρ‚ΠΎΠΊΠΎΠ²Π΅Π΄ΡƒΡ‰ΠΈΠΌΠΈ элСмСнтами, ΠΊΠΎΠ³Π΄Π° Ρ‚ΠΎΠΊ Ρ‚Π΅Ρ‡Π΅Ρ‚ Π² ΠΎΠ΄Π½ΠΎΠΌ Π½Π°ΠΏΡ€Π°Π²Π»Π΅Π½ΠΈΠΈ ΠœΠ°Π³Π½ΠΈΡ‚Π½Π°Ρ сила Π½Π° Π΅Π΄ΠΈΠ½ΠΈΡ†Ρƒ Π΄Π»ΠΈΠ½Ρ‹ Π‘Π»ΡƒΡ‡Π°ΠΉ (a) y = 10 см Π‘Π»ΡƒΡ‡Π°ΠΉ ( Π°) Ρƒ = 10 см

72 ΠœΠ°Π³Π½ΠΈΡ‚Π½Ρ‹Π΅ ΠΌΠ°Ρ‚Π΅Ρ€ΠΈΠ°Π»Ρ‹ ΠΈ Π³Ρ€Π°Π½ΠΈΡ‡Π½Ρ‹Π΅ условия

73 ΠžΡ‚Π½ΠΎΡΠΈΡ‚Π΅Π»ΡŒΠ½Π°Ρ ΠΏΡ€ΠΎΠ½ΠΈΡ†Π°Π΅ΠΌΠΎΡΡ‚ΡŒ для Ρ€Π°Π·Π»ΠΈΡ‡Π½Ρ‹Ρ… ΠΌΠ°Ρ‚Π΅Ρ€ΠΈΠ°Π»ΠΎΠ².
ΠœΠ°Π³Π½ΠΈΡ‚Π½Ρ‹Π΅ ΠΌΠ°Ρ‚Π΅Ρ€ΠΈΠ°Π»Ρ‹ ΠœΡ‹ Π·Π½Π°Π΅ΠΌ, Ρ‡Ρ‚ΠΎ Ρ‚ΠΎΠΊ, ΠΏΡ€ΠΎΡ‚Π΅ΠΊΠ°ΡŽΡ‰ΠΈΠΉ Ρ‡Π΅Ρ€Π΅Π· ΠΊΠ°Ρ‚ΡƒΡˆΠΊΡƒ с ΠΏΡ€ΠΎΠ²ΠΎΠ»ΠΎΠΊΠΎΠΉ, Π±ΡƒΠ΄Π΅Ρ‚ ΡΠΎΠ·Π΄Π°Π²Π°Ρ‚ΡŒ ΠΌΠ°Π³Π½ΠΈΡ‚Π½ΠΎΠ΅ ΠΏΠΎΠ»Π΅, ΠΏΠΎΠ΄ΠΎΠ±Π½ΠΎΠ΅ полю стСрТнСвого ΠΌΠ°Π³Π½ΠΈΡ‚Π°. ΠœΡ‹ Ρ‚Π°ΠΊΠΆΠ΅ Π·Π½Π°Π΅ΠΌ, Ρ‡Ρ‚ΠΎ ΠΌΠΎΠΆΠ½ΠΎ Π·Π½Π°Ρ‡ΠΈΡ‚Π΅Π»ΡŒΠ½ΠΎ ΡƒΠ»ΡƒΡ‡ΡˆΠΈΡ‚ΡŒ ΠΏΠΎΠ»Π΅, Π½Π°ΠΌΠΎΡ‚Π°Π² ΠΏΡ€ΠΎΠ²ΠΎΠ»ΠΎΠΊΡƒ Π½Π° ΠΆΠ΅Π»Π΅Π·Π½Ρ‹ΠΉ сСрдСчник. Π£Ρ‚ΡŽΠ³ считаСтся ΠΌΠ°Π³Π½ΠΈΡ‚Π½Ρ‹ΠΌ ΠΌΠ°Ρ‚Π΅Ρ€ΠΈΠ°Π»ΠΎΠΌ, ΠΏΠΎΡΠΊΠΎΠ»ΡŒΠΊΡƒ ΠΎΠ½ ΠΌΠΎΠΆΠ΅Ρ‚ Π²Π»ΠΈΡΡ‚ΡŒ Π½Π° ΠΌΠ°Π³Π½ΠΈΡ‚Π½ΠΎΠ΅ ΠΏΠΎΠ»Π΅, Π° Π² Π΄Π°Π½Π½ΠΎΠΌ случаС ΡƒΡΠΈΠ»ΠΈΠ²Π°Ρ‚ΡŒ Π΅Π³ΠΎ. ΠžΡ‚Π½ΠΎΡΠΈΡ‚Π΅Π»ΡŒΠ½Π°Ρ ΠΏΡ€ΠΎΠ½ΠΈΡ†Π°Π΅ΠΌΠΎΡΡ‚ΡŒ для Ρ€Π°Π·Π»ΠΈΡ‡Π½Ρ‹Ρ… ΠΌΠ°Ρ‚Π΅Ρ€ΠΈΠ°Π»ΠΎΠ². ΠœΠ°Ρ‚Π΅Ρ€ΠΈΠ°Π» mr Π”ΠΈΠ°ΠΌΠ°Π³Π½ΠΈΡ‚Π½Ρ‹ΠΉ висмут, Π·ΠΎΠ»ΠΎΡ‚ΠΎ, сСрСбро, мСдь, Π²ΠΎΠ΄Π°, ΠΏΠ°Ρ€Π°ΠΌΠ°Π³Π½Π΅Ρ‚ΠΈΠΊ, Π²ΠΎΠ·Π΄ΡƒΡ…, алюминий, ΠΏΠ»Π°Ρ‚ΠΈΠ½Π°, 1.0003, Ρ„Π΅Ρ€Ρ€ΠΎΠΌΠ°Π³Π½ΠΈΡ‚Π½Ρ‹ΠΉ (Π½Π΅Π»ΠΈΠ½Π΅ΠΉΠ½Ρ‹ΠΉ), ΠΊΠΎΠ±Π°Π»ΡŒΡ‚, никСль, ΠΆΠ΅Π»Π΅Π·ΠΎ (99.8% -Π½ΠΎΠ΅ ΠΆΠ΅Π»Π΅Π·ΠΎ (чистота 99,96%) супСрсплав Mo / Ni 250 600 5000 280 000 1 000 000 Π‘Ρ‚Π΅ΠΏΠ΅Π½ΡŒ, Π² ΠΊΠΎΡ‚ΠΎΡ€ΠΎΠΉ ΠΌΠ°Ρ‚Π΅Ρ€ΠΈΠ°Π» ΠΌΠΎΠΆΠ΅Ρ‚ Π²Π»ΠΈΡΡ‚ΡŒ Π½Π° ΠΌΠ°Π³Π½ΠΈΡ‚Π½ΠΎΠ΅ ΠΏΠΎΠ»Π΅, опрСдСляСтся Π΅Π³ΠΎ ΠΎΡ‚Π½ΠΎΡΠΈΡ‚Π΅Π»ΡŒΠ½ΠΎΠΉ ΠΏΡ€ΠΎΠ½ΠΈΡ†Π°Π΅ΠΌΠΎΡΡ‚ΡŒΡŽ r, Π°Π½Π°Π»ΠΎΠ³ΠΈΡ‡Π½ΠΎΠΉ ΠΎΡ‚Π½ΠΎΡΠΈΡ‚Π΅Π»ΡŒΠ½ΠΎΠΉ диэлСктричСской проницаСмости ο₯r для диэлСктриков. Π’ свободном пространствС (Π²Π°ΠΊΡƒΡƒΠΌΠ΅) r = 1 ΠΈ влияниС Π½Π° ΠΏΠΎΠ»Π΅ отсутствуСт.

74 ΠŸΠ»ΠΎΡ‚Π½ΠΎΡΡ‚ΡŒ ΠΌΠ°Π³Π½ΠΈΡ‚Π½ΠΎΠ³ΠΎ ΠΏΠΎΡ‚ΠΎΠΊΠ° Π’ присутствии внСшнСго ΠΌΠ°Π³Π½ΠΈΡ‚Π½ΠΎΠ³ΠΎ поля ΠΌΠ°Π³Π½ΠΈΡ‚Π½Ρ‹ΠΉ ΠΌΠ°Ρ‚Π΅Ρ€ΠΈΠ°Π» намагничиваСтся (ΠΏΠΎΠ΄ΠΎΠ±Π½ΠΎ ΠΆΠ΅Π»Π΅Π·Π½ΠΎΠΌΡƒ сСрдСчнику).Π­Ρ‚ΠΎ свойство называСтся Π½Π°ΠΌΠ°Π³Π½ΠΈΡ‡Π΅Π½Π½ΠΎΡΡ‚ΡŒΡŽ M, опрСдСляСмой ΠΊΠ°ΠΊ Π³Π΄Π΅ m (Β«Ρ‡ΠΈΒ») — магнитная Π²ΠΎΡΠΏΡ€ΠΈΠΈΠΌΡ‡ΠΈΠ²ΠΎΡΡ‚ΡŒ ΠΌΠ°Ρ‚Π΅Ρ€ΠΈΠ°Π»Π°. Полная ΠΏΠ»ΠΎΡ‚Π½ΠΎΡΡ‚ΡŒ ΠΌΠ°Π³Π½ΠΈΡ‚Π½ΠΎΠ³ΠΎ ΠΏΠΎΡ‚ΠΎΠΊΠ° Π²Π½ΡƒΡ‚Ρ€ΠΈ ΠΌΠ°Π³Π½ΠΈΡ‚Π½ΠΎΠ³ΠΎ ΠΌΠ°Ρ‚Π΅Ρ€ΠΈΠ°Π»Π°, Π²ΠΊΠ»ΡŽΡ‡Π°Ρ эффСкт намагничивания M Π² присутствии внСшнСго ΠΌΠ°Π³Π½ΠΈΡ‚Π½ΠΎΠ³ΠΎ поля H, ΠΌΠΎΠΆΠ΅Ρ‚ Π±Ρ‹Ρ‚ΡŒ записана ΠΊΠ°ΠΊ ΠŸΠΎΠ΄ΡΡ‚Π°Π½ΠΎΠ²ΠΊΠ° Π³Π΄Π΅ Π³Π΄Π΅  — ΠΏΡ€ΠΎΠ½ΠΈΡ†Π°Π΅ΠΌΠΎΡΡ‚ΡŒ ΠΌΠ°Ρ‚Π΅Ρ€ΠΈΠ°Π»Π°, связанная с диэлСктричСской ΠΏΡ€ΠΎΠ½ΠΈΡ†Π°Π΅ΠΌΠΎΡΡ‚ΡŒΡŽ свободного пространства с ΠΏΠΎΠΌΠΎΡ‰ΡŒΡŽ Ρ„Π°ΠΊΡ‚ΠΎΡ€Π° r, Π½Π°Π·Ρ‹Π²Π°Π΅ΠΌΠΎΠ³ΠΎ ΠΎΡ‚Π½ΠΎΡΠΈΡ‚Π΅Π»ΡŒΠ½Π°Ρ ΠΏΡ€ΠΎΠ½ΠΈΡ†Π°Π΅ΠΌΠΎΡΡ‚ΡŒ.

75 ΠœΠ°Π³Π½ΠΈΡ‚ΠΎΡΡ‚Π°Ρ‚ΠΈΡ‡Π΅ΡΠΊΠΈΠ΅ Π³Ρ€Π°Π½ΠΈΡ‡Π½Ρ‹Π΅ условия
Π‘ΡƒΠ΄Π΅ΠΌ ΠΈΡΠΏΠΎΠ»ΡŒΠ·ΠΎΠ²Π°Ρ‚ΡŒ Π·Π°ΠΊΠΎΠ½ АмпСра ΠΈ Π·Π°ΠΊΠΎΠ½ Гаусса для получСния Π½ΠΎΡ€ΠΌΠ°Π»ΡŒΠ½Ρ‹Ρ… ΠΈ ΠΊΠ°ΡΠ°Ρ‚Π΅Π»ΡŒΠ½Ρ‹Ρ… Π³Ρ€Π°Π½ΠΈΡ‡Π½Ρ‹Ρ… условий для магнитостатики.Π—Π°ΠΊΠΎΠ½ АмпСра: ΠŸΡƒΡ‚ΡŒ 1 ΠŸΡƒΡ‚ΡŒ 4 ΠŸΡƒΡ‚ΡŒ 2 Π’ΠΎΠΊ, Π·Π°ΠΊΠ»ΡŽΡ‡Π΅Π½Π½Ρ‹ΠΉ Π½Π° ΠΏΡƒΡ‚ΠΈ, — ΠŸΡƒΡ‚ΡŒ 3 ΠœΡ‹ ΠΌΠΎΠΆΠ΅ΠΌ Ρ€Π°Π·Π±ΠΈΡ‚ΡŒ Ρ†ΠΈΡ€ΠΊΡƒΠ»ΡΡ†ΠΈΡŽ H Π½Π° Ρ‡Π΅Ρ‚Ρ‹Ρ€Π΅ ΠΈΠ½Ρ‚Π΅Π³Ρ€Π°Π»Π°: ΠŸΡƒΡ‚ΡŒ 1: ΠŸΡƒΡ‚ΡŒ 2:

76 ΠœΠ°Π³Π½ΠΈΡ‚ΠΎΡΡ‚Π°Ρ‚ΠΈΡ‡Π΅ΡΠΊΠΈΠ΅ Π³Ρ€Π°Π½ΠΈΡ‡Π½Ρ‹Π΅ условия
ΠŸΡƒΡ‚ΡŒ 3: ΠŸΡƒΡ‚ΡŒ 4: Π’Π΅ΠΏΠ΅Ρ€ΡŒ, комбинируя наши Ρ€Π΅Π·ΡƒΠ»ΡŒΡ‚Π°Ρ‚Ρ‹ (Ρ‚ΠΎ Π΅ΡΡ‚ΡŒ, ΠŸΡƒΡ‚ΡŒ 1 + ΠŸΡƒΡ‚ΡŒ 2 + ΠŸΡƒΡ‚ΡŒ 3 + ΠŸΡƒΡ‚ΡŒ 4), ΠΌΡ‹ ΠΏΠΎΠ»ΡƒΡ‡Π°Π΅ΠΌ ACL: Π£Ρ€Π°Π²Π½Π΅Π½ΠΈΠ΅ Tangential BC: Π±ΠΎΠ»Π΅Π΅ ΠΎΠ±Ρ‰Π΅Π΅ Π²Ρ‹Ρ€Π°ΠΆΠ΅Π½ΠΈΠ΅ для ΠΏΠ΅Ρ€Π²ΠΎΠ³ΠΎ магнитостатичСского Π³Ρ€Π°Π½ΠΈΡ‡Π½ΠΎΠ³ΠΎ условия ΠΌΠΎΠΆΠ½ΠΎ Π·Π°ΠΏΠΈΡΠ°Ρ‚ΡŒ ΠΊΠ°ΠΊ Π³Π΄Π΅ a21 — Π½ΠΎΡ€ΠΌΠ°Π»ΡŒ Π΅Π΄ΠΈΠ½ΠΈΡ‡Π½ΠΎΠ³ΠΎ Π²Π΅ΠΊΡ‚ΠΎΡ€Π°, идущая ΠΎΡ‚ срСды 2 ΠΊ срСдС 1.

77 ΠœΠ°Π³Π½ΠΈΡ‚ΠΎΡΡ‚Π°Ρ‚ΠΈΡ‡Π΅ΡΠΊΠΈΠ΅ Π³Ρ€Π°Π½ΠΈΡ‡Π½Ρ‹Π΅ условия
ΠžΡΠΎΠ±Ρ‹ΠΉ случай: Ссли ΠΏΠ»ΠΎΡ‚Π½ΠΎΡΡ‚ΡŒ повСрхностного Ρ‚ΠΎΠΊΠ° K = 0, ΠΌΡ‹ ΠΏΠΎΠ»ΡƒΡ‡Π°Π΅ΠΌ If K = 0 ΠΠ°ΠΏΡ€ΡΠΆΠ΅Π½Π½ΠΎΡΡ‚ΡŒ Ρ‚Π°Π½Π³Π΅Π½Ρ†ΠΈΠ°Π»ΡŒΠ½ΠΎΠ³ΠΎ ΠΌΠ°Π³Π½ΠΈΡ‚Π½ΠΎΠ³ΠΎ поля Π½Π΅ΠΏΡ€Π΅Ρ€Ρ‹Π²Π½Π° Ρ‡Π΅Ρ€Π΅Π· Π³Ρ€Π°Π½ΠΈΡ†Ρƒ, ΠΊΠΎΠ³Π΄Π° ΠΏΠ»ΠΎΡ‚Π½ΠΎΡΡ‚ΡŒ повСрхностного Ρ‚ΠΎΠΊΠ° Ρ€Π°Π²Π½Π° Π½ΡƒΠ»ΡŽ. Π’Π°ΠΆΠ½ΠΎΠ΅ ΠΏΡ€ΠΈΠΌΠ΅Ρ‡Π°Π½ΠΈΠ΅: ΠΌΡ‹ Π·Π½Π°Π΅ΠΌ, Ρ‡Ρ‚ΠΎ (ΠΈΠ»ΠΈ) Π˜ΡΠΏΠΎΠ»ΡŒΠ·ΡƒΡ ΡƒΠΊΠ°Π·Π°Π½Π½ΠΎΠ΅ Π²Ρ‹ΡˆΠ΅ ΡΠΎΠΎΡ‚Π½ΠΎΡˆΠ΅Π½ΠΈΠ΅, ΠΌΡ‹ ΠΏΠΎΠ»ΡƒΡ‡Π°Π΅ΠΌ Π‘Π»Π΅Π΄ΠΎΠ²Π°Ρ‚Π΅Π»ΡŒΠ½ΠΎ, ΠΌΡ‹ ΠΌΠΎΠΆΠ΅ΠΌ ΡΠΊΠ°Π·Π°Ρ‚ΡŒ, Ρ‡Ρ‚ΠΎ Ρ‚Π°Π½Π³Π΅Π½Ρ†ΠΈΠ°Π»ΡŒΠ½Π°Ρ ΡΠΎΡΡ‚Π°Π²Π»ΡΡŽΡ‰Π°Ρ плотности ΠΌΠ°Π³Π½ΠΈΡ‚Π½ΠΎΠ³ΠΎ ΠΏΠΎΡ‚ΠΎΠΊΠ° B Π½Π΅ являСтся Π½Π΅ΠΏΡ€Π΅Ρ€Ρ‹Π²Π½ΠΎΠΉ Ρ‡Π΅Ρ€Π΅Π· Π³Ρ€Π°Π½ΠΈΡ†Ρƒ.

78 ΠœΠ°Π³Π½ΠΈΡ‚ΠΎΡΡ‚Π°Ρ‚ΠΈΡ‡Π΅ΡΠΊΠΈΠ΅ Π³Ρ€Π°Π½ΠΈΡ‡Π½Ρ‹Π΅ условия
Π—Π°ΠΊΠΎΠ½ Гаусса для магнитостатичСских ΠΏΠΎΠ»Π΅ΠΉ: Ρ‡Ρ‚ΠΎΠ±Ρ‹ Π½Π°ΠΉΡ‚ΠΈ Π²Ρ‚ΠΎΡ€ΠΎΠ΅ Π³Ρ€Π°Π½ΠΈΡ‡Π½ΠΎΠ΅ условиС, ΠΌΡ‹ Ρ†Π΅Π½Ρ‚Ρ€ΠΈΡ€ΡƒΠ΅ΠΌ гауссовский Π΄ΠΎΡ‚ ΠΏΠΎ Π³Ρ€Π°Π½ΠΈΡ†Π΅ Ρ€Π°Π·Π΄Π΅Π»Π°, ΠΊΠ°ΠΊ ΠΏΠΎΠΊΠ°Π·Π°Π½ΠΎ Π½Π° рисункС. ΠœΡ‹ ΠΌΠΎΠΆΠ΅ΠΌ ΡƒΠΌΠ΅Π½ΡŒΡˆΠΈΡ‚ΡŒ h Ρ‚Π°ΠΊ, Ρ‡Ρ‚ΠΎΠ±Ρ‹ ΠΏΠΎΡ‚ΠΎΠΊ, исходящий ΠΈΠ· Π΄ΠΎΡ‚Π°, Π±Ρ‹Π» Π½Π΅Π·Π½Π°Ρ‡ΠΈΡ‚Π΅Π»ΡŒΠ½Ρ‹ΠΌ. Π—Π°Ρ‚Π΅ΠΌ Ρƒ нас Π΅ΡΡ‚ΡŒ ΠΠΎΡ€ΠΌΠ°Π»ΡŒΠ½Ρ‹ΠΉ BC:

79 ΠœΠ°Π³Π½ΠΈΡ‚ΠΎΡΡ‚Π°Ρ‚ΠΈΡ‡Π΅ΡΠΊΠΈΠ΅ Π³Ρ€Π°Π½ΠΈΡ‡Π½Ρ‹Π΅ условия
ΠΠΎΡ€ΠΌΠ°Π»ΡŒΠ½Ρ‹ΠΉ BC: Π’Π°ΠΊΠΈΠΌ ΠΎΠ±Ρ€Π°Π·ΠΎΠΌ, ΠΌΡ‹ Π²ΠΈΠ΄ΠΈΠΌ, Ρ‡Ρ‚ΠΎ Π½ΠΎΡ€ΠΌΠ°Π»ΡŒΠ½Π°Ρ ΡΠΎΡΡ‚Π°Π²Π»ΡΡŽΡ‰Π°Ρ плотности ΠΌΠ°Π³Π½ΠΈΡ‚Π½ΠΎΠ³ΠΎ ΠΏΠΎΡ‚ΠΎΠΊΠ° Π΄ΠΎΠ»ΠΆΠ½Π° Π±Ρ‹Ρ‚ΡŒ Π½Π΅ΠΏΡ€Π΅Ρ€Ρ‹Π²Π½ΠΎΠΉ Ρ‡Π΅Ρ€Π΅Π· Π³Ρ€Π°Π½ΠΈΡ†Ρƒ.Π’Π°ΠΆΠ½ΠΎΠ΅ ΠΏΡ€ΠΈΠΌΠ΅Ρ‡Π°Π½ΠΈΠ΅: ΠΌΡ‹ Π·Π½Π°Π΅ΠΌ, Ρ‡Ρ‚ΠΎ, ΠΈΡΠΏΠΎΠ»ΡŒΠ·ΡƒΡ ΠΏΡ€ΠΈΠ²Π΅Π΄Π΅Π½Π½ΠΎΠ΅ Π²Ρ‹ΡˆΠ΅ ΡΠΎΠΎΡ‚Π½ΠΎΡˆΠ΅Π½ΠΈΠ΅, ΠΌΡ‹ ΠΏΠΎΠ»ΡƒΡ‡Π°Π΅ΠΌ Π‘Π»Π΅Π΄ΠΎΠ²Π°Ρ‚Π΅Π»ΡŒΠ½ΠΎ, ΠΌΡ‹ ΠΌΠΎΠΆΠ΅ΠΌ ΡΠΊΠ°Π·Π°Ρ‚ΡŒ, Ρ‡Ρ‚ΠΎ Π½ΠΎΡ€ΠΌΠ°Π»ΡŒΠ½Π°Ρ ΡΠΎΡΡ‚Π°Π²Π»ΡΡŽΡ‰Π°Ρ напряТСнности ΠΌΠ°Π³Π½ΠΈΡ‚Π½ΠΎΠ³ΠΎ поля Π½Π΅ являСтся Π½Π΅ΠΏΡ€Π΅Ρ€Ρ‹Π²Π½ΠΎΠΉ Ρ‡Π΅Ρ€Π΅Π· Π³Ρ€Π°Π½ΠΈΡ†Ρƒ (Π½ΠΎ ΠΏΠ»ΠΎΡ‚Π½ΠΎΡΡ‚ΡŒ ΠΌΠ°Π³Π½ΠΈΡ‚Π½ΠΎΠ³ΠΎ ΠΏΠΎΡ‚ΠΎΠΊΠ° Π½Π΅ΠΏΡ€Π΅Ρ€Ρ‹Π²Π½Π°).

80 ΠœΠ°Π³Π½ΠΈΡ‚ΠΎΡΡ‚Π°Ρ‚ΠΈΡ‡Π΅ΡΠΊΠΈΠ΅ Π³Ρ€Π°Π½ΠΈΡ‡Π½Ρ‹Π΅ условия
ΠŸΡ€ΠΈΠΌΠ΅Ρ€ 3.11: ΠΠ°ΠΏΡ€ΡΠΆΠ΅Π½Π½ΠΎΡΡ‚ΡŒ ΠΌΠ°Π³Π½ΠΈΡ‚Π½ΠΎΠ³ΠΎ поля задаСтся ΠΊΠ°ΠΊ h2 = 6ax + 2ay + 3az (А / ΠΌ) Π² срСдС с r1 = 6000, которая сущСствуСт для z <0.ΠœΡ‹ Ρ…ΠΎΡ‚ΠΈΠΌ Π½Π°ΠΉΡ‚ΠΈ h3 Π² срСдС с r2 = 3000 ΠΏΡ€ΠΈ z> 0. Π¨Π°Π³ (a) ΠΈ (b): ΠŸΠ΅Ρ€Π²Ρ‹ΠΉ шаг — Ρ€Π°Π·Π±ΠΈΡ‚ΡŒ h2 Π½Π° Π΅Π³ΠΎ Π½ΠΎΡ€ΠΌΠ°Π»ΡŒΠ½ΡƒΡŽ ΡΠΎΡΡ‚Π°Π²Π»ΡΡŽΡ‰ΡƒΡŽ (a) ΠΈ Ρ‚Π°Π½Π³Π΅Π½Ρ†ΠΈΠ°Π»ΡŒΠ½ΡƒΡŽ ΡΠΎΡΡ‚Π°Π²Π»ΡΡŽΡ‰ΡƒΡŽ (b). Π¨Π°Π³ (c): ΠΏΡ€ΠΈ отсутствии Ρ‚ΠΎΠΊΠ° Π½Π° Π³Ρ€Π°Π½ΠΈΡ†Π΅ Ρ€Π°Π·Π΄Π΅Π»Π° Ρ‚Π°Π½Π³Π΅Π½Ρ†ΠΈΠ°Π»ΡŒΠ½Π°Ρ ΡΠΎΡΡ‚Π°Π²Π»ΡΡŽΡ‰Π°Ρ ΠΎΠ΄ΠΈΠ½Π°ΠΊΠΎΠ²Π° с ΠΎΠ±Π΅ΠΈΡ… сторон Π³Ρ€Π°Π½ΠΈΡ†Ρ‹. Π¨Π°Π³ (d): Π—Π°Ρ‚Π΅ΠΌ ΠΌΡ‹ Π½Π°Ρ…ΠΎΠ΄ΠΈΠΌ BN1, умноТая HN1 Π½Π° ΠΏΡ€ΠΎΠ½ΠΈΡ†Π°Π΅ΠΌΠΎΡΡ‚ΡŒ Π² срСдС 1. Π¨Π°Π³ (e): Π­Ρ‚ΠΎΡ‚ Π½ΠΎΡ€ΠΌΠ°Π»ΡŒΠ½Ρ‹ΠΉ ΠΊΠΎΠΌΠΏΠΎΠ½Π΅Π½Ρ‚ B ΠΎΠ΄ΠΈΠ½Π°ΠΊΠΎΠ² с ΠΎΠ±Π΅ΠΈΡ… сторон Π³Ρ€Π°Π½ΠΈΡ†Ρ‹. Π¨Π°Π³ (f): Π—Π°Ρ‚Π΅ΠΌ ΠΌΡ‹ ΠΌΠΎΠΆΠ΅ΠΌ Π½Π°ΠΉΡ‚ΠΈ HN2, Ρ€Π°Π·Π΄Π΅Π»ΠΈΠ² BN2 Π½Π° ΠΏΡ€ΠΎΠ½ΠΈΡ†Π°Π΅ΠΌΠΎΡΡ‚ΡŒ срСды 2.Π¨Π°Π³ (g): ПослСдний шаг — ΡΡƒΠΌΠΌΠΈΡ€ΠΎΠ²Π°Ρ‚ΡŒ поля.

81 Π³ΠΎΠ΄ ΠΠ°ΠΌΠ°Π³Π½ΠΈΡ‡Π΅Π½Π½ΠΎΡΡ‚ΡŒ ΠΈ ΠΏΡ€ΠΎΠ½ΠΈΡ†Π°Π΅ΠΌΠΎΡΡ‚ΡŒ
M ΠΌΠΎΠΆΠ½ΠΎ Ρ€Π°ΡΡΠΌΠ°Ρ‚Ρ€ΠΈΠ²Π°Ρ‚ΡŒ ΠΊΠ°ΠΊ Π½Π°ΠΏΡ€ΡΠΆΠ΅Π½Π½ΠΎΡΡ‚ΡŒ ΠΌΠ°Π³Π½ΠΈΡ‚Π½ΠΎΠ³ΠΎ поля ΠΈΠ·-Π·Π° Π΄ΠΈΠΏΠΎΠ»ΡŒΠ½Ρ‹Ρ… ΠΌΠΎΠΌΠ΅Π½Ρ‚ΠΎΠ² ΠΏΡ€ΠΈ ΠΏΡ€ΠΈΠ»ΠΎΠΆΠ΅Π½ΠΈΠΈ внСшнСго поля H Π‘Π»Π΅Π΄ΠΎΠ²Π°Ρ‚Π΅Π»ΡŒΠ½ΠΎ, общая Π½Π°ΠΏΡ€ΡΠΆΠ΅Π½Π½ΠΎΡΡ‚ΡŒ ΠΌΠ°Π³Π½ΠΈΡ‚Π½ΠΎΠ³ΠΎ поля Π²Π½ΡƒΡ‚Ρ€ΠΈ ΠΌΠ°Ρ‚Π΅Ρ€ΠΈΠ°Π»Π° Ρ€Π°Π²Π½Π° M + H ΠŸΠ»ΠΎΡ‚Π½ΠΎΡΡ‚ΡŒ ΠΌΠ°Π³Π½ΠΈΡ‚Π½ΠΎΠ³ΠΎ поля Π²Π½ΡƒΡ‚Ρ€ΠΈ ΠΌΠ°Ρ‚Π΅Ρ€ΠΈΠ°Π»Π° Ρ€Π°Π²Π½Π° B = mo (M + H) Но M зависит ΠΎΡ‚ H, ΠžΠΏΡ€Π΅Π΄Π΅Π»ΠΈΡ‚Π΅ ΠΌΠ°Π³Π½ΠΈΡ‚Π½ΡƒΡŽ Π²ΠΎΡΠΏΡ€ΠΈΠΈΠΌΡ‡ΠΈΠ²ΠΎΡΡ‚ΡŒ m.Π‘Π»Π΅Π΄ΠΎΠ²Π°Ρ‚Π΅Π»ΡŒΠ½ΠΎ, M = m H ΠžΡ‚ΡΡŽΠ΄Π° ΠΏΠΎΠ»ΡƒΡ‡Π°Π΅ΠΌ B = mo (m H + H) = B = moH (m +1) НаконСц, ΠΎΠΏΡ€Π΅Π΄Π΅Π»ΠΈΠΌ ΠΎΡ‚Π½ΠΎΡΠΈΡ‚Π΅Π»ΡŒΠ½ΡƒΡŽ ΠΏΡ€ΠΎΠ½ΠΈΡ†Π°Π΅ΠΌΠΎΡΡ‚ΡŒ mr = (m +1) B = mo mr H = m ЧАБ

82 ΠŸΡ€ΠΈΡ€ΠΎΠ΄Π° ΠΌΠ°Π³Π½ΠΈΡ‚Π½Ρ‹Ρ… ΠΌΠ°Ρ‚Π΅Ρ€ΠΈΠ°Π»ΠΎΠ²
ΠœΠ°Ρ‚Π΅Ρ€ΠΈΠ°Π»Ρ‹ ΠΈΠΌΠ΅ΡŽΡ‚ Ρ€Π°Π·Π½ΠΎΠ΅ ΠΏΠΎΠ²Π΅Π΄Π΅Π½ΠΈΠ΅ Π² ΠΌΠ°Π³Π½ΠΈΡ‚Π½Ρ‹Ρ… полях. Для Ρ‚ΠΎΡ‡Π½ΠΎΠ³ΠΎ описания трСбуСтся квантовая тСория. Однако Π½Π°ΠΌ достаточно простой ΠΌΠΎΠ΄Π΅Π»ΠΈ Π°Ρ‚ΠΎΠΌΠ° (Ρ†Π΅Π½Ρ‚Ρ€Π°Π»ΡŒΠ½ΠΎΠ΅ ядро, ΠΎΠΊΡ€ΡƒΠΆΠ΅Π½Π½ΠΎΠ΅ элСктронами). ΠœΡ‹ Ρ‚Π°ΠΊΠΆΠ΅ ΠΌΠΎΠΆΠ΅ΠΌ ΡΠΊΠ°Π·Π°Ρ‚ΡŒ, Ρ‡Ρ‚ΠΎ B пытаСтся ΡΠΎΠ·Π΄Π°Ρ‚ΡŒ ΠΌΠΎΠΌΠ΅Π½Ρ‚ ΠΌΠ°Π³Π½ΠΈΡ‚Π½ΠΎΠ³ΠΎ диполя. ΠΌΠΌ Π² Ρ‚ΠΎΠΌ ΠΆΠ΅ Π½Π°ΠΏΡ€Π°Π²Π»Π΅Π½ΠΈΠΈ, Ρ‡Ρ‚ΠΎ ΠΈ B

83 ΠœΠ°Π³Π½ΠΈΡ‚Π½Ρ‹Π΅ Π΄ΠΈΠΏΠΎΠ»ΡŒΠ½Ρ‹Π΅ ΠΌΠΎΠΌΠ΅Π½Ρ‚Ρ‹ Π² Π°Ρ‚ΠΎΠΌΠ΅
Π•ΡΡ‚ΡŒ 3 ΠΌΠ°Π³Π½ΠΈΡ‚Π½Ρ‹Ρ… Π΄ΠΈΠΏΠΎΠ»ΡŒΠ½Ρ‹Ρ… ΠΌΠΎΠΌΠ΅Π½Ρ‚Π°: ΠœΠΎΠΌΠ΅Π½Ρ‚ ΠΈΠ·-Π·Π° вращСния элСктронов ΠœΠΎΠΌΠ΅Π½Ρ‚ ΠΈΠ·-Π·Π° спина элСктронов ΠœΠΎΠΌΠ΅Π½Ρ‚ ΠΈΠ·-Π·Π° спина ядра Π’Ρ€ΠΈ вращСния — это 3 Ρ‚ΠΎΠΊΠ° ΠΏΠ΅Ρ‚Π»ΠΈ ΠŸΠ΅Ρ€Π²Ρ‹Π΅ Π΄Π²Π° Π·Π½Π°Ρ‡ΠΈΡ‚Π΅Π»ΡŒΠ½Ρ‹ Π±ΠΎΠ»Π΅Π΅ эффСктивный Π­Π»Π΅ΠΊΡ‚Ρ€ΠΎΠ½Π½Ρ‹ΠΉ спин ΠΏΠΎΠΏΠ°Ρ€Π½ΠΎ, Π² Π΄Π²ΡƒΡ… ΠΏΡ€ΠΎΡ‚ΠΈΠ²ΠΎΠΏΠΎΠ»ΠΎΠΆΠ½Ρ‹Ρ… направлСниях Π‘Π»Π΅Π΄ΠΎΠ²Π°Ρ‚Π΅Π»ΡŒΠ½ΠΎ, Ρ€Π΅Π·ΡƒΠ»ΡŒΡ‚ΠΈΡ€ΡƒΡŽΡ‰ΠΈΠΉ ΠΌΠΎΠΌΠ΅Π½Ρ‚ ΠΈΠ·-Π·Π° элСктронного спина Π²ΠΎΠ·Π½ΠΈΠΊΠ°Π΅Ρ‚ Ρ‚ΠΎΠ»ΡŒΠΊΠΎ Ρ‚ΠΎΠ³Π΄Π°, ΠΊΠΎΠ³Π΄Π° Π΅ΡΡ‚ΡŒ нСзаполнСнная ΠΎΠ±ΠΎΠ»ΠΎΡ‡ΠΊΠ° (ΠΈΠ»ΠΈ ΠΎΡ€Π±ΠΈΡ‚Π°) .ΠšΠΎΠΌΠ±ΠΈΠ½Π°Ρ†ΠΈΡ ΠΌΠΎΠΌΠ΅Π½Ρ‚Π° опрСдСляСт ΠΌΠ°Π³Π½ΠΈΡ‚Π½Ρ‹Π΅ характСристики ΠΌΠ°Ρ‚Π΅Ρ€ΠΈΠ°Π»Π°

84 Π’ΠΈΠΏ ΠΌΠ°Π³Π½ΠΈΡ‚Π½Ρ‹Ρ… ΠΌΠ°Ρ‚Π΅Ρ€ΠΈΠ°Π»ΠΎΠ²
ΠœΡ‹ Π±ΡƒΠ΄Π΅ΠΌ ΠΈΠ·ΡƒΡ‡Π°Ρ‚ΡŒ 6 Ρ‚ΠΈΠΏΠΎΠ²: Π”ΠΈΠ°ΠΌΠ°Π³Π½ΠΈΡ‚Π½Ρ‹ΠΉ ΠŸΠ°Ρ€Π°ΠΌΠ°Π³Π½ΠΈΡ‚Π½Ρ‹ΠΉ Π€Π΅Ρ€Ρ€ΠΎΠΌΠ°Π³Π½Π΅Ρ‚ΠΈΠΊ АнтифСрромагнитный Π€Π΅Ρ€Ρ€ΠΈΠΌΠ°Π³Π½ΠΈΡ‚Π½Ρ‹ΠΉ Π‘ΡƒΠΏΠ΅Ρ€ΠΏΠ°Ρ€Π°ΠΌΠ°Π³Π½ΠΈΡ‚Π½Ρ‹ΠΉ

85 Π”ΠΈΠ°ΠΌΠ°Π³Π½ΠΈΡ‚Π½Ρ‹Π΅ ΠΌΠ°Ρ‚Π΅Ρ€ΠΈΠ°Π»Ρ‹
Π‘Π΅Π· внСшнСго ΠΌΠ°Π³Π½ΠΈΡ‚Π½ΠΎΠ³ΠΎ поля Π΄ΠΈΠ°ΠΌΠ°Π³Π½ΠΈΡ‚Π½Ρ‹Π΅ ΠΌΠ°Ρ‚Π΅Ρ€ΠΈΠ°Π»Ρ‹ Π½Π΅ ΠΈΠΌΠ΅ΡŽΡ‚ чистого ΠΌΠ°Π³Π½ΠΈΡ‚Π½ΠΎΠ³ΠΎ поля ΠŸΡ€ΠΈ Π½Π°Π»ΠΈΡ‡ΠΈΠΈ внСшнСго ΠΌΠ°Π³Π½ΠΈΡ‚Π½ΠΎΠ³ΠΎ поля ΠΎΠ½ΠΈ ΡΠΎΠ·Π΄Π°ΡŽΡ‚ нСбольшоС ΠΌΠ°Π³Π½ΠΈΡ‚Π½ΠΎΠ΅ ΠΏΠΎΠ»Π΅ Π² ΠΏΡ€ΠΎΡ‚ΠΈΠ²ΠΎΠΏΠΎΠ»ΠΎΠΆΠ½ΠΎΠΌ Π½Π°ΠΏΡ€Π°Π²Π»Π΅Π½ΠΈΠΈ. Π’Π΅Π»ΠΈΡ‡ΠΈΠ½Π° этого ΠΏΡ€ΠΎΡ‚ΠΈΠ²ΠΎΠΏΠΎΠ»ΠΎΠΆΠ½ΠΎΠ³ΠΎ поля зависит ΠΎΡ‚ внСшнСго поля ΠΈ Π΄ΠΈΠ°ΠΌΠ°Π³Π½ΠΈΡ‚Π½ΠΎΠ³ΠΎ ΠΌΠ°Ρ‚Π΅Ρ€ΠΈΠ°Π»Π°. ΠΌΠ°Ρ‚Π΅Ρ€ΠΈΠ°Π»Ρ‹ Π΄ΠΈΠ°ΠΌΠ°Π³Π½ΠΈΡ‚Π½Ρ‹ (с Ρ€Π°Π·Π½Ρ‹ΠΌΠΈ ΠΏΠ°Ρ€Π°ΠΌΠ΅Ρ‚Ρ€Π°ΠΌΠΈ). ΠœΡ‹ ΡƒΠ²ΠΈΠ΄ΠΈΠΌ, Ρ‡Ρ‚ΠΎ ΠΎΡ‚Π½ΠΎΡΠΈΡ‚Π΅Π»ΡŒΠ½Π°Ρ ΠΏΡ€ΠΎΠ½ΠΈΡ†Π°Π΅ΠΌΠΎΡΡ‚ΡŒ mr, Π½ΠΎ  1

86 ΠŸΠΎΡ‡Π΅ΠΌΡƒ Π΄ΠΈΠ°ΠΌΠ°Π³Π½ΠΈΡ‚Π½Ρ‹Π΅ ΠΌΠ°Ρ‚Π΅Ρ€ΠΈΠ°Π»Ρ‹, 1
ΠšΠ°ΠΆΠ΄Ρ‹ΠΉ Π°Ρ‚ΠΎΠΌ ΠΈΠΌΠ΅Π΅Ρ‚ Π½ΡƒΠ»Π΅Π²ΠΎΠΉ ΠΎΠ±Ρ‰ΠΈΠΉ ΠΌΠ°Π³Π½ΠΈΡ‚Π½Ρ‹ΠΉ Π΄ΠΈΠΏΠΎΠ»ΡŒΠ½Ρ‹ΠΉ ΠΌΠΎΠΌΠ΅Π½Ρ‚ НСт крутящСго ΠΌΠΎΠΌΠ΅Π½Ρ‚Π° ΠΈΠ·-Π·Π° внСшнСго поля ΠΈ Π½Π΅ добавляСт Π½ΠΈΠΊΠ°ΠΊΠΎΠ³ΠΎ поля Но Ссли Π½Π΅ΠΊΠΎΡ‚ΠΎΡ€Ρ‹Π΅ элСктроны ΠΈΠΌΠ΅ΡŽΡ‚ свой ΠΌΠ°Π³Π½ΠΈΡ‚Π½Ρ‹ΠΉ Π΄ΠΈΠΏΠΎΠ»ΡŒΠ½Ρ‹ΠΉ ΠΌΠΎΠΌΠ΅Π½Ρ‚ с внСшним ΠΏΠΎΠ»Π΅ΠΌ Π’Π½Π΅ΡˆΠ½Π΅Π΅ ΠΏΠΎΠ»Π΅ Π²Ρ‹Π·ΠΎΠ²Π΅Ρ‚ Π½Π΅Π±ΠΎΠ»ΡŒΡˆΡƒΡŽ внСшнюю силу Π½Π° элСктроны, ΠΊΠΎΡ‚ΠΎΡ€Ρ‹Π΅ Π΄ΠΎΠ±Π°Π²Π»ΡΡŽΡ‚ ΠΊ ΠΈΡ… Ρ†Π΅Π½Ρ‚Ρ€ΠΎΠ±Π΅ΠΆΠ½ΠΎΠΉ силС. Π­Π»Π΅ΠΊΡ‚Ρ€ΠΎΠ½Ρ‹ Π½Π΅ ΠΌΠΎΠ³ΡƒΡ‚ ΠΏΠΎΠΊΠΈΠ½ΡƒΡ‚ΡŒ ΠΎΠ±ΠΎΠ»ΠΎΡ‡ΠΊΠΈ ΠΈ ΠΏΠ΅Ρ€Π΅ΠΉΡ‚ΠΈ ΠΊ ΡΠ»Π΅Π΄ΡƒΡŽΡ‰Π΅ΠΉ ΠΎΠ±ΠΎΠ»ΠΎΡ‡ΠΊΠ΅ (нСдостаточно энСргии). Π‘ΠΈΠ»Π° притяТСния ΠΊΡƒΠ»ΠΎΠ½ΠΎΠ² с ядром ΠΎΠ΄ΠΈΠ½Π°ΠΊΠΎΠ²Π°. Π§Ρ‚ΠΎΠ±Ρ‹ ΠΎΡΡ‚Π°Π²Π°Ρ‚ΡŒΡΡ Π½Π° Ρ‚ΠΎΠΉ ΠΆΠ΅ ΠΎΡ€Π±ΠΈΡ‚Π΅, цСнтробСТная сила Π΄ΠΎΠ»ΠΆΠ½Π° ΡƒΠΌΠ΅Π½ΡŒΡˆΠ°Ρ‚ΡŒΡΡ.Π‘Π»Π΅Π΄ΠΎΠ²Π°Ρ‚Π΅Π»ΡŒΠ½ΠΎ, ΡΠΊΠΎΡ€ΠΎΡΡ‚ΡŒ ΡƒΠΌΠ΅Π½ΡŒΡˆΠ°Π΅Ρ‚ΡΡ. ΠœΠ°Π³Π½ΠΈΡ‚Π½Ρ‹ΠΉ Π΄ΠΈΠΏΠΎΠ»ΡŒΠ½Ρ‹ΠΉ ΠΌΠΎΠΌΠ΅Π½Ρ‚ Π°Ρ‚ΠΎΠΌΠ° ΡƒΠΌΠ΅Π½ΡŒΡˆΠ°Π΅Ρ‚ΡΡ. БоздаСтся чистый ΠΌΠ°Π³Π½ΠΈΡ‚Π½Ρ‹ΠΉ Π΄ΠΈΠΏΠΎΠ»ΡŒΠ½Ρ‹ΠΉ ΠΌΠΎΠΌΠ΅Π½Ρ‚ Π°Ρ‚ΠΎΠΌΠ°, ΠΏΡ€ΠΎΡ‚ΠΈΠ²ΠΎΠΏΠΎΠ»ΠΎΠΆΠ½Ρ‹ΠΉ B.

87 ΠŸΠΎΡ‡Π΅ΠΌΡƒ Π΄ΠΈΠ°ΠΌΠ°Π³Π½ΠΈΡ‚Π½Ρ‹Π΅ ΠΌΠ°Ρ‚Π΅Ρ€ΠΈΠ°Π»Ρ‹, 2
Π’Π°ΠΊΠΆΠ΅, Ссли Π½Π΅ΠΊΠΎΡ‚ΠΎΡ€Ρ‹Π΅ элСктроны ΠΈΠΌΠ΅ΡŽΡ‚ свой ΠΌΠ°Π³Π½ΠΈΡ‚Π½Ρ‹ΠΉ Π΄ΠΈΠΏΠΎΠ»ΡŒΠ½Ρ‹ΠΉ ΠΌΠΎΠΌΠ΅Π½Ρ‚, ΠΏΡ€ΠΎΡ‚ΠΈΠ²ΠΎΠΏΠΎΠ»ΠΎΠΆΠ½Ρ‹ΠΉ Π²Π½Π΅ΡˆΠ½Π΅ΠΌΡƒ полю Π’Π½Π΅ΡˆΠ½Π΅Π΅ ΠΏΠΎΠ»Π΅ Π²Ρ‹Π·ΠΎΠ²Π΅Ρ‚ Π½Π΅Π±ΠΎΠ»ΡŒΡˆΡƒΡŽ Π²Π½ΡƒΡ‚Ρ€Π΅Π½Π½ΡŽΡŽ силу Π½Π° элСктроны, которая ΡƒΠΌΠ΅Π½ΡŒΡˆΠ°Π΅Ρ‚ Ρ†Π΅Π½Ρ‚Ρ€ΠΎΠ±Π΅ΠΆΠ½ΡƒΡŽ силу Π­Π»Π΅ΠΊΡ‚Ρ€ΠΎΠ½Ρ‹ Π½Π΅ ΠΌΠΎΠ³ΡƒΡ‚ ΠΏΠΎΠΊΠΈΠ½ΡƒΡ‚ΡŒ ΠΎΠ±ΠΎΠ»ΠΎΡ‡ΠΊΠΈ ΠΈ ΠΏΠ΅Ρ€Π΅ΠΉΡ‚ΠΈ ΠΊ ΡΠ»Π΅Π΄ΡƒΡŽΡ‰Π΅ΠΉ ΠΎΠ±ΠΎΠ»ΠΎΡ‡ΠΊΠ΅ (нСдостаточно энСргии) Π‘ΠΈΠ»Π° притяТСния ΠΊΡƒΠ»ΠΎΠ½ΠΎΠ² с ядром ΠΎΠ΄ΠΈΠ½Π°ΠΊΠΎΠ²Π°. Π§Ρ‚ΠΎΠ±Ρ‹ ΠΎΡΡ‚Π°Π²Π°Ρ‚ΡŒΡΡ Π½Π° Ρ‚ΠΎΠΉ ΠΆΠ΅ ΠΎΡ€Π±ΠΈΡ‚Π΅, цСнтробСТная сила Π΄ΠΎΠ»ΠΆΠ½Π° ΡƒΠ²Π΅Π»ΠΈΡ‡ΠΈΠ²Π°Ρ‚ΡŒΡΡ. ΠœΠ°Π³Π½ΠΈΡ‚Π½Ρ‹ΠΉ Π΄ΠΈΠΏΠΎΠ»ΡŒΠ½Ρ‹ΠΉ ΠΌΠΎΠΌΠ΅Π½Ρ‚ Π°Ρ‚ΠΎΠΌΠ° увСличиваСтся. БоздаСтся чистый ΠΌΠ°Π³Π½ΠΈΡ‚Π½Ρ‹ΠΉ Π΄ΠΈΠΏΠΎΠ»ΡŒΠ½Ρ‹ΠΉ ΠΌΠΎΠΌΠ΅Π½Ρ‚ Π°Ρ‚ΠΎΠΌΠ°, ΠΏΡ€ΠΎΡ‚ΠΈΠ²ΠΎΠΏΠΎΠ»ΠΎΠΆΠ½Ρ‹ΠΉ B

88 Π”ΠΈΠ°ΠΌΠ°Π³Π½ΠΈΡ‚Π½Ρ‹Π΅ ΠΌΠ°Ρ‚Π΅Ρ€ΠΈΠ°Π»Ρ‹
ΠžΠ±Ρ€Π°Ρ‚ΠΈΡ‚Π΅ Π²Π½ΠΈΠΌΠ°Π½ΠΈΠ΅, Ρ‡Ρ‚ΠΎ mr = 1 + Π²ΠΎΡΠΏΡ€ΠΈΠΈΠΌΡ‡ΠΈΠ²ΠΎΡΡ‚ΡŒ Π§ΡƒΠ²ΡΡ‚Π²ΠΈΡ‚Π΅Π»ΡŒΠ½ΠΎΡΡ‚ΡŒ ΠΌΠ°Ρ‚Π΅Ρ€ΠΈΠ°Π»Π° x 10-5 Висмут -16.6 Π£Π³Π»Π΅Ρ€ΠΎΠ΄ (Π°Π»ΠΌΠ°Π·) -2,1 Π£Π³Π»Π΅Ρ€ΠΎΠ΄ (Π³Ρ€Π°Ρ„ΠΈΡ‚) -1,6 МСдь -1,0 Π‘Π²Π΅Ρ€Ρ…ΠΏΡ€ΠΎΠ²ΠΎΠ΄Π½ΠΈΠΊ -105

89 ΠŸΠ°Ρ€Π°ΠΌΠ°Π³Π½ΠΈΡ‚Π½Ρ‹Π΅ ΠΌΠ°Ρ‚Π΅Ρ€ΠΈΠ°Π»Ρ‹
Атомы ΠΈΠΌΠ΅ΡŽΡ‚ нСбольшой суммарный ΠΌΠ°Π³Π½ΠΈΡ‚Π½Ρ‹ΠΉ Π΄ΠΈΠΏΠΎΠ»ΡŒΠ½Ρ‹ΠΉ ΠΌΠΎΠΌΠ΅Π½Ρ‚. Блучайная ориСнтация Π°Ρ‚ΠΎΠΌΠΎΠ² Π΄Π΅Π»Π°Π΅Ρ‚ срСдний Π΄ΠΈΠΏΠΎΠ»ΡŒΠ½Ρ‹ΠΉ ΠΌΠΎΠΌΠ΅Π½Ρ‚ Π² ΠΌΠ°Ρ‚Π΅Ρ€ΠΈΠ°Π»Π΅ Ρ€Π°Π²Π½Ρ‹ΠΌ Π½ΡƒΠ»ΡŽ. Π‘Π΅Π· внСшнСго поля Π½Π΅Ρ‚ ΠΌΠ°Π³Π½ΠΈΡ‚Π½Ρ‹Ρ… свойств. Π°Ρ‚ΠΎΠΌΡ‹ ΠΈ ΠΎΠ½ΠΈ Π²Ρ‹Ρ€Π°Π²Π½ΠΈΠ²Π°ΡŽΡ‚ΡΡ с ΠΏΠΎΠ»Π΅ΠΌ Π‘Π»Π΅Π΄ΠΎΠ²Π°Ρ‚Π΅Π»ΡŒΠ½ΠΎ, Π²Π½ΡƒΡ‚Ρ€ΠΈ ΠΌΠ°Ρ‚Π΅Ρ€ΠΈΠ°Π»Π° Π°Ρ‚ΠΎΠΌΡ‹ Π΄ΠΎΠ±Π°Π²Π»ΡΡŽΡ‚ своС собствСнноС ΠΏΠΎΠ»Π΅ ΠΊ Π²Π½Π΅ΡˆΠ½Π΅ΠΌΡƒ полю. Π”ΠΈΠ°ΠΌΠ°Π³Π½Π΅Ρ‚ΠΈΠ·ΠΌ ΠΈΠ·-Π·Π° ΠΎΡ€Π±ΠΈΡ‚Π°Π»ΡŒΠ½Ρ‹Ρ… элСктронов Ρ‚Π°ΠΊΠΆΠ΅ дСйствуСт.

90 ΠŸΠ°Ρ€Π°ΠΌΠ°Π³Π½ΠΈΡ‚Π½Ρ‹Π΅ ΠΌΠ°Ρ‚Π΅Ρ€ΠΈΠ°Π»Ρ‹
ΠžΠ±Ρ€Π°Ρ‚ΠΈΡ‚Π΅ Π²Π½ΠΈΠΌΠ°Π½ΠΈΠ΅, Ρ‡Ρ‚ΠΎ mr = 1 + Π²ΠΎΡΠΏΡ€ΠΈΠΈΠΌΡ‡ΠΈΠ²ΠΎΡΡ‚ΡŒ Π§ΡƒΠ²ΡΡ‚Π²ΠΈΡ‚Π΅Π»ΡŒΠ½ΠΎΡΡ‚ΡŒ ΠΌΠ°Ρ‚Π΅Ρ€ΠΈΠ°Π»Π° x 10-5 Оксид ΠΆΠ΅Π»Π΅Π·Π° (FeO) 720 Π–Π΅Π»Π΅Π·ΠΎ-Π°ΠΌΠΎΠ½ΠΈΠ΅Π²Ρ‹Π΅ квасцы 66 Π£Ρ€Π°Π½ 40 ΠŸΠ»Π°Ρ‚ΠΈΠ½Π° 26 Π’ΠΎΠ»ΡŒΡ„Ρ€Π°ΠΌ 6.8

91 Π€Π΅Ρ€Ρ€ΠΎΠΌΠ°Π³Π½ΠΈΡ‚Π½Ρ‹Π΅ ΠΌΠ°Ρ‚Π΅Ρ€ΠΈΠ°Π»Ρ‹
Атомы ΠΎΠ±Π»Π°Π΄Π°ΡŽΡ‚ большим Π΄ΠΈΠΏΠΎΠ»ΡŒΠ½Ρ‹ΠΌ ΠΌΠΎΠΌΠ΅Π½Ρ‚ΠΎΠΌ, ΠΎΠ½ΠΈ Π²Π»ΠΈΡΡŽΡ‚ Π΄Ρ€ΡƒΠ³ Π½Π° Π΄Ρ€ΡƒΠ³Π° ВзаимодСйствиС ΠΌΠ΅ΠΆΠ΄Ρƒ Π°Ρ‚ΠΎΠΌΠ°ΠΌΠΈ ΠΏΡ€ΠΈΠ²ΠΎΠ΄ΠΈΡ‚ ΠΊ Π²Ρ‹Ρ€Π°Π²Π½ΠΈΠ²Π°Π½ΠΈΡŽ ΠΈΡ… ΠΌΠ°Π³Π½ΠΈΡ‚Π½Ρ‹Ρ… Π΄ΠΈΠΏΠΎΠ»ΡŒΠ½Ρ‹Ρ… ΠΌΠΎΠΌΠ΅Π½Ρ‚ΠΎΠ² Π²Π½ΡƒΡ‚Ρ€ΠΈ областСй, Π½Π°Π·Ρ‹Π²Π°Π΅ΠΌΡ‹Ρ… Π΄ΠΎΠΌΠ΅Π½Π°ΠΌΠΈ. ΠšΠ°ΠΆΠ΄Ρ‹ΠΉ Π΄ΠΎΠΌΠ΅Π½ ΠΈΠΌΠ΅Π΅Ρ‚ ΡΠΈΠ»ΡŒΠ½Ρ‹ΠΉ ΠΌΠ°Π³Π½ΠΈΡ‚Π½Ρ‹ΠΉ Π΄ΠΈΠΏΠΎΠ»ΡŒΠ½Ρ‹ΠΉ ΠΌΠΎΠΌΠ΅Π½Ρ‚. Π€Π΅Ρ€Ρ€ΠΎΠΌΠ°Π³Π½ΠΈΡ‚Π½Ρ‹ΠΉ ΠΌΠ°Ρ‚Π΅Ρ€ΠΈΠ°Π», ΠΊΠΎΡ‚ΠΎΡ€Ρ‹ΠΉ Π½ΠΈΠΊΠΎΠ³Π΄Π° Ρ€Π°Π½ΡŒΡˆΠ΅ Π½Π΅ намагничивался, Π±ΡƒΠ΄Π΅Ρ‚ ΠΈΠΌΠ΅Ρ‚ΡŒ ΠΌΠ°Π³Π½ΠΈΡ‚Π½Ρ‹ΠΉ Π΄ΠΈΠΏΠΎΠ»ΡŒΠ½Ρ‹Π΅ ΠΌΠΎΠΌΠ΅Π½Ρ‚Ρ‹ Π²ΠΎ ΠΌΠ½ΠΎΠ³ΠΈΡ… направлСниях. Π‘Ρ€Π΅Π΄Π½ΠΈΠΉ эффСкт — это Π°Π½Π½ΡƒΠ»ΠΈΡ€ΠΎΠ²Π°Π½ΠΈΠ΅.Чистый эффСкт Ρ€Π°Π²Π΅Π½ Π½ΡƒΠ»ΡŽ

92 Π€Π΅Ρ€Ρ€ΠΎΠΌΠ°Π³Π½ΠΈΡ‚Π½Ρ‹Π΅ ΠΌΠ°Ρ‚Π΅Ρ€ΠΈΠ°Π»Ρ‹
ΠŸΡ€ΠΈ ΠΏΡ€ΠΈΠ»ΠΎΠΆΠ΅Π½ΠΈΠΈ внСшнСго ΠΌΠ°Π³Π½ΠΈΡ‚Π½ΠΎΠ³ΠΎ поля B Π΄ΠΎΠΌΠ΅Π½Ρ‹ с ΠΌΠ°Π³Π½ΠΈΡ‚Π½Ρ‹ΠΌ Π΄ΠΈΠΏΠΎΠ»ΡŒΠ½Ρ‹ΠΌ ΠΌΠΎΠΌΠ΅Π½Ρ‚ΠΎΠΌ Π² Ρ‚ΠΎΠΌ ΠΆΠ΅ Π½Π°ΠΏΡ€Π°Π²Π»Π΅Π½ΠΈΠΈ B ΡƒΠ²Π΅Π»ΠΈΡ‡ΠΈΠ²Π°ΡŽΡ‚ свой Ρ€Π°Π·ΠΌΠ΅Ρ€ Π·Π° счСт Π΄Ρ€ΡƒΠ³ΠΈΡ… Π΄ΠΎΠΌΠ΅Π½ΠΎΠ². Π’Π½ΡƒΡ‚Ρ€Π΅Π½Π½Π΅Π΅ ΠΌΠ°Π³Π½ΠΈΡ‚Π½ΠΎΠ΅ ΠΏΠΎΠ»Π΅ Π·Π½Π°Ρ‡ΠΈΡ‚Π΅Π»ΡŒΠ½ΠΎ увСличиваСтся Когда внСшнСС ΠΌΠ°Π³Π½ΠΈΡ‚Π½ΠΎΠ΅ ΠΏΠΎΠ»Π΅ удаляСтся, остаточноС ΠΌΠ°Π³Π½ΠΈΡ‚Π½ΠΎΠ΅ ΠΏΠΎΠ»Π΅ Π΄ΠΈΠΏΠΎΠ»ΡŒΠ½Ρ‹ΠΉ ΠΌΠΎΠΌΠ΅Π½Ρ‚ остаСтся, вызывая постоянный ΠΌΠ°Π³Π½ΠΈΡ‚. ЕдинствСнными Ρ„Π΅Ρ€Ρ€ΠΎΠΌΠ°Π³Π½ΠΈΡ‚Π½Ρ‹ΠΌΠΈ ΠΌΠ°Ρ‚Π΅Ρ€ΠΈΠ°Π»Π°ΠΌΠΈ ΠΏΡ€ΠΈ ΠΊΠΎΠΌΠ½Π°Ρ‚Π½ΠΎΠΉ Ρ‚Π΅ΠΌΠΏΠ΅Ρ€Π°Ρ‚ΡƒΡ€Π΅ ΡΠ²Π»ΡΡŽΡ‚ΡΡ ΠΆΠ΅Π»Π΅Π·ΠΎ, никСль ΠΈ ΠΊΠΎΠ±Π°Π»ΡŒΡ‚. Они Ρ‚Π΅Ρ€ΡΡŽΡ‚ Ρ„Π΅Ρ€Ρ€ΠΎΠΌΠ°Π³Π½Π΅Ρ‚ΠΈΠ·ΠΌ ΠΏΡ€ΠΈ Ρ‚Π΅ΠΌΠΏΠ΅Ρ€Π°Ρ‚ΡƒΡ€Π΅> Ρ‚Π΅ΠΌΠΏΠ΅Ρ€Π°Ρ‚ΡƒΡ€Ρ‹ ΠšΡŽΡ€ΠΈ (Ρ‡Ρ‚ΠΎ составляСт 770 Β° C для ΠΆΠ΅Π»Π΅Π·Π°).

93 Π€Π΅Ρ€Ρ€ΠΎΠΌΠ°Π³Π½ΠΈΡ‚Π½Ρ‹Π΅ ΠΌΠ°Ρ‚Π΅Ρ€ΠΈΠ°Π»Ρ‹
Π’Π΅ΠΌΠΏΠ΅Ρ€Π°Ρ‚ΡƒΡ€Π° ΠšΡŽΡ€ΠΈ: Π–Π΅Π»Π΅Π·ΠΎ: 770 Β° C НикСль: 354 Β° C ΠšΠΎΠ±Π°Π»ΡŒΡ‚: 1115 Β° C ΠžΡ‚Π½ΠΎΡΠΈΡ‚Π΅Π»ΡŒΠ½Π°Ρ ΠΏΡ€ΠΎΠ½ΠΈΡ†Π°Π΅ΠΌΠΎΡΡ‚ΡŒ срСды ΞΌr Mu ΠœΠ΅Ρ‚Π°Π»Π» 20000 ΠŸΠ΅Ρ€ΠΌΠ°Π»Π»ΠΎΠΉ 8000 Π­Π»Π΅ΠΊΡ‚Ρ€ΠΎΡΡ‚Π°Π»ΡŒ 4000 Π€Π΅Ρ€Ρ€ΠΈΡ‚ (никСль-Ρ†ΠΈΠ½ΠΊ) 16-640 Π€Π΅Ρ€Ρ€ΠΈΡ‚ (ΠΌΠ°Ρ€Π³Π°Π½Ρ†Π΅Π²Ρ‹ΠΉ Ρ†ΠΈΠ½ΠΊ)> 640 Π‘Ρ‚Π°Π»ΡŒ 100 НикСль

94 АнтифСрромагнитныС ΠΌΠ°Ρ‚Π΅Ρ€ΠΈΠ°Π»Ρ‹
Атомы ΠΎΠ±Π»Π°Π΄Π°ΡŽΡ‚ суммарным Π΄ΠΈΠΏΠΎΠ»ΡŒΠ½Ρ‹ΠΌ ΠΌΠΎΠΌΠ΅Π½Ρ‚ΠΎΠΌ Однако ΠΌΠ°Ρ‚Π΅Ρ€ΠΈΠ°Π» Ρ‚Π°ΠΊΠΎΠ², Ρ‡Ρ‚ΠΎ Π΄ΠΈΠΏΠΎΠ»ΡŒΠ½Ρ‹Π΅ ΠΌΠΎΠΌΠ΅Π½Ρ‚Ρ‹ Π°Ρ‚ΠΎΠΌΠΎΠ² Π²Ρ‹ΡΡ‚Ρ€Π°ΠΈΠ²Π°ΡŽΡ‚ΡΡ Π² ΠΏΡ€ΠΎΡ‚ΠΈΠ²ΠΎΠΏΠΎΠ»ΠΎΠΆΠ½ΠΎΠΌ Π½Π°ΠΏΡ€Π°Π²Π»Π΅Π½ΠΈΠΈ. ΠžΠ±Ρ‰ΠΈΠΉ Π΄ΠΈΠΏΠΎΠ»ΡŒΠ½Ρ‹ΠΉ ΠΌΠΎΠΌΠ΅Π½Ρ‚ Ρ€Π°Π²Π΅Π½ Π½ΡƒΠ»ΡŽ. НСт большой Ρ€Π°Π·Π½ΠΈΡ†Ρ‹ ΠΏΡ€ΠΈ Π½Π°Π»ΠΈΡ‡ΠΈΠΈ внСшнСго ΠΌΠ°Π³Π½ΠΈΡ‚Π½ΠΎΠ³ΠΎ поля. Π―Π²Π»Π΅Π½ΠΈΠ΅ происходит ΠΏΡ€ΠΈ Ρ‚Π΅ΠΌΠΏΠ΅Ρ€Π°Ρ‚ΡƒΡ€Π΅ Π·Π½Π°Ρ‡ΠΈΡ‚Π΅Π»ΡŒΠ½ΠΎ Π½ΠΈΠΆΠ΅ ΠΊΠΎΠΌΠ½Π°Ρ‚Π½ΠΎΠΉ. Π’ настоящСС врСмя Π½Π΅ ΠΈΠΌΠ΅Π΅Ρ‚ ΠΈΠ½ΠΆΠ΅Π½Π΅Ρ€Π½ΠΎΠ³ΠΎ значСния

95 Π€Π΅Ρ€Ρ€ΠΈΠΌΠ°Π³Π½Π΅Ρ‚ΠΈΠΊΠΈ
Подобно Π°Π½Ρ‚ΠΈΡ„Π΅Ρ€Ρ€ΠΎΠΌΠ°Π³Π½Π΅Ρ‚ΠΈΠΊΠ°ΠΌ, Π΄ΠΈΠΏΠΎΠ»ΡŒΠ½Ρ‹Π΅ ΠΌΠΎΠΌΠ΅Π½Ρ‚Ρ‹ Π°Ρ‚ΠΎΠΌΠΎΠ² Π²Ρ‹ΡΡ‚Ρ€Π°ΠΈΠ²Π°ΡŽΡ‚ΡΡ Π² ΠΏΡ€ΠΎΡ‚ΠΈΠ²ΠΎΠΏΠΎΠ»ΠΎΠΆΠ½ΠΎΠΌ Π½Π°ΠΏΡ€Π°Π²Π»Π΅Π½ΠΈΠΈ. Однако Π΄ΠΈΠΏΠΎΠ»ΡŒΠ½Ρ‹Π΅ ΠΌΠΎΠΌΠ΅Π½Ρ‚Ρ‹ Π½Π΅ Ρ€Π°Π²Π½Ρ‹.Π‘Π»Π΅Π΄ΠΎΠ²Π°Ρ‚Π΅Π»ΡŒΠ½ΠΎ, сущСствуСт суммарный Π΄ΠΈΠΏΠΎΠ»ΡŒΠ½Ρ‹ΠΉ ΠΌΠΎΠΌΠ΅Π½Ρ‚. Π€Π΅Ρ€Ρ€ΠΈΠΌΠ°Π³Π½Π΅Ρ‚ΠΈΠΊΠΈ Π²Π΅Π΄ΡƒΡ‚ сСбя ΠΊΠ°ΠΊ Ρ„Π΅Ρ€Ρ€ΠΎΠΌΠ°Π³Π½ΠΈΡ‚Π½Ρ‹Π΅ ΠΌΠ°Ρ‚Π΅Ρ€ΠΈΠ°Π»Ρ‹, Π½ΠΎ ΡƒΠ²Π΅Π»ΠΈΡ‡Π΅Π½ΠΈΠ΅ ΠΌΠ°Π³Π½ΠΈΡ‚Π½ΠΎΠ³ΠΎ поля Π½Π΅ Ρ‚Π°ΠΊ Π²Π΅Π»ΠΈΠΊΠΎ. Π­Ρ„Ρ„Π΅ΠΊΡ‚ исчСзаСт Π²Ρ‹ΡˆΠ΅ Ρ‚Π΅ΠΌΠΏΠ΅Ρ€Π°Ρ‚ΡƒΡ€Ρ‹ ΠšΡŽΡ€ΠΈ.

96 Π€Π΅Ρ€Ρ€ΠΈΠΌΠ°Π³Π½ΠΈΡ‚Π½Ρ‹Π΅ ΠΌΠ°Ρ‚Π΅Ρ€ΠΈΠ°Π»Ρ‹
Π“Π»Π°Π²Π½ΠΎΠ΅ прСимущСство Π² Ρ‚ΠΎΠΌ, Ρ‡Ρ‚ΠΎ ΠΎΠ½ΠΈ ΠΎΠ±Π»Π°Π΄Π°ΡŽΡ‚ высоким сопротивлСниСм. Π‘Π»Π΅Π΄ΠΎΠ²Π°Ρ‚Π΅Π»ΡŒΠ½ΠΎ, ΠΌΠΎΠΆΠ΅Ρ‚ ΠΈΡΠΏΠΎΠ»ΡŒΠ·ΠΎΠ²Π°Ρ‚ΡŒΡΡ Π² качСствС сСрдСчника трансформаторов, особСнно Π½Π° высоких частотах. Π’Π°ΠΊΠΆΠ΅ ΠΈΡΠΏΠΎΠ»ΡŒΠ·ΡƒΠ΅Ρ‚ΡΡ Π² Ρ€Π°ΠΌΠΎΡ‡Π½Ρ‹Ρ… Π°Π½Ρ‚Π΅Π½Π½Π°Ρ… Π² Ρ€Π°Π΄ΠΈΠΎΠΏΡ€ΠΈΠ΅ΠΌΠ½ΠΈΠΊΠ°Ρ… AM. Π’ этом случаС ΠΏΠΎΡ‚Π΅Ρ€ΠΈ Π½Π° Π²ΠΈΡ…Ρ€Π΅Π²Ρ‹Π΅ Ρ‚ΠΎΠΊΠΈ Π½Π°ΠΌΠ½ΠΎΠ³ΠΎ мСньшС, Ρ‡Π΅ΠΌ Ρƒ ΠΆΠ΅Π»Π΅Π·Π½ΠΎΠ³ΠΎ сСрдСчника. ΠŸΡ€ΠΈΠΌΠ΅Ρ€ ΠΌΠ°Ρ‚Π΅Ρ€ΠΈΠ°Π»Π°: оксид ΠΆΠ΅Π»Π΅Π·Π° (Fe3O4) ΠΈ Ρ„Π΅Ρ€Ρ€ΠΈΡ‚ никСля (NiFe2O4)

97 Π‘ΡƒΠΏΠ΅Ρ€ΠΏΠ°Ρ€Π°ΠΌΠ°Π³Π½ΠΈΡ‚Π½Ρ‹Π΅ ΠΌΠ°Ρ‚Π΅Ρ€ΠΈΠ°Π»Ρ‹
Бостоят ΠΈΠ· Ρ„Π΅Ρ€Ρ€ΠΎΠΌΠ°Π³Π½ΠΈΡ‚Π½Ρ‹Ρ… частиц Π²Π½ΡƒΡ‚Ρ€ΠΈ Π½Π΅Ρ„Π΅Ρ€Ρ€ΠΎΠΌΠ°Π³Π½ΠΈΡ‚Π½Ρ‹Ρ… ΠΌΠ°Ρ‚Π΅Ρ€ΠΈΠ°Π»ΠΎΠ². Π”ΠΎΠΌΠ΅Π½Ρ‹ Π²ΠΎΠ·Π½ΠΈΠΊΠ°ΡŽΡ‚, Π½ΠΎ Π½Π΅ ΠΌΠΎΠ³ΡƒΡ‚ Ρ€Π°ΡΡˆΠΈΡ€ΡΡ‚ΡŒΡΡ ΠΏΠΎΠ΄ воздСйствиСм внСшнСго поля. Π˜ΡΠΏΠΎΠ»ΡŒΠ·ΡƒΠ΅Ρ‚ΡΡ Π² ΠΌΠ°Π³Π½ΠΈΡ‚Π½Ρ‹Ρ… Π»Π΅Π½Ρ‚Π°Ρ… для Π°ΡƒΠ΄ΠΈΠΎ- ΠΈ видСозаписи.

98 ΠΠ°ΠΌΠ°Π³Π½ΠΈΡ‡Π΅Π½Π½ΠΎΡΡ‚ΡŒ ΠΈ ΠΏΡ€ΠΎΠ½ΠΈΡ†Π°Π΅ΠΌΠΎΡΡ‚ΡŒ
Π’Π΅ΠΏΠ΅Ρ€ΡŒ Π΄Π°Π²Π°ΠΉΡ‚Π΅ обсудим ΠΌΠ°Π³Π½ΠΈΡ‚Π½Ρ‹ΠΉ эффСкт ΠΌΠ°Π³Π½ΠΈΡ‚Π½ΠΎΠ³ΠΎ ΠΌΠ°Ρ‚Π΅Ρ€ΠΈΠ°Π»Π° количСствСнным ΠΎΠ±Ρ€Π°Π·ΠΎΠΌ. ΠŸΠΎΠ·Π²ΠΎΠ»ΡŒΡ‚Π΅ Π½Π°ΠΌ Π½Π°Π·Ρ‹Π²Π°Ρ‚ΡŒ Ρ‚ΠΎΠΊ Π²Π½ΡƒΡ‚Ρ€ΠΈ ΠΌΠ°Ρ‚Π΅Ρ€ΠΈΠ°Π»Π°, обусловлСнный элСктронной ΠΎΡ€Π±ΠΈΡ‚ΠΎΠΉ, элСктронным спином ΠΈ спином Π°Ρ‚ΠΎΠΌΠ° связанным Ρ‚ΠΎΠΊΠΎΠΌ Ib. ΠœΠ°Ρ‚Π΅Ρ€ΠΈΠ°Π» Π²ΠΊΠ»ΡŽΡ‡Π°Π΅Ρ‚ Π² сСбя мноТСство Π΄ΠΈΠΏΠΎΠ»ΡŒΠ½Ρ‹Ρ… ΠΌΠΎΠΌΠ΅Π½Ρ‚ΠΎΠ² m (Π΅Π΄ΠΈΠ½ΠΈΡ†Ρ‹ A Β· ΠΌ2), ΠΊΠΎΡ‚ΠΎΡ€Ρ‹Π΅ ΡΠΊΠ»Π°Π΄Ρ‹Π²Π°ΡŽΡ‚ΡΡ ΠžΠΏΡ€Π΅Π΄Π΅Π»ΠΈΡ‚Π΅ Π½Π°ΠΌΠ°Π³Π½ΠΈΡ‡Π΅Π½Π½ΠΎΡΡ‚ΡŒ M ΠΊΠ°ΠΊ ΠΌΠ°Π³Π½ΠΈΡ‚Π½Ρ‹ΠΉ Π΄ΠΈΠΏΠΎΠ»ΡŒΠ½Ρ‹ΠΉ ΠΌΠΎΠΌΠ΅Π½Ρ‚ Π½Π° Π΅Π΄ΠΈΠ½ΠΈΡ†Ρƒ объСма M, ΠΈΠΌΠ΅ΡŽΡ‰ΠΈΠΉ Π΅Π΄ΠΈΠ½ΠΈΡ†Ρƒ A / ΠΌ (которая Π°Π½Π°Π»ΠΎΠ³ΠΈΡ‡Π½Π° Π΅Π΄ΠΈΠ½ΠΈΡ†Π°ΠΌ H)


201

— Π—Π°ΠΊΠΎΠ½ Π›Π΅Π½Ρ†Π°, сила Π›ΠΎΡ€Π΅Π½Ρ†Π° ΠΈ сила Ρ‚ΠΎΠΊΠ°, ΡΠΎΠΎΡ‚Π½ΠΎΡˆΠ΅Π½ΠΈΠ΅ энСргии Π² процСссС Ρ€Π΅Π·ΠΊΠΈ ΠΌΠ°Π³Π½ΠΈΡ‚Π½ΠΎΠΉ ΠΈΠ½Π΄ΡƒΠΊΡ†ΠΈΠΎΠ½Π½ΠΎΠΉ ΠΏΡ€ΠΎΠ²ΠΎΠ»ΠΎΠΊΠΈ

Π­Ρ‚Π° ΡΡ‚Π°Ρ‚ΡŒΡ ΠΏΡ€Π΅Π΄Π½Π°Π·Π½Π°Ρ‡Π΅Π½Π° для ΡΡ‚Π°Ρ€ΡˆΠ΅ΠΊΠ»Π°ΡΡΠ½ΠΈΠΊΠΎΠ², Ρ‡Ρ‚ΠΎΠ±Ρ‹ ΠΎΠ±ΡŠΡΡΠ½ΠΈΡ‚ΡŒ ΡΠ»Π΅ΠΊΡ‚Ρ€ΠΎΠΌΠ°Π³Π½ΠΈΡ‚Π½ΡƒΡŽ ΠΈΠ½Π΄ΡƒΠΊΡ†ΠΈΡŽ, ΠΏΡ‹Ρ‚Π°ΡΡΡŒ ΠΎΠ±ΡŠΡΡΠ½ΠΈΡ‚ΡŒ Π·Π°ΠΊΠΎΠ½Ρ‹ элСктромагнитной ΠΈΠ½Π΄ΡƒΠΊΡ†ΠΈΠΈ ΠΏΠΎΠ΄ понятным для ΡΡ‚Π°Ρ€ΡˆΠ΅ΠΊΠ»Π°ΡΡΠ½ΠΈΠΊΠ° ΡƒΠ³Π»ΠΎΠΌ. 2} {R} RE2 οΌ›

Ξ¦ B \ Phi_B Ξ¦B прСдставляСт собой ΠΌΠ°Π³Π½ΠΈΡ‚Π½Ρ‹ΠΉ ΠΏΠΎΡ‚ΠΎΠΊ ΠΌΠ°Π³Π½ΠΈΡ‚Π° Π½Π° ΠΊΠ°Ρ‚ΡƒΡˆΠΊΠ΅. Ξ¦ B Π·Π½Π°ΠΊ Ρ€Π°Π²Π½ΠΎ B β‹… S Π·Π½Π°ΠΊ Ρ€Π°Π²Π½ΠΎ B S ΠΏΠΎΡ‚ΠΎΠΌΡƒ Ρ‡Ρ‚ΠΎ ⁑ ΞΈ \ Phi_ {B} = \ mathbf {B} \ cdot \ mathbf {S} = B S \ cos \ theta Ξ¦B = Bβ‹…S = BScosΞΈ , βˆ‚ Ξ¦ B βˆ‚ Ρ‚ \ frac {\ partial \ Phi _ {\ mathrm {B}}} {\ partial t} βˆ‚tβˆ‚Ξ¦B ΠŸΡ€Π΅Π΄ΡΡ‚Π°Π²Π»ΡΠ΅Ρ‚ ΠΈΠ·ΠΌΠ΅Π½Π΅Π½ΠΈΠ΅ ΠΌΠ°Π³Π½ΠΈΡ‚Π½ΠΎΠ³ΠΎ ΠΏΠΎΡ‚ΠΎΠΊΠ° Π²ΠΎ Π²Ρ€Π΅ΠΌΠ΅Π½ΠΈ;

ΠžΡ‚Ρ€ΠΈΡ†Π°Ρ‚Π΅Π»ΡŒΠ½Ρ‹ΠΉ Π·Π½Π°ΠΊ Напротив, Π½Π°ΠΏΡ€Π°Π²Π»Π΅Π½ΠΈΠ΅ ΠΌΠ°Π³Π½ΠΈΡ‚Π½ΠΎΠ³ΠΎ поля, создаваСмого ΠΈΠ½Π΄ΡƒΡ†ΠΈΡ€ΠΎΠ²Π°Π½Π½Ρ‹ΠΌ Ρ‚ΠΎΠΊΠΎΠΌ , ΠΏΠΎ ΡΡ€Π°Π²Π½Π΅Π½ΠΈΡŽ с Π½Π°ΠΏΡ€Π°Π²Π»Π΅Π½ΠΈΠ΅ΠΌ ΠΌΠ°Π³Π½ΠΈΡ‚Π½ΠΎΠ³ΠΎ поля .


Π£Π³ΠΎΠ» сохранСния энСргии β€”β€” ПослС Ρ‚ΠΎΠ³ΠΎ, ΠΊΠ°ΠΊ ΠΌΠ°Π³Π½ΠΈΡ‚ приблизится ΠΊ ΠΊΠ°Ρ‚ΡƒΡˆΠΊΠ΅, ΠΊΠ°Ρ‚ΡƒΡˆΠΊΠ° вырабатываСтся ЭлСктричСская энСргия , ЭлСктричСская энСргия, гСнСрируСмая ΠΊΠ°Ρ‚ΡƒΡˆΠΊΠΎΠΉ, Ρ€Π°Π²Π½Π° ΡƒΠΌΠ΅Π½ΡŒΡˆΠ΅Π½Π½ΠΎΠΉ кинСтичСской энСргии ΠΌΠ°Π³Π½ΠΈΡ‚Π° Π‘Π»Π΅Π΄ΠΎΠ²Π°Ρ‚Π΅Π»ΡŒΠ½ΠΎ, Π·Π°ΠΊΠΎΠ½ Π›Π΅Π½Ρ†Π° описываСт процСсс Π²Π·Π°ΠΈΠΌΠ½ΠΎΠ³ΠΎ прСобразования кинСтичСской энСргии ΠΌΠ°Π³Π½ΠΈΡ‚Π° ΠΈ элСктричСской энСргии Π² ΠΊΠ°Ρ‚ΡƒΡˆΠΊΠ΅.

ЀизичСский процСсс β€”β€” ΠœΠ°Π³Π½ΠΈΡ‚ находится Π±Π»ΠΈΠ·ΠΊΠΎ ΠΊ ΠΊΠ°Ρ‚ΡƒΡˆΠΊΠ΅ ΠΈΠ»ΠΈ ΠΎΡ‚ Π½Π΅Π΅, ΠΌΠ°Π³Π½ΠΈΡ‚Π½Ρ‹ΠΉ ΠΏΠΎΡ‚ΠΎΠΊ Π² ΠΊΠ°Ρ‚ΡƒΡˆΠΊΠ΅ увСличиваСтся, ΠΈ создаСтся ΠΈΠ½Π΄ΡƒΡ†ΠΈΡ€ΠΎΠ²Π°Π½Π½ΠΎΠ΅ Π²ΠΈΡ…Ρ€Π΅Π²ΠΎΠ΅ элСктричСскоС ΠΏΠΎΠ»Π΅ (Ρ‚. Π•. Π˜Π½Π΄ΡƒΡ†ΠΈΡ€ΠΎΠ²Π°Π½Π½Π°Ρ элСктродвиТущая сила).Π˜Π½Π΄ΡƒΡ†ΠΈΡ€ΠΎΠ²Π°Π½Π½ΠΎΠ΅ элСктричСскоС ΠΏΠΎΠ»Π΅ Π³Π΅Π½Π΅Ρ€ΠΈΡ€ΡƒΠ΅Ρ‚ ΠΈΠ½Π΄ΡƒΡ†ΠΈΡ€ΠΎΠ²Π°Π½Π½Ρ‹ΠΉ Ρ‚ΠΎΠΊ, Π° ΠΈΠ½Π΄ΡƒΡ†ΠΈΡ€ΠΎΠ²Π°Π½Π½Ρ‹ΠΉ Ρ‚ΠΎΠΊ создаСт ΠΈΠ½Π΄ΡƒΡ†ΠΈΡ€ΠΎΠ²Π°Π½Π½ΠΎΠ΅ ΠΌΠ°Π³Π½ΠΈΡ‚Π½ΠΎΠ΅ ΠΏΠΎΠ»Π΅. Π­Ρ‚ΠΎ ΠΌΠ°Π³Π½ΠΈΡ‚Π½ΠΎΠ΅ ΠΏΠΎΠ»Π΅ дСйствуСт Π½Π° ΠΌΠ°Π³Π½ΠΈΡ‚ ΠΈ Π²Ρ‹Π·Ρ‹Π²Π°Π΅Ρ‚ ΡƒΠΌΠ΅Π½ΡŒΡˆΠ΅Π½ΠΈΠ΅ кинСтичСской энСргии ΠΌΠ°Π³Π½ΠΈΡ‚Π°.


Π‘ΠΈΠ»Π° Π›ΠΎΡ€Π΅Π½Ρ†Π° ΠžΠΏΠΈΡΡ‹Π²Π°Π΅Ρ‚ силу заряТСнных частиц Π² ΠΌΠ°Π³Π½ΠΈΡ‚Π½ΠΎΠΌ ΠΏΠΎΠ»Π΅,
F Π·Π½Π°ΠΊ Ρ€Π°Π²Π½ΠΎ q v Γ— B Π·Π½Π°ΠΊ Ρ€Π°Π²Π½ΠΎ q v B s я ΠΏ ΞΈ F = q \ mathbf {v} \ times \ mathbf {B} = qvBsin \ theta F = qv Γ— B = qvBsinΞΈq — заряд частицы, v \ mathbf {v} vΠ‘ΠΊΠΎΡ€ΠΎΡΡ‚ΡŒ двиТСния частицы, B \ mathbf {B} BIs магнитная индукция, ΞΈ \ theta ΞΈΠ­Ρ‚ΠΎ ΡƒΠ³ΠΎΠ» ΠΌΠ΅ΠΆΠ΄Ρƒ ΡΠΊΠΎΡ€ΠΎΡΡ‚ΡŒΡŽ двиТСния частицы ΠΈ ΠΈΠ½Ρ‚Π΅Π½ΡΠΈΠ²Π½ΠΎΡΡ‚ΡŒΡŽ ΠΌΠ°Π³Π½ΠΈΡ‚Π½ΠΎΠΉ ΠΈΠ½Π΄ΡƒΠΊΡ†ΠΈΠΈ.

Π‘ΠΈΠ»Π° АмпСра описываСт силу элСктричСского Ρ‚ΠΎΠΊΠ° Π² ΠΌΠ°Π³Π½ΠΈΡ‚Π½ΠΎΠΌ ΠΏΠΎΠ»Π΅,
F Π·Π½Π°ΠΊ Ρ€Π°Π²Π½ΠΎ B я L s я ΠΏ ΞΈ F = BILsin \ theta F = BILsinΞΈ я я I прСдставляСт Ρ‚Π΅ΠΊΡƒΡ‰ΠΈΠΉ Ρ€Π°Π·ΠΌΠ΅Ρ€, L L L прСдставляСт Π΄Π»ΠΈΠ½Ρƒ Ρ‚ΠΎΠΊΠ° (ΠΏΡ€ΠΎΠ²ΠΎΠ΄Π°), ΞΈ \ theta ΞΈ Π£Π³ΠΎΠ» ΠΌΠ΅ΠΆΠ΄Ρƒ ΠΌΠ°Π³Π½ΠΈΡ‚Π½Ρ‹ΠΌ ΠΏΠΎΠ»Π΅ΠΌ ΠΈ Ρ‚ΠΎΠΊΠΎΠΌ.

ЭлСктричСский Ρ‚ΠΎΠΊ прСдставляСт собой Π½Π°ΠΏΡ€Π°Π²Π»Π΅Π½Π½ΠΎΠ΅ Π΄Π²ΠΈΠΆΠ΅Π½ΠΈΠ΅ заряТСнных частиц, поэтому сила Π² Π°ΠΌΠΏΠ΅Ρ€Π°Ρ…, ΠΊΠΎΠ³Π΄Π° ΠΏΡ€ΠΎΠ²ΠΎΠ΄ находится Π² состоянии покоя, Π½Π° самом Π΄Π΅Π»Π΅ являСтся макроскопичСским Π²Ρ‹Ρ€Π°ΠΆΠ΅Π½ΠΈΠ΅ΠΌ силы Π›ΠΎΡ€Π΅Π½Ρ†Π°. Когда ΠΏΡ€ΠΎΠ²ΠΎΠ»ΠΎΠΊΠ° двиТСтся, ампСрная сила являСтся ΡΠΎΡΡ‚Π°Π²Π»ΡΡŽΡ‰Π΅ΠΉ силы Π›ΠΎΡ€Π΅Π½Ρ†Π°.


ΠŸΠΎΡΠΊΠΎΠ»ΡŒΠΊΡƒ сила Π›ΠΎΡ€Π΅Π½Ρ†Π° всСгда пСрпСндикулярна Π½Π°ΠΏΡ€Π°Π²Π»Π΅Π½ΠΈΡŽ двиТСния частицы, ΠΈΡΠΏΠΎΠ»ΡŒΠ·ΡƒΠΉΡ‚Π΅ W Π·Π½Π°ΠΊ Ρ€Π°Π²Π½ΠΎ F L c ΠΎ s ΞΈ W = FLcos \ theta W = FLcosΞΈ МоТно ΡΡƒΠ΄ΠΈΡ‚ΡŒ, Ρ‡Ρ‚ΠΎ Π›ΠΎΡ€Π΅Π½Ρ† Π½Π΅ Ρ€Π°Π±ΠΎΡ‚Π°Π΅Ρ‚.

Когда ΠΏΡ€ΠΎΠ²ΠΎΠ΄Π½ΠΈΠΊ Π½Π°Ρ‡ΠΈΠ½Π°Π΅Ρ‚ Π΄Π²ΠΈΠ³Π°Ρ‚ΡŒΡΡ, ΡΠΊΠΎΡ€ΠΎΡΡ‚ΡŒ заряда ΠΈΠΌΠ΅Π΅Ρ‚ Π΄Π²Π΅ ΡΠΎΡΡ‚Π°Π²Π»ΡΡŽΡ‰ΠΈΠ΅: ΠΎΠ΄Π½Π° — это ΡΠΊΠΎΡ€ΠΎΡΡ‚ΡŒ двиТСния с ΠΏΡ€ΠΎΠ²ΠΎΠ΄Π½ΠΈΠΊΠΎΠΌ, Π° другая — Π½Π°ΠΏΡ€Π°Π²Π»Π΅Π½Π½ΠΎΠ΅ Π΄Π²ΠΈΠΆΠ΅Π½ΠΈΠ΅ вдоль ΠΏΡ€ΠΎΠ²ΠΎΠ΄Π½ΠΈΠΊΠ°, ΠΊΠΎΡ‚ΠΎΡ€ΠΎΠ΅ проявляСтся Π² Π²ΠΈΠ΄Π΅ Ρ‚ΠΎΠΊΠ°.

ΠŸΠΎΠ»ΡƒΡ‡Π΅Π½Π½Π°Ρ сила Π›ΠΎΡ€Π΅Π½Ρ†Π° состоит ΠΈΠ· Π΄Π²ΡƒΡ… ΠΊΠΎΠΌΠΏΠΎΠ½Π΅Π½Ρ‚ΠΎΠ².Π‘ΠΈΠ»Π°, пСрпСндикулярная ΠΏΡ€ΠΎΠ²ΠΎΠ΄Π½ΠΈΠΊΡƒ, макроскопичСски выраТаСтся ΠΊΠ°ΠΊ сила Π² Π°ΠΌΠΏΠ΅Ρ€Π°Ρ…, ΡΠΎΠ·Π΄Π°ΡŽΡ‰Π°Ρ ΠΎΡ‚Ρ€ΠΈΡ†Π°Ρ‚Π΅Π»ΡŒΠ½ΠΎΠ΅ Ρ€Π°Π±ΠΎΡ‡Π΅Π΅ сопротивлСниС, ΠΏΡ€Π΅Π΄ΠΎΡ‚Π²Ρ€Π°Ρ‰Π°ΡŽΡ‰Π΅Π΅ ΠΏΠ΅Ρ€Π΅ΠΌΠ΅Ρ‰Π΅Π½ΠΈΠ΅ ΠΏΡ€ΠΎΠ²ΠΎΠ΄Π½ΠΈΠΊΠ°, Π° сила, Π΄Π΅ΠΉΡΡ‚Π²ΡƒΡŽΡ‰Π°Ρ вдоль ΠΏΡ€ΠΎΠ²ΠΎΠ΄Π½ΠΈΠΊΠ°, макроскопичСски выраТаСтся ΠΊΠ°ΠΊ элСктродвиТущая сила источника питания, Π²Ρ‹ΠΏΠΎΠ»Π½ΡΡŽΡ‰Π°Ρ ΠΏΠΎΠ»ΠΎΠΆΠΈΡ‚Π΅Π»ΡŒΠ½ΡƒΡŽ Ρ€Π°Π±ΠΎΡ‚Ρƒ. ΠŸΠΎΡΠΊΠΎΠ»ΡŒΠΊΡƒ сила Π›ΠΎΡ€Π΅Π½Ρ†Π° Π½Π΅ Ρ€Π°Π±ΠΎΡ‚Π°Π΅Ρ‚, Π΄Π²Π΅ части Ρ€Π°Π±ΠΎΡ‚Ρ‹ ΡΠ²Π»ΡΡŽΡ‚ΡΡ ΠΏΠΎΠ»ΠΎΠΆΠΈΡ‚Π΅Π»ΡŒΠ½ΠΎΠΉ ΠΈ ΠΎΡ‚Ρ€ΠΈΡ†Π°Ρ‚Π΅Π»ΡŒΠ½ΠΎΠΉ, Ρ‡Ρ‚ΠΎ ΠΎΠ·Π½Π°Ρ‡Π°Π΅Ρ‚, Ρ‡Ρ‚ΠΎ Ρ€Π°Π±ΠΎΡ‚Π°, выполняСмая силой Ρ‚ΠΎΠΊΠ°, Ρ€Π°Π²Π½Π° ΠΏΠΎΡ‚Ρ€Π΅Π±Π»Π΅Π½ΠΈΡŽ Ρ†Π΅ΠΏΠΈ.

Π—Π½Π°ΠΉ ΠΎΡ‚Π²Π΅Ρ‚-Π‘Π°ΠΉ Π ΡƒΠ±ΠΈΠ½

PHY_10_55_V2_TG_Ampere force.Π‘ΠΈΠ»Π° Π›ΠΎΡ€Π΅Π½Ρ†Π°

ВСорСтичСский ΠΌΠ°Ρ‚Π΅Ρ€ΠΈΠ°Π» ΠΊ ΡƒΡ€ΠΎΠΊΡƒ, опрСдСлСния понятий

АмпСра Π—Π°ΠΊΠΎΠ½ силы

Π‘ΠΈΠ»Π° АмпСра Π—Π°ΠΊΠΎΠ½ гласит, Ρ‡Ρ‚ΠΎ сила притяТСния ΠΈΠ»ΠΈ отталкивания ΠΌΠ΅ΠΆΠ΄Ρƒ двумя ΠΏΡ€ΠΎΠ²ΠΎΠ΄Π°ΠΌΠΈ, нСсущими Ρ‚ΠΎΠΊΠΈ ΠΏΡ€ΠΎΠΏΠΎΡ€Ρ†ΠΈΠΎΠ½Π°Π»ΡŒΠ½Ρ‹ ΠΈΡ… Π΄Π»ΠΈΠ½Π΅ ΠΈ интСнсивности проходящСго Ρ‚ΠΎΠΊΠ° Ρ‡Π΅Ρ€Π΅Π· Π½ΠΈΡ…. Если Ρ‚ΠΎΠΊΠΈ Ρ‚Π΅ΠΊΡƒΡ‚ Π² ΠΎΠ΄Π½ΠΎΠΌ Π½Π°ΠΏΡ€Π°Π²Π»Π΅Π½ΠΈΠΈ, ΠΎΡ‚Ρ‚Π°Π»ΠΊΠΈΠ²Π°Π½ΠΈΠ΅ ΠΏΡ€ΠΈΠ½ΠΈΠΌΠ°Π΅Ρ‚ мСсто. Если Ρ‚ΠΎΠΊΠΈ Ρ‚Π΅ΠΊΡƒΡ‚ Π² ΠΏΡ€ΠΎΡ‚ΠΈΠ²ΠΎΠΏΠΎΠ»ΠΎΠΆΠ½Ρ‹Ρ… направлСниях, происходит притяТСниС. Π—Π°ΠΊΠΎΠ½ основан Π½Π° этих Π΄Π²ΡƒΡ… основных понятиях элСктростатики:

Β· Π—Π°ΠΊΠΎΠ½ Π‘ΠΈΠΎ-Π‘Π°Π²Π°Ρ€Π° гласит, Ρ‡Ρ‚ΠΎ ΠΊΠ°ΠΆΠ΄Ρ‹ΠΉ Ρ‚ΠΎΠΊ нСсущий ΠΏΡ€ΠΎΠ²ΠΎΠ΄ создаСт Π²ΠΎΠΊΡ€ΡƒΠ³ сСбя ΠΌΠ°Π³Π½ΠΈΡ‚Π½ΠΎΠ΅ ΠΏΠΎΠ»Π΅, ΠΊΠ°ΠΊ ΠΏΠΎΠΊΠ°Π·Π°Π½ΠΎ Π½Π° Рис. 1 .

Β· Π›ΠΎΡ€Π΅Π½Ρ† сила относится ΠΊ силС, ΠΊΠΎΡ‚ΠΎΡ€ΡƒΡŽ ΠΊΠ°ΠΆΠ΄ΠΎΠ΅ ΠΌΠ°Π³Π½ΠΈΡ‚Π½ΠΎΠ΅ ΠΏΠΎΠ»Π΅ ΠΎΠΊΠ°Π·Ρ‹Π²Π°Π΅Ρ‚ Π½Π° Π»ΡŽΠ±ΡƒΡŽ ΡΠ»Π΅ΠΊΡ‚Ρ€ΠΈΡ‡Π΅ΡΠΊΡƒΡŽ заряд двиТСтся Π² своСм ΠΏΠΎΠ»Π΅.

Рисунок 1: ΠŸΡ€Π°Π²ΠΈΠ»ΠΎ большого ΠΏΠ°Π»ΡŒΡ†Π° для поиска ΠΌΠ°Π³Π½ΠΈΡ‚Π½ΠΎΠ³ΠΎ поля Π²ΠΎΠΊΡ€ΡƒΠ³ Ρ‚ΠΎΠΊΠΎΠ²Π΅Π΄ΡƒΡ‰Π΅Π³ΠΎ ΠΏΡ€ΠΎΠ²ΠΎΠ΄Π°

На основании Π·Π°ΠΊΠΎΠ½Π° Π‘ΠΈΠΎ-Π‘Π°Π²Π°Ρ€Π° ΠΈ силы Π›ΠΎΡ€Π΅Π½Ρ†Π°, сущСствуСт связь ΠΌΠ΅ΠΆΠ΄Ρƒ ΠΌΠ°Π³Π½ΠΈΡ‚Π½Ρ‹ΠΌ ΠΏΠΎΠ»Π΅ΠΌ ΠΈ элСктричСским зарядом / Ρ‚ΠΎΠΊΠΎΠΌ. Π­Ρ‚ΠΎ это ΡΠΎΠΎΡ‚Π½ΠΎΡˆΠ΅Π½ΠΈΠ΅, ΠΊΠΎΡ‚ΠΎΡ€ΠΎΠ΅ АмпСр пытался ΡƒΡΡ‚Π°Π½ΠΎΠ²ΠΈΡ‚ΡŒ с ΠΏΠΎΠΌΠΎΡ‰ΡŒΡŽ экспСримСнтов. Π’ Π‘Π°ΠΌΡ‹ΠΌ основным ΠΈΠ· этих экспСримСнтов Π±Ρ‹Π»ΠΎ ΠΈΠ·ΡƒΡ‡Π΅Π½ΠΈΠ΅ силы ΠΌΠ΅ΠΆΠ΄Ρƒ двумя Ρ‚ΠΎΠΊΠΎΠ²Π΅Π΄ΡƒΡ‰ΠΈΠ΅ ΠΏΡ€ΠΎΠ²ΠΎΠ΄Π°, ΠΊΠ°ΠΊ ΠΏΠΎΠΊΠ°Π·Π°Π½ΠΎ Π½Π° Рисунок 2 .Π­Ρ‚ΠΎΡ‚ экспСримСнт ΠΈ ΠΏΠΎΡΠ»Π΅Π΄ΡƒΡŽΡ‰ΠΈΠ΅ Ρ‚Π΅ΠΎΡ€ΠΈΠΈ для объяснСния Π΅Π³ΠΎ Ρ€Π΅Π·ΡƒΠ»ΡŒΡ‚Π°Ρ‚ΠΎΠ² Π·Π°Π»ΠΎΠΆΠΈΠ» Ρ„ΡƒΠ½Π΄Π°ΠΌΠ΅Π½Ρ‚ элСктромагнСтизма ΠΊΠ°ΠΊ области Π² Ρ„ΠΈΠ·ΠΈΠΊΠ° сама ΠΏΠΎ сСбС.

Рисунок 2: ΠœΠ°Π³Π½ΠΈΡ‚Π½ΠΎΠ΅ ΠΏΠΎΠ»Π΅ ΠΌΠ΅ΠΆΠ΄Ρƒ Ρ‚ΠΎΠΊΠΎΠ²Π΅Π΄ΡƒΡ‰ΠΈΠΌΠΈ ΠΏΡ€ΠΎΠ²ΠΎΠ΄Π°ΠΌΠΈ

АмпСр, Π΅Π΄ΠΈΠ½ΠΈΡ†Π° измСрСния элСктричСского Ρ‚ΠΎΠΊΠ° Π² систСмС БИ, опрСдСляСтся ΠΊΠ°ΠΊ элСктромагнит. сила Π½Π° Π΅Π΄ΠΈΠ½ΠΈΡ†Ρƒ Π΄Π»ΠΈΠ½Ρ‹ ΠΌΠ΅ΠΆΠ΄Ρƒ двумя ΠΏΡ€ΠΎΠ²ΠΎΠ΄Π°ΠΌΠΈ бСсконСчной Π΄Π»ΠΈΠ½Ρ‹, ΠΈΠΌΠ΅ΡŽΡ‰Π°Ρ ΠΏΡ€Π΅Π½Π΅Π±Ρ€Π΅ΠΆΠΈΠΌΠΎ ΠΌΠ°Π»ΡƒΡŽ Π΄ΠΈΠ°ΠΌΠ΅Ρ‚Ρ€ΠΎΠΌ ΠΈ Ρ€Π°Π·ΠΌΠ΅Ρ‰Π΅Π½Ρ‹ Π½Π° расстоянии 1 ΠΌ Π² Π²Π°ΠΊΡƒΡƒΠΌΠ΅. ОсновноС ΠΏΡ€Π΅Π΄ΠΏΠΎΠ»ΠΎΠΆΠ΅Π½ΠΈΠ΅ здСсь состоит Π² Ρ‚ΠΎΠΌ, Ρ‡Ρ‚ΠΎ ΠΏΡ€ΠΎΠ²ΠΎΠ΄Π° находятся Π² свободном мСстС, Ρ‚.Π΅.Π΅. Π½Π΅Ρ‚ Π½ΠΈΡ‡Π΅Π³ΠΎ, Ρ‡Ρ‚ΠΎ ΠΌΠΎΠΆΠ΅Ρ‚ Π±Ρ‹Ρ‚ΡŒ Π½Π°ΠΌΠ°Π³Π½ΠΈΡ‡Π΅Π½Π½Ρ‹ΠΉ. Если ΠΊΠ°ΠΊΠΎΠ΅-Π»ΠΈΠ±ΠΎ вСщСство, ΠΏΡ€ΠΈΡΡƒΡ‚ΡΡ‚Π²ΡƒΡŽΡ‰Π΅Π΅ Π² ΠΎΠΊΡ€ΡƒΠΆΠ°ΡŽΡ‰Π΅ΠΉ срСдС, намагнитится, ΠΎΠ½ΠΎ Π±ΡƒΠ΄Π΅Ρ‚ ΠΏΡ€ΠΎΡΠ²Π»ΡΡ‚ΡŒ ΡΠΎΠ±ΡΡ‚Π²Π΅Π½Π½ΡƒΡŽ ΠΌΠ°Π³Π½ΠΈΡ‚Π½ΡƒΡŽ силу, ΠΊΠΎΡ‚ΠΎΡ€ΡƒΡŽ Π½Π΅ΠΎΠ±Ρ…ΠΎΠ΄ΠΈΠΌΠΎ ΠΏΡ€ΠΈΠ½ΠΈΠΌΠ°Ρ‚ΡŒ Π²ΠΎ Π²Π½ΠΈΠΌΠ°Π½ΠΈΠ΅, Ρ‚Π°ΠΊ Ρ‡Ρ‚ΠΎ это Π΄ΠΎΠ»ΠΆΠ½ΠΎ Π±Ρ‹Ρ‚ΡŒ сдСлано ΠΏΡ€Π΅Π΄ΠΏΠΎΠ»ΠΎΠΆΠ΅Π½ΠΈΠ΅.

Π˜ΡΠΏΠΎΠ»ΡŒΠ·ΡƒΡ Π·Π°ΠΊΠΎΠ½ силы АмпСра, ΠΌΠ°Π³Π½ΠΈΡ‚Π½ΠΎΠ΅ ΠΏΠΎΠ»Π΅ Π²ΠΎΠΊΡ€ΡƒΠ³ бСсконСчный ΠΏΡ€ΠΎΠ²ΠΎΠ΄, бСсконСчный лист, Ρ‚ΠΎΡ€ΠΎΠΈΠ΄, солСноид ΠΈΠ»ΠΈ любая другая ΠΏΡ€Π°Π²ΠΈΠ»ΡŒΠ½Π°Ρ Ρ„ΠΎΡ€ΠΌΠ° ΠΌΠΎΠΆΠ΅Ρ‚ ΠΌΠΎΠΆΠ½ΠΎ Ρ€Π°ΡΡΡ‡ΠΈΡ‚Π°Ρ‚ΡŒ, ΠΊΠ°ΠΊ ΠΏΠΎΠΊΠ°Π·Π°Π½ΠΎ Π½Π° рисунках 3 ΠΈ 4 Π½ΠΈΠΆΠ΅.

Рисунок 3: ΠœΠ°Π³Π½ΠΈΡ‚Π½Ρ‹ΠΉ ПолС Π²ΠΎΠΊΡ€ΡƒΠ³ солСноида Рис. 4: ΠœΠ°Π³Π½ΠΈΡ‚Π½ΠΎΠ΅ ΠΏΠΎΠ»Π΅ Π²ΠΎΠΊΡ€ΡƒΠ³ Ρ‚ΠΎΡ€ΠΎΠΈΠ΄Π°

Π—Π°ΠΊΠΎΠ½ силы АмпСра оказался Π½Π°ΡΡ‚ΠΎΠ»ΡŒΠΊΠΎ Ρ„ΡƒΠ½Π΄Π°ΠΌΠ΅Π½Ρ‚Π°Π»ΡŒΠ½Ρ‹ΠΌ Π·Π°ΠΊΠΎΠ½ΠΎΠΌ, Ρ‡Ρ‚ΠΎ послС Π½Π΅Π³ΠΎ ΠΌΠ½ΠΎΠ³ΠΈΠ΅ Ρ„ΠΈΠ·ΠΈΠΊΠΈ, Ρ‚Π°ΠΊΠΈΠ΅ ΠΊΠ°ΠΊ ДТСймс ΠšΠ»Π΅Ρ€ΠΊ МаксвСлл, Π’ΠΈΠ»ΡŒΠ³Π΅Π»ΡŒΠΌ Π’Π΅Π±Π΅Ρ€, Π‘Π΅Ρ€Π½Π°Ρ€ Π ΠΈΠΌΠ°Π½ ΠΈ Π΄Ρ€.Ρ€Π°ΡΡˆΠΈΡ€ΠΈΠ» Π΅Π³ΠΎ, Ρ‡Ρ‚ΠΎΠ±Ρ‹ Π½Π°ΠΉΡ‚ΠΈ Π±Π°Π·ΠΎΠ²ΠΎΠ΅ ΠΎΠΏΡ€Π΅Π΄Π΅Π»Π΅Π½ΠΈΠ΅ самой силы. Π’ΠΎΠ·Π²Ρ€Π°Ρ‰Π°ΡΡΡŒ ΠΊ Ρ€Π°Π±ΠΎΡ‚Π΅ АмпСра, Π—Π°ΠΊΠΎΠ½ силы. ΡƒΡ‚Π²Π΅Ρ€ΠΆΠ΄Π°Π΅Ρ‚, Ρ‡Ρ‚ΠΎ сила ΠΌΠ΅ΠΆΠ΄Ρƒ Ρ‚ΠΎΠΊΠΎΠ²Π΅Π΄ΡƒΡ‰ΠΈΠΌΠΈ ΠΏΡ€ΠΎΠ²ΠΎΠ΄Π°ΠΌΠΈ ΠΏΡ€ΠΎΠΏΠΎΡ€Ρ†ΠΈΠΎΠ½Π°Π»ΡŒΠ½Π° ΠΈΡ… Π΄Π»ΠΈΠ½Π° ΠΈ сила ΠΏΡ€ΠΎΡ‚Π΅ΠΊΠ°ΡŽΡ‰Π΅Π³ΠΎ Ρ‚ΠΎΠΊΠ°. Π­Ρ‚ΠΎ ΠΎΠ·Π½Π°Ρ‡Π°Π΅Ρ‚, Ρ‡Ρ‚ΠΎ Ρ‡Π΅ΠΌ Π²Ρ‹ΡˆΠ΅ Ρ‚ΠΎΠΊ, большС притяТСниС ΠΈΠ»ΠΈ ΠΎΡ‚Ρ‚Π°Π»ΠΊΠΈΠ²Π°Π½ΠΈΠ΅ ΠΌΠ΅ΠΆΠ΄Ρƒ ΠΏΡ€ΠΎΠ²ΠΎΠ΄Π°ΠΌΠΈ.

Π”ΠΎΠΏΠΎΠ»Π½ΠΈΡ‚Π΅Π»ΡŒΠ½Ρ‹Π΅ инструкции ΠΏΠΎ ΠΎΡ€Π³Π°Π½ΠΈΠ·Π°Ρ†ΠΈΠΈ ΡƒΡ€ΠΎΠΊ

Π£Ρ€ΠΎΠΊ начинаСтся с ввСдСния Π² Ρ‚Π΅ΠΌΡƒ дня ΠΈ ΠΈΠ·Π»Π°Π³Π°Π΅Ρ‚ Ρ€Π΅Π·ΡƒΠ»ΡŒΡ‚Π°Ρ‚Ρ‹ обучСния, ΠΊΠΎΡ‚ΠΎΡ€Ρ‹Π΅ ΠΎΠ½ΠΈ ΠΏΠΎΠ»ΡƒΡ‡Π°Ρ‚ послС изучСния.Знакомство студСнтов со ΡΠ»Π΅Π΄ΡƒΡŽΡ‰ΠΈΠΌΠΈ ΠΏΡ€ΠΎΠ±Π»Π΅ΠΌΠ°ΠΌΠΈ:

β€’ Π’Π΅ΠΌΠ° ΡƒΡ€ΠΎΠΊΠ°

β€’ Π—Π°Π΄Π°Ρ‡ΠΈ ΡƒΡ€ΠΎΠΊΠ°

β€’ ΠšΡ€ΠΈΡ‚Π΅Ρ€ΠΈΠΈ ΡƒΡΠΏΠ΅ΡˆΠ½ΠΎΡΡ‚ΠΈ ΡƒΡ€ΠΎΠΊΠ°

β€’ План мСроприятий ΠΊ ΡƒΡ€ΠΎΠΊΡƒ

β€’ ΠŸΡ€Π΅Π΄Π²Π°Ρ€ΠΈΡ‚Π΅Π»ΡŒΠ½ΠΎ ΠΎΠ±ΡƒΡ‡ΠΈΡ‚Π΅ ΠΏΡ€Π΅Π΄ΠΌΠ΅Ρ‚Π½ΠΎΠΌΡƒ ΡΠ»ΠΎΠ²Π°Ρ€ΡŽ.

Π—Π°Ρ‚Π΅ΠΌ студСнты ΠΌΠΎΠ³ΡƒΡ‚ вывСсти Ρ‚Π΅ΠΌΡƒ ΡƒΠΌΠ΅Π½ΡŒΡˆΠΈΡ‚ΡŒ ΠΈ Ρ†Π΅Π»ΠΈ, для пояснСния Π²Ρ‹ ΠΌΠΎΠΆΠ΅Ρ‚Π΅ ΠΏΠΎΠΊΠ°Π·Π°Ρ‚ΡŒ Ρ‚Π΅ΠΌΡƒ ΠΈ ΠΈΠ·ΡƒΡ‡Π΅Π½ΠΈΠ΅ Ρ†Π΅Π»ΠΈ Π½Π° ΠΏΡ€Π΅Π·Π΅Π½Ρ‚Π°Ρ†ΠΈΠΈ.

Π—Π°Ρ‚Π΅ΠΌ ΠŸΡ€Π΅Π΄ΠΌΠ΅Ρ‚Π½Ρ‹ΠΉ ΡΠ»ΠΎΠ²Π°Ρ€ΡŒ ΠΈ тСрминология Π±ΡƒΠ΄Π΅Ρ‚ прСдставлСна ​​студСнтам ΠΈ ΠΈΡ… Π΄Π΅ΡΡ‚Π΅Π»ΡŒΠ½ΠΎΡΡ‚ΠΈ Π²ΠΎ врСмя Π±ΡƒΠ΄Π΅Ρ‚ объяснСна ΠΈΡΡΠ»Π΅Π΄ΠΎΠ²Π°Ρ‚Π΅Π»ΡŒΡΠΊΠ°Ρ Ρ€Π°Π±ΠΎΡ‚Π°.

учащихся подСлятся своим ΠΎΠΏΡ‹Ρ‚ΠΎΠΌ с ΠΏΡ€Π°Π²ΠΈΠ»ΠΎΠΌ Π»Π΅Π²ΠΎΠΉ Ρ€ΡƒΠΊΠΈ, силой АмпСра ΠΈ силой Π›ΠΎΡ€Π΅Π½Ρ†Π°. Π‘ΠΌΠΎΡ‚Ρ€Π΅Ρ‚ΡŒ Π²ΠΈΠ΄Π΅ΠΎ. Π’Ρ‹ΠΏΠΎΠ»Π½ΡΡ‚ΡŒ дСмонстрации ΠΈ симуляции, ΠΊΠ°ΠΊ объяснил ΠΈΡ… ΡƒΡ‡ΠΈΡ‚Π΅Π»ΡŒ.

Π£Ρ‡ΠΈΡ‚Π΅Π»ΡŒ, ΠΈΡΠΏΠΎΠ»ΡŒΠ·ΡƒΡ ppt, ΠΎΠ±ΡŠΡΡΠ½ΡΠ΅Ρ‚ ΠΏΡ€Π°Π²ΠΈΠ»ΠΎ Π»Π΅Π²ΠΎΠΉ Ρ€ΡƒΠΊΠΈ, описываСт, обсуТдаСт ΠΈ ΠΎΠ±ΡŠΡΡΠ½ΡΠ΅Ρ‚ ΠΏΡ€Π°Π²ΠΈΠ»ΠΎ Π»Π΅Π²ΠΎΠΉ Ρ€ΡƒΠΊΠΈ, силу АмпСра ΠΈ силу Π›ΠΎΡ€Π΅Π½Ρ†Π°. сила, Π·Π°Π΄Π°Π΅Ρ‚ вопросы Π½Π° Ρ€Π°Π±ΠΎΡ‡Π΅ΠΌ листС.

учащихся обсудят свой ΠΎΠΏΡ‹Ρ‚ с ΠΏΡ€Π°Π²ΠΈΠ»ΠΎΠΌ Π»Π΅Π²ΠΎΠΉ Ρ€ΡƒΠΊΠΈ, силой АмпСра ΠΈ силой Π›ΠΎΡ€Π΅Π½Ρ†Π°. Π‘ΠΌΠΎΡ‚Ρ€ΠΈΡ‚Π΅ Π²ΠΈΠ΄Π΅ΠΎ слСва ΠΏΡ€Π°Π²ΠΈΠ»ΠΎ, сила АмпСра ΠΈ сила Π›ΠΎΡ€Π΅Π½Ρ†Π°.ΠžΡ‚Π²Π΅Ρ‡Π°Π΅Ρ‚ Π½Π° вопросы ΠΎ Π²ΠΈΠ΄Π΅ΠΎ.

ΠŸΠΎΠΏΡ€ΠΎΠ±ΡƒΠΉΡ‚Π΅ Π·Π°Π΄Π°Ρ‚ΡŒ вопросы Π½Π° листС.

ΠŸΠΎΠ»Π΅Π·Π½Ρ‹ΠΉ рСсурс BBC Bitesize для пСрСсмотра прСдставлСний ΠΎ ΠΌΠ°Π³Π½ΠΈΡ‚Π½Ρ‹Ρ… полях ΠΈ ΠΈΡ… прСдставлСния с ΠΏΠΎΠΌΠΎΡ‰ΡŒΡŽ поля:

http://www.bbc.co.uk/schools/gcsebitesize/science/ocr_gateway_pre_2011/living_future/5_mintage_field1.shtml

Π’ этом Π²ΠΈΠ΄Π΅ΠΎ ΠΏΠΎΠΊΠ°Π·Π°Π½ простой дСмонстрация Β«ΠΌΠΎΡ‚ΠΎΡ€Π½ΠΎΠ³ΠΎ эффСкта» ΠΈ ΠΏΠΎΠΊΠ°Π·Ρ‹Π²Π°Π΅Ρ‚, ΠΊΠ°ΠΊ ΠΏΡ€ΠΈΠΌΠ΅Π½ΡΡ‚ΡŒ Π»Π΅Π²ΡƒΡŽ Ρ€ΡƒΠΊΡƒ Π€Π»Π΅ΠΌΠΈΠ½Π³Π° ΠŸΡ€Π°Π²ΠΈΠ»ΠΎ:

http: // www.youtube.com/watch?v=U9RezsWnPYs

Π­Ρ‚Ρƒ ΠΏΡ€ΠΎΡΡ‚ΡƒΡŽ Π΄Π΅ΠΌΠΎΠ½ΡΡ‚Ρ€Π°Ρ†ΠΈΡŽ ΠΌΠΎΠΆΠ½ΠΎ использован ΠΊΠ°ΠΊ классный экспСримСнт ΠΏΠΎ Π΄Π²ΠΈΠ³Π°Ρ‚Π΅Π»ΡŒΠ½ΠΎΠΌΡƒ эффСкту:

На этом ΠΊΠ»ΠΈΠΏΠ΅ ΠΏΠΎΠΊΠ°Π·Π°Π½ простой источник постоянного Ρ‚ΠΎΠΊΠ°. ΠΌΠΎΡ‚ΠΎΡ€ со Ρ‰Π΅Ρ‚ΠΊΠ°ΠΌΠΈ ΠΈ ΠΊΠΎΠΌΠΌΡƒΡ‚Π°Ρ‚ΠΎΡ€ΠΎΠΌ:

Анимация Β«ΠŸΡ€Π°Π²ΠΈΠ»Π° Π»Π΅Π²ΠΎΠΉ Ρ€ΡƒΠΊΠΈΒ»:

https://www.vascak.cz/data/android/physicsatschool/templateimg.php?s=mag_fleming&l=ru

Π’ этом ΠΊΠΎΡ€ΠΎΡ‚ΠΊΠΎΠΌ Π²ΠΈΠ΄Π΅ΠΎ ΠΏΠΎΠΊΠ°Π·Π°Π½ΠΎ, ΠΊΠ°ΠΊ ΠΈΠ·ΠΎΠ±Ρ€Π°ΠΆΠ΅Π½ΠΈΠ΅ Π½Π° старый Ρ‚Π΅Π»Π΅Π²ΠΈΠ·ΠΎΡ€ с Π­Π›Π’ искаТаСтся ΠΈΠ·-Π·Π° наличия ΠΌΠ°Π³Π½ΠΈΡ‚Π½ΠΎΠ³ΠΎ поля:

http: // www.youtube.com/watch?v=Se8UtAflums

На этом Π²ΠΈΠ΄Π΅ΠΎ ΠΏΠΎΠΊΠ°Π·Π°Π½ΠΎ ΠΎΡ‚ΠΊΠ»ΠΎΠ½Π΅Π½ΠΈΠ΅ элСктронный ΠΏΡƒΡ‡ΠΎΠΊ Π² ΠΎΠ΄Π½ΠΎΡ€ΠΎΠ΄Π½ΠΎΠΌ ΠΌΠ°Π³Π½ΠΈΡ‚Π½ΠΎΠΌ ΠΏΠΎΠ»Π΅, создаваСмый ΠΏΠ°Ρ€ΠΎΠΉ Π“Π΅Π»ΡŒΠΌΠ³ΠΎΠ»ΡŒΡ†Π° ΠΊΠ°Ρ‚ΡƒΡˆΠΊΠΈ:

Π˜Π½Ρ‚Π΅Ρ€Π°ΠΊΡ‚ΠΈΠ²Π½Π°Ρ модСль двиТСния частицы Π² ΠΌΠ°Π³Π½ΠΈΡ‚Π½ΠΎΠΌ ΠΏΠΎΠ»Π΅:

https://www.vascak.cz/data/android/physicsatschool/templateimg.php?s=mag_wehnelt&l=ru

Π£Ρ‡ΠΈΡ‚Π΅Π»ΡŒ выдСляСт ΠΊΠ»ΡŽΡ‡Π΅Π²Ρ‹Π΅ понятия, опрСдСлСния ΠΈ уравнСния, ΠΈΠ·ΡƒΡ‡Π΅Π½Π½Ρ‹Π΅ с ΠΏΠΎΠΌΠΎΡ‰ΡŒΡŽ ΠΊΠ°Ρ€Ρ‚Ρ‹ понятий.ΠŸΡ€ΠΎΡΠΈΡ‚ студСнтов ΡΠ΄Π΅Π»Π°Ρ‚ΡŒ вопросы ΠΏΠΎ прСдоставлСнному листу. Π‘ Π½Π΅Ρ‚Π΅Ρ€ΠΏΠ΅Π½ΠΈΠ΅ΠΌ ΠΆΠ΄Ρƒ ΡΠ»Π΅Π΄ΡƒΡŽΡ‰Π΅Π³ΠΎ ΡƒΡ€ΠΎΠΊΠ°.

Π‘Ρ‚ΡƒΠ΄Π΅Π½Ρ‚Ρ‹ ΠΏΠΎΠΏΡ‹Ρ‚Π°ΡŽΡ‚ΡΡ ΠΎΡ‚Π²Π΅Ρ‚ΠΈΡ‚ΡŒ Π½Π° Π·Π°Π΄Π°Π½Π½Ρ‹Π΅ вопросы ΡƒΡ‡ΠΈΡ‚Π΅Π»Π΅ΠΌ. ΠžΠ±ΠΎΠ±Ρ‰ΠΈΡ‚Π΅ основныС ΠΈΠ·ΡƒΡ‡Π΅Π½Π½Ρ‹Π΅ ΠΊΠΎΠ½Ρ†Π΅ΠΏΡ†ΠΈΠΈ, опрСдСлСния ΠΈ уравнСния. Π Π°Π·ΠΌΡ‹ΡˆΠ»ΡΠΉΡ‚Π΅ ΠΎ собствСнном ΠΎΠ±ΡƒΡ‡Π΅Π½ΠΈΠΈ. ΠžΡ†Π΅Π½ΠΈΡ‚Π΅ свою Ρ€Π°Π±ΠΎΡ‚Ρƒ ΠΈ Ρ€Π°Π±ΠΎΡ‚Ρƒ своих одноклассники.

ΠžΠ±ΠΎΠ±Ρ‰Π΅Π½ΠΈΠ΅: студСнтов спросят, ΠΎΠ½ΠΈ достигли Ρ†Π΅Π»Π΅ΠΉ ΡƒΡ€ΠΎΠΊΠ°.

Π’ ΠΊΠΎΠ½Ρ†Π΅ ΡƒΡ€ΠΎΠΊΠ° учащиСся ΡΠΏΡ€Π°ΡˆΠΈΠ²Π°ΡŽΡ‚ вопрос, Π΅ΡΡ‚ΡŒ Π»ΠΈ Ρƒ Π½ΠΈΡ….

Π£Ρ‡ΠΈΡ‚Π΅Π»ΡŒ собираСт Ρ€Π°Π·ΠΌΡ‹ΡˆΠ»Π΅Π½ΠΈΡ.

Π’ ΠΊΠΎΠ½Ρ†Π΅ ΡƒΡ‡ΠΈΡ‚Π΅Π»ΡŒ Π΄Π°Π΅Ρ‚ домашнСС Π·Π°Π΄Π°Π½ΠΈΠ΅: Π²Ρ‹ΠΏΠΎΠ»Π½ΠΈΡ‚ΡŒ ΡƒΠΊΠ°Π·Π°Π½Π½Ρ‹Π΅ Π·Π°Π΄Π°Ρ‡ΠΈ Π½Π° ΠΌΡ‹ΡˆΠ»Π΅Π½ΠΈΠ΅ для этого ΡƒΡ€ΠΎΠΊΠ°.

Π”ΠΎΠΏΠΎΠ»Π½ΠΈΡ‚Π΅Π»ΡŒΠ½Ρ‹ΠΉ ΠΌΠ½ΠΎΠ³ΠΎΡƒΡ€ΠΎΠ²Π½Π΅Π²Ρ‹ΠΉ (Π½Π° диффСрСнциация) Π·Π°Π΄Π°Ρ‡ΠΈ

Π£Ρ‡ΠΈΡ‚Π΅Π»ΡŒ Π½Π°Π·Π½Π°Ρ‡Π°Π΅Ρ‚ вопросы 1, 2, 3, 4, 5, 6 слабым ΡƒΡ‡Π΅Π½ΠΈΠΊΠ°ΠΌ, вопросы 7, 8, 9, 10, 11 срСдним ΡƒΡ‡Π΅Π½ΠΈΠΊΠ°ΠΌ, вопросы 4, 7 (Π°, Π±), 12, 13 ΡΠΈΠ»ΡŒΠ½Ρ‹ΠΌ ΡƒΡ‡Π΅Π½ΠΈΠΊΠ°ΠΌ.

Π Π΅ΠΊΠΎΠΌΠ΅Π½Π΄Π°Ρ†ΠΈΠΈ ΠΏΠΎ Ρ„ΠΎΡ€ΠΌΠΈΡ€ΡƒΡŽΡ‰Π΅ΠΌΡƒ ΠΎΡ†Π΅Π½ΠΈΠ²Π°Π½ΠΈΡŽ

студСнтов выполнят эту Ρ€Π°Π±ΠΎΡ‚Ρƒ ΡΠ°ΠΌΠΎΡΡ‚ΠΎΡΡ‚Π΅Π»ΡŒΠ½ΠΎ послС Ρ‚ΠΎΠ³ΠΎ, ΠΊΠ°ΠΊ ΡƒΡ‡ΠΈΡ‚Π΅Π»ΡŒ ΠΏΠΎΠΊΠ°ΠΆΠ΅Ρ‚ ΠΎΡ‚Π²Π΅Ρ‚Ρ‹ Π½Π° доска, учащиСся провСрят сСбя.

ΠžΡ‚Π²Π΅Ρ‚Ρ‹, ΠΊΡ€ΠΈΡ‚Π΅Ρ€ΠΈΠΈ Π·Π°Π΄Π°Π½ΠΈΠΉ, Π΄ΠΎΠΏΠΎΠ»Π½ΠΈΡ‚Π΅Π»ΡŒΠ½Ρ‹Π΅ ΠΌΠ°Ρ‚Π΅Ρ€ΠΈΠ°Π»Ρ‹ ΠΊ ΡƒΡ€ΠΎΠΊΡƒ

ΠŸΠΎΡΡ‚ΠΎΡΠ½Π½Ρ‹Π΅ ΠΌΠ°Π³Π½ΠΈΡ‚Ρ‹

Π‘ΠΈΠ»Π°, ΠΊΠΎΡ‚ΠΎΡ€ΡƒΡŽ ΠΌΠ°Π³Π½ΠΈΡ‚Π½ΠΎΠ΅ ΠΏΠΎΠ»Π΅ ΠΎΠΊΠ°Π·Ρ‹Π²Π°Π΅Ρ‚ Π½Π° ΠŸΠΎΠ΄Π²ΠΈΠΆΠ½Ρ‹ΠΉ заряд

1. E

2. D

3. E

4. A

5. B

6. A

7. C

8. D

9. C

10. А

11. E

Π—Π°ΠΊΠΎΠ½ АмпСра. ΠœΠ°Π³Π½ΠΈΡ‚Π½Ρ‹Π΅ ΠΌΠ°Ρ‚Π΅Ρ€ΠΈΠ°Π»Ρ‹

12. D

13. B

ΠžΡ‚Π²Π΅Ρ‚ΠΈΠ»ΠΎ ΠΈ сработало Ρ€Π΅ΡˆΠ΅Π½ΠΈΡ

1 (Π°) F = Π‘Π˜Π›

(Π±) B = F / IL

B = [2,0 x 10 9 Β· 1020 -3 x 9,81] / [4 x 0,15] = 0,0327 T (Π½Π΅ Π·Π°Π±ΡƒΠ΄ΡŒΡ‚Π΅ ΠΈΠ·ΠΌΠ΅Π½ΠΈΡ‚ΡŒ Π³Ρ€Π°ΠΌΠΌΡ‹ Π½Π° ΠΊΠ³ ΠΈ сантимСтры Π² ΠΌΠ΅Ρ‚Ρ€Ρ‹)

2 ВрСбуСмая масса = BIL

Π‘Π»Π΅Π΄ΠΎΠ²Π°Ρ‚Π΅Π»ΡŒΠ½ΠΎ: 1.5 Ρ… 10 -3 x 9,81 = 0,5 x I x 0,06, Ρ‡Ρ‚ΠΎ Π΄Π°Π΅Ρ‚ I = 0,49 A

3 F = BIL = 0,2 x 2 x 0,5 = 0,2 Н

4 F = 2,5 x10 -5 N

5 Для Π΄Π»ΠΈΠ½Ρ‹ 20 см

6. Если ΠΊΠ°ΠΆΠ΄Ρ‹ΠΉ ΠΏΡ€ΠΎΠ²ΠΎΠ΄ ΠΈΠΌΠ΅Π΅Ρ‚ Ρ‚ΠΎΠΊ 3,0 А, это Ρ‚ΠΎ ΠΆΠ΅ самоС, Ρ‡Ρ‚ΠΎ ΠΈ эффСктивный Ρ‚ΠΎΠΊ 9,0 А,

, поэтому сила F = 3 * 0,625 = 1,9 Н.

7 (a) ΠŸΡ€ΠΎΠ²ΠΎΠ΄ Π·Π²ΡƒΠΊΠΎΠ²ΠΎΠΉ ΠΊΠ°Ρ‚ΡƒΡˆΠΊΠΈ располоТСн ΠΏΠΎΠ΄ прямым ΡƒΠ³Π»ΠΎΠΌ ΠΊ ΠΏΠΎΠ»Π΅.Когда ΠΏΠ΅Ρ€Π΅ΠΌΠ΅Π½Π½Ρ‹ΠΉ Ρ‚ΠΎΠΊ ΠΎΡ‚ усилитСля Ρ‚Π΅Ρ‡Π΅Ρ‚ Π² ΠΊΠ°Ρ‚ΡƒΡˆΠΊΠ΅, ΠΎΠ½Π° испытываСт силу, которая ΠΏΠ΅Ρ€Π΅ΠΌΠ΅Ρ‰Π°Π΅Ρ‚ ΠΊΠ°Ρ‚ΡƒΡˆΠΊΡƒ Π²ΠΏΠ΅Ρ€Π΅Π΄ ΠΈ Π½Π°Π·Π°Π΄ ΠΏΠΎ мягкому ΠΆΠ΅Π»Π΅Π·Π½ΠΎΠ΅ ядро.

(б) F = nBIL = 0,013 Н

(ΠŸΡ€ΠΈΠΌΠ΅Ρ‡Π°Π½ΠΈΠ΅: см ΠΊ ΠΌ ΠΈ мА ΠΊ А)


Π‘ΠΊΠ°Ρ‡Π°Π½ΠΎ с www.znanio.ru

Π‘ΠΈΠ»Π° Π›ΠΎΡ€Π΅Π½Ρ†Π°, сила АмпСра ΠΈ Π·Π°ΠΊΠΎΠ½ сохранСния ΠΈΠΌΠΏΡƒΠ»ΡŒΡΠ° ΠšΠΎΠ»ΠΈΡ‡Π΅ΡΡ‚Π²Π΅Π½Π½Ρ‹ΠΉ. Анализ ΠΈ слСдствия., Архив элСктронной ΠΏΠ΅Ρ‡Π°Ρ‚ΠΈ viXra.org, viXra: 1407.0066

Π‘ΠΈΠ»Π° Π›ΠΎΡ€Π΅Π½Ρ†Π°, сила АмпСра ΠΈ Π·Π°ΠΊΠΎΠ½ сохранСния количСства двиТСния. Анализ ΠΈ слСдствия., Архив элСктронной ΠΏΠ΅Ρ‡Π°Ρ‚ΠΈ viXra.org, viXra: 1407.0066

Π‘ΠΈΠ»Π° Π›ΠΎΡ€Π΅Π½Ρ†Π°, сила АмпСра ΠΈ Π·Π°ΠΊΠΎΠ½ сохранСния ΠΈΠΌΠΏΡƒΠ»ΡŒΡΠ° ΠšΠΎΠ»ΠΈΡ‡Π΅ΡΡ‚Π²Π΅Π½Π½Ρ‹ΠΉ. Анализ ΠΈ слСдствия.

Авторы: Π₯мСльник Π‘ΠΎΠ»ΠΎΠΌΠΎΠ½ Π˜Π²Π°Π½ΠΎΠ²ΠΈΡ‡

Π˜Π·Π²Π΅ΡΡ‚Π½ΠΎ, Ρ‡Ρ‚ΠΎ сила Π›ΠΎΡ€Π΅Π½Ρ†Π° ΠΈ сила АмпСра ΠΏΡ€ΠΎΡ‚ΠΈΠ²ΠΎΡ€Π΅Ρ‡Π°Ρ‚ Ρ‚Ρ€Π΅Ρ‚ΡŒΠ΅ΠΌΡƒ Π·Π°ΠΊΠΎΠ½Ρƒ ΠΡŒΡŽΡ‚ΠΎΠ½Π°, Π½ΠΎ Π½Π΅ ΠΏΡ€ΠΎΡ‚ΠΈΠ²ΠΎΡ€Π΅Ρ‡Π°Ρ‚ Π±ΠΎΠ»Π΅Π΅ ΠΎΠ±Ρ‰Π΅ΠΌΡƒ Π·Π°ΠΊΠΎΠ½Ρƒ сохранСния ΠΈΠΌΠΏΡƒΠ»ΡŒΡΠ°, ΠΏΠΎΡΠΊΠΎΠ»ΡŒΠΊΡƒ элСктромагнитноС ΠΏΠΎΠ»Π΅ ΠΈΠΌΠ΅Π΅Ρ‚ ΠΈΠΌΠΏΡƒΠ»ΡŒΡ.Из этого слСдуСт, Ρ‡Ρ‚ΠΎ силы Π›ΠΎΡ€Π΅Π½Ρ†Π° ΠΈ АмпСра Π΄ΠΎΠ»ΠΆΠ½Ρ‹ ΡƒΡ€Π°Π²Π½ΠΎΠ²Π΅ΡˆΠΈΠ²Π°Ρ‚ΡŒΡΡ ΠΏΠΎΡ‚ΠΎΠΊΠΎΠΌ ΠΈΠΌΠΏΡƒΠ»ΡŒΡΠ° элСктромагнитного поля. Однако, насколько извСстно Π°Π²Ρ‚ΠΎΡ€Ρƒ, ΡΠΎΠΎΡ‚Π²Π΅Ρ‚ΡΡ‚Π²ΡƒΡŽΡ‰Π΅Π³ΠΎ количСствСнного сравнСния Π½Π΅Ρ‚, поэтому ΠΎΠ½ΠΎ обсуТдаСтся Π½ΠΈΠΆΠ΅. Π’ частности, ΠΏΠΎΠΊΠ°Π·Π°Π½ΠΎ, Ρ‡Ρ‚ΠΎ ΠΌΠΎΠΆΠ½ΠΎ Π½Π°ΠΉΡ‚ΠΈ Π½Π΅ΠΊΠΎΡ‚ΠΎΡ€Ρ‹Π΅ слСдствия ΠΈΠ· Π·Π°ΠΊΠΎΠ½Π° сохранСния ΠΈΠΌΠΏΡƒΠ»ΡŒΡΠ°.

ΠšΠΎΠΌΠΌΠ΅Π½Ρ‚Π°Ρ€ΠΈΠ΅Π²: 8 стр.

Π—Π°Π³Ρ€ΡƒΠ·ΠΈΡ‚ΡŒ: PDF

Π˜ΡΡ‚ΠΎΡ€ΠΈΡ прСдставлСний

[v1] 09.07.2014, 09:39:08
[v2] 17.09.2014 06:50:12

Π—Π°Π³Ρ€ΡƒΠ·ΠΎΠΊ Π΄ΠΎΠΊΡƒΠΌΠ΅Π½Ρ‚ΠΎΠ² с ΡƒΠ½ΠΈΠΊΠ°Π»ΡŒΠ½Ρ‹ΠΌ IP: 255 Ρ€Π°Π·

Vixra.org — это скорСС Ρ€Π΅ΠΏΠΎΠ·ΠΈΡ‚ΠΎΡ€ΠΈΠΉ Π΄ΠΎΠΏΠ΅Ρ‡Π°Ρ‚Π½ΠΎΠΉ Π΄ΠΎΠΊΡƒΠΌΠ΅Π½Ρ‚Π°Ρ†ΠΈΠΈ, Ρ‡Π΅ΠΌ ΠΆΡƒΡ€Π½Π°Π». Π Π°Π·ΠΌΠ΅Ρ‰Π΅Π½Π½Ρ‹Π΅ ΡΡ‚Π°Ρ‚ΡŒΠΈ ΠΌΠΎΠ³ΡƒΡ‚ Π΅Ρ‰Π΅ Π½Π΅ ΠΏΡ€ΠΎΠΉΡ‚ΠΈ Ρ€Π΅Ρ†Π΅Π½Π·ΠΈΡ€ΠΎΠ²Π°Π½ΠΈΠ΅ ΠΈ Π΄ΠΎΠ»ΠΆΠ½Ρ‹ Ρ€Π°ΡΡΠΌΠ°Ρ‚Ρ€ΠΈΠ²Π°Ρ‚ΡŒΡΡ ΠΊΠ°ΠΊ ΠΏΡ€Π΅Π΄Π²Π°Ρ€ΠΈΡ‚Π΅Π»ΡŒΠ½Ρ‹Π΅. Π’ частности, слСдуСт ΠΎΡ‚Π½ΠΎΡΠΈΡ‚ΡŒΡΡ с Π΄ΠΎΠ»ΠΆΠ½ΠΎΠΉ ΠΎΡΡ‚ΠΎΡ€ΠΎΠΆΠ½ΠΎΡΡ‚ΡŒΡŽ ΠΊΠΎ всСму, Ρ‡Ρ‚ΠΎ ΠΌΠΎΠΆΠ΅Ρ‚ Π²ΠΊΠ»ΡŽΡ‡Π°Ρ‚ΡŒ финансовыС ΠΈΠ»ΠΈ ΡŽΡ€ΠΈΠ΄ΠΈΡ‡Π΅ΡΠΊΠΈΠ΅ ΠΊΠΎΠ½ΡΡƒΠ»ΡŒΡ‚Π°Ρ†ΠΈΠΈ ΠΈΠ»ΠΈ ΠΏΡ€Π΅Π΄Π»Π°Π³Π°Π΅ΠΌΡ‹Π΅ мСдицинскиС ΠΏΡ€ΠΎΡ†Π΅Π΄ΡƒΡ€Ρ‹. Vixra.org Π½Π΅ нСсСт отвСтствСнности Π·Π° Π»ΡŽΠ±Ρ‹Π΅ послСдствия дСйствий, Π²ΠΎΠ·Π½ΠΈΠΊΡˆΠΈΡ… Π² Ρ€Π΅Π·ΡƒΠ»ΡŒΡ‚Π°Ρ‚Π΅ использования Π»ΡŽΠ±Ρ‹Ρ… Π΄ΠΎΠΊΡƒΠΌΠ΅Π½Ρ‚ΠΎΠ² Π½Π° этом Π²Π΅Π±-сайтС Π² любой Ρ„ΠΎΡ€ΠΌΠ΅.

Π”ΠΎΠ±Π°Π²ΡŒΡ‚Π΅ свой ΠΎΡ‚Π·Ρ‹Π² ΠΈ вопросы здСсь:
Π’Ρ‹ Ρ‚Π°ΠΊΠΆΠ΅ ΠΌΠΎΠΆΠ΅Ρ‚Π΅ ΠΏΠΎΠ»ΠΎΠΆΠΈΡ‚Π΅Π»ΡŒΠ½ΠΎ ΠΈΠ»ΠΈ ΠΎΡ‚Ρ€ΠΈΡ†Π°Ρ‚Π΅Π»ΡŒΠ½ΠΎ ΠΎΡ‚Π½ΠΎΡΠΈΡ‚ΡŒΡΡ ΠΊ любой ΡΡ‚Π°Ρ‚ΡŒΠ΅, Π½ΠΎ Π±ΡƒΠ΄ΡŒΡ‚Π΅ Π²Π΅ΠΆΠ»ΠΈΠ²Ρ‹.Если Π²Ρ‹ ΠΊΡ€ΠΈΡ‚ΠΈΠΊΡƒΠ΅Ρ‚Π΅, Π²Ρ‹ Π΄ΠΎΠ»ΠΆΠ½Ρ‹ ΡƒΠΊΠ°Π·Π°Ρ‚ΡŒ хотя Π±Ρ‹ ΠΎΠ΄Π½Ρƒ ΠΊΠΎΠ½ΠΊΡ€Π΅Ρ‚Π½ΡƒΡŽ ΠΎΡˆΠΈΠ±ΠΊΡƒ, ΠΈΠ½Π°Ρ‡Π΅ ваш ΠΊΠΎΠΌΠΌΠ΅Π½Ρ‚Π°Ρ€ΠΈΠΉ Π±ΡƒΠ΄Π΅Ρ‚ ΡƒΠ΄Π°Π»Π΅Π½ ΠΊΠ°ΠΊ бСсполСзный.

alexxlab

Π”ΠΎΠ±Π°Π²ΠΈΡ‚ΡŒ ΠΊΠΎΠΌΠΌΠ΅Π½Ρ‚Π°Ρ€ΠΈΠΉ

Π’Π°Ρˆ адрСс email Π½Π΅ Π±ΡƒΠ΄Π΅Ρ‚ ΠΎΠΏΡƒΠ±Π»ΠΈΠΊΠΎΠ²Π°Π½. ΠžΠ±ΡΠ·Π°Ρ‚Π΅Π»ΡŒΠ½Ρ‹Π΅ поля ΠΏΠΎΠΌΠ΅Ρ‡Π΅Π½Ρ‹ *